Инфоурок Алгебра КонспектыПоурочные планы по алгебре 7 класса

Поурочные планы по алгебре 7 класса

Скачать материал

Урок1                                                                                                                      Дата ______________
Тема:      Числовые выражения.

Цели: ввести понятия числового выражения, значения числового выражения; формировать умение находить значение числового выражения, выполняя действия над числами и используя скобки.

I. Устная работа.

Вычислите.

а) 13 – 18,5;                     б) –19 + 21,3;       в) –14 – 71,03;    

г) 17 – (–21,3);                 д) – (–3 – 2,8);                 е) 3 · 15 – 7;

ж) 12 – 16 : 4;                  з) (15 – 2) · (–3);              и) (–2) ∙  ;             к) 7 : .

II. Объяснение нового материала.

1. Для введения понятия «числовое выражение» целесообразно сообщить  учащимся  следующую  информацию.  При  решении многих задач приходится над заданными числами производить арифметические действия: сложение, вычитание, умножение и деление. Но часто, прежде чем доводить до конца каждое из этих действий, удобно заранее указать порядок (план), следуя которому надо производить эти действия. Этот план сводится к тому, что по данным задачи с помощью чисел, знаков действий и скобок составляется числовое выражение.

2. Разбираем задачу со с. 3 учебника и показываем на примере полученное числовое выражение.

следует привести достаточное число различных числовых выражений:

43 : 5;                            9,6 – 3 · 1,2;                        5 · (7,4 – 6,1);

;                  (39 – 15) : 23 + .

3. Если в числовом выражении выполнить все указанные в нем действия, то в результате получим действительное число, про которое говорят, что оно равно данному числовому выражению и называется значением выражения.

Подчеркнем, что числовое выражение дает указание, какие арифметические действия и в каком порядке мы должны произвести над данными числами. Скобки помогают установить порядок действий.

Задание. расставить над знаками арифметических действий порядковые номера их выполнения.

3,5 – 8 · 2,7 + 2,5 : 3 – 112 · 5;

(3,5 – 8) · 2,7 + 2,5 : (3 – 112) · 5;

3,5 – 8 · (2,7 + 2,5 : 3) – 112 · 5;

3,5 – 8 · (2,7 + 2,5 : (3 – 112)) · 5.

4. № 1 (а, г, ж).

Решение:

а) 6,965 + 23,3 = 30, 265;

г) 6,5 · 1,22 = 7,93;

ж) 53,4 : 15 = 3,56.

5. Мы, конечно, предполагаем, что все действия возможно осуществить. Поясним эти слова. Всегда возможно произвести сложение, вычитание и умножение любых чисел. А вот делить числа одно на другое возможно, только если делитель не равен нулю: на нуль делить нельзя. Если в данном выражении на некотором его этапе требуется делить на нуль, то это требование неосуществимо. Такое выражение не имеет смысла.

Например, выражения 35 : (4 · 2 – 8) и 0,37 –  не имеют смысла, потому что при выполнении указанных в них действий появляется необходимость делить на нуль.

6. Замечаем,  что  числовое  выражение  может  состоять  и  из  одного числа.

III. Формирование умений и навыков.

Все  упражнения,  выполняемые  на  этом  уроке,  можно  разбить  на группы:

1-я группа. Нахождение значения числового выражения, представляющего собой сумму или разность, произведение или частное.

2-я группа. Нахождение значения числового выражения, содержащего в записи два и более арифметических действия, а также скобки.

3-я группа. Задания на составление числовых выражений, отвечающих заданным условиям (наличие или отсутствие смысла, равенство определенному значению).

1-я группа

1. № 1 (б; д; з). Самостоятельно.

2. Найдите сумму или разность.

а) ;             б) ;                            в) ;

г) ;                     д) ;                           е) ;

ж) ;                   з) ;                     и) .

3. Найдите значение выражения.

а) 7 + 5,31 + 9 + 13,49;

б) 62,7 + 8,31 + 5,79 + 0,07.

4. № 4 (д, е, ж, з); № 5 (а, г, ж); № 6 (а, г, ж).

2-я группа

1. № 3 (а, б).

2. Найдите значение выражения.

а) ;                      б) ;

в) ;                г) .

3. Вычислите.

а) (0,008 + 0,992) : (5 · 0,6 – 1,4);

б) .

3-я группа

1. № 13.

2. Записать несколько числовых выражений, значение которых равно:

а) 8;   б) 0;                      в) –14;   г) 3,76.

3. Придумать два примера числовых выражений, где бы участвовали все арифметические действия, причем одно из них имело бы смысл, а второе нет.

IV. Итоги урока.

– Что называется значением числового выражения?

– Для чего в записи числового выражения присутствуют скобки?

– Когда числовое выражение имеет смысл? Приведите пример такого выражения.

– Когда числовое выражение не имеет смысла? Приведите пример такого выражения.

Домашнее задание.

1. № 1  (в, е, и);  № 2;  № 4  (а, б, в, г);  № 5  (б, в, д, е, з, и) (устно);  № 6 (б, д, з).

Урок 2                                                             Дата __________
Тема: Числовые выражения

Цели: продолжить формировать умение находить значение числового выражения; формировать умение составлять числовое выражение по условию задачи и находить его значение.

Ход урока

I. Устная работа.

1. Найдите значение числового выражения.

а) ;               б) ;     в) ;                г) ;

д) –0,3 · (–0,2);        е) –3,7 – 2,3;              ж) –8,1 : (–9);               з) –3,4 + 7,5.

2. Найдите:

а) 10 %  от  480;              г) 25 %  от  36;

б) 1 %  от  500;                д) 2 %  от  150;

в) 50 %  от  23;                е) 20 %  от  45.

II. Проверочная работа.

Вариант 1

1. Вычислите.

а) ;                           б) .

2. Найдите значение выражения.

0,7 · 1,3 + 5,1 : 0,17.

Вариант 2

1. Вычислите.

а) ;                    б) .

2. Найдите значение выражения.

1,8 · 0,4 + 6,4 : 0,16.

III. Актуализация знаний.

На этом уроке учащиеся составляют числовое выражение по условию задачи на проценты, на работу, на движение. Следует повторить понятия «процент», нахождение процента от числа и числа по его проценту.

1. № 7 (в; г).

2. Найдите число, если 17 % его равны:

а) 340;                              б) 2,89.

3. Сколько процентов число 8 составляет от числа:

а) 16;                                б) 0,8?

4. Выразите десятичной дробью числа его процент:

а) 43 %;                            б) 11,4 %.

5. Выразите в процентах дробь числа:

а) 0,5;                               б) 1,35.

IV. Формирование умений и навыков.

При решении упражнений следует обращать внимание учащихся на обоснование  получаемого  числового  выражения,  а  также  на  грамотную запись.

1. № 8.

Решение:

Для нахождения количества жира в молоке необходимо найти 3,2 % от числа 200. Выразим 3,2 % обыкновенной или десятичной дробью:

3,2 % =  = 0,032.

Умножим  данное  число  на  дробь  и  получим  числовое  выражение: 0,032 · 200. Найдем значение данного выражения, оно равно 6,4.

Аналогично находим количество белка и углеводов:

2,5 % =  = 0,025;     0,025 · 200 = 5.

4,7 % =  = 0,047;     0,047 · 200 = 9,4.

Ответ: 6,4 г; 5 г; 9,4 г.

2. Завод по плану должен был изготовить 537 000 изделий. План был выполнен на 102,5 %. Установите:

а) сколько изделий выпустил завод;

б) сколько изделий выпустил завод сверх плана.

Решение:

а) Выразим 102,5 % обыкновенной или десятичной дробью:

102,5 % =  = 1,025 = .

Умножим данное число на дробь и найдем значение выражения.

537 000 ∙   = 537 ∙  1025 = 550 425.

б) По плану завод должен был изготовить 537 000, а изготовил 550 425 изделий, значит, сверх плана он изготовил:

550 425 – 537 000 = 13 425.

Ответ: а) 550 425; б) 13 425.

Замечание. Обращаем  внимание  учащихся,  что  перед  вычислением  значения числового выражения следует подумать, нельзя ли применить свойства действий для более удобных вычислений. Записывать промежуточные результаты, получаемые от применения свойств действий, следует только тогда, когда становится затруднительным их запоминание. Данные навыки являются неотъемлемой частью вычислительной культуры учащихся.

3*. Цена изделия сначала возросла на 20 %, а затем на столько же процентов снизилась. Как и на сколько процентов изменилась цена по сравнению с первоначальной?

Решение:Цена выросла на 20 %, то есть составляет 120 % от первоначальной. Выразим 120 % в виде обыкновенной дроби:  = 1,2. Следующее изменение составляет уменьшение на 20 %, то есть цена составит 80 % от предыдущей.

80 % =  = 0,8.

Значит, изменение цены составило 1,2 · 0,8 = 0,96. Таким образом, новая цена составляет 0,96 от исходной, или, в процентах, 96 %. Значит, цена уменьшилась по сравнению с первоначальной на 4 %.

Ответ: уменьшилась на 4 %.

4. № 15.

Решение:

За  3  часа  первый  пешеход  прошел  3 · 4  (км),  а  второй  пешеход
3 · 5 (км), значит, вместе они прошли 3 · 4 + 3 · 5 (км).

Зная, что расстояние между пунктами составляет 40 км, найдем расстояние между пешеходами:

40 – (3 · 4 + 3 · 5) = 13.

Ответ: 13 км.

5. Следующая группа заданий направлена на формирование грамотной математической речи и закрепление навыков по нахождению значения числовых выражений.

№ 12; № 17 (устно); № 18.

V. Итоги урока.

– Что называется значением числового выражения?

– Каков порядок выполнения действий при нахождении значения числового выражения?

– Как выражается 18 % в виде обыкновенной или десятичной дроби?

– Какие термины используются при прочтении выражения  (3,7 – 4) :
: (7 + 35,8)?

Домашнее задание: 1. № 9, № 10, № 16.

2. № 12, № 11* (дополнительное задание).

 

 

 

 

 

 

 

 

 

 

 

 

 

Урок 3                                                                                         Дата __________________
Тема: Выражение с переменной и его числовое значение

Цели: ввести понятия «переменная», «выражение с переменной», «числовое значение выражения с переменной»; формировать умение находить значение выражения с переменной, используя различные формы записи («если … , то …», таблица).

Ход урока

I. Устная работа.

1. Назовите числовые выражения, не имеющие смысла.

а)  + 8 : 4 – 2 ∙  2;                б) ;              в) ;

г) 3,4 : 8 ∙  (–2) + 16;              д) 3 : (3 ∙  0,9 – 2,7) + 2; е) .

2. Найдите значение числового выражения.

а)  ∙  (–9);          б) ;              в) ;               г) ;

д) 33;                     е) (–8)2;                          ж) ;               з) (–0,2)2.

II. Объяснение нового материала.

1. Мотивация изучения.

При решении многих практических задач удобно для обозначения различных чисел использовать буквы.

Например,  если  а  и  b – длины  сторон  прямоугольника,  то  выражение а · b показывает способ вычисления его площади. Это утверждение носит общий характер, оно относится к любому прямоугольнику, имеющему любые значения длин сторон; а и bпеременные, входящие в запись выражения.

Затем рассматриваем задачу со с. 5 учебника. Выражение 60t обозначает путь, пройденный автомобилем за некоторый промежуток времени. Подчеркиваем, что в этом выражении t является переменной, подставляя вместо t различные значения, мы можем находить путь, пройденный автомобилем за различные промежутки времени.

2. Определение 1. Если в числовом выражении некоторые (или все) входящие в него числа заменить буквами, то получим выражение с переменными (переменной).

Определение 2. Если в выражение с переменными подставить вместо каждой переменной какое-либо её значение, то получится числовое выражение. Его называют значением выражения с переменными при выбранных значениях переменных.

3. Необходимо ввести понятие допустимых значений переменных, входящих в выражения с переменными. Рассматриваем различные примеры выражений с переменными, имеющих смысл при любых значениях переменных (всех значениях) и не имеющих смысла при некоторых значениях переменной.

III. Формирование умений и навыков.

На  этом  уроке  отрабатываются  умения  выполнять  в  буквенных  выражениях  числовые  подстановки  и  производить  соответствующие  вычисления.

Все задания можно условно разделить на группы:

1-я группа. Нахождение значения выражения с одной переменной с использованием записи «если… , то…» и таблицы.

2-я группа. Нахождение значения выражения с несколькими переменными с использованием записи «если… , то…» и таблицы.

3-я группа. Нахождение значения выражения с переменными при заданных значениях других выражений с переменными.

1-я группа

1. Найдите значение выражения.

а) х + 3,2 при х = –6,8; –3,2; 1;

б) –5у при у = –2,6; 0; 1; 2;

в) 12а – 7 при а = –1; 0; –7,6; 0,05;

г) 3 – 1,5т при т = 4; –2; –; 0,8.

При  выполнении  задания  обращаем  внимание  учащихся  на  запись решения.

Решение:

а) если х = –6,8, то х + 3,2 = –6,8 + 3,2 = –3,6;

б) если х = –3,2, то х + 3,2 = –3,2 + 3,2 = 0;

в) если х = 1, то x + 3,2 = 1 + 3,2 =  + 3 = =
= .

2. № 21.

Решение:

у

–3

–1

0

2

3

4

6

10 – 2у

16

12

10

6

4

2

–2

10 + 2у

4

8

10

14

16

18

22

Данное задание можно вынести на доску. Каждый ученик самостоятельно выполняет все задания в тетради, а затем «по цепочке» ученики выходят к доске и заполняют соответствующую ячейку таблицы. Также данное задание можно выполнить устно.

3. Заполните таблицу.

 

–3

–2

–1

0

1

2

3

х (3 – 5х)

–54

–26

–8

0

–2

–14

–36

2-я группа

1. № 22 (устно); № 23.

2. Найдите значение выражения.

а) 8т + 3п + 1, при т = –4 и п = 10; т = –6,5 и n = 4.

б) (а + b) · (аb), при а = 1,7 и b = –1,3;

в) 2 – 0,3 · (b + 3а), при а = –0,2 и b = 0,6;

г) , при а = 2,8 и b = 0.

3-я группа

1. Пусть х + у = 5 и z = –8. Найдите:

а) х + уz;                       в) x – 5z + y;               д) ;

б) 2z – (х + у);                  г) 3 (х + у) + 2z;                     е) z (х + у + 5z).

2. № 27.

IV. Проверочная работа.

Вариант 1

1. Заполните таблицу:

р

0

–1

2

–3

3

t

–7

–2

3

0

9

p (3tp)

 

 

 

 

 

 

2. Найдите значение выражения х + у – 2z, если х + у = 3 и z = –2.

Вариант 2

1. Заполните таблицу:

т

0

–1

3

2

–2

п

–2

–3

6

0

1

т (п – 2т)

 

 

 

 

 

 

2. Найдите значение выражения аb + 3c, если аb = 11 и с = –6.

V. Итоги урока.

– Что называется выражением с переменной?

– Может ли выражение состоять из одной буквы?

– Как найти значение выражения с переменной при определенном значении переменной?

– Какие способы записи можно использовать при нахождении значения выражения с переменной?

Домашнее задание: № 19, № 20, № 24 (а; в), № 26 (а; в), № 28.

 

 

Урок                                                    Дата _________
Тема: Выражения с переменными

Цели: продолжить формировать умение находить значение выражения с переменными; формировать умение составлять выражение с переменными  по  условию  задачи,  в  том  числе  формулы,  и  находить  их значение.

Ход урока

I. Устная работа.

1. Назовите выражения, не имеющие смысла.

а) 2 · 4 – 8;                       б) 3 · 2 : (6 – 1,5 · 4);      в) ;

г) 3 : 3 – 7 · 2;                  д) ;                         е) ;

ж) 2 : 4 – 2;                      з) 3 : .

2. Найдите значение выражения 3аb, если:

а) а = 2 и b = –4;              б) а = 0 и b = ;

в) а = –4 и b = 5;              г) а = – и b = .

3. Сколько процентов составляет:

а) 50 от 200;                     б) 13 от 260;

в) 1,5 от 20;                     г) 240 от 80?

II. Объяснение нового материала.

На этом уроке учащиеся по аналогии с числовым выражением формулируют понятие выражения с переменной, не имеющего смысла. Затем вводится понятие формулы. Следует привести примеры различных формул, применяемых на практике (вычисление площадей, объемов, числовые формулы и т. п.). Также следует объяснить учащимся, что есть стабильные формулы, которые уже выведены и могут использоваться для расчетов. А есть задачи, для решения которых необходимо самостоятельно выявить закономерности (зависимости), описанные в условии, ввести переменные, составить выражение с переменными (формулу) и использовать его для вычисления искомого задачи при конкретных исходных данных.

III. Формирование умений и навыков.

Упражнения, решаемые на этом уроке, можно разбить на две группы:

1-я группа. Задачи практического характера, для решения которых необходимо составить формулу и вычислить по ней результат.

2-я группа. Задания на формирование грамотной математической речи (использование терминов «сумма», «разность», «произведение» и «частное») при чтении и записи выражений с переменными.

1-я группа

1. № 29.

 

 

Решение:

Если площадь первого участка а га, а с каждого га собрали 32 ц пшеницы, то со всего участка собрали 32а ц пшеницы. Аналогично получаем для  второго  участка  урожай  40b  ц  пшеницы.  Тогда  с  обоих  участков  был  собран  урожай  32а + 40b (ц).  Если  а = 120  и  b = 80, то 32а + + 40b = 32 · 120 + 40 · 80 = 3840 + 3200 = 7040.

Ответ: 32а + 40b (ц); 7040 ц.

2. № 31.

Решение:

Фигура состоит из отдельных частей. её площадь можно найти двумя способами:

1-й способ. «Разбить» фигуру на отдельные фигуры, для которых можно легко найти площадь, и, сложив полученные результаты, получить общую площадь.

Площадь состоит из суммы площадей трех прямоугольников со сторонами:  d и сd и са и bс.  Их  площади  соответственно  равны: сd; сd; а (bс). Значит, площадь искомой фигуры составляет:

сd + сd + а (bс)    или    2сd + а (bс).

2-й способ. Представить фигуру в виде прямоугольника со сторонами а и b с «вырезанным» прямоугольником со сторонами с и а – 2d. Их площади соответственно равны аb и с (а – 2d). Значит, площадь искомой фигуры составляет аbс (а – 2d).

Ответ: 2сd + а (bс) (см2) или аbс (а – 2d) (см2).

3. № 33.

Решение:

После  добавления   5 г  соли  в  раствор  масса   его  стала  равна 255 г. Масса  чистой  соли  в  растворе  также  увеличилась  на  5 г  и  стала  составлять  (х + 5) г.  Концентрация   соли,   таким   образом,  составляет  ∙  100 %.

Ответ:  ∙  100 %.

4. № 35 (устно); № 36 (устно).

2-я группа

1. № 37 (устно); № 38.

2. № 39 (устно); № 40 (устно).

3. № 41 (устно); № 42.

iV. Проверочная работа.

Вариант 1

Составьте выражение для вычисления площади пола, уложенного п квадратными  плитками  со  стороной  а  см.  Вычислите  эту  площадь, если а = 20 и п = 500.

Вариант 2

Составьте выражение для вычисления пути, пройденного велосипедистом за время t ч со скоростью х км/ч. Вычислите путь велосипедиста, если х = 25, t = 1,2.

V. Итоги урока.

– Что называется значением выражения с переменными?

– В каком случае выражение с переменными не имеет смысла? Назовите выражение, которое содержит переменную х и которое не имеет смысла при х = –3,5.

– Назовите выражение, имеющее смысл при любых значениях входящей в него переменной у.

– Что представляет собой формула? Назовите формулу четного числа, нечетного числа.

Домашнее задание: 1. № 30, № 32, № 34, № 43.

2*. Запишите трехзначное число, содержащее:

а) 6 сотен, а десятков, b единиц;

б) х сотен, 7 десятков, у единиц;

в) 8 сотен, р десятков, р единиц.

 

 

 

 

 

 

 

 

 

 

 

 

 

Урок                                                Дата ____________  
Тема:  выражения с переменными

Цели: формировать умение сравнивать значения числовых выражений, а также буквенных выражений при заданных значениях входящих в них переменных; применять свойства действий над числами при нахождении значений числовых выражений; ввести понятие двойного неравенства; формировать умение записывать результат сравнения выражений в виде двойного неравенства.

Ход урока

I. Устная работа.

1. Проанализируйте порядок выполнения действий в каждом из данных выражений и объясните, как оно читается:

а) а + b;                б) а · b;                       в) 2аb;             г) а + (bс);

д) ;                  е) 2х – 3у;                   ж) ak + p;                   з) .

2. От куска проволоки длиной а м первый раз отрезали b м, а второй раз – с м проволоки. Какой смысл имеют следующие выражения:

а) b + с;                 б) а – (b + с);  в) аb;                      г) аbс?

3. Поставьте вместо звездочек такое число, чтобы получилось верное равенство.

а) –(–12) = *;        б) 1,5 = –(*);   в) = –8 = –(*); г) 0 = –(*).

II. Объяснение нового материала.

1. Данная тема входит в пропедевтическое изучение неравенств. Уже в начальной школе учащиеся приобретают первые представления о неравенствах – сравнивают числа, решают задачи на установление знака «>» или «<» между двумя числовыми выражениями: 3 + 7 и 7 · 2; 5 · 3 и 4 + 9. Затем сведения о неравенствах повторяются и закрепляются. Так, при изучении темы «Больше или меньше» отмечается, что результат сравнения двух чисел записывается в виде неравенства с использованием символов «<» (меньше) или «>» (больше). Здесь же вводится и двойное неравенство. Запись 3 < 5 < 7 означает, что число меньше 5, а число 5 в свою очередь меньше 7.

Использование данных символов осмысливается учащимися в процессе выполнения достаточного числа упражнений на сравнение чисел (с активным использованием координатной прямой) и сравнение значений величин.

После ознакомления учащихся с буквенными выражениями задания усложняются.

2. Рассмотрим задачу со с. 10 учебника. Она носит чисто практический характер, и её решение служит мотивацией изучения темы. Показываем учащимся, что при записи неравенства слева или справа (или в обеих частях) может стоять числовое выражение. Просим назвать неравенства.

Здесь  следует  напомнить,  что  неравенства  бывают  верные  или  неверные.

Задание. Определите, верно ли неравенство.

а) 3 · 15 > 8 : 2;                           б) 14 : 2 < –3 · 2;

в) 1,7 · 10 > 7 : 10;                      г)  < –2,5 ∙  2.

3. Показываем на конкретных примерах, что если выражения содержат переменные, то для разных значений переменных результат сравнения значений этих выражений может оказаться различным.

Рассматриваем пример со с. 10 учебника. Также целесообразно попросить учащихся подобрать несколько значений переменной а, при которых будут верны либо не верны неравенства 2а > а + 4 и 2а < а + 4.

4. Вводим понятие двойного неравенства. Обращаем внимание на различные формулировки прочтения двойного неравенства.

III. Формирование умений и навыков.

При выполнении упражнений всегда имеется в виду, что работа с буквенными данными является естественным продолжением работы с конкретными числами и числовыми выражениями. Учащиеся на первых порах испытывают затруднения в усвоении буквенной символики: в усвоении наименования и их записи. Поэтому при выполнении всех упражнений на этом уроке следует побуждать учащихся проговаривать как результаты сравнения, так и саму запись выражений.

1-я группа. Сравнение числовых выражений.

1. Сравните значение выражений:

а)  и ;                     б)  и ;

в) 0,5 и ;                           г)  и –1,6;

д) 3,2 · 6,01 и 77,2 : 4;    е) 38,4 : 6 и 12 – 5,6.

2. № 50.

3. № 48 (а; в); № 49 (а; б).

При  выполнении  этих  заданий  ученики  должны  грамотно  формулировать  обоснование  полученного  результата.  Например:  6,16 – 7,44 < 7,23 + 8,11. Слева стоит выражение, значением которого является отрицательное  число  (из  меньшего  числа  вычитаем  большее),  а справа значением выражения является положительное число (сумма двух положительных  чисел).  Следовательно,  отрицательное  число  меньше  положительного.

2-я группа. Сравнение буквенных выражений.

1. № 51; № 52 (устно).

2. Сравните значения выражений:

а) 2х + 5 при х = 0 и х = –1;

б) 3 – 3а при а = 1 и а = –1;

в) 3х + 5у при х = –0,3, у = 0,6 и х = 1,2,  у = –0,3;

г) 7а + b – 2c при а = 2, b = –4, с = 3 и а = –1,2,  b = 0,4, с = 1.

3. Для выражений 25х + 1 и 800 : х – 99 составьте таблицу значений при х = 1; 2; 4; 5; 8. При  каких из этих значений х:

а) первое выражение меньше второго;

б) первое выражение равно второму;

в) первое выражение больше второго?

Решение:

х

1

2

4

5

8

25х + 1

26

51

101

126

201

800 : х – 99

701

301

101

61

1

Ответ: а) 1; 2;  б) 4;  в) 5; 8.

3-я группа. Сравнение выражений в виде двойного неравенства.

1. № 56, № 57.

2. Какие числа, кратные 5, удовлетворяют неравенству:

а) 64 < х < 78;                  б) 405 < у < 450?

3. Запишите все числа х, у которых знаменатель дробной части 10, если .

4. № 59.

IV. Итоги урока.

– В каком отношении могут находиться числовые выражения?

– Каким образом сравниваются выражения, содержащие переменные?

– Верны ли неравенства:

а) 3х + 5 > –7х + 11 при х = –1; х = 2?

б) 3х – 2 = – 5х + 6 при х = –2; х = 1?

в) –2х – 1,4 > х + 5 при х = 1; х = 0?

– Прочитайте неравенство:

а) –5 < х < –8;                  б) 15,7 < 15,9 < 16,2;             в) –1 < 3 < 5,85.

Домашнее задание: № 47; № 48 (б; г); № 49 (в; г); № 53; № 54; № 58.

 

 

 

 

 

 

 

 

 

 

 

 

 

 

 

 

 

Урок                                                     Дата ___________
Тема: Сравнение значений выражений

Цели: продолжить формировать умение сравнивать значения числовых выражений, а также выражений с переменными при заданных значениях входящих в них переменных; ввести понятие строгого и нестрогого неравенства; формировать умение составлять выражения по условию задачи и сравнивать их значения.

Ход урока

I. Устная работа.

1. Сколько процентов составляет: а) число 8 от числа 200; б) число 15 от числа 1500; в) число 24 от числа 12; г) число  от ?

2. Замените звездочку знаком: >, < или =.

а)  * 3;                         г) 32,5 – 12 * 4,01;    

б)  * 5 – 2,5;                               д) (5 – 2) · 7,5 * 5 – 2 · 7,5;

в) (–2) ∙   ∙  7 * – 3,5;                   е) –3,7 – 2,4 * –6,2.

3. Прочитайте неравенство:

а) 3,7 < 3,8 < 3,95;                                  в) –b < –a < – c;

б) k < p < 2k;                                           г) .

II. Проверочная работа.

Вариант 1

1. Сравните значения выражений:

1) 3х – 6,2 и 2х – 1,8 при х = –4; х = 4,4;

2) 2a – 3b и 3a – 2b при а = –2 и b = 3.

2. Запишите  в  виде  двойного  неравенства:  t  положительно  и  меньше 45.

Вариант 2

1. Сравните значения выражений:

1) 5х + 11 и 3х – 6 при х = 2; х = –8,5;

2) 3a + 2b и 2a – 3b при а = –2 и b = 4.

2. Запишите в виде двойного неравенства: р отрицательно и больше –18.

III. Объяснение нового материала.

Вводится понятие строгого и нестрогого неравенства на конкретных примерах (число дней в месяце, количество пассажиров в автобусе, предельные температуры и т. п.).

Определение. Неравенства, составленные с помощью знаков > и <, называют строгими неравенствами, а неравенства, составленные с помощью знаков и , называют нестрогими.

Необходимо подчеркнуть, что нестрогое неравенство является верным, если выполняется хотя бы одно соотношение:

18 ≥ 14 – верно (выполняется 18 > 14);

–35 ≤ –35 – верно (выполняется –35 = –35).

Если не выполняется ни одно из соотношений, то неравенство является неверным:

–35 ≥ –34.

Двойные  неравенства  также  могут  быть  записаны  с  помощью  знаков ≥ и ≤:

18 ≤ х ≤ 19;  1,7 < п ≤ 1,8;  .

IV. Формирование умений и навыков.

Все упражнения, выполняемые на этом уроке, можно условно разделить на две группы:

1-я группа. упражнения на запись и чтение строгих и нестрогих неравенств.

2-я группа. решение практических задач на составление выражений и их сравнение.

1-я группа

1. № 60 (устно); № 61 (устно).

2. Задание по вариантам.

Запишите каждое предложение с помощью знаков неравенства. Подберите три значения переменной, при которых данное неравенство верно, и три, при которых неверно.

Вариант 1

1) а) t меньше 5;

    б) р больше или равно –11,3;

    в) т – неотрицательное число;

2) а) х меньше 5 и больше или равно 4;

    б) а больше 0,01 и меньше 0,02;

    в) с больше или равно –0,7 и отрицательно.

Вариант 2

1) а) t больше 7;

    б) х меньше или равно –1,17;

    в) р – неположительное число;

2) а) b меньше 8 и больше или равно –7;

    б) а меньше 0,07 и больше 0,06;

    в) q меньше или равно 0,1 и положительно.

3. Расположите числа в порядке возрастания.

.

4. Расположите числа в порядке убывания.

(0,3)2; 0,3; (0,3)3.

2-я группа

1. Один сплав состоит из 5 кг олова и 15 кг меди, другой – из 3 кг олова и 7 кг меди. В каком из сплавов процентное содержание меди больше?

При  решении  задач  на  проценты  нужно  использовать  наглядное изображение данных, что в дальнейшем позволит учащимся грамотно выполнять анализ условия текстовых задач, решаемых алгебраическим методом.

Решение:

   20 кг                              10 кг

1) Масса первого сплава равна 20 кг, второго – 10 кг.

2) Выразим  процентное  содержание  меди  в  первом  и  во  втором сплавах:

 ∙  100 % = 75 %  и   ∙  100 % = 70 %.

3) 75 > 70,  значит,  в  первом  сплаве  процентное  содержание  меди больше.

Ответ: в первом сплаве.

2. № 65.

Решение:Средняя скорость автомобиля «Жигули» равна  км/ч, а автомобиля «Москвич» –  км/ч. Сравним средние скорости автомобилей:

а) Если х = 12,5, у =10,5, то  = 56, а = 60. То есть при данных значениях переменных верно неравенство  < .

б) Если х = у = 14, то  = 50, а  = 45. То есть при данных значениях переменных верно неравенство  > .

Ответ: а) Средняя скорость автомобиля «Жигули» меньше. б) Средняя скорость автомобиля «Жигули» больше.

3. Цену товара понизили сначала на 20 %, а через 5 лет еще на 25 %. При каком снижении цена понизилась больше?

Решение:

Обозначим за х цену товара. При первом снижении цена уменьшилась на 20 %, то есть стала равна 80 % от х. Выразим её в виде десятичной дроби: 0,8х. Значит, снижение составило х – 0,8х = 0,2х.

При втором снижении цена снизилась на 25 %, то есть равна 75 % от 0,8х. Выразим ее в виде десятичной дроби: 0,75 · 0,8х = 0,6х. Значит, снижение составило 0,8х – 0,6х = 0,2х.

Таким образом, в обоих случаях цена снижалась одинаково.

Ответ:одинаково.                                                                                                                                              V. Итоги урока.

– Какое неравенство называется строгим? Приведите примеры.

– Какое неравенство называется нестрогим? Приведите примеры.

– Когда верно нестрогое неравенство? Когда оно не верно? Приведите примеры.                                  Домашнее задание: 1. № 62, № 63, № 64.

2. Один сплав состоит из 21,9 кг цинка, 6 кг алюминия и 2,1 кг магния, другой сплав – из 22 кг цинка, 12 кг алюминия и 2 кг магния. В каком из сплавов процентное содержание магния меньше?

3. № 68 (а; в).

Урок
свойства ДЕЙСТВИИ НАД ЧИСЛАМИ

Цели: актуализировать знания основных свойств сложения и умножения чисел (переместительное, сочетательное и распределительное свойства); формировать умение применять свойства действий над числами при нахождении значений числовых выражений.

Ход урока

I. Устная работа.

1. Объясните следующие записи:

а) +(2x – 3y + 5) = 2x – 3y + 5;        б) –(2x – 3y + 5) = –2x + 3y – 5.

2. Раскройте скобки.

а) a ∙  (–b + c);                  г) 2 ∙  (a + bc);                    ж) (2x + 4y – 5z – 3) ∙  7;

б) (–a + b) ∙  c;                 д) –5 ∙  (ab + c);                 з) –0,5 ∙  (4a – 3b – 2c + 7).

в) (1 + b) ∙  (–4);   е) (a + b – 4) ∙  (–5);

3. Следующие выражения заключите в скобки двумя способами:

1) поставив перед скобкой знак «плюс»;

2) поставив перед скобкой знак «минус»:

а) а + b;                     б) 1 – b;                     в) 0,5 – 2х;        г) –1,3х + 2,4;

д) –2 + аb;             е) –ху + 5;             ж) 6 – 5а + b;               з) –15 – 7х – 2у.

4. Вынесите за скобки общий множитель.

а) ax + bx + cx;                   б) 10a – 5b – 15c;           в) ayby + 3y;

г) 6xy – 12x + 9xz;              д) –8ab – 29ac + 16a;                 е) 8abc – 24abd – 6ab.

II. Актуализация знаний.

Выполнение устной работы позволит вспомнить основные свойства сложения и умножения чисел, которые целесообразно записать в буквенной форме для любых чисел и оформить в виде плаката.

Переместительное свойство

Для любых чисел а и b верны равенства:

а + b = b + а;     а · b = b · а.

Сочетательное свойство

Для любых чисел а, b и с верны равенства:

(а + b) + с = а + (b + с);     (аb) с = а ().

Распределительное свойство

Для любых чисел а, b и с верно равенство:

а (b + с) = аb + ас.

Также следует отметить, что комбинация данных свойств позволяет сделать вычисление числовых выражений более простым и рациональным. Иными словами, речь идет о формировании вычислительной культуры учащихся.

В то же время основная трудность заключается в том, чтобы научить учащихся «видеть» возможности применения свойств действий над числами и осознанно их применять.

Например:

1. Найдите значение выражения 928 · 36 + 72 · 36.

Для нахождения значения выражения целесообразно преобразовать его, применив распределительное свойство:

928 · 36 + 72 · 36 = (928 + 72) · 36 = 1000 · 36 = 36 000.

Заметим здесь, что если приучать школьников при выполнении аналогичных упражнений рассуждать таким образом: «Для любых чисел а, b и с справедливо распределительное свойство (а + b) с = ас + , значит, и для наших чисел оно верно, то есть…», то тем самым будем развивать у учащихся умения выполнять отдельные виды дедуктивных умозаключений. Так на простом учебном примере воспитывается потребность в обосновании выполняемых действий и в доказательстве, что, в свою очередь, явится хорошей пропедевтикой для проведения более сложных дедукций при изучении систематического курса алгебры и геометрии.

2. Вычислите сумму 1,23 + 13,5 + 4,27.

В учебнике указано, что «удобно объединить первое слагаемое с третьим». Учащиеся должны объяснить, в чем это удобство (в сумме получается десятичная дробь с одним разрядом после запятой):

1,23 + 13,5 + 4,27 = (1,23 + 4,27) + 13,5 = 5,5 + 13,5 = 19.

3. 1,8 · 0,25 · 64 · 0,5 = (1,8 · 0,5) · (64 · 0,25).

Такое распределение целесообразно потому, что 0,5 =  и 0,25 = . То есть следует понимать, что, умножая число на , мы получаем половину, а умножая на , – четверть. Поэтому удобно найти половину от 1,8 и четверть от 64.

Аналогично комментируем все примеры со с. 15 учебника.

III. Формирование умений и навыков.

При выполнении упражнений на этом уроке следует требовать от учащихся обоснования своих действий с проговариванием основных свойств действий над числами.

1. № 70 (устно).

2. № 71.

Решение:

а) 3,17 + 10,2 + 0,83 + 9,8 = (3,17 + 0,83) + (10,2 + 9,8) = 4 + 20 = 24;

б) 4,11 + 15,5 + 0,89 + 4,4 = (4,11 + 0,89) + (15,5 + 4,4) = 5 + 19,9 = 24,9;

в) 15,21 – 3,9 – 4,7 + 6,79 = (15,21 + 6,79 + (–3,9 – 4,7) = 22 + (–8,6) =
= 13,4;

г) –4,27 + 3,8 – 5,73 – 3,3 = (–4,27 – 5,73) + (3,8 – 3,3) = –10 + 0,5 = –9,5.

3. Вычислите наиболее рациональным способом.

а) 527 – 825 + 925;

б) –5,37 + 9,27 + 4,37.

Решение:

а) 527 – 825 + 925 = 527 + (925 – 825) = 527 + 100 = 627;

б) –5,37 + 9,27 + 4,37 = (4,37 – 5,37) + 9,27 = –1 + 9,27 = 8,27.

4. № 73.

5. № 75 (а; в); № 76 (а; в); № 77.

IV. Итоги урока.

– Сформулируйте переместительное свойство сложения и умножения. Приведите примеры.

– Сформулируйте сочетательное свойство сложения и умножения. Приведите примеры.

– Сформулируйте распределительное свойство умножения. Приведите примеры.

– Какие свойства действий позволяют, не выполняя вычислений, утверждать, что верно равенство:

а) 3 · 17,8 = 17,8 · 3;                               б) 35 + 73 = 73 + 35;

в) 32 + (14 + 3) = (32 + 14) + 3;                            г) 13 · (5 + 11) = 13 · 5 + 13 · 11?

Домашнее задание: № 72; № 74; № 75 (б; г); № 76 (б; г); № 78.

 

 

Урок 8
Решение задач по теме
«Свойства действий над числами»

Цель: продолжить формирование умений применять основные свойства действий над числами (переместительное, сочетательное, распределительное) при нахождении значений числовых выражений.

Ход урока

I. Устная работа.

1. Вычислите:

а) ;                   б) ;                     в)

г) ;                   д) ;                        е) ;

ж) ;               з) .

2. Вычислите:

а)  ∙  2;          б)  : 3;               в) ;             г) 5 : ;

д) ;          е) ;            ж) ;               з) .

II. Актуализация знаний.

Вычислить значение каждого выражения наиболее простым способом, проговорив при этом используемое свойство действий над числами:

а) 405 · 82 + 405 · 18;

б) 707 · 13 + х · 13 при х = 293;

в) 417р – 217 · 163 при р = 163;

г) 24а – 48 · 15 при а = 33;

д) (64 · 37 + 64 · 23) : 5.

III. Формирование умений и навыков.

На этом уроке решаются задания более высокого уровня сложности.

1. № 79.

Решение:

а) 24 · 17 + 17 · 6 = 17 · (24 + 6) = 17 · 30 = 17 · 6 · 5, значит, выражение делится на 5.

б) 34 · 85 + 34 · 36 = 34 · (85 + 36) = 34 · 121 = 34 · 11 · 11, значит, выражение делится на 11.

2. № 223.

Решение:

а) 5,9 · 2,6 + 5,9 · 3,2 + 5,8 · 4,1 = 5,9 (2,6 + 3,2) + 5,8 · 4,1 = 5,9 · 5,8 +
+ 5,8 · 4,1 = 5,8 (5,9 + 4,1) = 5,8 · 10 = 58;

б) 6,8 · 8,4 – 1,6 · 8,4 + 5,2 · 1,6 = 8,4 (6,8 – 1,6) + 5,2 · 1,6 = 8,4 · 5,2 +
+ 5,2 · 1,6 = 5,2 (8,4 + 1,6) = 5,2 · 10 = 52.

3. Вычислите наиболее рациональным способом.

а) ;    б) .

Решение:

а) Выполняем сперва умножение первой дроби на вторую, затем полученный результат – на третью дробь и т. д. Получим .

б)

.

4. Найдите последовательно значение каждой из разностей:

, а затем значение суммы .

Решение:

;

;

;

;

;

.

.

5. Разберите, как выполнено умножение.

5 · 424 = 5 · 2 · 212 = 10 · 212 = 2120.

Используя данный прием, выполните вычисления устно.

а) 5 · 822;                 б) 5 · 412;            в) 5 · (–724);

г) 822,2 · 5;               д) 43,6 · 5;           е) (–0,626) · 5.

Решение:

Суть приема заключается в том, чтобы разложить четный сомножитель на произведение 2 · х, тогда выражение примет вид 5 · 2 · х = 10 · х, что позволит выполнить действие устно.

а) 5 · 822 = 5 · 2 · 411 = 10 · 411 = 4110;

б) 5 · 412 = 5 · 2 · 206 = 10 · 206 = 2060;

в) 5 · (–724) = 5 · 2 · (–362) = 10 · (–362) = –3620;

г) 822,2 · 5 = 411,1 · 2 · 5 = 411,1 · 10 = 4111;

д) 43,6 · 5 = 21,8 · 2 · 5 = 21,8 · 10 = 218;

е) (–0,626) · 5 = (–0,313) · 2 · 5 = (–0,313) · 10 = –3,13.

6. № 224*.

Решение:

а) (1,25 ∙  1,7 ∙  0,8 – 1,7) ∙  3,45 = 1,7 ∙  (1,25 ∙  0,8 – 1) ∙  3,45 =
= 1,7 ∙   ∙  3,45 = 1,7 ∙  (1 – 1) ∙  3,45 = 0;

б) 3,947 : (3,6 – 2,6 · 4 · 0,25) = 3,947 : (3,6 – 2,6 · 1) =
= 3,947 : (3,6 – 2,6) = 3,947 : 1 = 3,947.

IV. Проверочная работа.

Вариант 1

Вычислите наиболее рациональным способом:

1. .

2. 28 · 3,9 · .                  3. 5 · .

Вариант 2

Вычислите наиболее рациональным способом:

1. .

2. 36 · 2,7 · .                  3. 8 · .

V. Итоги урока.

– Сформулируйте переместительное и сочетательное свойства сложения и умножения.

– Сформулируйте распределительное свойство умножения. Приведите пример.

– Запишите названные свойства в буквенной форме.

Домашнее задание:

1. Вычислите наиболее рациональным способом:

а) 6,89 + 5,37 + 3,11 + 4,63;                  д) ;  

б) –321 + 457 + 921;                    е)  · (–28);

в) –4,83 + 3,99 + 2,83;                 ж) 9 · 7.

г)  · 37,4 · 15;

2. № 80, № 82.

 

 

Урок 9
Понятие тождества. Доказательство тождеств

Цели: ввести понятия тождественно равных выражений и тождества; формировать умение определять тождественное равенство выражений на основе выражения основных свойств действий над числами.

Ход урока

I. Устная работа.

1. Найдите значение числового выражения.

а) 3 + 15 : (–5);                                               г) ;

б) (–18 – 2) : (–4);                                 д) 9 · 0,1 – 0,1;

в) 7 · 2 + (–4) : 2;                                  е) ;

ж) 0,5 · 3 + 0,5 · 8 – 0,5 · 10;                 з) 0,86 : 2 · 100.

2. Какие свойства действий позволяют, не выполняя вычислений, утверждать, что верно равенство?

а) –368 + 2,54 = 2,54 – 368;                            г) (1,5 · 3) · 10 = 1,5 · (3 · 10);

б) ;        д) ;

в) 3 ·  – 3 · 2;             е) (2,8 – 10) · 5 = 2,8 · 5 – 10 · 5.

II. Объяснение нового материала.

1. Значение темы «Тождественные преобразования» состоит в следующем:

– ученики должны понимать, что в алгебре все действия только обозначаются, а затем выражения преобразуются в более простые заменой суммы, произведения тождественно равным выражением;

– тождественные преобразования не самоцель; они используются для удобства нахождения числовых значений выражений, решения уравнений, доказательства неравенств и выявления свойств функций.

Это значит, что с тождественными преобразованиями связаны все линии курса алгебры.

2. Мотивация изучения.

Предлагаем  учащимся  для  выполнения  следующую  лабораторную работу.

Заполните таблицу.

х

1

1

2

–3

у

2

–2

0

2

2 (х + у)

6

–2

4

–2

2х + 2у

6

–2

4

–2

х – (2 + у)

–3

1

0

–7

(х – 2) + у

1

–3

0

–3

(х – 2) – у

–3

1

0

–7

Задания:

1) Назовите выражения, равные при всех наборах значений х и у.

2) Назовите выражения, равные при одних наборах х и у и не равные при других наборах значений х и у.

3) Из каких свойств действий над числами следует равенство этих выражений (или не следует)?

3. Введение определений.

Определение 1. Два выражения, значения которых равны при любых значениях переменных, называются тождественно равными.

Определение 2. Равенство, верное при любых значениях переменных, называется тождеством.

Следует помнить, что в 8 классе с введением дробно-рациональных выражений авторы учебника вернутся к понятию тождества и определят тождество как равенство, верное при всех допустимых значениях входящих в него переменных.

4. Рассматриваем примеры тождеств со с. 18 учебника. Подчеркиваем, что равенства, выражающие основные свойства действий над числами, являются тождествами.

Отмечаем, что замена выражения тождественно равным позволяет часто упростить вычисление значения исходного выражения.

III. Формирование умений и навыков.

все упражнения, решаемые на этом уроке, направлены на усвоение определений тождества и тождественно равных выражений, а также на закрепление навыка применения основных свойств действий над числами для преобразования выражений в тождественно равные.

1. № 85 (устно).

При выполнении этого упражнения ученики должны четко проговаривать свойство действий, которое позволило им сделать соответствующий вывод.

2. № 86, № 87.

3. № 88, № 89.

4. Упростите выражение.

а) 2,8 · 5а;       в) 3,6 · 0,8а;         д) 8х · (–3а);        ж) –0,25у · 8b;

б) –3,5а · 4;     г) –8а · (–12);       е) 3,5х · 2у;           з) .

5. № 92, № 94.

IV. Проверочная работа.

Вариант 1

1. Упростите сумму.

а) –8 + х + (–22);                б) –10 + а + 34.

2. Выполните вычисления, выбирая удобный порядок действий:

–25 · 123,7 · 4.

3. Представьте выражение в виде произведения.

а) 27 · 41 + 41 · х;               б) 31а + 14а.

Вариант 2

1. Упростите сумму.

а) –17 + с + 47;                   б) –16 + р + (–21).

2. Выполните вычисления, выбирая удобный порядок действий:

–50 · 12,1 · 4.

3. Представьте выражение в виде произведения.

а) 38 · 54 + 54у;                 б) 34х + 15х.

Решение заданий проверочной работы

Вариант 1

1. а) –8 + х + (–22) = (–8 + (–22)) + х = –30 + х = х – 30;

    б) –10 + а + 34 = (–10 + 34) + а = 24 + а = а + 24.

2. –25 · 123,7 · 4 = (–25 · 4) · 123,7 = –100 · 123,7 = –12370.

3. а) 27 · 41 + 41 · х = 41 · (27 + х);

    б) 31а + 14а = (31 + 14) · а = 45а.

Вариант 2

1. а) –17 + с + 47 = (–17 + 47) + с = 30 + с = с + 30;

    б) –16 + р + (–21) = (–16 + (–21)) + р = –37 +р = р – 37.

2. –50 · 12,1 · 4 = (–50 · 4) · 12,1 = –100 · 12,1 = –1210.

3. а) 38 · 54 + 54у = 54 · (38 + у);

    б) 34х + 15х = (34 + 15) · х = 49х.

V. Итоги урока.

– Какие выражения называются тождественно равными? Приведите пример тождественно равных выражений.

– Какое  равенство  называется  тождеством?  Приведите  пример  тождества.

– Для чего необходимо заменять выражения тождественно равными?

Домашнее задание: № 90, № 91, № 93, № 108.

 

 

 

 

 

 

 

 

 

 

Урок                                                                                          Дата
Тема: Тождественные преобразования

Цели: закрепить усвоение понятий тождественно равных выражений и тождества; ввести понятие тождественного преобразования выражения; формировать умения выполнять основные тождественные преобразования (приведение подобных слагаемых, раскрытие скобок).

Ход урока

I. Устная работа.

1. Сравните значения выражений, не вычисляя их:

а) 35,8 +  и 35,8 + ;             г) –2,8 +  и  – 2,8;

б)  и ;                                д) 19,7 ·  и 19,7 : ;

в)  и ;                            е) 3,8 : 2,1 и 3,8 · 2,1.

2. Является ли тождеством равенство:

а) х + 4 = (3 + х) + 1;                               г) 3а – 4 = (2а – 4) – а;

б) 5у – 35 = 5 (у – 7);                  д) –2 (b – 3) = –2b – 6;

в) 7х – 42 = (х – 6) · 7;                е) 25 (а а) = 25?

II. Объяснение нового материала.

1. Объяснение проводить согласно пункту 5 учебника. Особое внимание следует уделить конкретным примерам тождественных преобразований. Для лучшего закрепления необходимо использовать как буквенные, так и числовые выражения.

2. Изучение всей темы направлено на то, чтобы добиться от учащихся соблюдения следующей системы требований:

1) Если в задачнике (или со стороны учителя) нет указаний, каким способом производить вычисления, то там, где это посильно, их следует выполнять устно.

2) Перед вычислением значения числового выражения или выражения с переменными подумать, нельзя ли применить свойства действий для более удобных вычислений. Записывать промежуточные результаты, получаемые от применения свойств действий, следует только тогда, когда затруднительно их запоминание.

3) Запись цифр и букв должна быть правильной, аккуратной.

4) Результат  вычислений  считать  правильным  только  после  проверки; после введения уточняющих понятий о тождественно равных выражениях, тождестве и тождественных преобразованиях эти требования расширяются.

5) Если нужно выполнить тождественные преобразования заданного выражения, то не следует сразу же руководствоваться возникшей догадкой – надо мысленно поискать возможные варианты преобразований и выбрать тот из них, который покажется наиболее выгодным.

3. Важно добиться, чтобы учащиеся хорошо понимали, что такие виды тождественных преобразований, как раскрытие скобок, приведение подобных членов, сокращение дробей, приведение дробей к общему знаменателю и т. д., являются следствиями определений и свойств соответствующих действий.

III. Формирование умений и навыков.

Все задания можно условно разбить на три группы:

1-я группа. Упражнения на узнавание и применение основных видов тождественных преобразований выражений (приведение подобных слагаемых и раскрытие скобок).

2-я группа. Комбинированные упражнения на применение основных видов тождественных преобразований.

3-я группа. Практические задачи.

1-я группа

1. № 95.

Образец оформления:

в) 6х – 14 – 13х + 26 = (6х – 13х) + (–14 + 26) = (6 – 13) х + 12 =
= –7х + 12.

2. № 96 (в; г); № 97 (в; г).

3. № 98, № 100.

2-я группа

1. № 102 (б; г).

Образец оформления:

г) 37 – (х – 16) + (11х – 53) = 37 – х + 16 + 11х – 53 = (–х + 11х) +
+ (37 + 16 – 53) = (–1 + 11) х + 0 = 10х.

Если х = –0,03, то 10х = 10 · (–0,03) = –0,3.

Ответ: –0,3.

2. № 103 (а; б; в) (самостоятельно).

3. № 104, № 105, № 106.

3-я группа

1. № 107 (а).

Решение:

В первом альбоме а марок, тогда во втором – (а + 15) марок, а в третьем – 3 · (а + 15) марок.

Всего марок у Игоря: а + (а + 15) + 3 · (а + 15). Упростим данное выражение:

а + (а + 15) + 3 · (а + 15) = а + а + 15 + 3а + 45 = (1 + 1 + 3) а +
+ (15 + 45) = 5а + 60.

Ответ: всего 5а + 60 марок.

Напоминаем учащимся, что удобно отмечать подобные слагаемые подчеркиванием их одинаковыми линиями:

а + а + 15 + 3а + 45.

2. В  магазине  товар  стоит  а рублей. На распродаже его цена упала на 30 %. На сколько полученная прибыль магазина меньше предполагаемой первоначальной прибыли, если закупочная цена товара составляет 0,6а?

Решение:

Предполагаемая прибыль: а – 0,6а.

Новая цена: 0,7а.

Полученная прибыль: 0,7а – 0,6а.

Составим разность:

(а – 0,6а) – (0,7а – 0,6а) = а – 0,6а – 0,7а + 0,6а = а – 0,7а = 0,3а.

Ответ: 0,3а.

На этом примере показываем, что если подобные слагаемые имеют противоположные коэффициенты, то их сумма равна нулю и такие слагаемые можно «сокращать».

– 0,6а + 0,6а = (–0,6 + 0,6) а = 0 · а = 0.

IV. Итоги урока.

– Какие выражения называются тождественно равными?

– Какие преобразования выражений называются тождественными? Приведите примеры.

– Каким способом приводятся подобные слагаемые?

– Назовите правило раскрытия скобок, перед которыми стоит знак «плюс». На каком свойстве действий основывается это правило?

– Назовите правило раскрытия скобок, перед которыми стоит знак «минус». На каком свойстве действий основывается это правило?

Домашнее задание: № 96 (а; б); № 97 (а; б); № 99; № 101; № 102 (а; в).

 

 

 

 

 

 

 

 

 

 

 

 

 

 

 

 

 

 

 

 

 

 

Урок                                                                                                        Дата
ТЕМА: Обобщающий урок по теме «Выражения. тождества»

Цели: обобщить и систематизировать знания: свойства действий над числами, термины «числовое выражение», «выражение с переменными», «значение выражения», «тождество», «тождественные преобразования»; актуализировать умения: выполнять в буквенных выражениях числовые подстановки и производить соответствующие вычисления; сравнивать значения буквенных выражений при заданных значениях входящих в них переменных; применять свойства действий над числами.

Ход урока

I. Проверочная работа.

Вариант 1

1. Приведите подобные слагаемые.

а) 8b + 12b – 21b + b;                 б) 1,2c + 1 – 0,6y – 0,8 – 0,2c.

2. Раскройте скобки и приведите подобные слагаемые.

(1 – 9y) – (22y – 4) – 5.

Вариант 2

1. Приведите подобные слагаемые.

а) 9a + 17a – 30a + 4a;                б) 1,8y + 3 – 2,8c – 0,2 – 2y.

2. Раскройте скобки и приведите подобные слагаемые.

(2 – 4b) – (31b – 6) – 11.

II. Повторение материала.

Повторение целесообразно организовать в форме практикума по решению задач. Все задания можно разбить на три группы.

1-я группа. Нахождение значения числового выражения и выражения с переменными.

1. Устная работа.

1) Используя термины «сумма», «разность», «произведение» и «частное», прочитайте выражение:

а) ;             г) 3,72 · 8,02;             ж) 3,12 · (5,3 + 2,7);

б) 6,8 : 34;                        д) ;                              з)  + 11;

в) 5,3 + 7,2;                      е) (10 – 18) : 3,4;                   и) 3,11 · (12 : 3,5).

2) Из данных выражений выберите выражение, не имеющее смысла:

а) 32 : (7 · 2 – 3,5 · 3);                 в) ;

б) ;                           г) (3,8 · 2 – 7,6) : 4.

2. Письменная работа.

1) Найдите значение выражения.

а) 13 + 27,13 + 40 + 50,07;                     в) 4,24 – 17,05 : 12,5;

б) 5,47 – (8,32 – 5,311);              г) (0,018 + 0,982) : (8 · 0,5 – 0,8).

При выполнении этих упражнение учащиеся должны обосновывать, почему они выбирают тот или иной порядок действий.

2) Найдите значение данного выражения:

а) 2m + 6n – 11 при т = –12 и п = 4; т = –3,5 и п = 3;

б) 8 – 0,7 (3b – 5a) при а = –3,3 и b = 5,5;

в)  при а = 0 и b = 2,3;

г) пусть ху = 3 и z = –5. Найдите .

2-я группа. Сравнение значений выражений.

1. Устная работа.

Не выполняя вычислений, сравните значения выражений:

а) 3,5 · 0,24 и 3,5;                                   г) 0,57 : 6 и 0,57 : ;

б) 3,5 · 0,24 и 0,24;                                 д) –0,57 :  и –0,57;

в) –3,5 · 0,24 и –3,5;                               е) 94 : (–2,1) и 64 : (–2,1).

2. Письменная работа.

1) Сравните значения выражений:

а)  и ;                                б) 0,5 и ;

в) 5 – 2х  при х = 2 и х = –2;

г) 4х + 10у  при х = –0,7, у = 0,9 и х = 1,4, у = –1,37.

2) Расположите числа в порядке убывания:

2,07; 2,007; –1,65; –1,66; 0.

3-я группа. Преобразование выражений на основе свойств действий, приведение подобных слагаемых и раскрытие скобок.

1. Устная работа.

Какие свойства действий позволяют, не выполняя вычислений, утверждать, что верно равенство?

а)  + 354 = 354 + ;               в) ;

б) 85 · 11 = 11 · 85;                                 г) .

2. Письменная работа.

1) Вычислите наиболее рациональным способом.

а) 6,83 + 7,81 + 3,17 + 8,19;                   в) ;

б)  ∙  13,5 ∙  19;                                  г) –4,83 + 3,99 + 2,83.

2) Раскройте скобки и приведите подобные слагаемые.

а) 2a + (3a – 8b);                         в) 9x + 3 (15 – 8x);

б) (2a – 7y) – (5a – 7y);               г) 33 – 8 (11b – 1) – 2b.

3) Найдите значение данного выражения:

а) 1,7 (а – 11) – 16,3  при а = 3,8;

б) 0,6 (4х – 14) – 0,4 (5х – 1) при x = 4.

При выполнении всех заданий учащиеся должны проговаривать правила, на которые опираются, а также обосновывать рациональность вычислений. Следует приветствовать устный счет.

III. Итоги урока.

– Что называется значением числового выражения?

– Как находится значение выражения с переменными?

– Каким образом сравниваются выражения с переменными?

– Какие  свойства  действий  используются  при  преобразовании  выражений?

– Сформулируйте правила приведения подобных слагаемых и раскрытия скобок.

Домашнее задание:  повторить  п. 1–5;  № 210;  № 109,  № 217  (а; г), № 230 (а).

 

 

 

 

 

 

 

 

 

 

 

 

 

 

 

 

Урок                                                                                                  Дата
ТЕМА: Контрольная работа по теме «Выражения. Тождества».

Вариант 1

1. Найдите значение выражения 6х – 8у  при x = , y = .

2. Сравните значения выражений –0,8х – 1 и 0,8х – 1  при х = 6.

3. Упростите выражение.

а) 2х – 3у – 11х + 8у;

б) 5(2а + 1) – 3;

в) 14х – (х – 1) + (2х + 6).

4. Упростите выражение и найдите его значение.

–4 (2,5a – 1,5) + 5,5a – 8 при a = –.

5. Из двух городов, расстояние между которыми s км, одновременно навстречу друг другу выехали легковой автомобиль и грузовик и встретились через t ч. Скорость легкового автомобиля х км/ч. Найдите скорость грузовика. Ответьте на вопрос задачи, если s = 200, t = 2, х = 60.

6. Раскройте скобки: 3x – (5x – (3x – 1)).

Вариант 2

1. Найдите значение выражения 16а + 2у при a = , y = .

2. Сравните значения выражений 2 + 0,3а и 2 – 0,3а при а = –9.

3. Упростите выражение.

а) 5a + 7b – 2a – 8b;

б) 3 (4х + 2) – 5;

в) 20b – (b – 3) + (3b – 10).

4. Упростите выражение и найдите его значение.

–6 (0,5x – 1,5) – 4,5x – 8 при x = .

5. Из двух городов одновременно навстречу друг другу выехали автомобиль и мотоцикл и встретились через t ч. Найдите расстояние между городами, если скорость автомобиля х1 км/ч, а скорость мотоцикла х2 км/ч. Ответьте на вопрос задачи, если t = 3,  х1 = 80,  х2 = 60.

6. Раскройте скобки: 2p – (3p – (2pc)).

Рекомендации по оцениванию контрольной работы

Первые три задания соответствуют обязательному уровню усвоения материала. Их выполнение оценивается на «3». Для получения отметки «4» достаточно выполнить правильно 5 любых заданий, для получения отметки «5» – все шесть.

Для слабого класса можно рассматривать одно из двух последних заданий как резервное и на отметку «5» достаточно выполнить 5 заданий.

Решение заданий контрольной работы

Вариант 1

1. Если x = , y = , то 6x – 8y = 6 ·  – 8 ·  = 4 – 5 = –1.

Ответ: –1.

2. Если х = 6, то –0,8х – 1 = –0,8 · 6 – 1 = –4,8 – 1 = –5,8;

0,8х – 1 = 0,8 · 6 – 1 = 4,8 – 1 =3,8.

–5,8 < 3,8, значит, –0,8х – 1 < 0,8х – 1 при х = 6.

Ответ: –0,8х – 1 < 0,8х – 1 при х = 6.

3. а) 2x – 3y – 11x + 8y = (2 – 11) x + (–3 + 8) y = –9x + 5y;

    б) 5 (2а + 1) – 3 = 10а + 5 – 3 = 10а + 2;

    в) 14x – (x – 1) + (2x + 6) = 14xx + 1 + 2x + 6 = (14 – 1 + 2) x +
+ (1 + 6) = 15x + 7.

Ответ: а) –9х + 5у;  б) 10а + 2;  в) 15х + 7.

4. –4 (2,5a – 1,5) + 5,5a – 8 = –10a + 6 + 5,5a – 8 = (–10 + 5,5) a +
+ (6 – 8) = –4,5a – 2.

Если a = , то –4,5 ·  – 2 = 1 – 2 = –1.

Ответ: –1.

5.

Автомобиль проехал х · t км, значит, грузовик проехал (sх · t) км. Скорость грузовика равна (sх · t) : t км /ч.

Если  s = 200,  t = 2,  х = 60,  то (sх · t) : t = (200 – 60 · 2) : 2 =
= 80 : 2 = 40.

Ответ: 40 км/ч.

6. 3x – (5x – (3x – 1)) = 3x – (5x – 3x + 1) = 3x – 5x + 3x – 1 =
= (3 – 5 + 3) x – 1 = x – 1.

Ответ: х – 1.

Вариант 2

1. Если a = , y = , то 16а + 2у = 16 ·  + 2 · .

Ответ: 1.

2. Если а = –9, то 2 + 0,3а = 2 + 0,3 · (–9) = 2 – 2,7 = –0,7;

2 – 0,3а = 2 – 0,3 · (–9) = 2 + 2,7 = 4,7.

–0,7 < 4,7, значит, 2 + 0,3а < 2 – 0,3а  при а = –9.

Ответ: 2 + 0,3а < 2 – 0,3а  при а = –9.

3. а) 5a + 7b – 2a – 8b = (5 – 2) a + (7 – 8) b = 3ab;

    б) 3 (4х + 2) – 5 = 12х + 6 – 5 = 12х + 1;

    в) 20b – (b – 3) + (3b – 10) = 20bb + 3 + 3b – 10 = (20 – 1 + 3) b +
+ (3 – 10) = 22b – 7.

Ответ: 3аb; б) 12х + 1; в) 22b – 7.

4. –6 (0,5x – 1,5) – 4,5x – 8 = –3x + 9 – 4,5x – 8 = (–3 – 4,5) x + (9 – 8) =
= –7,5x + 1.

Если x = , то

Ответ: –4.

5.

Автомобиль проехал х1 · t км, мотоцикл – х2 · t км, значит, расстояние между городами равно х1t + х2t км.

Если t = 3, х1 = 80, х2 = 60, то х1t + х2t = 80 · 3 + 60 · 3 = (80 + 60) · 3 =
= 140 · 3 = 420 км.

Ответ: 420 км.

6. 2p – (3p – (2pc)) = 2p – (3p – 2p + c) = 2p – 3p + 2pc =
= (2 – 3 + 2) pc = pc.

Ответ: рс.

 

 

 

 

 

 

 

 

 

 

 

 

 

 

 

 

 

 

 

 

 

 

Урок 12

Уравнение и его корни

Цели: проанализировать задания контрольной работы, выявить типичные  ошибки,  допущенные  учащимися,  провести  работу  над  ошибками; ввести понятия «уравнение с одной переменной», «корень уравнения», «решить уравнение», «равносильные уравнения»; формировать умение заменять уравнение равносильным на основе некоторых свойств уравнения.

Ход урока

I. Анализ результатов контрольной работы.

II. Устная работа.

Найдите значение выражения.

а) ;                   г) 72 + 5;                        ж) | 3 | + 0,5;

б) ;                 д) (–3)2 + ;                  з) | –5 | – 7;

в) ;              е) (–2)3;                  и) .

III. Объяснение нового материала.

1. Значимость изучаемого материала.

Уравнение как общематематическое понятие многоаспектно. Оно функционирует как:

џ средство решения текстовых задач;

џ особого рода формула, служащая в алгебре объектом изучения;

џ формула, которой косвенно определяются числа или координаты точек плоскости (пространства), служащие его решением.

Линия уравнений тесно связана со всеми содержательными линиями школьного курса математики. На данном этапе важно показать учащимся тесную связь линии уравнений именно с линией тождественных преобразований. Последняя приобретает новое содержание и смысл именно при изучении равносильных преобразований уравнений и, в дальнейшем, неравенств. В свою очередь, владение содержанием линии уравнений и неравенств позволяет расширить список выполнимых преобразований.

2. Мотивация изучения.

Уравнение, будучи математической моделью реальных процессов, первоначально возникает как обобщение метода решения задач арифметическим методом, а затем используется при решении текстовых задач, фабула которых отражает многообразные реальные процессы окружающего мира. Данный аспект линии уравнений обеспечивает мотивацию изучения школьного курса математики в целом. Демонстрируем учащимся, что при решении текстовых задач ведущим аппаратом является математическое моделирование, а одним из средств построения модели и решения ситуации в её рамках – уравнение.

Разбираем задачу со с. 22–23 учебника и вводим понятие «уравнение с одной переменной».

3. Введение основных определений.

Вводим четкое определение понятия корня уравнения.

Определение 1. Корнем уравнения называется значение переменной, при котором уравнение обращается в верное числовое равенство.

Показываем на примерах, что уравнение с одной переменной может иметь как конечное число корней, так и бесконечное, а также может не иметь корней вообще. Отсюда вытекает следующее определение.

Определение 2. Решить уравнение – значит найти все его корни или доказать, что корней нет.

4. Понятие равносильных уравнений.

Определение 3. Уравнения, имеющие одни и те же корни, называются равносильными уравнениями.

Следует подчеркнуть, что замена исходного уравнения равносильным позволяет более просто и рационально решать исходное уравнение.

Показываем на примерах, с помощью каких приемов можно получать уравнения, равносильные данному. Отмечаем, что такие свойства уравнений опираются на свойства числовых равенств.

IV. Формирование умений и навыков.

Задания, решаемые на этом уроке, направлены на усвоение понятия «корень уравнения» (1-я группа), на нахождение корней некоторых уравнений (2-я группа) и на применение свойств уравнений, позволяющих заменять их равносильными уравнениями (3-я группа).

1-я группа

№ 111, № 112, № 114.

При выполнении этих заданий ученики проговаривают определение корня уравнения и способ проверки.

2-я группа

№ 116.

Решение:

а) 1,4 (у + 5) = 7 + 1,4у;

1,4у + 1,4 · 5 = 7 + 1,4у;

1,4у + 7 = 1,4у + 7.

При любом значении у равенство верное, значит, корнем уравнения является любое число.

б) у – 3 = у.

При любом значении у левая часть уравнения на 3 меньше его правой части, значит, уравнение не имеет корней.

№ 118.

Решение:

1. 2(х + 3) = 2х + 6;

    2х + 6 = 2х + 6.

Корнем уравнения является любое число.

2. 2у = 4у;

    2у – 4у = 0;

    – 2у = 0;

    у = 0.

3. 4 (с – 2) = 3с – 6;

    4с – 8 = 3с – 6;

    4с – 3с = 8 – 6;

    с = 2.

4. 3х + 11 = 3(х + 4);

    3х + 11 = 3х + 12.

Уравнение не имеет корней.

Ответ: 4.

№ 120.

Решение:

а) | x | = 1, если х = 1 или х = –1, значит, уравнение имеет 2 корня.

б) | x | = 0, если х = 0, значит, уравнение имеет один корень.

в) | x | = –5. Уравнение не имеет корней, так как, по определению модуля, модуль любого числа есть число неотрицательное.

г) | x | = 1,3,  если  х = 1,3  или  х = –1,3,  значит,  уравнение  имеет  два корня.

3-я группа

1. а) Составить уравнение, корнем которого является число 5;

    б) составить уравнение, корнями которого являются числа –3 и 0;

    в) составить уравнение, которое не имеет корней.

2. № 121.

Решение:

а) 0,3х = –4   | · 10;             б) 5х – 4 = 21;

    0,3х · 10 = –4 · 10;              5х = 21 + 4;

    3х = –40.                            5х = 25.

При выполнении этих упражнений ученики должны проговаривать свойства уравнений, позволяющие получить уравнение, равносильное исходному.

V. Итоги урока.

– Дайте  определение  уравнения  с  одной  переменной.  Приведите пример.

– Дайте определение корня уравнения. Является ли число 8 корнем уравнения 7х – 11 = х + 37?

– Что значит «решить уравнение»?

– Какие уравнения называются равносильными? Сформулируйте свойства уравнений.

– Приведите пример уравнения, равносильного уравнению 3х – 16 = 18; 3,8х = –11.

Домашнее задание: № 113, № 115, № 117, № 119.

 

Урок 13
  линейное уравнение с одной переменной

Цели: ввести определение линейного уравнения с одной переменной (общий вид); выяснить, сколько корней может иметь линейное уравнение; формировать умение решать линейное уравнение переходом к равносильному уравнению, применяя свойства уравнений и выполняя тождественные преобразования.

Ход урока

I. Устная работа.

1. Какие из чисел 3; –2; 2 являются корнями следующих уравнений:

а) 3х = –6;                          г) 4х – 4 = х + 5;

б) 3х + 2 = 10 – х;               д) 10х = 5(2х + 3);

в) х + 3 = 6;                        е) 10 + х = 13?

2. Являются ли уравнения равносильными? Если да, то сформулируйте, по какому свойству уравнений.

а) 3х + 4 = 2                 и     3х = –2;

б) –3х + 12 + 2х = 4     и     2х + 12 = 3х + 4;

в) 3х + 15 = 0               и     3х = 15;

г) 0,5х = 0,08               и     50х = 8;

д) 120х = –10               и     12х = 1;

е) x = 11                   и     3х = 44.

II. Объяснение нового материала.

1. Актуализация знаний.

Напомним  учащимся  свойства  верных  неравенств  (запись в виде таблицы):

Для чисел,
обозначенных
цифрами

Для чисел,
обозначенных
буквами

Словесная
формулировка

1

2

3

1. 7 = 7

    7 + 2 = 7 + 2

    7 – 2 = 7 – 2

а = b

a + l = b + l

a – l = b – l

l – любое число

Если к обеим частям верного равенства прибавить одно и то же число или
из обеих частей верного равенства вычесть одно и то же число, то получится верное равенство


Окончание табл.

1

2

3

2. 27 = 27

    27 · 3 = 27 · 3

    27 : 3 = 27 : 3

    3 ¹ 0

а = b

a · m = b · m

a : m = b : m

m ¹ 0

Если обе части верного равенства умножить или разделить на одно и то же
не равное нулю число, то получится
верное равенство

Используя  данные  таблицы,  учащиеся  формулируют  свойства  уравнений.

2. Мотивация изучения.

Рассмотрим уравнение 9х – 23 = 5х – 11. Применим известные свойства уравнений и получим равносильные уравнения:

9х – 5х = – 11 + 23;

       4х = 12;

        х = 3.

Уравнение, равносильное исходному, имеет единственный корень 3, значит, исходное уравнение также имеет единственный корень 3.

Используя свойства уравнений, многие из них всегда можно привести к виду ax = b, где х – переменная, а a и b – некоторые числа. Уравнения такого вида называются линейными.

Важно подчеркнуть учащимся, что, используя буквенные обозначения, мы записали целый класс уравнений.

3. Организация исследовательской деятельности учащихся.

На этом этапе востребуется логический прием мышления – обобщение.

Задание. Привести уравнение к линейному виду, используя свойства уравнений:

а) 3х – 11 = 5х + 7;

б) 2 (х + 1) = 2х + 2;

в) –8х + 11 = 8 (3 – х).

Решение:

а) 3х – 11 = 5х + 7;                       б) 2 (х + 1) = 2х + 2;

    3х – 5х = 7 + 11;                          2х + 2 = 2х + 2;

    –2х = 18.                                      2х – 2х = 2 – 2;

                                                        0 · х = 0.

в) –8х + 11 = 8 (3 – х);

    –8х + 11 = 24 – 8х;

    –8х + 8х = 24 – 11;

    0 · х = 13.

Теперь, глядя на линейное уравнение, записать, чему равны коэффициенты a и b и сколько корней имеет уравнение. как это определили?

а) a = –2; b = 18 – один корень х = –9, определили, разделив обе части на (–2).

б) a = 0; b = 0 – бесконечно много корней, так как равенство 0 · х = 0 верно при любом значении х.

в) a = 0; b = 13 – нет корней, так как равенство 0 · х = 13 неверно ни при каком значении х.

Обобщая полученные данные, заполняем таблицу решения линейного уравнения в общем виде:

Линейное уравнение

ax = b, где х – переменная, a, b – любое число.

Если a ¹ 0, то x = ;

если а = 0 и b = 0, то х – любое;

если а = 0 и b ¹ 0, то нет корней.

4. Создание алгоритма  решения  уравнений,  сводящихся к линейным.

Анализируя решенные примеры, приходим к выводу, что решение многих уравнений сводится к решению линейных.

Учащиеся могут сами создать алгоритм:

1-й шаг. Если выражения, стоящие в левой или правой части уравнения, содержат скобки, то раскрываем их по правилам.

2-й шаг. Переносим слагаемые с переменными в левую часть уравнения, а без переменных в правую.

3-й шаг. Приводим подобные слагаемые в обеих частях уравнения, приводя его к виду ax = b.

4-й шаг. Решаем получившееся линейное уравнение, равносильное исходному, в зависимости от значений коэффициентов a и b.

III. Формирование умений и навыков.

Задания, решаемые на этом уроке, направлены на усвоение определения линейного уравнения и решение линейных уравнений в зависимости от значений коэффициентов a и b.

1. (Устно.) Назовите коэффициенты a и b линейного уравнения ax = b. Сколько корней имеет уравнение:

а) 3х = 12;                 в) 1x = –14;                д) 0 · х = 0;

б) –3х = 18;               г) 0 ∙  x = ;                            е) –18х = –2?

2. Решите уравнение.

а) –8х = 24;               г) –3x = ;                    ж) –6 = x;

б) 50х = –5;               д) –x = –1;                  з) ;

в) –18х = 1;               е)  = –5x;                    и) –0,81х = 72,9.

3. Определите значение х, при котором значение выражения –3х равно:

а) 0;        б) 6;        в) –12;        г) ;        д) ;        е) 2.

3. (Устно.) На доске было записано решение линейного уравнения, но правую часть данного уравнения стерли. Восстановите ее:

а) 3х = ;         б) 5х = ;              в) x = ;

    х = 11.                        х = 0.                             х = 14.

4. При каких значениях а уравнение ах = 8:

а) имеет корень, равный –4; ; 0;

б) не имеет корней;

в) имеет отрицательный корень?

IV. Итоги урока.

– Дайте определение линейного уравнения с одной переменной. Приведите примеры.

– В каком случае уравнение ax = b имеет единственный корень? бесконечно много корней? Не имеет корней?

– Сформулируйте  алгоритм  решения  уравнения,  сводящегося  к  линейному.

Домашнее задание: № 126, № 127, № 245, № 142.

 

 

 

 

 

 

 

 

 

Урок 14
линейное уравнение с одной переменной

Цель: формировать умение решать по алгоритму уравнения, сводящиеся к линейным.

Ход урока

I. Устная работа.

1. Вычислите.

а) ;               в) 8 : ;             д)  · (–16);

б) (–3) · ;              г) ;              е)  · 16.

2. Является ли число –11 корнем уравнения:

а) –3х = 33;               в) 0 · х = 0;           д) x = –9;

б) 4х = 44;                г) x = 7;        е) 0 · х = –11?

II. Проверочная работа.

Вариант 1

1. Сколько корней имеет уравнение:

а) –2х = 17;               б) 0 · х = –6;         в) 0 · х = 0?

2. Найдите корень уравнения.

а) 26х = –78;             б) 0,2х = 2,8;        в) x = 24;        г) –3x = .

Вариант 2

1. Сколько корней имеет уравнение:

а) 0 · х = –72;            б) x = 11;          в) 0 · х = 0?

2. Найдите корень уравнения.

а) 21х = 84;          б) –1,2х = 0,36;         в) x = 21;          г) –2x = .

III. Формирование умений и навыков.

1. Актуализация знаний.

Предлагаем учащимся вспомнить свойства уравнений и основывающийся на них алгоритм решения уравнений, сводящихся к линейным.

2. Задания, решаемые на уроке, выстроены по нарастанию уровня сложности: сначала на применении одного свойства уравнений, затем – их комбинации и, наконец, на полном применении алгоритма с преобразованием выражений, стоящих в обеих частях уравнения.

№ 128 (а; б; е; ж; и); № 129; № 131.

3. № 131, № 132.

№ 131.

Решение:

а) (у + 4) – (у – 1) = 6у;

    у + 4 – у + 1 = 6у;

    у у – 6у = –4 – 1;

    – 6у = –5;

    у = (–5) : (–6);

    у = ;

б) 3р – 1 – (р + 3) = 1;

    3р – 1 – р – 3 = 1;

    3р р = 1 + 1 + 3;

    2р = 5;

    р = 5 : 2;

    р = 2,5;

в) 6х – (7х – 12) = 101;

    6х – 7х + 12 = 101;

    6х – 7х = 101 – 12;

    –х = 89;

    х = –89.

г) 20х = 19 – (3 + 12х);

    20х = 19 – 3 – 12х;

    20х + 12х = 19 – 3;

    32х = 16;

    х = 16 : 32;

    х = 0,5.

№ 132.

Решение:

а) (13х – 15) – (9 + 6х) = –3х;

    13х – 15 – 9 – 6х = –3х;

    13х – 6х + 3х = 15 + 9;

    10х = 24;

    х = 24 : 10;

    х = 2,4.

б) 12 – (4х – 18) = (36 + 4х) + (18 – 6х);

    12 – 4х + 18 = 36 + 4х + 18 – 6х;

    – 4х – 4х + 6х = 36 + 18 – 12 – 18;

    – 2х = 24;

    х = 24 : (–2);

    х = –12.

в) 1,6х – (х – 2,8) = (0,2х + 1,5) – 0,7;

    1,6хх + 2,8 = 0,2х + 1,5 – 0,7;

    1,6хх – 0,2х = 1,5 – 0,7 – 2,8;

    0,4х = –2;

    х = (–2) : 0,4;

    х = –5.

г) (0,5х + 1,2) – (3,6 – 4,5х) = (4,8 – 0,3х) + (10,5х + 0,6);

    0,5х + 1,2 – 3,6 + 4,5х = 4,8 – 0,3х + 10,5х + 0,6;

    0,5х + 4,5х + 0,3х – 10,5х = 4,8 + 0,6 – 1,2 + 3,6;

    –5,2х = 7,8;

    х = 7,8 : (–5,2);

    х = –1,5.

4. № 134.

Решение:

а) 8b – 27 = 5;

    8b = 5 + 27;

    8b = 32;

    b = 32 : 8;

    b = 4.

б) 8b – 27 = –11;

    8b = –11 + 27;

    8b = 16;

    b = 16 : 8;

    b = 2.

в) 8b – 27 = 1,8;

    8b = 1,8 + 27;

    8b = 28,8;

    b = 28,8 : 8;

    b = 3,6.  

г) 8b – 27 = –1;

    8b = –1 + 27;

    8b = 26;

    b = 26 : 8;

    b = 3,25.

5. При каком значении t:

а) значение выражения 5t + 11 равно значению выражения 7t + 31;

б) значение  выражения  8t + 3  в  три  раза  больше  значения  выражения 5t – 6;

в) значение  выражения  5t + 1  в два раза меньше значения выражения 10t + 18;

г) значение  выражения  0,25t – 31  на  5  больше  значения  выражения t – 18;

д) значение  выражения  13t – 7  на  8  меньше  значения  выражения
12t + 11;

е) разность выражений 1,5t – 37 и 1,5t – 73 равна 36?

Основную трудность при составлении равенств у учащихся вызывают задания б) – д). Следует разобрать принцип составления равенства с использованием наглядности.

Решение:

б) 8t + 3                    5t – 6                    8t + 3                    3 (5t – 6)

                               

(8t + 3) = 3 (5t – 6);

8t + 3 = 15t – 18;

8t – 15t = – 18 – 3;

–7t = –21;

t = 3.

в) 5t + 1                    10t + 18               5t + 1                    (10t + 18) : 2

                              

5t + 1 = (10t + 18) : 2;

5t + 1 = 5t + 9;

5t – 5t = 9 – 1;

0 · t = 8 – нет решений.

г) 0,25t – 31              t – 18                0,25t – 31             + 5

                               

0,25t – 31 = t – 18 + 5;

0,25tt = – 18 + 5 + 31;

0 · t = 18 – нет решений.

д) 13t – 7 = (12t + 11) – 8   или   (13t – 7) + 8 = 12t + 11.

На этом примере можно показать, что, составив равенство по принципу «из большего вычитаем меньшее и получаем разность», мы получаем два равносильных уравнения. Просим объяснить учащихся почему.

е) (1,5t – 37) – (1,5t – 73) = 36;

    1,5t – 37 – 1,5t + 73 = 36;

    1,5t – 1,5t = 36 + 37 – 73;

    0 · t = 0                 – t – любое число.

IV. Итоги урока.

– Какое  уравнение  называется  линейным  уравнением  с  одной  переменной?

– Если а = 0 и b ¹ 0, то сколько корней имеет уравнение вида ах = b? Если а = 0 и b = 0?

– Сформулируйте  алгоритм  решения  уравнения,  сводящегося  к  линейному.

Домашнее задание: № 128 (в; г; д; з); № 130; № 133; № 135.

 

Урок 15
Решение задач по теме
«Линейное уравнение с одной переменной»

Цели: продолжить формировать умение решать уравнения, сводящиеся к линейным.

Ход урока

I. Устная работа.

1. Показать, что следующие уравнения не имеют решений, и объяснить почему:

а) х + 3 = х;               в) 2х = 2(х + 1);             д) (–х)2 + 1 = 0.

б) х – 1 = х + 1;         г) х2 + 4 = 0;        

2. Определить, равносильны ли уравнения и почему:

а) 5х + 1 = 2              и       10х + 2 = 4;

б) 2х – 1 = 4              и       2х = 6;

в) 3х + 1 = 10            и       х = 3;

г) 2х + 3 = 2х – 4       и       х + 5 = х;

д)               и       21х = –6.

II. Математический диктант.

Вариант 1

1. Придумайте и запишите какое-нибудь линейное уравнение с одним неизвестным х.

2. Как называется уравнение –2х = 17?

3. При каком условии уравнение сх = 5 имеет единственный корень? Запишите этот корень.

4. Решите уравнение 0,2х = –1.

5. К обеим частям уравнения прибавили число –3. Какими являются полученное и исходное уравнения?

6. Решите уравнение 2х + 1 = 3хх.

7. Решите уравнение 5 – х = 2х + 2.

Вариант 2

1. Придумайте и запишите какое-нибудь линейное уравнение с одним неизвестным у.

2. Как называется уравнение 17х = –2?

3. При каком условии уравнение ау = 3 не имеет корней?

4. Решите уравнение –0,3х = 1.

5. Обе части уравнения умножим на число –7. Какими являются полученное и исходное уравнения?

6. Решите уравнение х + 3 = 5 + х – 2.

7. Решите уравнение 2 – 2х = –2х + 3.

III. Формирование умений и навыков.

Все задания, решаемые на этом уроке, относятся к более высокому уровню овладения изучаемым материалом. Их выполнение требует более продвинутых технических навыков, нестандартных приемов решения, определенной сообразительности.

1. Решите уравнение.

а) (5х – 3) + (7х – 4) = 8 – (15 – 11х);

б) (4х + 3) – (10х + 11) = 7 + (13 – 4х);

в) (7 – 5х) – (8 – 4х) + (5х + 6) = 8;

г) (3 – 2х) + (4 – 3х) + (5 – 5х) = 12 + 7х.

Решение:

а) (5х – 3) + (7х – 4) = 8 – (15 – 11х);

    5х – 3 + 7х – 4 = 8 – 15 + 11х;

    5х + 7х – 11х = 8 – 15 + 3 + 4;

    х = 0.

б) (4х + 3) – (10х + 11) = 7 + (13 – 4х);

    4х + 3 – 10х – 11 = 7 + 13 – 4х;

    4х – 10х + 4х = 7 + 13 – 3 + 11;

    –2х = 28;

    х = 28 : (–2);

    х = –14.

в) (7 – 5х) – (8 – 4х) + (5х + 6) = 8;

    7 – 5х – 8 + 4х + 5х + 6 = 8;

    – 5х + 4х + 5х = 8 – 7 + 8 – 6;

    4х = 3;

    х = .

г) (3 – 2х) + (4 – 3х) + (5 – 5х) = 12 + 7х;

    3 – 2х + 4 – 3х + 5 – 5х = 12 + 7х;

    – 2х – 3х – 5х – 7х = 12 – 3 – 4 – 5;

    –17х = 0;

    х = 0.

2. Среди данных уравнений выберите те, которые имеют тот же корень, что и уравнение 2х – 3 = 5х + 6:

а) 19 (2х – 3) = 19 (5х + 6);

б) 5х – 2х = 6 – 3;

в) .

Решение:

2х – 3 = 5х + 6;

2х – 5х = 6 + 3;

–3х = 9;

х = –3.

а) 19 (2х – 3) = 19 (5х + 6); | : 19

    2х – 3 = 5х + 6;

    х = –3, так как уравнение равносильно исходному.

При  решении  данного  уравнения  важно  заметить,  что  разделить обе части уравнения на 19 рационально, а выполнить умножение числа на скобку – нет.

б) 5х – 2х = 6 – 3;               в)    | · 11;

    3х = 3;                                 2х – 3 = 5х + 6;

    х = 1.                                  2х – 5х = 6 + 3;

                                              х = –3,

                                          так как уравнение равносильно исходному.

Ответ: а);  в);  х = –3.

3. Среди данных уравнений укажите те, которые не имеют корней:

а) 5х – 10 = 4х;                   в) 5 – х = 6 – х;              д) | x | + 1 = 0.

б) 3х + 7 = 3х + 11;                      г) | x | = 8;            

Решение:

а) 5х – 10 = 4х;                   б) 3х + 7 = 3х + 11;

    5х – 4х = 10;                       3х – 3х = 11 – 7;

    х = 10.                                 0 · х = 4 – нет корней.

в) 5 – х = 6 – х;                   г) | x | = 8;                      д) | x | + 1 = 0.

    –х + х = 6 – 5;                     х = 8 или х = –8.            | x | = –1 –

    0 · х = 1 – нет корней.                                             нет решений,

                                                                                    так как | x | ≥ 0.

4. № 238*, № 239*, № 242*.

Данные задания повышенной трудности.

№ 238. Решение:

Если т ¹ 0, то тх = 5 имеет единственный корень х = 5 : т.

Если т = 0, то уравнение примет вид 0 · х = 5, оно не имеет корней.

Не существует такое значение т, чтобы уравнение имело бесконечно много корней.

№ 239. Решение:

Если х = –5, то р · (–5) = 10 – верное равенство.

Найдем р:                 р = 10 : (–5);

                                 р = –2.

Если х = 1, то           р · (–1) = 10;

                                 р = 10 : (–1);

                                 р = –10.

Если х = 20, то         р · 20 = 10;

                                 р = 10 : 20;

                                 р = 0,5.

Ответ: –2; –10; 0,5.

Обращаем внимание учащихся, что это уравнение с параметром р.

№ 242. Решение:

а) (х + 5) (х + 6) + 9 = 0;

    х2 + 6х + 5х + 30 + 9 = 0;

    х2 + 11х + 39 = 0;

    х2 = –11х – 39.

Слева стоит выражение, значение которого не отрицательно. если х – положительное число, то –11х < 0 и –11х – 39 < 0, значит, х2 = –11х – 39 – неверно для любого положительного х, значит, уравнение не может иметь положительный корень.

б) х2 + 3х + 1 = 0.

Если х > 0, то каждое слагаемое в левой части уравнения положительно, значит, и вся сумма положительна, следовательно, х > 0 не может являться корнем данного уравнения.

IV. Итоги урока.

– Дайте определение линейного уравнения.

– Когда линейное уравнение имеет единственный корень? Бесконечно много корней? Не имеет решений?

– Назовите шаги решения уравнения, сводящегося к линейному.

– Решите уравнение .

Домашнее задание: № 136, № 137, № 138, № 246*.

 

 

 

 

 

 

 

 

 

Урок 15
РЕШЕНИЕ ЗАДАЧ С ПОМОЩЬЮ УРАВНЕНИЙ

Цели: обеспечить понимание уравнения в качестве математической модели некоторой жизненной ситуации, описанной в текстовой задаче; выделить этапы решения задач алгебраическим методом; формировать умение составлять уравнение по условию задачи и решать его.

Ход урока

I. Устная работа.

1. Найдите корни уравнения.

а) 8х = 16;                 б) 3x = ;                    в) x = 5;

г) 3x – 15 = 0;           д) –х – 4 = 0;                  е) х + 7 = –11;

ж) 0 ∙  x = ;           з) 2x = 2x – 4;                и) 2(x + 3) = 2x + 6.

2. Найдите:

а) 50 % от 80;           г) 20 % от 25;                ж) 50 % от 17;

б) 10 % от 300;         д) 25 % от 400;              з) 40 % от 10;

в) 1 % от 30;             е) 5 % от 200;                и) 9 % от 500.

II. Объяснение нового материала.

1. Решение текстовых задач способствует развитию логического мышления учащихся, более глубокому усвоению идеи функциональной зависимости, повышает вычислительную культуру. В процессе решения текстовых задач у учащихся формируются умения и навыки моделирования реальных объектов и явлений.

В курсе математики рассматриваются два основных способа решения текстовых задач: арифметический и алгебраический. При изучении предыдущих пунктов 6–7 учащиеся пользовались арифметическим способом: неизвестную величину находили посредством составления числового выражения (числовой формулы) и подсчета результата. Алгебраический способ основан на использовании уравнений и систем уравнений, составленных при решении задач.

В методике математики общепринято следующее деление процесса решения задач:

1) анализ текста задачи;

2) поиск способа решения задачи и составление плана решения;

3) осуществление найденного плана;

4) изучение (анализ) найденного решения.

Основную трудность представляет для учащихся первый этап. Поэтому на  этом  уроке  следует  максимально  использовать  средства  наглядности (таблицы, схемы, чертежи) при анализе условия.

2. Объяснение лучше начать с решения конкретной (приведенной в учебнике) задачи № 1.

Можно воспользоваться таблицей:

Сперва в таблице стрелками обозначаем и подписываем все зависимости, затем видим, что неизвестны все четыре клеточки, значит, обозначить переменной удобно главный вопрос задачи, например, количество яблок в корзине первоначально. Затем, по стрелкам, заполняем все клеточки. Последняя стрелка даст уравнение: 5(х – 10) = 2х + 10.

Аналогичную таблицу можно составить для задачи № 2:

х + 2х + (х + 12) = 78.

При решении второй задачи особое внимание уделяется последнему этапу – интерпретации полученного результата.

III. Формирование умений и навыков.

При  решении  задач  особое  внимание  уделяем  анализу  условия  задачи, выбору переменной и выбору основной зависимости для составления уравнения. При решении уравнения используем соответствующий алгоритм.

1. № 143.

Решение:

Пусть в одной кассе было х билетов, тогда во второй – (х + 36) билетов. Зная, что всего было продано 392 билета, составим уравнение:

х + (х + 36) = 392;

х + х + 36 = 392;

2х = 356;

х = 178.

Следовательно, в первой кассе было продано 178 билетов.

Так как х + 36 = 178 + 36 = 214, то во второй кассе было продано 214 билетов.

Ответ: 178 и 214 билетов.

2. № 146.

Решение:

Анализ условия:

Пусть х м – длина одного тоннеля, тогда (х + 17) м – длина другого. Так как наземная часть составляет 703 м, а вся трасса – 6940 м, то длина тоннелей в сумме составляет (6940 – 703) м. Зная, что длина тоннелей равна х + (х + 17) м, составим уравнение:

х + (х + 17) = 6940 – 703;

х + х + 17 = 6237;

х + х = 6237 – 17;

2х = 6220;

х = 3110.

Значит, длина одного тоннеля равна 3110 м. Так как х + 17 = = 3110 + 17 = 3127, то длина другого тоннеля равна 3127 м.

Ответ: 3110 м и 3127 м.

Обращаем внимание учащихся, что для анализа условия можно использовать не только таблицы, но и рисунки-схемы. Если ученик делает удобный ему чертеж, соответствующий условию задачи, то стоит его поощрить.

3. № 147.

Решение:

Анализ условия:

Пусть первый жертвователь дал х рупий, тогда второй дал 2х рупий, третий – 3 · 2х рупий, четвертый – 4 · (3 · 2х) рупий. Зная, что все вместе они дали 132 рупии, составим уравнение:

х + 2х + 3 · 2х + 4 · (3 · 2х) = 132;

х + 2х + 6х + 24х = 132;

33х = 132;

х = 132 : 33;

х = 4.

Значит, первый жертвователь дал 4 рупии. Так как 2х = 2 · 4 = 8, то второй дал 8 рупий. Так как 3 · 2х = 3 · 8 = 24, то третий дал 24 рупии. Так как 4 · (3 · 2х) = 4 · 24 = 96, то четвертый дал 96 рупий.

Ответ: 4; 8; 24 и 96 рупий.

4. № 148.

Решение:

Анализ условия:

Пусть  х  деталей  изготовил  второй  рабочий,  тогда  первый  изготовил (х + 0,15х) деталей. Зная, что вместе они изготовили 86 деталей, составим уравнение:

х + (х + 0,15х) = 86;

х + х + 0,15х = 86;

2,15х = 86;

х = 86 : 2,15;

х = 40.

Значит, второй рабочий изготовил 40 деталей. Так как х + 0,15х = 40 +
+ 0,15 · 40 = 40 + 6 = 46, то первый рабочий изготовил 46 деталей.

Ответ: 46 деталей и 40 деталей.

Учащиеся могут сами заметить, что во всех задачах мы за переменную обозначаем меньшее неизвестное. Это более удобно. Можно спросить: почему? Это связано с тем, что если мы обозначим за переменную большее число, то в уравнении появятся дробные коэффициенты, а это несколько усложняет его решение.

IV. Итоги урока.

– Какими способами можно решать текстовые задачи?

– Из  каких  этапов  состоит  алгебраический  метод  решения  текстовой задачи?

– Чем является уравнение для описываемой в тексте задачи ситуации?

– Какие способы наглядного представления условия задачи мы можем использовать?

– Для чего необходимо истолковывать полученный корень уравнения в соответствии с условием задачи?

Домашнее задание: № 144; № 145; № 149; № 165.

 

 

 

Урок 16
РЕШЕНИЕ ЗАДАЧ С ПОМОЩЬЮ УРАВНЕНИЯ

Цели: продолжить формировать умение решать текстовые задачи алгебраическим методом – с помощью составления уравнений, сводящихся к линейным.

Ход урока

I. Устная работа.

1. Вычислите.

а) 0,35 · 0,2 + 0,35 · 0,8;     в) ;                д) ;

б)  · 0,5 · 8;                     г) ;            е) (–3)2 – 9,2.

2. Выразите:

а) t     из   s = х · t;              в) y   из   х = 2ay;

б) p   из   N = p : t;              г) x   из   y = .

II. Проверочная работа.

Вариант 1

1. Двое рабочих изготовили 657 деталей, причем первый изготовил на 63 детали больше второго. Сколько деталей изготовил каждый рабочий?

2. Папе и дедушке вместе 111 лет. Сколько лет каждому, если папа в 2 раза моложе дедушки?

Вариант 2

1. В двух седьмых классах 67 учеников, причем в одном на 3 ученика больше, чем в другом. Сколько учеников в каждом классе?

2. У Коли и Пети вместе 98 марок, причем у Коли в 6 раз больше марок, чем у Пети. Сколько марок у каждого мальчика?

Решение заданий проверочной работы

Вариант 1

1. Пусть х деталей изготовил второй рабочий, тогда (х + 63) деталей – первый. Зная, что вместе они изготовили 657 деталей, составим уравнение:

х + (х + 63) = 657;

х + х + 63 = 657;

2х = 657 – 63;

2х = 594;

х = 297.

Значит, второй рабочий изготовил 297 деталей. Так как х + 63 = 297 +
+ 63 = 360, то первый рабочий изготовил 360 деталей.

Ответ: 360 деталей и 297 деталей.

2. Пусть папе х лет, тогда дедушке 2х лет. Зная, что папе и дедушке вместе 111 лет, составим уравнение:

х + 2х = 111;

3х = 111;

х = 37.

Значит, папе 37 лет. Так как 2х = 2 · 37 = 74, то дедушке 74 года.

Ответ: 37 лет и 74 года.

Вариант 2

1. Пусть в одном классе х учеников, тогда во втором (х + 3) ученика. Зная, что в двух классах 67 учеников, составим уравнение:

х + (х + 3) = 67;

х + х + 3 = 67;

2х = 67 – 3;

2х = 64;

х = 32.

Значит, в одном классе 32 ученика. Так как х + 3 = 32 + 3 = 35, то во втором классе 35 учеников.

Ответ: 32 ученика и 35 учеников.

2. Пусть у Пети было х марок, тогда у Коли – 6х марок. Зная, что вместе у них было 98 марок, составим уравнение:

х + 6х = 98;

7х = 98;

х = 14.

Значит, у Пети было 14 марок. Так как 6х = 6 · 14 = 84, то у Коли было 84 марки.

Ответ: 14 марок и 84 марки.

III. Формирование умений и навыков.

При решении задач замечаем, что неизвестную величину не обязательно  обозначаем  за  х.  Наоборот,  если  в  задаче  используется  формула, например, s = х · t, то и переменную удобно обозначать соответствующей буквой.

1. № 151.

Решение:

Анализ условия:

Пусть х г шерсти ушло на шапку, тогда на свитер ушло 5х г, а на шарф – (х – 5) г шерсти. Зная, что на все изделия ушло 555 г шерсти, составим уравнение:

х + 5х + (х – 5) = 555;

х + 5х + х – 5 = 555;

7х = 560;

х = 80.

Значит, на шапку ушло 80 г шерсти. Так как 5х = 5 · 80 = 400, то на свитер ушло 400 г шерсти.

Так как х – 5 = 80 – 5 = 75, то на шарф ушло 75 г шерсти.

Ответ: 400 г; 80 г; 75 г.

2. № 152.

Решение:

Анализ условия:

Пусть  на  первой  полке  расположено  п  книг,  тогда  на  второй  полке – (п + 8), а на третьей – (п – 5) книг. Зная, что на трех полках необходимо расположить всего 158 книг, составим уравнение:

п + (п + 8) + (п – 5) = 158;

п + п + 8 + п – 5 = 158;

3п + 3 = 158;

3п = 155;

п = 51.

Интерпретация результата: так как п – число книг, то п дол-жно быть натуральным числом. 51 – дробное, значит, указанным способом нельзя разместить книги на полках.

Ответ: нельзя.

На примере этой задачи видно, что важен этап интерпретации полученного решения.

3. № 154.

Решение:

Анализ условия:

Пусть х кустов малины было на втором садовом участке, тогда на первом было 5х кустов. После пересадки на первом участке осталось (5х – 22) кустов малины, а на втором стало (х + 22) куста малины. Зная, что после пересадки  на  обоих  участках  стало  кустов  малины  поровну,  составим уравнение:

5х – 22 = х + 22;

5хх = 22 + 22;

4х = 44;

х = 11.

Значит,  на  втором  участке  было  11  кустов  малины.  Так  как  5х =
= 5 · 11 = 55, то на первом участке было 55 кустов малины.

Ответ: 55 и 11 кустов малины.

4. № 155.

Решение:

Анализ условия:

 

х (км/ч)

t (ч)

s (км)

По течению

хc + 2

9

  9 · (хc + 2)

Против течения

хc – 2

11

  11 · (хc – 2)

Пусть хc км/ч – собственная скорость теплохода, тогда по течению он шел со скоростью (хc + 2) км/ч и за 9 часов прошел 9 · (хc + 2) км. Против течения он шел со скоростью (хc – 2) км/ч и прошел 11 · (хc – 2) км. Зная, что он прошел по течению и против одинаковое расстояние, составим уравнение:

9 · (хc + 2) = 11 · (хc – 2);

9 хc + 18 = 11 хc – 22;

9 хc – 11 хc = – 22 – 18;

–2 хc = –40;

хc = 20.

Значит, собственная скорость теплохода равна 20 км/ч.

Ответ: 20 км/ч.

При обозначении переменной можно не ставить индекс хc, а просто обозначить х. Не возбраняется использовать любую букву латинского алфавита.

5. № 157.

Решение:

Анализ условия:

 

х (верст/день)

t (день)

s (верст)

I

40

п + 1

40 (п + 1)

II

45

п

45п

Пусть второй человек догонит первого через п дней, тогда за эти дни он пройдет 45п верст. Первый человек, так как он шел на день дольше, пройдет 40 (п + 1) верст. Зная, что они пройдут одинаковое расстояние, составим уравнение:

45п = 40 (п + 1);

45п = 40п + 40;

45п – 40п = 40;

5п = 40;

п = 8

Значит, через 8 дней второй догонит первого.

IV. Итоги урока.

– Какие этапы выделяют при решении задачи алгебраическим методом?

– В чем состоит интерпретация полученного решения задачи?

– Когда полученное решение может противоречить условию задачи?

– Какие решения, полученные на сегодняшнем уроке, вы интерпретировали как противоречащие условию?

Домашнее задание: № 150, № 153,

 

 

 

 

 

 

 

 

Урок 17
РЕШЕНИЕ ЗАДАЧ НА СОСТАВЛЕНИЕ УРАВНЕНИЯ

Цель: продолжить формировать умение решать текстовые задачи алгебраическим методом – с помощью составления уравнений, сводящихся к линейным.

Ход урока

I. Актуализация знаний.

1. Запишите в виде выражения:

а) сумма чисел а и b;

б) сумма числа х и произведения чисел а и b;

в) разность числа k и частного чисел х и у;

г) произведение суммы чисел а и b и числа с;

д) произведение числа с и разности чисел х и z;

е) сумма частного чисел а и b и числа с;

ж) частное разности х и у и числа z.

2. Запишите в виде равенства:

а) х в 2 раза больше у;

б) сумма чисел а и b в 2 раза меньше числа с;

в) а на 2,5 больше b;

г) х меньше у на 18;

д) а больше b на 18 %;

е) х составляет 25 % от у.

II. Формирование умений и навыков.

Все задачи, решаемые на этом уроке, требуют составления более сложного уравнения. Основная трудность при их решении заключается в умении «увидеть» основную зависимость и правильно её записать в виде равенства.

На уроке продолжаем использовать схемы, таблицы, графики для наглядного представления условия задачи.

1. № 158.

Решение:

Анализ условия:

Пусть х плотников было в бригаде, тогда маляров было 2,5х. После переводов в бригаде стало (2,5х + 4) маляров и (х – 2) плотников. Зная, что маляров стало в 4 раза больше плотников, составим уравнение:

(2,5х + 4) = 4 · (х – 2);

2,5х + 4 = 4х – 8;

2,5х – 4х = –8 – 4;

–1,5х = –12;

х = (–12) : (–1,5);

х = 8.

Значит, в бригаде было 8 плотников. Так как 2,5х = 2,5 · 8 = 20, то в бригаде было 20 маляров.

Ответ: 20 маляров и 8 плотников.

В таблице основную зависимость, по которой формируем равенство, можно выделить другим цветом или более жирной линией.

2. № 161.

Решение:

Анализ условия:

Пусть х кг – масса первого арбуза, тогда второй арбуз весит (х + 2) кг, а третий – 5х кг. Первый и третий арбуз вместе весят х + 5х, то есть 6х кг. Зная, что в сумме они весят в 3 раза больше второго арбуза, составим уравнение:

3 · (х + 2) = 6х;

3х + 6 = 6х;

3х – 6х = –6;

–3х = –6;

х = 2.

Значит, первый арбуз весит 2 кг. Так как х + 2 = 2 + 2 = 4, то второй арбуз весит 4 кг. Так как 5 · х = 5 · 2 = 10, то третий арбуз весит 10 кг.

Ответ: 2 кг, 4 кг, 10 кг.

3. № 162.

Решение:

Анализ условия:

 

было

взяли

осталось

 

 

 

 

в 2 раза меньше

I

50

х

50 – х

II

50

3х

50 – 3х

Пусть х кг сахара взяли из первого мешка, тогда из второго мешка взяли 3х кг сахара. В первом мешке осталось (50 – х) кг сахара, а во втором – (50 – 3х) кг. Зная, что во втором мешке осталось в 2 раза меньше сахара, чем в первом, составим уравнение:

2 · (50 – 3х) = 50 – х;

100 – 6х = 50 – х;

–6х + х = 50 – 100;

–5х = –50;

х = (–50) : (–5);

х = 10.

Значит,  из  первого  мешка  взяли  10  кг  сахара.  Так  как  50 – х =
= 50 – 10 = 40, то в первом мешке осталось 40 кг сахара. Так как 50 – 3х =
= 50 – 3 · 10 = 50 – 30 = 20, то во втором мешке осталось 20 кг сахара.

Ответ: 40 кг и 20 кг.

На примере этой задачи показываем, что не всегда за переменную следует принимать главный вопрос задачи. В нашем случае выгоднее обозначить промежуточное неизвестное, а затем вычислить искомые величины. Кроме того, искомые можно вычислить по разным зависимостям. Например, как мы взяли алгебраические выражения из ячеек таблицы. А можно было найти оставшееся количество сахара в первом мешке, а во втором – вычислить по зависимости «в 2 раза меньше».

4. Федя на 7 лет старше Пети, а их папе в 3 раза больше лет, чем им обоим вместе. Сколько лет каждому из них, если папе было 36 лет, когда родился Петя?

Решение:

Анализ условия:

Пусть х лет Пете, тогда Феде (х + 7) лет, а папе (х + 36) лет. Пете и Феде вместе х + (х + 7) лет или 2х + 7 лет. Зная, что папе лет в 3 раза больше, чем им обоим вместе, составим уравнение:

(2х + 7) · 3 = х + 36;

6х + 21 = х + 36;

6хх = 36 – 21;

5х = 15;

х = 3.

Значит, Пете 3 года. Так как х + 7 = 3 + 7 = 10, то Феде 10 лет.

Ответ: Пете 3 года, Феде 10 лет.

III. Проверочная работа.

Вариант 1

Стоимость изделия третьего сорта в 3 раза меньше стоимости изделия первого сорта. Сколько стоит каждое изделие, если изделие первого сорта на 5000 р. дороже изделия третьего сорта?

Вариант 2

Мама весит в 5 раз больше дочери, а дочь на 40 кг легче мамы. Сколько весят мама и дочь в отдельности?

Решение заданий проверочной работы

Вариант 1

Анализ условия:

Пусть х р. стоит изделие III сорта, тогда изделие I сорта стоит 3х р. Зная, что изделие I сорта на 5000 р. дороже изделия III сорта, составим уравнение:

3х = х + 5000;

3хх = 5000;

2х = 5000;

х = 2500.

Значит, изделие III сорта стоит 2500 р. Так как 3х = 3 · 2500 = 7500, то изделие I сорта стоит 7500 р.

Ответ: 7500 р. и 2500 р.

Вариант 2

Анализ условия:

Пусть дочь весит х кг, тогда мама весит 5х кг. Зная, что дочь весит на 40 кг меньше, составим уравнение:

х + 40 = 5х;

х – 5х = –40;

–4х = –40;

х = 10.

Значит, дочь весит 10 кг. Так как 5х = 5 · 10 = 50, то мама весит 50 кг.

Ответ: 10 кг и 50 кг.

IV. Итоги урока.

– Какие этапы выделяют при решении задачи алгебраическим методом?

– Когда за переменную можно обозначать не главный вопрос задачи?

– В чем состоит интерпретация полученного решения задачи?

– Когда полученное решение может противоречить условию задачи?

Домашнее задание: 1. № 159, № 160, № 252.

2. Поезд был задержан в пути на 1 час. Увеличив скорость на 30 км/ч, он через 3 ч прибыл на станцию по расписанию. Какова скорость поезда до остановки?

 

 

 

 

 

 

 

 

 

 

 

 

 

 

 

 

 

 

Урок 18

Среднее арифметическое, размах и мода

Цели: ввести понятия таких статистических характеристик, как среднее арифметическое, размах и мода; формировать умение находить средние статистические характеристики различных рядов.

Ход урока

I. Устная работа.

Вычислите.

а) 1 · 3;                  б) 4 : 4;             в) ;          г) ;

д) 5 – 2;                 е) 0,9 · 6;              ж) 5,6 : 7;             з) 0,4 · 0,9;

и) 0,06 : ;             к) 8,2 · .

II. Объяснение нового материала.

1. Данный параграф (§ 4 учеб.) является первым в изучении стохастической линии. Одной из главных особенностей школьного изучения стохастики является тесная связь отвлеченных понятий и структур с окружающим миром. Поэтому математическая деятельность школьников не должна ограничиваться изучением только готовых статистических и вероятностных моделей. Напротив, процессы построения и истолкования моделей рассматриваются как ведущие формы ученической деятельности.

Сперва целесообразно провести беседу с учащимися, выяснить их представления о статистике как науке, о приложении статистики к практической деятельности человека. Речь будет идти об элементах так называемой «описательной» статистики, которая занимается вопросами сбора и представления первичной статистической информации в табличной и графической формах, вычисления числовых характеристик для совокупностей статистических данных.

2. Начинать обучение желательно с тех задач, в которых статистические сведения заданы изначально и требуется найти решение поставленной проблемы на фоне реальной ситуации.

Объяснение следует проводить согласно пункту 9 учебника.

Особое  внимание  следует  уделить  целесообразности  использования различных  средних  статистических  характеристик  в  зависимости  от ситуации.

Необходимо  подытожить,  какие  статистические  характеристики  теперь могут находить учащиеся. Для этого на доску можно вынести пример.

Упорядоченный ряд чисел:

1; 2; 2; 3; 4; 4; 5; 5; 5

1) Среднее арифметическое:

2) Размах: 5 – 1 = 4

3) Мода: 5

III. Формирование умений и навыков.

1. № 167, № 168.

Необходимо, чтобы учащиеся четко мотивировали свои ответы.

а) сложили все члены ряда и полученную сумму разделили на их количество. Значит, искали среднее арифметическое.

б) Нашли разность между наибольшим и наименьшим числом в ряду, то есть размах ряда.

в) Число … встречается наибольшее количество раз, значит, это мода ряда.

2. Даны упорядоченные ряды чисел:

а) 1; 1; 2; 3; 4; 5; 6;             б) .

Для каждого из них найти среднее арифметическое, размах и моду.

3. Найти среднее арифметическое, размах и моду рядов чисел:

а) 1; 2; 5; 2; 3; 4; 2;

б) 1; 2; 0; 2; 0; 1; 2; 1; 3; 1.

4. № 170.

5. № 171.

Решение:

Средний ежемесячный расход электроэнергии находим по формуле среднего арифметического:

x =  = 63.

Ответ: 63 кВт · ч.

6. Шесть сотрудников отдела обсуждали, кто сколько раз ходил на выборы за последние пять лет.

Соответствующие данные приведены в таблице:

Фамилия

Сколько раз участвовал в выборах

1. Андреев

8

2. Борисов

0

3. Васильев

2

4. Григорьев

0

5. Дмитриев

0

6. Евдокимов

2

Определите:

а) Сколько раз в среднем участвовали в выборах сотрудники этого отдела (среднее арифметическое)?

б) Как чаще всего поступали сотрудники отдела (мода)?

Решение:

а) среднее арифметическое равно X =  = 2.

б) Мода равна 2.

Ответ: 2; 2.

7. № 173*.

Решение:

Сумма прежних десяти членов ряда равна 15 · 10 = 150; после добавления числа 37 она станет равна 187, а количество членов ряда станет 11, поэтому среднее арифметическое нового ряда равно:

X =  = 17.

Ответ: 17.

8. № 176*.

Решение:

а) Обозначим неизвестное число через х:

 = 18;

 = 18;     х + 80 = 18 · 6;     х = 108 – 80;     х = 28.

б) A = xmaxxmin. Возможны два варианта решения:

    если считать хmax = 30, то xmin = Ахmax = –10;

    если считать xmin = 3, то хmax = А + xmin = 43.

в) Числа в ряду не повторяются, поэтому для того, чтобы 24 стало модой, нужно его повторить, то есть пропущенное число должно быть 24.

Ответ: а) 28; б) 43 или –10; в) 24.

IV. Итоги урока.

– Какие существуют средние статистические характеристики ряда?

– Какой ряд называется упорядоченным?

– Что называется размахом ряда? Приведите пример.

– Что такое мода ряда? Приведите пример.

– Как найти среднее арифметическое ряда?

 

 

 

Урок 19
Среднее арифметическое, размах и мода

Цель: продолжить формировать умения находить среднестатистические характеристики ряда (среднее арифметическое, размах, мода) при решении различных задач.

Ход урока

I. Устная работа.

Для упорядоченных рядов:

а) 0; 0; 1; 2; 3;           б) 1; 2; 2; 2; 3; 3;            в) 1; 2; 3; 4; 5; 5

найдите размах, среднее арифметическое, моду.

II. Проверочная работа.

Вариант 1

1. В таблице приведен возраст сотрудников одного из отделов:

Фамилия

Возраст

1. Башмачкин

42

2. Галошев

24

3. Каблуков

30

4. Сапогов

24

5. Тапочкин

40

Найдите среднее арифметическое, размах и моду этого ряда.

2*. Постройте ряд из четырех чисел, у которого размах равен 2, а среднее арифметическое равно моде.

Вариант 2

1. В таблице приведено количество очков, набранных в чемпионате некоторыми баскетболистами.

Фамилия

Количество очков

1. Дождева

48

2. Градова

26

3. Лунева

20

4. Метелева

40

5. Снежкова

26

Найдите среднее арифметическое, размах и моду этого ряда.

2*. Постройте ряд из четырех чисел, у которого размах равен 2, а среднее арифметическое в два раза больше моды.

III. Формирование умений и навыков.

1. № 177.

Решение:

Среднее арифметическое равно:

X = 42,45.

Размах A = xmaxxmin = 48 – 36 = 12.

Мода М = 45 (встречается 3 раза).

Среднее арифметическое – это условная величина (она не целая, хотя число деталей может быть только «целым»); она показывает центр «рассеивания» наблюдаемых величин (сумма отклонений от неё равна нулю); также это можно назвать средней выработкой рабочими деталей.

Размах характеризует разброс наблюдаемых значений, а мода показывает, какое число изготовленных деталей встречается чаще всего в данной смене рабочих.

Ответ: » 42,45; 12; 45.

2. № 179.

Решение:

Найдем средний балл каждого выпускника по формуле среднего арифметического:

Ильин: X =  = 4,4;

Семенов: X = 3,5;

Романов: X =  = 3,8;

Попов: X = 4,7.

Чтобы выявить наиболее типичную оценку для каждого выпускника, найдем для каждой совокупности моду, то есть оценку, встречающуюся чаще других:

Ильин: М = 4 (9 раз из 15);

Семенов: М = 3 (9 раз из 15);

Романов: М = 4 (10 раз из 15);

Попов: М = 5 (10 раз из 15).

Использованы среднее арифметическое и мода.

Ответ: 4,4 и 4; 3,5 и 3; 3,8 и 4; 4,7 и 5.

3. № 180.

Решение:

Средняя  урожайность  пшеницы  в  хозяйстве  равна  общему  сбору зерна,  деленному  на  общую  площадь  полей;  общий  сбор  зерна  равен 18 ц/га · 12 га + 19 ц/га · 8 га + 23 ц/га · 6га = 506 ц, а общая площадь участков равна 12 га + 8 га + 6 га = 26 га. Средняя урожайность в хозяйстве 19,5 ц/га.

Нельзя находить среднюю урожайность как  = 20 (ц/га), так как значения 18, 19 и 23 характеризуют участки разной величины и их «вклад» в общую урожайность зависит от площади каждого участка.

Ответ: » 19,5 ц/га.

4. № 181.

Решение:

Среднее арифметическое равно: X =  = 1,7.

Размах равен: A = xmaxxmin = 3 – 0 = 3.

Мода равна: М = 4 (встречается 4 раза из 10).

Среднее арифметическое показывает среднее количество бракованных деталей.

Размах показывает разброс количества бракованных деталей в ящиках.

Мода показывает наиболее часто встречающееся количество бракованных деталей.

Ответ: 1,7; 3; 4.

5. № 183.

Решение:

Среднее значение находим по формуле среднего арифметического:

X =  = 0,9.

Составим таблицу отклонений от средней температуры воздуха в полдень в каждый из дней декады:

Число месяца

1

2

3

4

5

6

7

8

9

10

Отклонение
температуры
от среднего, С °

–2,9

–1,9

–3,9

–0,9

0,1

1,1

1,1

2,1

3,1

2,1

Обращаем внимание, что сумма всех отклонений (вторая строка таблицы) равна нулю.

Ответ: 0,9 °С; таблица отклонений.

IV. Итоги урока.

– Какие существуют средние статистические характеристики ряда?

– Как найти среднее арифметическое ряда?

– Что такое размах ряда? Что он характеризует?

– Что такое мода ряда? Что она характеризует?

Домашнее задание: № 178, № 182, № 253*.

 

                                                                                

 

 

Урок 20
Медиана как статистическая характеристика

Цели: ввести понятие медианы как статистической характеристики упорядоченного ряда; формировать умение находить медиану для упорядоченных рядов с четным и нечетным числом членов; формировать умение интерпретировать значения медианы в зависимости от практической ситуации.

Ход урока

I. Устная работа.

Даны ряды:

1) 4; 1; 8; 5; 1; 7.

2) ; 9; 3; 0,5; .

3) 6; 0,2; ; 4; 6; 7,3; 6.

Найдите:

а) наибольшее и наименьшее значения каждого ряда;

б) размах каждого ряда;

в) моду каждого ряда.

II. Объяснение нового материала.

Объяснение  проводить  согласно  пункту  10  учебника.  Следует подчеркнуть, что перед нахождением медианы нужно всегда упорядочить ряд данных.

На доску следует вынести правила нахождения медианы для рядов с четным и нечетным числом членов:

Медианой упорядоченного ряда чисел с нечетным числом членов называется число, записанное посередине, а медианой упорядоченного ряда чисел с четным числом членов называется среднее арифметическое двух чисел, записанных посредине.

Медианой произвольного ряда называется медиана соответствующего упорядоченного ряда.

Особое внимание следует уделить интерпретации значений медианы для различных задач. Учитель должен прививать критическое отношение к статистическим выводам и обобщениям.

III. Формирование умений и навыков.

1-я группа. Упражнения на применение формул нахождения медианы упорядоченного и неупорядоченного ряда.

1. № 186.

Решение:

а) число членов ряда п = 9; медиана есть среднее в упорядоченном ряду значение варианта Ме = 41;

б) п = 7, ряд упорядочен, Ме = 207;

в) п = 6, ряд упорядочен, Ме =  = 21;

г) п = 8, ряд упорядочен, Ме =  = 2,9.

Ответ: а) 41; б) 207; в) 21; г) 2,9.

Учащиеся должны обосновывать способ нахождения медианы.

2. Найдите среднее арифметическое и медиану ряда чисел:

а) 27, 29, 23, 31, 21, 34;               в) ; 1.

б) 56, 58, 64, 66, 62, 74.

Решение:

Для нахождения медианы необходимо каждый ряд упорядочить:

а) 21, 23, 27, 29, 31, 34.

    п = 6; X =  = 27,5;

    Ме =  = 28;

б) 56, 58, 62, 64, 66, 74.

    п = 6; X = 63,3;

    Ме =  = 63;

в) ; 1.

    п = 5;

    X = : 5 = 3 : 5 = 0,6;

    Ме = .

3. № 188 (устно).

Решение:

а) Может, если сумма членов не кратна числу членов.

б) Не может, так как разность двух натуральных чисел,
из которых уменьшаемое больше вычитаемого, есть натуральное число.

в) Не может, так как мода – один из членов ряда, а все члены ряда – натуральные числа.

г) Может, если число членов ряда четное и числа  и  не равны между собой.

Ответ: да; б) нет; в) нет; г) да.

4. Зная, что в упорядоченном ряду содержится т чисел, где т – нечетное число, укажите номер члена, являющегося медианой, если т равно:

а) 5;     б)     17;     в) 47;      г) 201.

Решение:

Номер находим как  + 1, где  – целая часть числа.

а)  + 1 = 2 + 1 = 3;                в)  + 1 = 23 + 1 = 24;

б)  + 1 = 8 + 1 = 9;               г)  + 1 = 100 + 1 = 101.

Ответ: а) 3; б) 9; в) 24; г) 101.

2-я группа. Практические задачи на нахождение медианы соответствующего ряда и интерпретацию полученного результата.

1. № 189.

Решение:

Число членов ряда п = 12. Для нахождения медианы ряд нужно упорядочить:

136, 149, 156, 158, 168, 174, 178, 179, 185, 185, 185, 194.

Медиана ряда Ме =  = 176.

Выработка  за  месяц  была  больше  медианы  у  следующих  членов артели:

1) Квитко;                             4) Бобков;

2) Баранов;                  5) Рылов;

3) Антонов;                 6) Астафьев.

Ответ: 176.

2. № 192.

Решение:

Упорядочим ряд данных:

30, 31, 32, 32, 32, 32, 32, 32, 33, 35,

35, 36, 36, 36, 38, 38, 38, 40, 40, 42;

число членов ряда п = 20.

Размах A = xmaxxmin = 42 – 30 = 12.

Мода Мо = 32 (это значение встречается 6 раз – чаще других).

Медиана Ме =  = 35.

Размах показывает наибольший разброс времени на обработку детали; мода показывает наиболее типическое значение времени обработки; медиана – время обработки, которое не превысили половина токарей.

Ответ: 12; 32; 35.

IV. Итоги урока.

– Что называется медианой ряда чисел?

– Может  ли  медиана  ряда  чисел  не  совпадать  ни  с  одним  из  чисел ряда?

– Какое  число  является  медианой  упорядоченного  ряда,  содержащего 2п чисел? 2п – 1 чисел?

– Как найти медиану неупорядоченного ряда?

Домашнее задание: № 187, № 190, № 191, № 254.

 

 

 

 

 

 

 

 

 

 

 

 

 

 

 

Урок 21
Медиана как статистическая характеристика

Цели: продолжить формировать умение использовать средние статистические характеристики (размах, мода, среднее арифметическое, медиана) при решении различных задач (вычисление и интерпретация).

Ход урока

I. Устная работа.

1. Педагогический стаж восьми учителей школы, работающих в старших классах одной школы, следующий:

5 лет, 8 лет, 15 лет, 12 лет, 8 лет, 14 лет, 18 лет, 9 лет.

Найдите моду и медиану этой выборки.

2. Найдите среднее арифметическое и размах ряда:

2; 3; 5; 6; 14; 15; 17; 18.

II. Проверочная работа.

Вариант 1

1. Найдите медиану упорядоченного ряда:

а) ;

б) 11, 12, 18, 23, 29, 31, 37, 42.

2. Найдите медиану неупорядоченного ряда:

8, 11, 4, 17, 35, 21, 19, 50.

Вариант 2

1. Найдите медиану упорядоченного ряда:

а) ;

б) 0,5; 1,2; 1,8; 2,5; 3,5; 4,8; 5,1; 5,9.

2. Найдите медиану неупорядоченного ряда:

21, 13, 18, 11, 27, 32, 23, 41.

III. Формирование умений и навыков.

На данном уроке обобщаются знания по теме «Статистические характеристики» и учащимся предлагаются задания на нахождение всех характеристик и их интерпретацию в зависимости от условия задачи.

Кроме того, сильным учащимся можно предложить для решения задачи повышенной сложности. В конце занятия целесообразно привести пример, показывающий необходимость критического отношения к полученным результатам.

1. В  таблице  показано  число  посетителей  выставки  в  разные  дни недели:

День недели

Пн

Вт

Ср

Чт

Пт

Сб

Вс

Число посетителей

604

638

615

636

625

710

724

Найдите медиану указанного ряда данных. В какие дни недели число посетителей выставки было больше медианы?

Решение:

Число членов в ряду п = 7. Для нахождения медианы упорядочим ряд: 604, 615, 625, 636, 638, 710, 724.

Медиана Ме = 636. Число посетителей было больше медианы во вторник, субботу и воскресенье.

Ответ: 636; вторник, суббота, воскресенье.

2. Ниже указана среднесуточная переработка сахара (в тыс. ц) заводами сахарной промышленности некоторого региона:

12,2;  13,2;  13,7;  18,0;  18,6;  12,2;  18,5;  12,4;  14,2;  17,8.

Для представленного ряда данных найдите среднее арифметическое, моду, размах и медиану. Что характеризует каждый из этих показателей?

Решение:

Число членов ряда п = 10. Упорядочим ряд:

12,2; 12,2; 12,4; 13,2; 13,7; 14,2; 17,8; 18,0; 18,5; 18,6.

Среднее арифметическое характеризует средний уровень значений и общую сумму всех значений:

х = 15,08.

Мода Мо = 12,2 показывает значение, встречающееся чаще других (в данном случае слабо выражена, значение 12,2 встречается только 2 раза).

Размах A = xmaxxmin = 18,6 – 12,2 = 6,4 характеризует величину разброса наблюдаемых значений.

Медиана Me =  = 13,95 показывает, что половина членов ряда не превосходит по величине 13,95.

Ответ: 15,08; 12,2; 6,4; 13,95.

3. Девочки седьмого класса на уроке физкультуры при прыжках взяли высоты, величины которых (в см) учитель записал в журнал:

90;  125;  125;  130;  130;  135;  135;  135;  140;  140;  140.

Какая высота прыжка наилучшим образом характеризует спортивную подготовку девочек класса?

Решение:

Ряд наблюдений упорядочен: п = 11.

Ряд имеет две моды: Мо1 = 135, Мо2 = 140.

Среднее арифметическое ряда равно х » 129,5.

Медиана Ме = 135.

Наилучшей характеристикой спортивной подготовки девочек следует признать медиану: мода неоднозначна (135 и 140), а среднее значение занижено за счет одного очень плохого результата 90 см (если этот результат отбросить, то х = 133,5 см).

Ответ: 135 см.

4. № 257*.

Решение:

а) Среднее арифметическое увеличится:  х1 =  = x + 0,5.

б) Размах увеличится:  A1 = (xmax + 6) – xmin = (xmaxxmin) + 6 = A = 6.

в) Мода не изменится.

г) Медиана не изменится, так как в упорядоченном ряду, соответствующем исходному, величина и порядок членов не изменится, кроме величины последнего члена, что не влияет на величину медианы.

Ответ: а) увеличится на 0,5; б) увеличится на 6; в) нет; г) нет.

5. Владелец одного частного предприятия уволил бльшую часть рабочих, а оставшимся снизил зарплату на 20 %. После этого он заявил, что средний заработок его рабочих повысился. Так ли это?

 

Заработок
до увольнения

Заработок
после увольнения

1000 р.

400 р.

800 р.

320 р.

Число рабочих

200

800

200

120

Решение:

Вычисляем средние статистические характеристики:

мода до увольнения Мо = 400;

мода после увольнения Мо = 800;

медиана до увольнения Ме = 400;

медиана после увольнения Ме = 800;

среднее арифметическое

              до увольнения X =  = 520;

              после увольнения X =  = 620.

Вычисления подтверждают, что средние характеристики действительно увеличились. Однако простой взгляд на таблицу подтверждает, что жизнь рабочих не улучшилась, а, наоборот, ухудшилась! Не говоря уже о тех, кто потерял работу. Здесь итоги решения математической задачи противоречат здравому смыслу. Математическая модель не всегда адекватна практической ситуации. В данном случае средние характеристики не являются типичными представителями статистических данных, поэтому их использование приводит к ложному выводу.

На примере этой задачи показываем учащимся, что необходимо не только формально вычислять средние характеристики, но и уметь правильно истолковывать статистическую информацию.

IV. Итоги урока.

– Какие средние статистические характеристики вы узнали?

– Как вычисляются размах ряда? мода? Что они характеризуют?

– Как вычисляется среднее арифметическое ряда? Для чего служит эта характеристика?

– Как вычисляется медиана упорядоченного ряда с четным числом членов? нечетным? Что она характеризует?

Домашнее задание:

1. Найдите размах, моду и медиану ряда:

а) 1; 3; –2; 4; –2; 0; 2; 3; 1; –2; 4;

б) 0,2; 0,4; 0,1; 0,5; 0,1; 0,2; 0,3; 0,5; 0,4; 0,6.

2. В вашем (или соседнем) классе соберите данные о месяцах рождения учеников. Месяцы удобнее перечислять не по названиям, а по номерам.

Найдите: а) размах; б) моду; в) среднее арифметическое для экспериментальной выборки.

3. Для упорядоченного ряда, содержащего т чисел, где т – четное число, укажите номера двух последовательных членов, между которыми заключена медиана, если т равно:

а) 6;       б) 18;          в) 56;          г) 240.

 

 

 

 

 

Урок 22

Обобщение материала по теме
«Уравнение с одной переменной»

Цели: обобщить и систематизировать материал по теме «Уравнение с одной переменной», подготовиться к написанию контрольной работы.

Ход урока

I. Устная работа.

Найдите корень уравнения.

а) –2х = –14;                      б) –2x = ;                    в) 48х = –16;

г) –3 = x;                         д) –х = –5,6;                  е) –25х = –1;

ж) 2(х – 3) = 2х – 6;            з) 4х = 4(х + 2);              и) .

II. Обобщение и систематизация изученного материала.

Во время выполнения устной работы учащиеся должны вспомнить, как решается линейное уравнение, сколько оно может иметь корней и от чего это зависит.

Затем следует вспомнить алгоритм решения уравнений, сводящихся к линейным.

Третий блок для повторения – решение текстовых задач с помощью уравнений. Напоминаем, что в этом случае линейное уравнение и числовые  данные  задачи  выступают  в  качестве  математической  модели  реального процесса. Математическое моделирование состоит из следующих этапов:

1. анализ условия задачи.

2. поиск способа решения задачи и составление плана решения.

3. осуществление найденного плана.

4. изучение (анализ) найденного решения.

III. Практикум по решению задач.

Подготовку к контрольной работе целесообразно организовать в виде практикума по решению задач. Следует предусмотреть наличие как заданий обязательного уровня, так и повышенной трудности для сильных учащихся.

1. Ответьте устно на вопрос, равносильны ли уравнения и почему:

а) 2х + 3 = 0                       и       2х = –3;

б) 3х – 7 = 4х – 3                и       0 = (4х – 3) – (3х – 7);

в) –3х – 7 = 0                      и       3х + 7 = 0;

г) –2х + 3 = 0                     и       2х + 3 = 0;

д) 3х – 7 + 2х – 3 = х           и       4х – 10 = 0?

2. Выполните задание самостоятельно по вариантам.

Вариант 1

Решите уравнение.

а) (7x + 1) – (6x + 3) = 5;              б) (8x + 11) – 13 = 9x – 5;

в) 2 = (3x – 5) – (7 – 4x);              г) 8x + 5 = 119 + (7 – 3x).

Вариант 2

а) (6x + 1) – (3 – 2x) = 14;            б) (6 – 2x) + 4 = –5x – 3;

в) 12 = (7x – 9) – (11 – x);            г) 11x + 103 = 1 + (12x – 31).

Решение заданий по вариантам

Вариант 1

а) (7x + 1) – (6x + 3) = 5;

    7x + 1 – 6x – 3 = 5;

    7x – 6x = 5 – 1 + 3;

    х = 7.

б) (8x + 11) – 13 = 9x – 5;

    8x + 11 – 13 = 9x – 5;

    8x – 9x = –5 – 11 + 13;

    –х = –3;

    х = 3.

в) 2 = (3x – 5) – (7 – 4x);

    2 = 3x – 5 – 7 + 4x;

    –3x – 4x = –5 – 7 – 2;

    –7х = –14;

    х = 2.

г) 8x + 5 = 119 + (7 – 3x);

    8x + 5 = 119 + 7 – 3x;

    8x + 3x = 119 + 7 – 5;

    11х = 121;

    х = 11.

Вариант 2

а) (6x + 1) – (3 – 2x) = 14;

    6x + 1 – 3 + 2x = 14;

    6x + 2x = 14 – 1 + 3;

    8х = 16;

    х = 2.

б) (6 – 2x) + 4 = –5x – 3;

    6 – 2x + 4 = –5x – 3;

    –2x + 5x = –3 – 6 – 4;

    3х = –13;

    х = –4.

в) 12 = (7x – 9) – (11 – x);

    12 = 7x – 9 – 11 + x;

    –7x x = –9 – 11 – 12;

    –8х = –32;

    х = 4.

г) 11x + 103 = 1 + (12x – 31);

    11x + 103 = 1 + 12x – 31;

    11x – 12x = 1 – 31 – 103;

    –х = –133;

    х = 133.

3. Решите уравнение.

а) (10x – 3) + (14x – 4) = 8 – (15 – 22x);

б) (2x + 3) – (5x + 11) = 7 + (13 – 2x);

в) (7 – 10x) – (8 – 8x) + (10x + 6) = –8;

г) (2x + 3) + (3x + 4) + (5x + 5) = 12 – 7x.

Решение:

а) (10x – 3) + (14x – 4) = 8 – (15 – 22x);

    10x – 3 + 14x – 4 = 8 – 15 + 22x;

    10x + 14x – 22x = 8 – 15 + 3 + 4;

    2х = 0;

    х = 0.

б) (2x + 3) – (5x + 11) = 7 + (13 – 2x);

    2x + 3 – 5x – 11 = 7 + 13 – 2x;

    2x – 5x + 2x = 7 + 13 – 3 + 11;

    –х = 28;

    х = –28.

в) (7 – 10x) – (8 – 8x) + (10x + 6) = –8;

    7 – 10x – 8 + 8x + 10x + 6 = –8;

    –10x + 8x + 10x = –8 – 7 + 8 – 6;

    8х = –13;

    х = –1,625.

г) (2x + 3) + (3x + 4) + (5x + 5) = 12 – 7x;

    2x + 3 + 3x + 4 + 5x + 5 = 12 – 7x;

    2x + 3x + 5x + 7x = 12 – 3 – 4 – 5;

    17х = 0;

    х = 0.

Ответ: а) 0; б) –28; в) –1,625; г) 0.

4. Масса ящика с яблоками 22 кг и еще половина его массы. Какова масса ящика с яблоками?

5. Моторная лодка развивает скорость в стоячей воде 15 км/ч. Рыбак проплыл на ней против течения реки 30 ч, а затем вернулся на прежнее место за 20 ч. Какова скорость течения реки?

Решение:

Анализ условия:

 

х (км/ч)

t (ч)

s (км)

По течению

15 + х

20

20(15 + х)

Против течения

15 – х

30

30(15 – х)

Пусть х км/ч – скорость течения реки, тогда (15 + х) км/ч – скорость лодки по течению и (15 – х) км/ч – против течения. По течению лодка проплыла 20(15 + х) км, а против течения – 30(15 – х) км. Зная, что по течению и против него лодка проплыла одинаковые расстояния, составим уравнение:

20(15 + х) = 30(15 – х);

20 · 15 + 20 · х = 30 · 15 – 30 · х;

300 + 20х = 450 – 30х;

20х + 30х= 450 – 300;

50х = 150;

х = 3.

Значит, скорость течения реки равна 3 км/ч.

Ответ: 3 км/ч.

6. Фермер планировал засевать в день по 9 га поля. Применив новую технику, он каждый день засевал на 3 га больше, и за 3 дня до намеченного срока осталось засеять 9 га. Какова площадь поля?

Решение:

Анализ условия:

 

Производительность (га/день)

Сроки (день)

Площадь (га)

По плану

9

х

Фактически

9 + 3 = 12

х – 9

Пусть х га – площадь поля, тогда намеченный срок был  дней. Так как фактически фермер засевал на 3 га/день больше, то есть 12 га/день, и осталось ему засеять 9 га (то есть он засеял (х – 9) га), то он работал  дней. Зная, что срок работы был на 3 дня меньше запланированного, составим уравнение:

 + 3 = ;      | · 36

3(х – 9) + 36 · 3 = 4 · х;

3х – 27 + 108 = 4х;

3х – 4х = 27 – 108;

х = –81;

х = 81.

Значит, площадь поля равна 81 га.

Ответ: 81 га.

IV. Итоги урока.

– Какое уравнение называется линейным? Сколько решений оно может иметь?

– Какие уравнения называются равносильными? Приведите пример.

– Назовите алгоритм решения уравнений, сводящихся к линейным.

– Назовите основные этапы решения текстовых задач алгебраическим методом.

Домашнее задание: повторить п. 6–8.

1. Решите уравнение.

а) 0,71x – 13 = 10 – 0,29x;                     б) 8c + 0,73 = 4,61 – 8c;

в) 48 = 11 – (9a + 2);                             г) 13 – (5x + 11) = 6x.

2. № 240 (а; в).

3. № 249; № 251.

 

 

 

 

 

Урок 23
Контрольная работа № 2

Вариант 1

1. Решите уравнение.

а) x = 12;                         в) 5x – 4,5 = 3x + 2,5;

б) 6x – 10,2 = 0;                  г) 2x – (6x – 5) = 45.

2. Таня в школу сначала едет на автобусе, а потом идет пешком. Вся дорога у неё занимает 26 мин. Идет она на 6 мин дольше, чем едет на автобусе. Сколько минут она едет на автобусе?

3. В двух сараях сложено сено, причем в первом сарае сена в 3 раза больше, чем во втором. После того как из первого сарая увезли 20 т сена, а во второй привезли 10 т, в обоих сараях сена стало поровну. Сколько всего тонн сена было в двух сараях первоначально?

4. Решите уравнение 7x – (x + 3) = 3(2x – 1).

 

Вариант 2

1. Решите уравнение.

а) x = 18;                         в) 6x – 0,8 = 3x + 2,2;

б) 7x + 11,9 = 0;                 г) 5x – (7x + 7) = 9.

2. Часть пути в 600 км турист пролетел на самолете, а часть проехал на автобусе. На самолете он проделал путь в 9 раз больший, чем на автобусе. Сколько километров турист проехал на автобусе?

3. На одном участке было в 5 раз больше саженцев смородины, чем на другом. После того как с первого участка увезли 50 саженцев, а на второй посадили еще 90, на обоих участках саженцев стало поровну. Сколько всего саженцев было на двух участках первоначально?

4. Решите уравнение 6x – (2x – 5) = 2(2x + 4).

 

 

 

 

 

 

 

 

Рекомендации по оцениванию контрольной работы.

Для получения отметки «3» достаточно выполнить первые два задания (обязательный уровень). Для получения отметки «4» достаточно выполнить любые три задания, для отметки «5» – все четыре задания.

Решения заданий контрольной работы

Вариант 1

1. а) x = 12;   | · 3                                     б) 6x – 10,2 = 0;

        х = 12 · 3;                                                               6х = 10,2;   | : 6

        х = 36.                                                     х = 1,7.

    в) 5x – 4,5 = 3x + 2,5;                           г) 2x – (6x – 5) = 45;

        5x – 3x = 2,5 + 4,5;                               2x – 6x + 5 = 45;

        2х = 7;                                                     2x – 6x = 45 – 5;

        х = 3,5.                                                    –4х = 40;

                                                                                          х = –10.

Ответ: а) 36; б) 1,7; в) 3,5; г) –10.

2. Анализ условия:

Пусть Таня едет на автобусе х мин, тогда пешком она идет (х + 6) мин. Зная, что вся дорога занимает 26 минут, составим уравнение:

х + (х + 6) = 26;

х + х + 6 = 26;

х + х = 26 – 6;

2х = 20;

х = 10.

Значит, на автобусе Таня едет 10 минут.

Ответ: 10 мин.

3. Анализ условия:

Пусть  во  втором  сарае  было х т сена, тогда в первом сарае было 3х т сена.  После  того  как  из  первого  сарая  вывезли  20 т  сена,  там  осталось (3х – 20) т сена, а после того как во второй сарай довезли 10 т сена, там стало (х + 10) т. Зная, что после этого сена в обоих сараях стало поровну, составим уравнение:

3х – 20 = х + 10;

3х х = 10 + 20;

2х = 30;

х = 15.

Значит, во втором сарае первоначально было 15 т сена.

Так как 3х = 3 · 15 = 45, то в первом сарае было 45 т сена.

Следовательно, всего в двух сараях первоначально было 15 + 45, то есть 60 т сена.

Ответ: 60 т.

4. 7x – (x + 3) = 3(2x – 1);

    7xx – 3 = 6x – 3;

    7xx – 6x = –3 + 3;

    0 · х = 0;

    х – любое число.

Ответ: х – любое число.

Вариант 2

1. а) х = 18;   | · 6                                 б) 7x + 11,9 = 0;

        х = 18 · 6;                                                               7х = –11,9;

        х = 108.                                                   х = (–11,9) : 7;

                                                                                          х = –1,7.

   в) 6x – 0,8 = 3x + 2,2;                            г) 5x – (7x + 7) = 9;

       6x – 3x = 2,2 + 0,8;                                5x – 7x – 7 = 9;

       3х = 3;                                                      5x – 7x = 9 + 7;

       х = 1.                                                         –2х = 16;

                                                                                          х = –8.

Ответ: а) 108; б) –1,7; в) 1; г) –8.

2. Анализ условия:

Пусть  турист  проехал  на  автобусе  х км,  тогда  на  самолете  он  пролетел 9х км.

Зная, что весь путь составил 600 км, составим уравнение:

х + 9х = 600;

10х = 600;

х = 60.

Значит, на автобусе турист проехал 60 км.

Ответ: 60 км.

3. Анализ условия:

Пусть на втором участке было х саженцев смородины, тогда на первом было 5х саженцев. После того как с первого участка увезли 50 саженцев, там осталось (5х – 50) саженцев смородины, а после того как на второй участок посадили еще 90, там стало (х + 90) саженцев смородины. Зная, что после этого на обоих участках стало поровну саженцев смородины, составим уравнение:

5х – 50 = х + 90;

5х х = 90 + 50;

4х = 140;

х = 35.

Значит, на втором участке первоначально было 35 кустов смородины.

Так как 5 · 35 = 175, то на первом участке было 175 кустов смородины.

Следовательно, всего на двух участках первоначально было 35 + 175, то есть 210 саженцев смородины.

Ответ: 210.

4. 6x – (2x – 5) = 2(2x + 4);

    6x – 2x + 5 = 4x + 8;

    6x – 2x – 4x = 8 – 5;

    0 · х = –3;                                нет корней.

Ответ: нет корней.

 

 

 

 

 

 

Вариант 3

1. Решите уравнение.

а) x = 5;                           в) 4x + 5,5 = 2x – 2,5;

б) 3x – 11,4 = 0;                  г) 2x – (6x + 1) = 9.

2. Саша решил две задачи за 35 минут. Первую задачу он решал на 7 мин дольше, чем вторую. Сколько минут Саша решал вторую задачу?

3. В первом мешке в 3 раза больше картофеля, чем во втором. После того как из первого мешка взяли 30 кг картофеля, а во второй насыпали еще 10 кг, в обоих мешках картофеля стало поровну. Сколько килограммов картофеля было в двух мешках первоначально?

4. Решите уравнение 8x – (2x + 4) = 2(3x – 2).

Вариант 4

1. Решите уравнение.

а) x = 8;                           в) 3x – 0,6 = x + 4,4;

б) 5x – 12,5 = 0;                  г) 4x – (7x – 2) = 17.

2. Длина отрезка АС равна 60 см. Точка В взята на отрезке АС так, что длина отрезка АВ в 4 раза больше длины отрезка ВС. Найдите длину отрезка ВС.

3. В первом контейнере в 5 раз больше моркови, чем во втором. Когда из первого контейнера взяли 25 кг моркови, а во второй засыпали еще 15 кг, то в обоих контейнерах моркови стало поровну. Сколько килограммов моркови было в двух контейнерах первоначально?

4. Решите уравнение 3x – (9x – 3) = 3(4 – 2x).

Рекомендации по оцениванию контрольной работы.

Для получения отметки «3» достаточно выполнить первые два задания (обязательный уровень). Для получения отметки «4» достаточно выполнить любые три задания, для отметки «5» – все четыре задания.

Решения заданий контрольной работы

Вариант 1

1. а) x = 12;   | · 3                      б) 6x – 10,2 = 0;

        х = 12 · 3;                                 6х = 10,2;   | : 6

        х = 36.                                      х = 1,7.

    в) 5x – 4,5 = 3x + 2,5;               г) 2x – (6x – 5) = 45;

        5x – 3x = 2,5 + 4,5;                   2x – 6x + 5 = 45;

        2х = 7;                                      2x – 6x = 45 – 5;

        х = 3,5.                                     –4х = 40;

                                                        х = –10.

Ответ: а) 36; б) 1,7; в) 3,5; г) –10.

2. Анализ условия:

Пусть Таня едет на автобусе х мин, тогда пешком она идет (х + 6) мин. Зная, что вся дорога занимает 26 минут, составим уравнение:

х + (х + 6) = 26;

х + х + 6 = 26;

х + х = 26 – 6;

2х = 20;

х = 10.

Значит, на автобусе Таня едет 10 минут.

Ответ: 10 мин.

3. Анализ условия:

Пусть  во  втором  сарае  было х т сена, тогда в первом сарае было 3х т сена.  После  того  как  из  первого  сарая  вывезли  20 т  сена,  там  осталось (3х – 20) т сена, а после того как во второй сарай довезли 10 т сена, там стало (х + 10) т. Зная, что после этого сена в обоих сараях стало поровну, составим уравнение:

3х – 20 = х + 10;

3х х = 10 + 20;

2х = 30;

х = 15.

Значит, во втором сарае первоначально было 15 т сена.

Так как 3х = 3 · 15 = 45, то в первом сарае было 45 т сена.

Следовательно, всего в двух сараях первоначально было 15 + 45, то есть 60 т сена.

Ответ: 60 т.

4. 7x – (x + 3) = 3(2x – 1);

    7xx – 3 = 6x – 3;

    7xx – 6x = –3 + 3;

    0 · х = 0;

    х – любое число.

Ответ: х – любое число.

Вариант 2

1. а) х = 18;   | · 6                      б) 7x + 11,9 = 0;

        х = 18 · 6;                                 7х = –11,9;

        х = 108.                                    х = (–11,9) : 7;

                                                        х = –1,7.

   в) 6x – 0,8 = 3x + 2,2;                г) 5x – (7x + 7) = 9;

       6x – 3x = 2,2 + 0,8;                    5x – 7x – 7 = 9;

       3х = 3;                                       5x – 7x = 9 + 7;

       х = 1.                                         –2х = 16;

                                                        х = –8.

Ответ: а) 108; б) –1,7; в) 1; г) –8.

2. Анализ условия:

Пусть  турист  проехал  на  автобусе  х км,  тогда  на  самолете  он  пролетел 9х км.

Зная, что весь путь составил 600 км, составим уравнение:

х + 9х = 600;

10х = 600;

х = 60.

Значит, на автобусе турист проехал 60 км.

Ответ: 60 км.

3. Анализ условия:

Пусть на втором участке было х саженцев смородины, тогда на первом было 5х саженцев. После того как с первого участка увезли 50 саженцев, там осталось (5х – 50) саженцев смородины, а после того как на второй участок посадили еще 90, там стало (х + 90) саженцев смородины. Зная, что после этого на обоих участках стало поровну саженцев смородины, составим уравнение:

5х – 50 = х + 90;

5х х = 90 + 50;

4х = 140;

х = 35.

Значит, на втором участке первоначально было 35 кустов смородины.

Так как 5 · 35 = 175, то на первом участке было 175 кустов смородины.

Следовательно, всего на двух участках первоначально было 35 + 175, то есть 210 саженцев смородины.

Ответ: 210.

4. 6x – (2x – 5) = 2(2x + 4);

    6x – 2x + 5 = 4x + 8;

    6x – 2x – 4x = 8 – 5;

    0 · х = –3;              нет корней.

Ответ: нет корней.

Вариант 3

1. а) x = 5;   | · 5                        б) 3x – 11,4 = 0;

        х = 25.                                      3х = 11,4;

                                                        х = 11,4 : 3;

                                                        х = 3,8.

   в) 4x + 5,5 = 2x – 2,5;                г) 2x – (6x + 1) = 9;

       4x – 2x = – 2,5 – 5,5;                 2x – 6x – 1 = 9;

       2х = –8;                                     2x – 6x = 9 + 1;

       х = (–8) : 2;                                –4х = 10;

       х = –4.                                       х = 10 : (–4);

                                                        х = –2,5.

Ответ: а) 25; б) 3,8; в) –4; г) –2,5.

2. Анализ условия:

Пусть х минут Саша решал вторую задачу, тогда первую задачу он решал (х + 7) минут. Зная, что две задачи Саша решил за 35 минут, составим уравнение:

х + (х + 7) = 35;

х + х + 7 = 35;

2х = 35 – 7;

2х = 28;

х = 28 : 2;

х = 14.

Значит, вторую задачу Саша решил за 14 минут.

Ответ: 14 минут.

3. Анализ условия:

Пусть во втором мешке было х кг картофеля, тогда в первом мешке было 3х кг картофеля. После того как из первого мешка взяли 30 кг картофеля, в нем осталось (3х – 30) кг, а после того как во второй мешок насыпали еще 10 кг, в нем стало (х + 10) кг картофеля. Зная, что после этого в обоих мешках стало поровну картофеля, составим уравнение:

3х – 30 = х + 10;

3х х = 10 + 30;

2х = 40;

х = 40 : 2;

х = 20.

Значит, во втором мешке было 20 кг картофеля.

Так как 3х = 3 · 20 = 60, значит, в первом мешке было 60 кг картофеля.

Следовательно,  всего  в  двух  мешках  было  20 + 60,  то  есть  80 кг картофеля.

Ответ: 80 кг.

4. 8x – (2x + 4) = 2(3x – 2);

    8x – 2x – 4 = 6x – 4;

    8x – 2x – 6x = –4 + 4;

    0 · х = 0;

    х – любое число.

Ответ: х – любое число.

Вариант 4

1. а) x = 8;   | · 4                        б) 5x – 12,5 = 0;

        х = 8 · 4;                                   5х = 12,5;

        х = 32.                                      х = 12,5 : 5;

                                                        х = 2,5.

     в) 3x – 0,6 = x + 4,4;                г) 4x – (7x – 2) = 17;

        3xx = 4,4 + 0,6;                     4x – 7x + 2 = 17;

        2х = 5;                                      –3х = 17 – 2;

        х = 5 : 2;                                   –3х = 15;

        х = 2,5.                                     х = 15 : (–3);

                                                        х = –5.

Ответ: а) 32; б) 2,5; в) 2,5; г) –5.

2. Анализ условия:

Пусть х см – длина отрезка ВС, тогда 4х см – длина отрезка АВ, зная, что сумма отрезков АВ и ВС равна длине отрезка АС, то есть 60 см, составим уравнение:

х + 4х = 60;

5х = 60;

х = 60 : 5;

х = 12.

Значит, длина отрезка ВС равна 12 см.

Ответ: 12 см.

3. Анализ условия:

Пусть х кг моркови было во втором контейнере, тогда в первом было 5х кг моркови. После того как из первого контейнера взяли 25 кг, в нем осталось (5х – 25) кг моркови, а во втором, после того как в него засыпали еще 15 кг моркови, стало (х + 15) кг моркови. Зная, что после этого в обоих контейнерах стало поровну моркови, составим уравнение:

5х – 25 = х + 15;

5х х = 15 + 25;

4х = 40;

х = 40 : 4;

х = 10.

Значит, во втором контейнере было 10 кг моркови.

Так  как  5х = 5 · 10 = 50,  значит,  в  первом  контейнере  было  50 кг моркови.

Следовательно, всего в двух контейнерах было 10 + 50, то есть 60 кг моркови.

Ответ: 60 кг.

4. 3x – (9x – 3) = 3(4 – 2x);

    3x – 9x + 3 = 12 – 6x;

    3x – 9x + 6x = 12 – 3;

    0 · х = 9;                нет корней.

Ответ: нет корней.

 

 

Урок 26
Анализ контрольной работы.
Обобщение материала по теме
«Уравнения с одной переменной»

Цели: проанализировать результаты контрольной работы, выявить типичные ошибки, допускаемые учащимися, выполнить работу над ошибками; обобщить изученный материал, прорешать задания повышенной сложности.

Ход урока

I. Анализ результатов контрольной работы.

При анализе обратить особое внимание на решение текстовых задач, моделирование условия в виде блок-схем, таблиц, чертежей. Выполнить работу над ошибками.

II. Обобщение  и  систематизация  материала,  углубление  знаний по теме.

1. Актуализация знаний.

Проводится по карточкам с последующим взаимоконтролем.

Карточка 1

1. Определение уравнения.

2. Записать в виде равенства:

а) сумма выражений 2х + 7 и –х + 12 равна 4;

б) число 56 в 8 раз больше числа 7;

в) произведение выражений 2х и 3х + 5 равно 0.

Карточка 2

1. Назвать элементы уравнения.

2. Заполнить таблицу:

Уравнение

Левая часть

Правая часть

Члены уравнений

а) х2 – 2х + 3 = 7

 

 

 

б)

7х – 8

3 – 5х

 

Карточка 3

1. Что называется корнем уравнения?

2. Какие из чисел 3; –2 являются корнем уравнений:

3х = –6;      | x | = 3;      5х – 8 = 2х + 4?

Карточка 4

1. Что значит «решить уравнение»? Сколько корней может иметь уравнение?

2. При каких значениях х уравнения 2х – 1 = 0; (х – 5)(х + 4) = 0; | x | = 5 обращаются в верное равенство?

Карточка 5

1. Общий вид линейного уравнения с одной переменной. Чем отличается линейное уравнение от других уравнений?

2. Назовите линейные уравнения:

х(х – 2) = 0;      ;      | x | = 10;      5.

Карточка 6

1. Обосновать  способ  решения  линейного  уравнения  с  одной  переменной  с  помощью  свойств  числовых  равенств  на  примере  уравнения 10 – х = 3х – 5.

2. Решить уравнение  5х – 3 = 7 + 3х.

Карточка 7

1. В каком случае уравнение ax = b имеет единственный корень?

2. Решить уравнения  9x = ;      –0,7х = 49.

Карточка 8

1. В каком случае уравнение ax = b не имеет корней?

2. Решить уравнение  2х + 1 = 2 + 2х.

Карточка 9

1. В  каком  случае  уравнение  ax = b  имеет  бесконечное  множество корней?

2. Решить уравнение  3 – 3х + 2 = 5 – 3х.

Карточка 10

1. В каком случае уравнение ax = b имеет корень, равный 0?

2. Решить уравнение  10х = 0;     –3х + 8 = 3х + 8.

Карточка 11

1. Как проверить: верно ли решено уравнение?

2. Верно ли решено уравнение?

4х + 4 = х + 5;

4хх = 5 – 4;

3х = 1;

х = 1 : 3;

х = .

Ответ: х = .

2. Решение заданий повышенной сложности.

№ 1184*; № 1185* (а, в); № 1187*; № 1193*.

№ 1184.

Решение:

(a – 1) x = 12;   | : (а – 1) ¹ 0

x = .

Чтобы х было натуральным числом, 12 должно нацело делится на а – 1. Возможны следующие варианты:

а – 1 = 1;                   а = 1 + 1;                  а = 2;

а – 1 = 2;                   а = 2 + 1;                  а = 3;

а – 1 = 3;                   а = 3 + 1;                  а = 4;

а – 1 = 4;                   а = 4 + 1;                  а = 5;

а – 1 = 6;                   а = 6 + 1;                  а = 7;

а – 1 = 12;                 а = 12 + 1;                а = 13.

Ответ: 2; 3; 4; 5; 7; 13.

№ 1185.

Решение:

а) | x – 3 | = 7, значит,        х – 3 = 7               или         х – 3 = –7;

                                          х = 7 + 3;                             х = –7 + 3;

                                          х = 10;                                 х = –4.

в) | 4 – x | = 1,5, значит,     4 – х = 1,5;           или         4 – х = –1,5;

                                          –х = 1,5 – 4;                         –х = –1,5 – 4;

                                          –х = –2,5;                            –х = –5,5;

                                          х = 2,5;                                х = 5,5.

Ответ: а) 10; –4; в) 2,5; 5,5.

№ 1187.

Решение:

Анализ условия:

Пусть в обеих бочках было по а литров воды. В первой бочке сперва уменьшили на 10 %, то есть воды стало 0,9а, а затем увеличили на 10 %, то есть воды стало 1,1 · (0,9а), то есть 0,99а.

Во второй бочке сперва увеличили на 10 %, то есть воды стало 1,1а, а затем уменьшили на 10 %, то есть воды стало 0,9 · (1,1а), то есть 0,99а. Значит, в бочках воды осталось поровну.

Ответ: поровну.

№ 1193.

Решение:

Пусть число записано цифрами , где х – число десятков, у – число единиц. Значит, = 10x + y. Сумма цифр равна х + у. Зная, что число в 4 раза больше суммы цифр, составим уравнение:

10х + у = 4(х + у);

10х + у = 4х + 4у;

10х – 4х = 4уу;

6х = 3у;

2х = у.

Так как х и у могут принимать значения от 0 до 9 (причем х ¹ 0), то возможны варианты:

х

у

1

2

3

4

2

4

6

8

Значит, число может быть 12, 24, 36 и 48.

Ответ: 12; 24; 36; 48.

Домашнее задание: № 1185* (б, г); № 1188*; № 243 (а, б); № 244.

 

Урок 24

Формулы

Цели: формировать умения вычислять значение переменной по формуле, выражать из формулы одну переменную через другую, применять формулы при решении практических задач.

Ход урока

I. Актуализация знаний.

1. Выразите в метрах:

а) 14 см;                    б) 12,8 км;                     в) 13 дм;

г) 330 мм;                 д) 0,32 км;                     е) 0,03 дм.

2. Выразите в м2:

а) 16 см2;                  б) 0,35 дм2;                    в) 1,2 км2;

г) 0,03 а;                   д) 13 га;                         е) 283 дм2.

II. Изучение нового материала.

1. Актуализация знаний.

Напоминаем учащимся, что для записи формул используются выражения с переменными. Например, т = 2п – формула четного числа и т. п.

2. Объяснение проводим согласно пункту 11 учебника, рассматривая примеры из художественной литературы, практической деятельности человека.

3. Первичное закрепление. Просим записать формулу перевода в  метры  длины,  выраженной  в  сантиметрах  (миллиметрах,  километрах и т. п.).

М = 0,01 · а,         где М – длина в метрах,

                              а – длина в сантиметрах;

М = 0,001 · b,       где b – длина в миллиметрах;

М = 1000 · с,        где с – длина в километрах и т. д.

III. Закрепление изученного материала.

Все задания данного пункта можно условно разбить на три группы:

1-я группа. Вычисление значения переменной по формуле.

2-я группа. Выражение  одной  переменной  через  другую  из  формулы.

3-я группа. Решение практических задач с использованием формул.

1-я группа

№ 196, № 197, № 198.

№ 197.

Решение:

Если т = 3, то р = 16,38 · 3 = 49,14;

если т = 20,5, то р = 16,38 · 20,5 = 335,79.

Ответ: 49,14 кг; 335,79 кг.

№ 198.

Решение:

а) Если f = 8, то с = 1,409 · 8 = 11,272.

б) Если f = 30,5, то с = 1,409 · 30,5 » 42,97.

Ответ: 11,272 кг; б) » 42,97 кг.

2-я группа

1. № 205.

Решение:

а) s = at;   | : а           б) х = х0 + at;                в) S =   h;

   = t;                         х х0 = at;    | : t             2S = (a + b) ∙  h;

    t = .                       = a;                       = a + b;

                                     a = .                     – a = b;

                                                                            b =  – a.

2. Выразите из формулы:

а) y = 3x – 15 переменную х;

б) y = ax + 2bx переменную х;

в) y =  переменную х.

Решение:

а) y = 3x – 15;                              б) y = ax + 2bx;

    y + 15 = 3x;                                  y = x(a + 2b);

    = x;                                   = x;

    x = .                                  x = .

в) y = ;

    y = x – 6ax;

    y = x – 6ax;

    y = x;

    y = x (0,2 – 6a);

    = x;

    x = .

Ответ: а) x = ;  б) x = ;  в)  x = .

3-я группа

1. № 199, № 200.

№ 199.

Решение:

Пусть длина прямоугольника равна а, ширина – b, а площадь – s.

По формуле площади прямоугольника находим s = ab.

а) После  уменьшения  длины  и  ширины  на  10 %  длина  будет  равна а – 0,1а = 0,9а, а ширина b – 0,1b = 0,9b. Тогда площадь будет равна

0,9а · 0,9b = 0,81аb, то есть уменьшится на ab – 0,81ab = 0,19ab.

Имеем:  · 100 % = 19 %. Значит, площадь уменьшится на 19 %.

б) После увеличения длина будет равна а + 0,3а = 1,3а, ширина после уменьшения будет равна b – 0,3b = 0,7b. Тогда площадь будет равна

1,3a · 0,7b = 0,91ab, то есть уменьшится на ab – 0,91ab = 0,09 · b.

Имеем:  · 100 % = 9 %. Значит, площадь уменьшится на 9 %.

Ответ: а) уменьшится на 19 %; б) уменьшится на 9 %.

2. После повышения цен (с) на обувь в 2,5 раза приняли решение о 30-процентном снижении цен на детскую обувь. Составьте формулу расчета новых цен на детскую обувь.

Решение:

После  повышения  цена  составила  2,5с,  а  после  снижения – 2,5с
– 0,3 · (2,5с) = 2,5с – 0,75с = 1,75с.

Новая  формула  с1 = 1,75с,  где  с – первоначальная  стоимость  обуви, а с1 – новая.

3. № 201, № 202.

№ 201.

Решение:

После повышения цена стала равна а + 0,15а = 1,15а (р.). После снижения цена составляла 1,15а – 0,15 · (1,15а) = 1,15а – 0,1725а = 0,9775а.

То есть окончательная цена b » 0,98а, значит, b < a.

Ответ: 1.

№ 202.

Решение:Пусть х р. – исходная цена костюма, тогда после снижения цены она стала  равна  х – 0,2х = 0,8х.  Чтобы  вернуться  к  первоначальной  цене, её надо увеличить на 0,2х. Вычислим, сколько процентов составляет 0,2х от 0,8х:

 · 100 % = 25 %. Значит, увеличить цену надо на 25 %.

Ответ: на 25 %.

4. № 203, № 204.

№ 203.Решение:Формула  позволит переводить температуру, выраженную в градусах Фаренгейта, в градусы Цельсия. Чтобы проводить обратную операцию, выразим из формулы f :

c = ;     | · 9

9c = 5 (f – 32);

9c = 5 f – 160;

9c + 160 = 5 f ;     | : 5

 = f .                    

а) Если с = 4°, то f =  = 39,2 °F;

    если с = –15°, то f =  = 5 °F;

    если с = 0°, то f =  = 32 °F;

б) Если f = 20°, то c =  °C;

    если f = –16°, то c =  °C;

    если f = 0°, то c =  °C.

Ответ: а) 39,2 ° F; 5 °F; 32 °F; б) –6 °C; –26 °C; –17 °C.

№ 204.Решение:а) не может, так как если f < 0, то f – 32 < 0 и  < 0;

б) может, так как если 0 < f < 32, то f – 32 < 0 и  < 0.

Ответ: а) не может; б) может.

IV. Итоги урока.

 

Урок
Определение степени
с натуральным показателем

Цели: ввести понятие степени числа а с натуральным показателем п; определить значение степени с натуральным показателем положительного и отрицательного числа в зависимости от четности / нечетности показателя степени; формировать умение вычислять значение степени и представлять число в виде степени с натуральным показателем.

Ход урока

I. Устная работа.

Вычислите.

а) 3 · 45;                          б)  · 120;                        в) ;

г) ;                       д)  · 49;                      е) –3 · (–16);

ж) –(–3) · 12;                   з) –(2 · (–9));                    и) ;

к) 18 ·  + 11;          л)  · (11 – 6);           м) .

II. Объяснение нового материала.

1. Объяснение проводить согласно пункту 18 учебника. Напоминаем, что вместо длинной записи произведения 5 · 5 · 5 · 5 · 5 · 5 · 5 можно записать выражение 57, где 5 – основание степени (повторяющийся множитель), а 7 – показатель степени (число повторяющихся множителей).

Понятие степени определяем для любого числа а в качестве основания и любого натурального показателя (аналитическая запись).

На доску выносится запись:

Степенью числа а с натуральным показателем п, большим 1, называется выражение ап, равное произведению п множителей, каждый из которых равен а. Степенью числа а с показателем 1 называется само число а.

Проговариваем  с  учащимися  правило  чтения  степени,  приводим примеры.

2. Мини-лабораторная работа.

Найдите значение степени.

33;   34;   35;   36;   01;

;  02;

(0,1)2;   (0,1)3;   (0,1)4;   (0,1)5;   03;

(–2)2;   (–2)3;   (–2)4;   (–2)5;   04;

;  05;

(–0,1)2;   (–0,1)3;   (–0,1)4;   (–0,1)5;   06.

Задания разбиваем либо по группам, либо раздаем индивидуально. Затем «по цепочке» ученики выходят к доске и записывают результаты.

После анализа полученных результатов на доску выносятся следующие правила:

При возведении в степень положительного числа получается положительное число.

При возведении в степень нуля получается нуль.

Степень отрицательного числа с четным показателем – положительное число.

Степень отрицательного числа с нечетным показателем – отрицательное число.

Обособленно выносим правило для квадратов чисел (пропедевтика изучения решения квадратных уравнений):

Квадрат любого числа есть положительное число либо нуль (а2 ≥ 0 при любом а).

3. Рассматриваем примеры 1–3 со с. 88–89 учебника.

III. Формирование умений и навыков.

Упражнения,  решаемые  на  этом  уроке,  можно  условно  разбить на группы:

1-я группа. Задания на усвоение понятия степени.

2-я группа. Задания на вычисление значения степени числа с натуральным показателем.

3-я группа. Задания на вычисление значения числового выражения, содержащего степень.

1-я группа

№ 374, № 375 (устно), № 376, № 378, № 380.

При выполнении этих заданий учащиеся должны четко называть степень, можно просить назвать их основание и показатель степени.

2-я группа

1. № 382, № 381 (а, б).

2. Не  выполняя  вычислений,  сравните  значение  данного  выражения с нулем:

а) (–4,1) · (–5,6)6;               б) (–3,3)3 : (–5,7);

в) –(4,8)2 · (–1,2)4;              г) –(–2,7)4 · (–6,4)5.

3. Сравните значения выражений:

а) (–6,5)4 и (–2,4)3;

б) (–0,2)6 и (–0,2)10;

в) (–1,5)7 и (–1,5)9.

3-я группа

№ 384, 385 (а, в, г), 386 (а, в, д, ж), 387 (а, б, в).

IV. Итоги урока.

– Сформулируйте определение степени числа с натуральным показателем. Приведите примеры и назовите в каждом из них основание и показатель степени.

– Чему равна первая степень любого числа?

– Какой знак имеет результат возведения положительного числа в натуральную степень?

– Какой знак имеет значение степени отрицательного числа с четным показателем? С нечетным показателем?

– Каков порядок действий при нахождении значения выражения, содержащего степени с натуральным показателем?

Домашнее задание: № 377; 379; 381 (в, г); 383; 385 (б, г, е); 386 (б, г, е, з).

 

Урок 44
Решение задач по теме «Определение степени
с натуральным показателем»

Цели:  продолжить  формировать  умение  вычислять  значение  числового  выражения,  содержащего  степень;  формировать  умение  вычислять значение буквенного выражения, содержащего степень, и решать практические задачи с использованием понятия степени с натуральным показателем.

Ход урока

I. Математический диктант.

Вариант 1

1. Запишите в виде произведения третью степень числа 4 и найдите её числовое значение.

2. Чему равна первая степень числа –5?

3. Вычислите значение выражения 23 · 0,5.

4. Чему равна сумма кубов чисел 5 и 3?

5. Вычислите значение выражения (–3)2 + (0,1)3.

Вариант 2

1. Запишите в виде произведения четвертую степень числа 3 и найдите её числовое значение.

2. Чему равна первая степень числа ?

3. Вычислите значение выражения 32 · 0,7.

4. Чему равен квадрат разности чисел 7 и 5?

5. вычислите значение выражения (–5)3 – (0,2)2.

II. Актуализация знаний.

№ 387 (г, д, е, ж, з, и), № 388.

№ 388.

Решение:

а) –13 + (–2)3 = –1 + (–8) = –9;

б) –62 – (–1)4 = –36 – 1 = –37;

в) –83 + (–3)3 = –512 + (–27) = –539;

г) 10 – 5 · 24 = 10 – 5 · 16 = 10 – 80 = –70;

д) 2 · 34 – 3 · 24 = 2 · 81 – 3 · 16 = 162 – 48 = 114;

е) 2 · 53 + 5 · 23 = 2 · 125 + 5 · 8 = 250 + 40 = 290;

ж) 34 = 81 – 1 = 80;

з) 0,2 · 32 – 0,4 · 24 = 0,2 · 32 – 0,2 · 2 · 24 = 0,2(32 – 2 · 24) =
= 0,2(9 – 2 · 16) = 0,2 · (9 – 32) = 0,2 · (–23) = –4,6;

и) 8 · 0,53 + 25 · 0,22 = 2 3 · 0,53 + 52 · 0,22 = (2 · 0,5)3 + (5 · 0,2)2 =
= 13 + 12 = 1 + 1 = 2.

При выполнении этого упражнения учащиеся выводят правило:

an · bn = (a · b)n, для любых a и b.

III. Формирование умений и навыков.

На этом уроке отрабатывается умение вычислять значение буквенного выражения, содержащего степень.

1-й блок

1. Найдите значения выражений х2; – х2; х2 – 4 для заданных значений х и заполните таблицу (используйте найденные значения выражения х2 для вычисления значений двух других выражений):

х

–5

–2,5

0

0,3

1

12

х2

 

 

 

 

 

 

х2

 

 

 

 

 

 

х2 – 4

 

 

 

 

 

 

2. Найдите значение выражений х3; 0,1х3; х3 + 10 для заданных значений х и заполните таблицу:

х

–4

–0,3

–1

0

9

х3

 

 

 

 

 

0,1х3

 

 

 

 

 

х3 + 10

 

 

 

 

 

3. № 392 (устно).

2-й блок

1. Найдите значение выражения.

а) (ху)2 при х = 12 и у = –0,5; х = –14 и у = –1;

б)  при х = –6 и у = 1,5; х = 0 и у = –23;

в) (х + у)4 при х = 0,7 и у = 0,3; х = –11 и у = 6;

г) (ух)3 при х = –14 и у = –10; х = 0,9 и у = 1,1.

2. № 393.

3. Сравните значения выражений.

а) –а2 и (–а)2 при а = 3; –5; 0;

б) –а3 и (–а)3 при а = 10; –2; 0.

4. № 395.

Решение:

а) а3 · а = (а · а · а) · а = а4;

б) а4 · а2 = (а · а · а · а) · (а · а) = а6;

в) а3 · а6 =  = а9;

г) а20 · а12 =  = а32.

№ 396, № 397.

3-й блок

1. № 389.

2. Сколько биений сделает сердце человека за сутки, если за 1 мин оно делает в среднем 75 биений?

3. Может  ли  школьник  поднять  1 м3  пробки?  (Масса  1 см3  пробки 0,2 г.)

Решение:

Рассчитаем, сколько см3 в 1 м3:

1 м3 = 1 · 1 · 1 ( в м) = 100 · 100 · 100 (в см) = 1 000 000 = 106 см3.

Масса  1 м3  пробки  равна  0,2 · 106 (г),  что  составляет  200 000 г или 200 кг. Значит, школьник не сможет поднять такую массу.

Ответ: нет.

4. Если разрезать кубический метр на кубические сантиметры и поставить их один на другой, то какой высоты получится столб?

При решении этой задачи следует использовать результаты предыдущей задачи.

IV. Итоги урока.

– Сформулируйте определение степени с натуральным показателем.

– Чему равна любая натуральная степень нуля?

– Каков порядок действий при нахождении числового и буквенного выражения, содержащего степень?

– Чему равно значение выражения 0,28 · 58? Как рационально вычислить? Каким правилом необходимо воспользоваться?

Домашнее задание: № 390; № 391; № 394; № 398.

 

 

Урок 45
Умножение и деление степеней
с одинаковыми основаниями

Цели: вывести правила умножения и деления степеней с одинаковым основанием; дать определение нулевой степени числа, не равного нулю; формировать умение выполнять указанные действия со степенями.

Ход урока

I. Устная работа.

1. Вычислите.

а) 32;                         б) ;               в) (0,1)3;               г) ;  

д) ;                 е) (–0,1)4;             ж) ;               з) –(–7)2;    

и) –(–2)3;                  к) 016;                   л) (–1)18;               м) –(–1)23.

2. Сравните значение двух выражений:

а) (–8,64)20 и 030;                                   б) (–1)76 и (–1)70;

в)  и (–3,82)13;                       г)  и .

II. Проверочная работа.

Вариант 1

1. Найдите значение выражения.

а)  – (0,5)2;          б) 3000 · (0,2)3 – (–2)6;       в) – (–3)3.

2. Вычислите значение выражения х3х2 при:

а) х = 0,3;                           б) х = –6.

Вариант 2

1. Найдите значение выражения.

а)  + (0,6)2;          б) 2000 · (0,3)4 – (–2)4;       в)  – (–4)3.

2. Вычислите значение выражения х2 + х3 при:

а) х = –0,4;                         б) х = 10.

III. Объяснение нового материала.

На этом уроке изучаем два важных свойства степени: сложение и умножение степеней с одинаковыми основаниями.

Вывод правил целесообразно осуществлять, работая сразу с числовыми и буквенными выражениями, результаты оформить в виде таблицы.

Свойство 1. При умножении степеней с одинаковыми основаниями  основание  оставляют  прежним,  а  показатели  степеней складывают.

 

по сочетательному свойству умножения

по определению степени с натуральным показателем

= 25

Итак,  22 · 23 = 22 + 3

= am + n

 

Свойство 2. При делении степеней с одинаковыми основаниями, основание оставляют прежним, а из показателя степени делимого вычитают показатель степени делителя.

 

5 > 3

35 : 33 =

m > n, a ¹ 0

am : an =

запишем частное в виде дроби


 

сократим дробь

по определению степени с натуральным показателем

= 32

Итак, 35 : 33 = 35 – 3

= amn

Замечаем, что  am : am = am – m = a0 = 1.

Определение. Степень числа а, не равного нулю, с нулевым показателем равна единице.

IV. Формирование умений и навыков.

На этом занятии можно отрабатывать только умение находить произведение степеней с одинаковым основанием.

1. № 403.

Решение:

а) x5x8 = x5 + 8 = x13;                       е) yy12 = y1 + 12 = y13;

ж) 2624 = 26 + 4 + 210;                     з) 757 = 75 + 1 = 76.

2. № 405.

Решение:

а) a15 = a6 + 9 = a6  a9;                  б) a15 = a9 + 6 = a9  a6;

в) a15 = a2 + 13 = a2  a13;               г) a15 = a14 + 1 = a14  a = a  a14.

3. № 407.

Решение:

Представим число 6 в виде суммы двух натуральных чисел всеми возможными способами:

6 = 1 + 5;                  6 = 2 + 4;              6 = 3 + 3.

Значитa6 = a  a5;    a6 = a2  a4;    a6 = a3  a3.

4. № 409.

Решение:

а) m3m2m8 = m3 + 2 + 8 = m13;           в) xx4x4x = x1 + 4 + 4 + 1 = x10;

д) 78  7  74 = 78 + 1 + 4 = 713;                  е) 5  52  53  55 = 51 + 2 + 3 + 4 = 511.

5. № 410.

При выполнении этого упражнения ученики сами определяют основание степени, которое будет являться общим для двух степеней.

Решение:

а) 58 ∙  25 = 58 ∙  52 = 58 + 2 = 510;

в) 615 ∙  36 = 615 ∙  62 = 615 + 2 = 617;

д) 0,45 ∙  0,16 = 0,45 ∙  0,42 = 0,45 + 2 = 0,47;

е) 0,001 ∙  0,14 = 0,13 ∙  0,14 = 0,13 + 4 = 0,17.

6. № 411.

Решение:

а) 24 ∙  2 = 24 + 1 = 25 = 32;

б) 26 ∙  4 = 26 ∙  22 = 26 + 2 = 28 = 256;

в) 8 ∙  27 = 23 ∙  27 = 23 + 7 = 210 = 1024;

г) 16 ∙  32 = 24 ∙  25 = 24 + 5 = 29 = 512.

7. № 413.

Решение:

а) (c4)2 = c4 ∙  c4 = c4 + 4 = c8;

б) (c2)4 = c2 ∙  c2 ∙  c2 ∙  c2 = c2 + 2 + 2 + 2 = c8.

V. Итоги урока.

– Дайте определение степени с натуральным показателем.

– Сформулируйте основное свойство степени.

– Сформулируйте правила умножения и деления степеней с одинаковыми основаниями. Приведите примеры.

– Дайте определение степени числа с нулевым показателем.

Домашнее задание: № 404; № 406; № 408;  412; № 533.

 

 

Урок 46
Решение задач по теме
«Умножение и деление степеней»

Цели: продолжить формировать умение выполнять действия со степенями с одинаковыми основаниями.

Ход урока

I. Устная работа.

1. Найдите значение выражения.

а) 43;                б) (0,7)2;               в) ;               г) 012;

д) (–6)2;           е) (–0,3)4;             ж) (–1)8;               з) .

2. Сравните с нулем значение выражения.

а) (–25)12 · (–25)9;

б) (–4)19 : (–4)7;

в) (–12)13 · (–12)8.

3. Замените звездочку степенью с основанием а так, чтобы стало верным равенство:

а) а4 · * = а12;                     б) * · а = а4;

в) а14 : * = а7;                     г) * : а9 = а10.

II. Формирование умений и навыков.

На этом занятии учащиеся отрабатывают умение делить степени с одинаковыми основаниями и решают комбинированные задачи.

1. № 414.

Решение:

а) x5 : x3 = x5 – 3 = x2;

в) a21 : a = a21 – 1 = a20;

з) 0,79 : 0,74 = 0,79 – 4 = 0,75.

2. № 416.

Решение:

а) 56 : 54 = 56 – 4 = 52 = 25;

б) 1015 : 1012 = 1015 – 12 = 103 = 1000;

в) 0,510 : 0,57 = 0,510 – 7 = 0,53 = 0,125;

г) ;

д) 2,7313 : 2,7312 = 2,7313 – 12 = 2,73;

е) .

3. Используя правила умножения и деления степеней, упростите выражение.

а) x8 ∙  x3 : x5;                      б) x20 : x10 ∙  x;

в) x7 : x3 : x3;                       г) x14 : x9 ∙  x5.

Решение:

а) x8 ∙  x3 : x5 = x8 + 3 : x5 = x11 : x5 = x11 – 5 = x6;

б) x20 : x10 ∙  x = x20 – 10 ∙  x = x10 ∙  x = x10 + 1 = x11;

в) x7 : x3 : x3 = x7 – 3 : x3 = x4 : x3 = x4 – 3 = x;

г) x14 : x9 ∙  x5 = x14 – 9 ∙  x5 = x5 ∙  x5 = x5 + 5 = x10.

4. № 417.

Решение:

а) = 86 : 84 = 86 – 4 = 82 = 64;

б) = 0,87 : 0,84 = 0,87 – 4 = 0,83 = 0,512;

в) = (–0,3)5 : (–0,3)3 = (–0,3)5 – 3 = (–0,3)2 = 0,09;

г) ;

д)

.

5. Найдите значение выражения.

а) ;                          б) ;

в) ;                 г) .

При выполнении этого упражнения уже не обязательно переписывать дробь в виде частного.

Желательно, чтобы учащиеся проговаривали не только правила действий над степенями, но и правила возведения в степень отрицательного числа при четном нечетном показателях.

Решение:

а)  = 821 – 18 = 83 = 512;

б)  = 1010 – 6 = 104 = 10 000;

в)  = (–2)11 – 8 = (–2)3 = –8;

г)  = (0,3)17 – 14 = (0,3)3 = 0,027.

6. № 419 (а, в, д).

Решение:

а) xn  x3 = xn + 3;

в) x  xn = x1 + n = xn + 1;

д) c9 : cm = c9 – m.

7. Представьте данное выражение сначала в виде произведения степеней, а затем в виде частного степеней.

а) am – 2;            б) a4n;          в) an.

Решение:

а) am – 2 = am – 4 ∙  a2;            am – 2 = am : a2;

б) a4n = a2n ∙  a2n;                 a4n = a5n : an;

в) an = an – 1 ∙  a;                  an = a2n : an.

Выполняя это упражнение, учащиеся могут предложить свои варианты разбиения на множители.

8. № 420 (а, в), № 421 (а, б).

№ 420.

Решение:

а) если х = 2,6, то 3х0 = 3 (при любом значении х);

в) 10a2b0 = 10a2, если а = 3, b = –8, то 10a2 = 10 · 32 = 10 · 9 = 90.

№ 421.

Решение:

а) b4 · b0 = b4 · 1 = b4;         б) c5 : c0 = c5 : 1 = c5.

При выполнении этого упражнения учащиеся могут воспользоваться правилом умножения и деления степеней.

III. Итоги урока.

– Дайте определение степени с натуральным показателем.

– Сформулируйте правило возведения отрицательного числа в четную степень, в нечетную степень.

– Какой знак имеет результат возведения любого числа в квадрат?

– Сформулируйте правила сложения и умножения степеней с одинаковыми основаниями.

– Чему равно значение выражения 20; (–1)1; ?

Домашнее задание:  № 415;  № 418;  № 419  (б, г, е);  № 420  (б, г);
№ 421 (в, г); № 422.

 

Урок 47
Решение практических задач по теме
«Умножение и деление степеней»

Цель: формировать умение использовать правила умножения и деления  степеней  с  одинаковыми  основаниями  при  решении  практических задач.

Ход урока

I. Проверочная работа.

Вариант 1

1. Представьте в виде степени произведение.

а) x6  x3  x7;                      б) (–7)3  (–7)2  (–7)9.

2. Представьте в виде степени частное.

а) x8 : x4;                             б) (–0,5)6 : (–0,5)8.

3. Найдите значение выражения.

а) ;                       б) .

Вариант 2

1. Представьте в виде степени произведение.

а) y5  y9  y2;                      б) (–6)8  (–6)2  (–6)3.

2. Представьте в виде степени частное.

а) z10 : z7;                            б) .

3. Найдите значение выражения.

а) ;                б) .

II. Мотивация изучения.

Данная тема предоставляет учителю возможность познакомить детей с числовыми величинами, которыми можно выразить количественные отношения реального мира. В этом плане особенно важны задачи, содержащие реальные величины, например задачи о Солнечной системе, планетах и других космических телах.

Полезно ознакомить учащихся с названиями классов принятой десятичной нумерации:

103 – тысяча

106 – миллион

109 – биллион (миллиард)

1012 – триллион

1015 – квадриллион

1018 – квинтиллион

1021 – секстиллион

1024 – септиллион

1027 – октиллион

1030 – нониллион

1033 – дециллион

1036 – андециллион

1039 – дуодециллион

1042 – тредециллион

1045 – кваттордециллион

 

10100 – гугол

Интересно для сравнения привести наименования классов старинной русской нумерации. Л. Магницкий в своей «Арифметике», изданной при Петре I, упоминает такие названия:

103 – тысяча

104 – тьма

105 – легион

106 – леодр

107 – вран

108 – колода.

Операции с числовыми великанами делают актуальными приближенные вычисления. Если исходные данные в задаче получены в результате измерений (например, астрономических) с точностью до 2–3 десятичных знаков, нет никакого смысла в последующих десятках цифр. Поэтому в этой теме уместно познакомить детей с правилами округления чисел.

III. Формирование умений и навыков.

1. Найдите отношение массы каждой из планет Солнечной системы к массе Земли.

Справка.

Планета

Солнце

Меркурий

Венера

Земля

Марс

масса, кг

2 · 1030

3,4 · 1023

4,9 · 1024

6 · 1024

6,4 · 1023

 

Планета

Юпитер

Сатурн

Уран

Нептун

Плутон

масса, кг

1,9 · 1027

5,7 · 1026

8,8 · 1025

1,0 · 1026

1,1 · 1021

2. В  астрономии  одной  из  единиц  длины  является  световой  год,
то есть расстояние,  которое  проходит  за  год  луч света. Скорость света
с = 300 000 км/с. Вычислите:

а) за какое время луч света доходит от Земли до Луны, от Солнца до Земли;  б) величину светового года в километрах;  в) расстояние от Земли до звезды Сириус в световых годах.

Справка. Среднее расстояние от Земли до Луны 384 000 км, от Земли до звезды Сириус 8,2 · 1013 км.

3. Ежегодно прирост древесины на опытном участке составляет 10 %. Какое количество древесины будет на участке через 10 лет, если сейчас её 105 м3?

4. В сберегательном банке вкладчику начисляется 20 % в год от сданной на хранение суммы. Через сколько лет первоначальная сумма увеличится более чем в 2 раза; в 5 раз?

5. Найдите  массу  мотка  медной  проволоки  сечением  2 мм  и  длиной 50 м.

Справка. Масса вычисляется по формуле m = ρ ∙  V, где ρ – плотность вещества. В частности, для меди ρ = 8,9 г/см3. А для вычисления объема цилиндра V нужно воспользоваться формулой V = πR2H.

6*. Какое наибольшее число абонентов может быть прикреплено к одной АТС при семизначной записи номеров телефона? Первые три цифры всех номеров данной АТС одинаковы.

IV. Итоги урока.

– Сформулируйте определение степени с натуральным показателем.

– В  каких  областях  используются  вычисления  больших  степеней числа 10?

Домашнее задание:  1. Во  сколько  раз  число  4,8 · 1019  больше  числа 1,2 · 1019?

2. Найдите расстояние от Солнца до планет Солнечной системы в астрономических единицах.

Справка.

Планета

Меркурий

Венера

Земля

Марс

Юпитер

Сатурн

Уран

Нептун

Плутон

Среднее
расстояние
от Солнца, млн км

58

108

150

228

778

1430

2870

4500

5900

Астрономическая единица (а. е.) – среднее расстояние от Солнца до Земли.

3. № 542; № 543.

 

Урок 48
Возведение в степень произведения

Цели: вывести  правило  возведения  в  степень  произведения  двух и  более  сомножителей;  формировать  умение  вычислять  степень  произведения, а также рационально преобразовывать выражения, содержащие степень произведения либо предполагающие использование данного свойства.

Ход урока

I. Устная работа.

Вычислите.

а) 23 · 53;                         в) 122;                   д) 53 · ;             ж) (bx)5;

б) 103;                   г) 32 · 42;                е) (2а)3;                   з) (ab)n.

II. Объяснение нового материала.

Конструкция примеров и их последовательность позволяют сделать обобщение. В результате появится следующая запись:

(ab)n = anbn.

Заготовленный лист с этим свойством закрепить на доску к ранее изученным. Это равенство можно доказать устно с подробной записью доказательства на доске:

Для любых а и b и произвольного натурального п верно равенство (ab)n = anbn.

Доказательство:

(ab)n = (ab) · (ab) · ... · (ab) по определению степени п раз;

(ab) · (ab) · ... · (ab) = (aa...a)(bb...b) по свойствам умножения п раз п раз; (ab)n = anbn.

Ученики пробуют самостоятельно сформулировать алгоритмическое правило возведения в степень произведения. Они приходят к выводу, что необходимо выполнить два шага:

1) каждый множитель возводить в эту степень;

2) результаты перемножить.

Следует записать выводы учащихся в виде алгоритма на доске и подчеркнуть глаголы. Глагол обозначает действие, которое необходимо выполнить. Ребята выясняют, можно ли поменять местами порядок выполнения действий. Далее идёт работа с учебником. Ребята сравнивают формулировку, которая получилась у них, с той, которая находится в учебнике на с. 97.

Такой подход даёт хороший результат быстрого заучивания формулировок свойств степени.

Последним можно предложить следующий пример:

(abсd)4 = ...

Решение:

(abcd)4 = a4b4c4d4.

Учащиеся могут самостоятельно доказать, что данная формула верна не только для двух сомножителей, но и большего их числа.

По окончании объяснения нового материала рассмотреть пример 1 со с. 97 учебника.

III. Формирование умений и навыков.

При выполнении упражнений на уроке ученики должны проговаривать правило и алгоритм возведения произведения в степень.

Кроме того, задания предполагают применение формулы как слева направо, так и справа налево. Необходимость того или иного способа обусловлена рациональностью преобразования выражения либо вычисления его значения.

1. № 428.

2. Выполните возведение в степень, представив предварительно основание степени в виде произведения множителей –1 и х:

а) (–х)2;            б) (–х)8;                в) (–х)100;              г) (–х)2п;

д) (–х)3;           е) (–х)9;                ж) (–х)71;              з) (–х)2п + 1.

Решение:

а) (–х)2 = ((–1) · х)2 = (–1)2 · х2 = 1 · х2 = х2;

е) (–х)9 = ((–1) · х)9 = (–1)9 · х9 = –1 · х9 = –х9;

г) (–х)2п = ((–1) · х)2п = (–1)2п · х2п = 1 · х2п = х2п;

з) (–х)2п + 1 = ((–1) · х)2п + 1 = (–1)2п + 1 · х2п + 1 = –1 · х2п + 1 = –х2п + 1.

3. № 431.

Решение:

а и –а – противоположные числа.

а2;

(–а)2 = ((–1) · а)2 = (–1)2 · а2 = 1 · а2 = а2,

значит, а2 = (–а2).

4. № 432.

Решение:

Пусть а – сторона квадрата, тогда площадь квадрата равна а2.

Если сторона квадрата увеличится в 2 раза, то станет равна 2а, а его площадь будет равна (2а) · (2а) =
= (2а)2 = 22 · а2 = 4а2, то есть увеличится в 4 раза.

Аналогично рассуждаем для остальных случаев.

5. № 433.

Решение:

Пусть а – ребро куба, тогда его объем равен а3.

Если ребро увеличить в 3 раза, то объем куба будет вычисляться по формуле (3а) · (3а) · (3а) = (3а)3 =
= 33 · а3 = 27а3, значит, объем увеличится в 27 раз.

6. № 434.

Для решения используем данные задачи № 432.

Решение:

Поверхность  куба  состоит  из  6  квадратов  площадью  а2,  то  есть равна 6а2.

Если ребро куба увеличить в 3 раза, то площадь боковой грани составит 9а2, а общая площадь поверхности равна 6 · 9а2 или 54а2.

Новая площадь больше в 9 раз, значит, и краски потребуется в 9 раз больше, то есть 40 · 9 = 360 г. Следовательно, 350 г краски на хватит.

Ответ: не хватит.

7. Представьте произведение в виде степени.

а) x5y5;                       б) 36a2b2;                      в) 0,001x3c3;

г) –х3;                       д) –8х3;                          е) –32a5b5;

ж) x5y5z5;                            з) 0,027a3b3c3;               и) x3a3z3.

8. Вычислите значение выражения, используя свойство степени произведения.

а) 53 · 23;                                      в) (0,5)3 · 603;

б)  · 204;                    г) (1,2)4 · .

IV. Итоги урока.

– Сформулируйте определение степени с натуральным показателем.

– Сформулируйте правило возведения в степень произведения.

– Сколько  сомножителей  может  стоять  в  формуле  степени  произведения?

– Чему равно значение выражения (3 · 5 · 78)0?

Домашнее задание: № 429; № 430; № 435; № 436; № 437.

 

 

Урок 49
Возведение степени в степень

Цели: вывести правило возведения степени в степень; формировать умение  выполнять  преобразование  выражений,  содержащих  степень в степени.

Ход урока

I. Проверочная работа.

Вариант 1

1. Возведите в степень произведение.

а) (xyz)8;              б) ;           в) (–2а)3;          г) .

2. Вычислите значение выражения.

а) 252 · 42;           б)  · 93;          в) (–0,5)3 · 403.

Вариант 2

1. Возведите в степень произведение.

а) (abc)10;           б) ;         в) (–4а)3;          г) .

2. Вычислите значение выражения.

а) 203 · 53;           б)  · 252;       в) (–0,2)4 · 504.

II. Объяснение нового материала.

1. Устная работа.

Представьте в виде степени.

а) (а5)3 = а5 · а5 · а5 = … ;                      б) (у2)5 = … ;

в) (ат)7 = … ;                                        г) (ат)п = … .

В результате появится запись:

(ат)п = ат п.

2. Доказательство свойства можно оформить в виде таблицы.

Свойство. При возведении степени в степень основание оставляют тем же, а показатели перемножают.

(23)2 = 23 · 23 =

по первому свойству степени

= 23 + 3 =

по определению умножения

= 23 · 2

Итак, (23)2 = 23 · 2

= am · n

Подчеркиваем, что формулу можно применять в следующем виде:

(am)n = am n = an m = (an)m.

III. Формирование умений и навыков.

Все задания, решаемые на этом уроке, можно разделить условно на две группы:

1-я группа. Закрепление навыков возведения степени в степень.

2-я группа. Решение заданий на правило возведения в степень произведения и степени.

1. № 438 (устно).

Решение:

а) (х3)2 = х3 · 2 = х6;

з) (b5)2 = b5 · 2 = b10.

2. № 440, № 441.

№ 441.

Решение:

а) ап · а3 = ап + 3;

г) (а2)т = а2т.

3. № 443, № 445, № 446.

№ 443.

Решение:

а) 220 = 22 · 10 = (22)10;          б) 220 = 24 · 5 = (24)5;

в) 220 = 25 · 4 = (25)4;            г) 220 = 210 · 2 = (210)2.

№ 445.

Решение:

12 = 1 · 12;               а12 = (а1)12;

12 = 2 · 6;                 а12 = (а2)6;

12 = 3 · 4;                 а12 = (а3)4;

12 = 4 · 3;                 а12 = (а4)3;

12 = 6 · 2;                 а12 = (а6)2;

12 = 12 · 1;               а12 = (а12)1.

№ 446. Решение:

а2 = т;

а6 = а2 · 3 = (а2)3 = т3.

4. Представьте выражение в виде квадрата числа.

а) а4;                   б) b6;                    в) d8;                   г) c10;

д) d20;                 е) ;                  ж) 1;              з) .

5. № 447, № 449 (а, б), № 450 (а, б).

№ 447.

Решение:

а) x3 · (x2)5 = x3 · x2 · 5 = x3 · x10 = x3 + 10 = x13;

б) (a3)2 · a5 = a3 · 2 · a5 = a6 · a5 = a6 + 5 = a11;

в) (a2)3 · (a4)2 = a2 · 3 · a4 · 2 = a6 · a8 = a6 + 8 = a14;

г) (x2)5 · (x5)2 = x2 · 5 · x5 · 2 = x10 · x10 = (x10)2 = x10 · 2 = x20;

д) (a3a3)2 = (a6)2 = a6 · 2 = a12;

е) (aa6)3 = a3 · (a6)3 = a3 · a6 · 3 = a3 · a18 = a3 + 18 = a21.

№ 449.

Решение:

а) x5 · (x2)3 = x5 · x6 = x11;

б) (x3)4 · x8 = x12 · x8 = x20.

№ 450.

Решение:

а)  = 24 = 16;

б)  = 5.

6. (Устно.) Найдите примеры, в которых допущена ошибка.

1) (ab)3 = a3b3;                    5) (–32)3 = 36;

2) (–2bc)2 = –4b2c;              6) (c4)2c3 = c9;

3) (2 · 5)4 = 10000;             7)  = a24;

4) (–33)2 = 36;                      8)  = 26a6b14.

IV. Итоги урока.

– Сформулируйте определение степени с натуральным показателем.

– Сформулируйте правило возведения степени в степень. приведите примеры.

– Каков алгоритм возведения степени в степень?

– Чему равно значение выражения: ; (x3)0?

Домашнее задание:  № 439;  № 442;  № 444;  № 448;  № 449  (в, г);
№ 450 (в, г).

 

 

 

Урок 50
Решение задач по теме «Возведение в степень
произведения и степени»

Цели: обобщить знания по теме «Степень и её свойства»; закрепить умения преобразовывать числовые и буквенные выражения, содержащие степень.

Ход урока

I. Обобщение и систематизация материала.

Повторяем и систематизируем теоретический материал и практическую часть.

Дана таблица. В левом столбце заполнить пропущенные места, в правом – выполнить задания.

Степенью числа а с натуральным
показателем п называется __________
п ________, каждый из которых равен а.

Степень числа а с показателем,
равным 1 ________________

1. Представьте в виде степени
произведение:

а) (–8) · (–8) · (–8) · (–8) · (–8);

б) (ху) · (ху) · (ху) · (ху).

2. Возведите в степень:  34; (–0,2)5;

Назовите основание и показатель записанных степеней

При умножении степеней с одина-
ковыми основаниями ____________
складывают, а ________ оставляют
прежним

Выполните действия:

а4 · а12;

b6 · b9 · b;

32 · 33

При делении степеней с одинаковыми основаниями ________ оставляют
прежним, а из ______ числителя
_________________ знаменателя

Выполните действия:

b8 : b2;        n7 : n6;

c9 : c;          57 : 54

При возведении степени в степень
_________ оставляют прежним,
а _________ перемножают

Выполните действия:

(m3)7;    (k4)5;    (22)3

При возведении в степень произведения возводят в эту степень _______________ и результаты перемножают

Выполните возведение в степень:

(–2a3b)5;   

Степень числа а, не равного нулю,
с нулевым показателем равна _________

Вычислите:

3х0    при   х = 2,6

II. Закрепление умений и навыков.

Индивидуальная проверочная работа с  кодированным ответом.

Каждый учащийся выполняет задания, к ним прилагается ключ, в котором использован весь алфавит, чтобы исключить угадывание ответов по буквам. В случае правильного решения – правильное слово.

Задания для каждого ряда индивидуальные. Желательно при наличии места разместить задания на доске, чтобы можно было проверить ответы, ход решения. Побеждает команда, набравшая наибольшее число баллов за правильные ответы.

Размещение материала на доске

РОМАШКА

 

 

это

У

Н

И

К

А

Л

Ь

Н

О

Г

Л

О

Б

А

Л

Ь

Н

О

Г

Е

Н

И

А

Л

Ь

Н

О

 

 

п/п

Задание

I ряд

Ответ

 

п/п

Задания

II ряд

Ответ

 

код

 

 

код

1

т3 · т2 · т8

т13

У

1

а4 · а3 · а2

а9

Г

2

р20 : р17

р3

Н

2

(24)5 : (27)2

64

Л

3

с5 : с0

с5

И

3

3 · 32 · 30

27

О

4

(3а)3

27а3

К

4

(2у)5

32у5

Б

5

т · т5 · т3 · т0

т9

А

5

(т2)4 · т

т9

А

6

214 : 28

64

Л

6

(23)2

64

Л

7

(–х)3 · х4

х7

Ь

7

(–х3) · (–х)4

х7

Ь

8

(р · р3)2 : р5

р3

Н

8

(р2 · р5) · р0 : р4

р3

Н

9

37 · (32)3 : 310

27

О

9

(35)2 · 37 : 314

27

О

 

п/п

Задание

III ряд

Ответ

 

код

1

а4 · а · а3а

а9

Г

2

(7х)2

49х2

Е

3

р · р2 · р0

р3

Н

4

с · с3 · с

с5

И

5

т · т4 · (т2)2 · т0

т9

А

6

(23)7 : (25)3

64

Л

7

х3 · (–х)4

х7

Ь

8

(р2)4 : р5

р3

Н

9

(34)2 · (32)3 : 311

27

О

Ключ

А

Б

В

Г

Д

Е

Ж

З

И

К

Л

т9

32у5

81

а9

х3

49х2

т5

р4

с5

27а3

64

 

М

Н

О

П

Р

С

Т

У

Ф

Х

Ц

34

р3

27

25

х7

р6

т3

т13

а8

81а3

с7

 

Ч

Ш

Щ

Ъ

Ь

Ы

Э

Ю

Я

 

 

16а4

25

10у5

9у7

х7

а2

32х5

49у3

х5

 

 

III. Итоги урока.

– Сформулируйте определение степени числа с натуральным показателем. Приведите примеры и назовите в каждом из них основание и показатель степени.

– Сформулируйте и докажите основное свойство степени.

– Сформулируйте правило умножения и правило деления степеней с одинаковыми основаниями.

– Дайте определение степени числа с нулевым показателем.

– Сформулируйте правило возведения в степень произведения, правило возведения в степень степени.

Домашнее задание: № 534; № 535; № 539; № 547; № 548.

 

 

Урок 51
Понятие одночлена и приведение его
к стандартному виду

Цели: ввести понятие одночлена и его стандартного вида; формировать умение приводить одночлен к стандартному виду путем его упрощения; формировать умение определять коэффициент и степень одночлена.

Ход урока

I. Устная работа.

1. Упростите выражение.

а) х3 · (–х4);                        б) х3 · (–х)4;                    в) (–х)3 · х4;

г) (–х3) · (–х)4;                    д) (а2)5 · а5;                    е) (а2 · а5)2.

2. Выполняя задания на преобразование выражений, содержащих степени, ученик допустил следующие ошибки:

а) 5 · 5 · 5 · 5 = 45;        б) (–3)2 = –3 · 3 = –9;               в) 71 = 1;

г) 00 = 1;                       д) 23 · 27 = 221;                          е) 23 · 28 = 410;

ж) 23 + 27 = 210;             з) 230 : 210 = 23;                         и) (2х)3 = 2х3;

к) (а3)2 = а9;                  л) (а2)3 · (а4)2 = (а6)5 = а30.

Какие определения, свойства, правила не знает ученик?

II. Объяснение нового материала.

1. При решении различных задач часто встречаются алгебраические выражения вида  a · b · a · b · c;  3 · a2 · b. Для сокращения записи этих выражений знак умножения «точка» обычно опускается, то есть пишут просто  ababc;  3a2b.  Каждое  из  этих  произведений  называют одночленом.

На доску выносится запись:

Произведение нескольких чисел, обозначенных цифрами или буквами, называют одночленом.

Например, одночленами являются выражения:

abc;     (–4)a3ab;     a(–0,3)bab;     172;     –.

Так как произведение равных множителей можно записать в виде степени с натуральным показателем, то степень числа и произведение степеней чисел также называют одночленами.

Например;     (–7)3;     c5;     4a2;     a2b.

Множители одночлена, записанные с помощью цифр, называют числовыми множителями одночлена, а множители, обозначенные буквами, называют буквенными множителями.

2. Одночлены можно упрощать, пользуясь переместительным и сочетательным законами умножения.

Стандартным видом одночлена называется его запись, когда на первом месте стоит числовой коэффициент, а затем степени различных переменных.

Обращаем внимание учащихся, что коэффициент одночлена может быть  равен  единице,  в  этом  случае  мы  его  не  пишем  перед  буквенной частью. Переменные принято записывать в алфавитном порядке, то есть не 3x2a4c, а 3a4cx2.

3. Вводим понятие степени одночлена.

Степенью одночлена называют сумму показателей степеней всех входящих в него переменных.

Если одночлен не содержит переменных и является числом, отличным от нуля, то степень этого одночлена считают равной нулю.

Число 0 является одночленом, степень которого не определена.

III. Формирование умений и навыков.

На этом занятии необходимо отработать следующие умения:

1) выявлять одночлен, используя определения;

2) выделять элементы одночлена: числовой коэффициент и буквенную часть;

3) определять, записан ли одночлен в стандартном виде;

4) приводить одночлен к стандартному виду;

5) вычислять значение одночлена в стандартном виде;

6) определять степень одночлена стандартного вида.

1. (Устно). Назовите числовые и буквенные множители одночлена.

а) 6a(0,3)b2c;                     в) 3p(–0,1)q7r;

б) 0,5ab3c;                     г) 2,5mn4k.

2. № 455 (устно).

3. Вместо  словесной  формулировки  запишите  алгебраическое  выражение:

а) удвоенное произведение чисел a и b;

б) утроенное произведение чисел b и с;

в) произведение квадратов чисел х и у;

г) произведение числа а и квадрата числа b;

д) произведение куба числа т и числа р;

е) утроенное произведение квадрата числа а и числа b.

4. № 456 (устно).

При выполнении этого упражнения ученики должны мотивировать свой ответ.

5. Среди одночленов  10,2a2b2c;  –7,3ab2c;  17a2bca;  –2,6ab2c;  –m;  3ab; –28a2b2c2;  3aabc;  –2ab;  –m4mm ∙  2;  17a2b2c2:

а) назвать одночлены стандартного вида;

б) указать одночлены, отличающиеся только коэффициентами.

6. № 457.

Решение:

а) 8x2x = 8x2 + 1 = 8x3;

б) 1,2abc ∙  5a = (1,2 ∙  5) ∙  (a ∙  a) ∙  bc = 6a2bc;

в) 3xy(–1,7)y = 3 ∙  (–1,7) ∙  x ∙  y ∙  y = –5,1xy2;

г) 6c2(–0,8)c = 6(–0,8)c2c = –4,8c3;

д) m2n ∙  4,5n3 =  ∙  m2 ∙  n ∙  n3 = 3m2n4;

е)  a2a3xx2 = –a5x3.

7. № 459.

Решение:

а) если у = –2, то –0,125у4 = –0,125 · (–2)4 = –0,125 · 16 = –2;

б) если х = –0,3, у = , то 12x2y = 12 · (–0,3)2 · = 2 · 0,09 = 0,18.

Ответ: а) –2;  б) 0,18.

8. № 461.

Решение:

               S = 5m · m = 5m2 (см2).

Ответ: 5m2 (см2).

9. Запишите одночлен в стандартном виде и определите его степень.

а) ac12c;                            г)  · 4;

б) a8b2ba3;                  д) –m3np;

в) –0,5xy2x3;                    е) a3d0x.

IV. Итоги урока.

– Сформулируйте определение одночлена.

– Приведите пример одночлена стандартного вида и назовите его коэффициент.

– Каким образом можно привести одночлен к стандартному виду?

– Сформулируйте определение степени одночлена. Чему равна степень одночлена, не содержащего переменных? Чему равна степень 0?

Домашнее задание: № 458; № 460; № 462; № 463; № 554; № 555.

 

 

Урок 52
Умножение одночленов

Цель: формировать умение умножать одночлен на одночлен, используя правило умножения степеней с одинаковыми основаниями.

Ход урока

I. Устная работа.

1. Назовите коэффициент одночлена.

а) 15a2b2c;       б) 18a3b2c;           в) –24ab2c3;          г) –35ab3c2;

д) nm2;             е) n3m;                 ж) –pqr2;              з) –pq2r.

2. Определите степень одночлена.

а) 37a2bx3;       б) xyz;               в) x2y;          г) –862.

II. Актуализация знаний.

Работа в парах с тестами с последующей взаимопроверкой.

Вариант 1

1. Степенью  числа  а  с  натуральным  показателем  п,  большим  1,  называется  произведение _______ множителей,  каждый  из  которых  равен _____ : ап = _____.

2. 43 = ________ = _______, здесь 4 – _______ степени, 3 – ______ степени, 64 – ________ степени 43.

3. При умножении степеней с одинаковыми основаниями основание остается прежним, а показатели степеней _________.

4. am : aп = __________? (m > n, a ¹ 0).

5. При возведении степени в степень основание остается прежним, а показатели степеней ___________________.

6. (ab)п = ___________________________________.

7. При возведении дроби в степень следует в эту степень _______________________________________________.

8. Произведение числовых и буквенных множителей называют __________________________________________.

9. Коэффициент одночлена (–a3b2) равен _______________.

Вариант 2

1. В выражении ап  число а называют _______ степени, число п – ___________ степени.

2. 54 = _______ = _______, здесь 5 – _______ степени, 4 – _______ степени, 625 – _______ степени 54.

3. am ∙  aп = __________________________________.

4. При делении степеней с одинаковыми основаниями основание остается прежним, а показатели степеней _____________.

5. (am)п = __________________________________.

6. При возведении в степень произведения, в эту степень возводится _____________________________________.

7. При возведении в степень дроби следует в эту степень _______________________________________________.

8. Числа, переменные и их степени называют ____________.

9. Числовой множитель одночлена, записанного в стандартном виде, называют ________________.

III. Объяснение нового материала.

1. Решим следующую задачу.

Объем  прямоугольного  параллелепипеда  вычисляется  по  формуле
V = abc, где а – длина, b – ширина и с – высота этого параллелепипеда.

Каким будет объем нового параллелепипеда, если длину данного увеличить в 5 раз, ширину – в 2п раз, высоту в 3п раз?

Решение:

Найдем измерения нового параллелепипеда:

длина – 5а;

ширина – 2пb;

высота – 3пс.

Тогда его объем равен (5а) · (2пb) · (3пс). Данное выражение является произведением трех одночленов. По правилам умножения можно записать равенство:

(5а) · (2пb) · (3пс) = 5а · 2пb · 3пс = (5 · 2 · 3) · (аппbс) = 30ап2 =
= 30аbсп2.

2. В результате умножения одночленов снова получается одночлен, который можно упростить, записав в стандартном виде:

(3a2b3c) · (4ab2) = (3 · 4) · (a2a) · (b3b2) · c = 12a3b5c.

3. Аналогично находим произведение трех и более одночленов.

IV. Формирование умений и навыков.

На уроке отрабатываются умения перемножать одночлены и раскладывать одночлен в виде произведения двух и более одночленов.

1. Выполните умножение.

1) а) 12у · 0,5у;                       б) 8x · ;                  в) –b3 · 3b2;

2) а) xy2 · 16y;             б) 1,6a2c · (–2ac2);              в) –x3y4 · 1,4x6y5.

Решение:

1) а) 12у · 0,5у = (12 · 0,5) (у · у) = 6у2;

    б) 8x2 · (x2y) = –6x2y;

    в) –b3 · 3b2 = (–1 · 3)(b3b2) = –3b5;

2) а) xy2 · 16y = (xy2y) = 12xy3;

    б) 1,6a2c · (–2ac2) = (1,6 (–2))(a2cac2) = –3,2a3c3;

    в) –x3y4 · 1,4x6y5 = (–1 · 1,4)(x3y4x6y5) = –1,4x9y9.

2. Перемножьте одночлены.

а) (–0,4x5y6z2) · (–1,2xyz3);

б) (–2,5n4m5k2) · (3nm2k5);

в) ;

г) .

Решение:

а) (–0,4x5y6z2) · (–1,2xyz3) = (–0,4 · (–1,2)) · (x5x) · (y6y) · (z2z3) =
= 0,48x6y7z5;

б) (–2,5n4m5k2) · (3nm2k5) = (–2,5 · 3) · (n4n) · (m5m2) · (k2k5) = 7,5n5m7k7;

в)  · (x2x) · (y3y2) · (zz3) =
= 2x3y5z4;

г)  · (a2a3) · (b5b2) · (c3c4) =
= –7,5a5b7c7.

3. Перемножьте одночлены.

1) –20х4,   0,5ху2    и    –0,3х2у3;

2) 12x2y2z,   xy2z2    и    –0,1x2yz2.

Решение:

1) (–20x4) · (0,5xy2) · (–0,3x2y3) = (–20 · 0,5 · (–0,3)) · (x4xx2) · (y2y3) =
= 3x7y5;

2) (12x2y2z) ·  · (–0,1x2yz2) =  · (x2xx2) ×
× (y2y2y) · (zz2z2) = 0,9x5y5z5.

4. Выполните умножение.

а) (–a) · (3b) · (4a2b) · (5ab2);

б) (5a) · (a2b2) · (–2b) · (–3a);

в) (–1,5ab) ·  · (–2ac) · (24ab).

Решение:

а) (–a) · (3b) · (4a2b) · (5ab2) = (–1 · 3 · 4 · 5) · (aa2a) · (bbb2) = –60a4b4;

б) (5a) · (a2b2) · (–2b) · (–3a) = (5 · 1 · (–2) · (–3)) · (aa2a) · (b2b) = 30a4b3;

в) (–1,5ab) ·  · (–2ac) · (24ab) =  ×
× (aaa) · (bbb) · (cc) = 18a3b3c2.

5. Замените значок * одночленом стандартного вида так, чтобы получившееся равенство было тождеством:

1) * · 4c2 = 30ac3;

2) 8a2b4 · * = –8a5b6.

Решение:

1) 7,5ac · 4c2 = 30ac3;        

2) 8a2b4 · (–a3b2) = –8a5b6.

6. Представьте двумя способами в виде произведения двух одночленов стандартного вида следующий одночлен:

а) 18x2y6z;                 б) a5b5c;                   в) –0,24a3b4z.

V. Итоги урока.

– Дайте определение одночлена. Приведите примеры.

– Приведите пример одночлена стандартного вида и назовите его коэффициент.

– Сформулируйте определение степени одночлена.

– Каким образом можно умножить одночлен на одночлен? На какие правила мы опираемся?

Домашнее задание: № 467; № 468; № 469; № 470; № 471.

 

Урок 53
Возведение одночлена в степень

Цели: формировать умение возводить одночлен в степень и приводить его к стандартному виду.

Ход урока

I. Устная работа.

Разгадайте кроссворд.

По вертикали:

2. Числовой множитель в одночлене стандартного вида.

3. Чему равен коэффициент одночлена a5bc5?

4. Чему равна степень одночлена 85?

5. Чему  равна  степень  одночлена 102xy5z2?

6. Чему равно (–2)2?

7. Какое число получается при возведении отрицательного числа в нечётную степень?

8. Сумма показателей всех переменных одночлена.

9. Вид одночлена, в котором на первом месте числовой множитель, а за ним степени различных переменных.

По горизонтали:

1. Выражение, которое содержит только числа, натуральные степени переменных и их произведения.

Ответы: 1. одночлен. 2. Коэффициент. 3. Единица. 4. Ноль. 5. Восемь. 6. Четыре. 7. Отрицательное. 8. Степень. 9. Стандартный.

II. Объяснение нового материала.

1. Актуализация знаний.

Выполните устно умножение одночленов.

а) a3 ∙  a4;                            б) a ∙  a2;                     в) –a ∙  a2 ∙  a4;

г) a ∙  (–x);                 д) (–x) ∙  (–y);                 е) (–x) ∙  ;

ж) (–2a) ∙  a2;            з) b2 ∙  (–3b3);                 и)  ∙  6y;

к) (0,2a) ∙  (–5b);       л)  ∙  (–4ab);       м) (–8m3) ∙  (–0,5n).

2. Теперь рассмотрим произведение двух или нескольких одинаковых одночленов, то есть степень одночлена. Например, (5a3b2c)2. Так как этот одночлен является произведением чисел 5, a3, b2, c, то по свойству возведения в степень произведения имеем:

(5a3b2c)2 = 52(a3)2(b2)2c2 = 25a6b4c2.

В результате возведения одночлена в натуральную степень снова получается одночлен.

III. Формирование умений и навыков.

1. № 472.

Решение:

а) ;

б) ;

в) ;

г) ;

д) ;

е) .

2. Выполните возведение одночлена в степень.

1) а) (6y)2;                 б) ;                    в) (0,1c5)4;

2) а) (5ax)3;               б) (4ac4)3;                      в) (5x5y3)3;

3) а) ;         б) (–10x2y6)3;                  в) (–a2b3c4)7;

4) а) –(3a2b)3;           б) –(–2ab4)3;                  в) –(–a3b2c)4.

Решение:

1) а) ;

    б) ;

    в) .

2) а) ;

    б) ;

    в) .

3) а) ;

    б) ;

    в) .

4) а) ;

    б) ;

    в) .

При выполнении этих упражнений впоследствии можно не записывать подробно возведение в степень каждого сомножителя. Можно выполнять устно.

Следующие задания направлены на формирование умения раскладывать одночлен на множители либо представлять в виде степени некоторого одночлена.

3. № 475, № 477.

№ 475.

Решение:

а) ;

б) ;

в) ;

г) .

№ 477.

Решение:

а) ;

    ;

б) ;

    .

4. № 479.

Решение:

а) ;

    .

б) ;

    .

5. Упростите выражение.

1) а) 35a ∙  (2a)2;                 б) –4x3 ∙  (5x2)3;              в) (–4y2)3 ∙  y5;

2) а) ;                    б) .

Решение:

1) а) ;

    б) ;

    в) .

2) а) ;

    б) .

IV. Проверочная работа.

Вариант 1

Выполните действия.

1)  ∙  (–24n) ∙  (4mn);          2) ;             3) (0,1a3b3)3.

Вариант 2

Выполните действия.

1) (–18n) ∙   ∙  (–5mn);        2) ;           3) (0,4a3b2)2.

V. Итоги урока.

– Дайте определение одночлена.

– В  каком  случае  мы  говорим,  что  одночлен  задан  в  стандартном виде?

– Сформулируйте определение степени одночлена. Приведите пример.

– Каким образом можно умножить одночлен на одночлен? Что получится в результате?

– Как возвести одночлен в степень? На какое правило мы при этом опираемся?

Домашнее задание: № 473; № 474; № 476; № 478; № 480.

 

 

Урок 54
Обобщение материала по теме «Умножение
одночленов. Возведение одночленов в степень»

Цели: закрепить навыки действий с одночленами (представление в стандартном виде, умножение одночленов и возведение одночлена в степень, разложение одночлена на множители и представление одночлена в виде степени).

Ход урока

I. Математический диктант.

Вариант 1

1. Запишите выражения:  (x + a)(xa);   x4y ∙  3xy;    x2 + x3 – 1. Подчеркните то, которое является одночленом.

2. Запишите одночлен bc2 ∙  (–0,5b2) ∙  (–8c). Перепишите его в стандартном виде и подчеркните коэффициент.

3. Является ли одночленом выражение 17x2y? Если да, то каков его коэффициент и какова его степень?

4. Является ли одночленом выражение –b? Если да, то каков его коэффициент и какова его степень?

5. Возведите в квадрат одночлен –3xy3.

6. Запишите в виде одночлена стандартного вида произведение одночленов 50a2bx и –7acx2.

Вариант 2

1. Запишите выражения:  3 + a4 + a;  (ab)(a + b);   7x3 ∙  x.  Подчеркните то, которое является одночленом.

2. запишите одночлен –2x2 ∙  3x3y. Перепишите его в стандартном виде и подчеркните коэффициент.

3. Является ли одночленом выражение –х? Если да, то каков его коэффициент и какова степень?

4. Является ли одночленом выражение 12ab2? Если да, то каков его коэффициент и какова его степень?

5. Возведите в куб одночлен –2ab2.

6. Запишите в виде одночлена стандартного вида произведение одночленов 3b2cd и –2b2yd.

II. Закрепление умений и навыков.

Тренажер по вариантам.

Вариант 1

1. Найдите значение одночлена.

1) 3,5х2   для   х = 4;   0,2;   0;   –1;   –10;

2) –4а3   для   а = –9;   –0,5;   0;   3;   10;

3) 6ху   для   х = 7 и у = 1,5;   x = 1 и у = –1,4;

4) –0,02а2с   для   а = –5 и с = –8;   а = –4 и с = 100;

5) 10abc   для   а = 1, b = –1 и с = 0,4; а = –2, b = –3 и с = 5.

2. Найдите с помощью калькулятора значение одночлена.

1) 0,4abc   для   а = 1,2, b = 0,6 и с = 2,3;

2) 1,5х3у   для   х = 12 и у = 1,6.

3. Найдите:

1) значение с, при котором значение одночлена 0,4с равно 0; 1; –1; 10;

2) какую-нибудь пару значений b и с, при которых значение одночлена 6bc равно 12; –60; 0; 3.

4. Верно ли, что одночлен:

1) 70а2 при любом а принимает положительные значения;

2) 0,04с2 при любом с принимает неотрицательные значения;

3) –25х2 при любом х принимает отрицательные значения;

4) 6у3 при любом у принимает положительные значения?

При утвердительном ответе обоснуйте свое заключение, при отрицательном – приведите опровергающий пример.

5. Представьте в виде одночлена стандартного вида.

1) а) (4ac2)3 ∙  (0,5a3c)2;                б)  ∙  (–9x4)2;

2) а) –(–x2y4)4 ∙  (6x4y)2;                б) (–10a3b2)5 ∙  (–0,2a2b)5.

6. Можно ли представить в виде квадрата одночлена выражение:

1) а) 81x2y4;                                 2) а) –5x3y5 ∙  ;

    б) –100x4y8;                                 б) –(–3xy)3 ∙  27y6?

Решение заданий 5,6 из варианта 1

5. 1) а) .

        б) ;

    2) а) ;

        б) (–10a3b2)5 ∙  (–0,2a2b)5 = (–10a3b2(–0,2)ab2)5 = (2a4b4)5 = 32a20b20.

6. 1) а) ;

        б) нельзя, так как –100x4y8 ≤ 0;

    2) а) ;

        б)  – нельзя, так как показатели степени 3 и 9 – нечетные.

Вариант 2

1. Найдите значение одночлена.

1) –1,5а2   для   а = 2; 0,8; 0; –1; –20;

2) 5у3   для   у = –10; –0,4; 0; 2; 8;

3)    для   а = –2,5 и b = 8; а = 1,75 и b = 1;

4) 0,04ху2   для   х = 15 и у = –2; х = –8 и у = –10;

5) 0,1хуz   для   х = –1, у = 1 и z = 20; х = 3, у = –4 и z = –2.

2. Найдите с помощью калькулятора значение одночлена:

1) 1,7хуz   х = 2,1, у = 0,8 и z = 5,6;

2) –0,8a2b3   для   а = 1,4 и b = 2,5.

3. Найдите:

1) значение а,  при  котором  значение  одночлена  0,3а  равно  0;  0,6; –0,8;   –1;

2) какую-нибудь пару значений а и b, при которых значение одночлена 5ab равно 30; –10; 0; 5.

4. Верно ли, что одночлен:

1) 2а3 при любом а принимает положительные значения;

2) –10х6 при любом х принимает отрицательные значения;

3) –0,03у2 при любом у принимает неположительные значения;

4) 2,7с2 при любом с принимает неотрицательные значения?

При утвердительном ответе обоснуйте свое заключение, при отрицательном приведите опровергающий пример.

5. Представьте в виде одночлена стандартного вида.

1) а) (10a2y)2 ∙  (3ay2)3;                 б)  ∙  (4y5)2;

2) а) –(3x6y2)3 ∙  (–x2y)4;                б) (–5ab6)4 ∙  (0,2a6b)4.

6. Можно ли представить в виде квадрата одночлена выражение:

1) а) 49a6b4;                                 2) а) –0,1a4b2 ∙  (–10a2b4);

    б) –25x2y4;                                   б) –(–2a4)3 ∙  2b8?

Решение заданий 5, 6 из варианта 2

5. 1) а) ;

        б) ;

    2) а) ;

        б) (–5ab6)4 ∙  (0,2a6b)4 = (–5 ∙  0,2 ∙  ab6a6b)4 = (–a7b7)4 = a28b28.

6. 1) а) ;

        б) нельзя, так как –25x2y4 ≤ 0;

    2) а) ;

        б) –(–2a4)3 ∙  2b8 = 8a12 ∙  2b8 = 16a12b8 = 42(a6)2(b4)2 = (4a6b4)2.

III. Итоги урока.

– Дайте определение одночлена.

– Как найти значение одночлена.

– Сформулируйте правило умножения одночленов и правило возведения одночлена в степень.

Домашнее задание:

1. Составьте таблицу значений одночлена:

1) 8х2  для  значений  х  из  промежутка  от  –0,5  до  0,5  с  шагом, равным 0,1;

2) 0,5х3 для значений х из промежутка от –10 до 10 с шагом, равным 2.

2. Представьте в виде:

1) квадрата одночлена выражение  a6;   0,16a4b10;

2) куба одночлена выражение  0,008x9;   –27a3b12.

3. № 556; № 559.

 

 

 

 

 

 

 

 

 

 

 

 

 

 

Урок
Функции
y = x2  и  у = х3  и их графики

Цели: изучить функциональные зависимости y = x2 и у = х3; формировать умение строить графики данных функций и работать с ними.

Ход урока

I. Устная работа.

1. Назовите область определения функции.

а) y = 3x;                            г) y = 2x2;                       ж) y = ;

б) y = ;                     д) y = х3;                     з) y = ;

в) y = –3x2 + 11;                  е) y = ;                   и) y = (3 – x)(x + 6).

2. Найдите значение функции y = x2 – 11, если:

а) х = 3;                              в) х = ;

б) х = 0;                              г) х = 0.

II. Объяснение нового материала.

Организуем самостоятельную работу по учебнику в парах.

С помощью учебника (пункт 23, с. 105–108) учащиеся должны ответить на вопросы, описанные в таблице (см. далее), и сравнить две функции: в чем схожи и в чем их отличие.

Для удобства выполнения работы можно заранее заготовить миллиметровую бумагу.

Также следует обсудить такие вопросы, как расположение графика, промежутки знакопостоянства, нули функции.

Вопросы

y = x2

y = x3

Заполните таблицу

x

–2

–1

0

1

2

y

 

 

 

 

 

x

–2

–1

0

1

2

y

 

 

 

 

 

По данным таблицы построить график

Свойства функции

1.

2.

3.

1.

2.

3.

Функция
возрастает

 

 

Функция
убывает

 

 

Название
графика

 

 

III. Формирование умений и навыков.

На данном уроке учащиеся строят графики функций и находят значение функции для заданного аргумента и наоборот. Также определяют принадлежность некоторой точки графику.

1. № 484, № 485.

При выполнении этих заданий учащиеся проговаривают пра-вила выполнения действий с графиком.

2. № 487. Решение:

а) А (6; 36)                36 = 62;

                                 36 = 36 – верно, значит, принадлежит;

б) В (–1,5; 2,25)        2, 25 = (–1,5)2;

                                 2,25 = 2,25 – верно, значит, принадлежит;

в) С (4; –2)               –2 = 42;

                                 –2 = 16 – неверно, значит, не принадлежит;

г) D (1,2; 1,44)          1,44 = (1,2)2;

                                 1,44 = 1,44 – верно, значит, принадлежит.

Ответ: а) да; б) да; в) нет; г) да.

3. № 489.

4. № 490. Решение:

а) А (–0,2; –0,008)    –0,008 = (–0,2)3;

                                 –0,008 = –0,008 – верно, значит, принадлежит;

б) B          ;

                                 ;

                                  – верно, значит, принадлежит;

в) C         ;

                                  – неверно, значит, не принадлежит.

Ответ: а) да; б) да; в) нет.

5. № 491.

6. № 492. Решение:

а) Р (а; 64)                64 = а2;

                                 82 = а2 – возможно в случае а = 8 или а = –8.

б) Р (а; 64)               64 = а3;

                                 43 = а3 возможно в случае а = 4.

Ответ: а) 8; –8; б) 4.

При решении этого упражнения учащиеся часто допускают ошибку: если 82 = а2, то а = 8, то есть забывают второй случай. Следует обратить внимание, что . Это тождество им уже знакомо.

IV. Итоги урока.

– Сформулируйте свойства функции y = x2. Как отражаются эти свойства на графике функции?

– Как называется график функции y = x2?

– Сформулируйте свойства графика функции y = x3. Как отражаются эти свойства на графике функции?

– Как называется график функции y = x3?

Домашнее задание: № 486; № 488; № 562; № 563.

 

 

 

Урок 56
Графическое решение уравнений вида
у = х2  и  у = х3

Цели: формировать понятие графического решения уравнения как нахождения абсциссы точек пересечения графиков двух функций; формировать умение решать графически уравнения вида у = х2 и у = х3.

Ход урока

I. Устная работа.

1. Заданы функции:

1) у = 2х;                            4) у = 3х + 2;                  7) у = ;

2) у = х;                 5) у = –3х + 2;                8) у = х2;

3) у = –3х;                 6) у = –3х – 2;                9) у = х3.

На рисунках а) – и) изображены графики этих функций. Заполните таблицу соответствия:

Формула

1

2

3

4

5

6

7

8

9

График

 

 

 

 

 

 

 

 

 

 

a)      б)      в) 

г)       д)       е) 

ж)   з)      и) 

2. Как называется функция вида y = kx?

3. Как называется функция вида y = kx + b?

4. Как называется график функции y = x2?

5. Как называется график функции вида y = x3?

II. Актуализация знаний.

Решить уравнение.

а) x2 = 16;                 б) x3 = 8;              в) x2 = ;

г) x3 = ;             д) x2 = 0;              е) x2 = –4.

III. Объяснение нового материала.

Необходимо разъяснить принцип графического решения уравнения.

Рассматриваем примеры 1, 2 со с. 109 учебника. Показываем, что равенство (аналитическое) x2 = x + 1 можно понимать как равенство значений двух функций y = x2 и y = x + 1. Графически, если графики этих функций пересекаются, то точка пересечения показывает значение х (абсцисса), при котором значения функций (ордината) равны.

Отсюда учащиеся могут сами вывести и сформулировать алгоритм графического решения уравнения:

1-й шаг. Преобразовать уравнение к равенству двух функций известного вида (y = kxy = kx + by = x2y = x3).

2-й шаг.  В  одной  системе  координат  построить  графики  этих функций.

3-й шаг. Определить  наличие  или  отсутствие  точки  (точек)  пересечения.

4-й шаг. Если точки пересечения есть, то найти по графику их абсциссы, которые и будут являться решениями уравнения. Если точек пересечения нет, то, значит, уравнение не имеет решений.

Подчеркиваем учащимся, что решение, полученное графически, может быть как точным, так и приближенным.

Проверить полученное значение можно, подставив в уравнение.

IV. Формирование умений и навыков.

1. № 493 (устно).

2. Решите графически уравнение.

а) x2 = 2x;                  б) x2 = x;                     в) x2 = –2x.

3. № 566.

В следующем упражнении от учащихся требуется сначала преобразовать уравнение к «удобному» виду, а затем решить его графически.

4. № 494.

Решение:

б) x2 + 2x – 3 = 0;

    x2 = –2x + 3.

Построим графики функций y = x2 и y = –2x + 3.

Ответ: х = –3; х = 1.

5. № 495 (устно).

6. № 496.

V. Итоги урока.

– В каком случае уравнение можно решить графически?

– Назовите алгоритм решения уравнения графическим способом.

– В каком случае уравнение не имеет корней?

– Как можно проверить точность корней уравнения, найденных графическим способом?

Домашнее задание:

1. Решите графически уравнение.

а) х = 3х;               б) 2x = x + 2; в) 3x = 3x + 4.

2. Решите графически уравнение.

а) x2 = 9;               б) x2 = ;                в) x2 = –3;             г) x3 = 8.

3. Решите уравнение графически.

а) x2 = 6 – x;          б) x2 + 4x = –3;          в) x2 – 4x = 0;       г) x3 + 2 = 3x.

 

 

Урок
Обобщающий урок по теме
«Степень с натуральным показателем».
Подготовка к контрольной работе

Цели: обобщить и систематизировать знания по теме «Степень с натуральным показателем»; оценить степень сформированности умений и навыков, провести коррекционную работу.

Ход урока

I. Устная работа.

1. Представьте в виде степени.

а) c7 ∙  c4;                               б) b ∙  b2 ∙  b3;               в) (–7)3 ∙  (–7)8 ∙  (–7)9;

г) a10 : a8;                     д) 214 : 29;                    е) (x5)2;

ж) (–a3)3;                     з) ;             и) (a2)5 ∙  a5.

2. Упростите.

а) (a5)2 ∙  (a2 ∙  a3)2;       в) (4xy)2;                     д) 94 : 37;

б) (y4)5 : (y4)2;               г) 20a3 ∙  (5a)2;            е) 1012 : (24 ∙  54).

3. Выполняя задания, ученик допустил ошибки. Какие свойства, правила не знает ученик?

35 ∙  38 = 340;                 81 = 1;                         24 + 22 = 26;

(2a)5 = 2a5;                  (x2)3 = x8.

4. Представьте в виде степени.

(–3)8 ∙  (–3)4;                (0,1)20 : (0,1)6;             (xn)2.

5. Найдите значение выражения.

(1014 ∙  107) : 1019;        53 ∙  23.

6. Представьте произведение в виде степени.

x5y5;                             36a2b2;                        a3b3c3.

II. Теоретический опрос.

1) Сформулируйте определение степени с натуральным показателем.

2) Каким числом является:

а) степень положительного числа;

б) степень отрицательного числа с четным показателем;

в) степень отрицательного числа с нечетным показателем?

3) Сформулируйте  правило  умножения  степеней  с  одинаковыми  показателями.

4) Сформулируйте  правило  деления  степеней  с  одинаковыми  показателями.

5) Дайте определение степени числа с нулевым показателем.

6) Сформулируйте правило возведения степени в степень.

7) Сформулируйте правило возведения в степень произведения.

III. Математический диктант.

Вариант 1

1. Упростите.

а) x2 ∙  x8 : x;              б) a10 : a6 ∙  a4.

2. Найдите значение выражения.

94 : 37.

3. Представьте в виде квадрата одночлена.

0,25х4;                      49т2п6.

4. Выполните умножение.

x2y3 ∙  16yx.

5. Вычислите.

(516 · 316) : 1515.

Вариант 2

1. Упростите.

а) b3 ∙  b7 : b;             б) y12 : y5 ∙  y2.

2. Найдите значение выражения.

44 : 26.

3. Представьте в виде квадрата одночлена.

0,36у6;                      100с2а6.

4. Выполните умножение.

a3b4 ∙  12ab2.

5. Вычислите.

(310 · 710) : 219.

IV. Работа по карточкам.

Карточка № 1

1. Вычислите.

(494 · 75) : 712.

2. Упростите выражения.

а) ;              б) am + 1 · a · a3 – m.

Карточка № 2

1. Вычислите.

(56 · 125) : 254.

2. Упростите выражения.

а) ;              б) x2n : (xn – 1)2.

V. Итоги урока.

Домашнее задание: 1. Повторить п. 18–23.

2. Ответьте на вопросы теста:

1) Выполните умножение:  0,5х2у · (–ху) =

а) –0,5х3у2;                б) 0,5у2х3;                      в) –0,5х2у3.

2) Упростите:  –0,4x4y3 · 2,5x2y7 =

а) x8y6;                       б) –10x6y7;                     в) –x6y7.

3) Преобразуйте выражение в одночлен стандартного вида:

20а3 · (–5а)2 =

а) 100а5;                   б) –500а6;                      в) 500а5.

4) Вычислите:  (25 · (23)4) : 213 =

а) 23;                         б) 16;                             в) 32.

 

Урок 58
Контрольная работа № 4

Вариант 1

1. Найдите значение выражения 1 – 5х2 при  х = –4.

2. Выполните действия.

а) y7 ∙  y12;                  б) y20 : y5;             в) (y2)8;                 г) (2y)4.

3. Упростите выражение.

а) –2ab3 ∙  3a2 ∙  b4;    б) (–2a5b2)3.

4. Постройте график функции y = x2. С помощью графика определите значение у при х = 1,5; х = –1,5.

5. Вычислите: .

6. Упростите выражение.

а) ;                      б) xn – 2 ∙  x3 – n ∙  x.

Вариант 2

1. Найдите значение выражения –9р3 при p = .

2. Выполните действия.

а) c3 ∙  c22;                  б) c18 : c6;             в) (c4)6;                 г) (3c)5.

3. Упростите выражение.

а) –4x5y2 ∙  3xy4;                  б) (3x2y3)2.

4. Постройте график функции y = x2. С помощью графика определите, при каких значения х значение у равно 4.

5. Вычислите: .

6. Упростите выражение.

а) ;                      б) (an + 1)2 : a2n.

Вариант 3

1. Найдите значение выражения –3х2 + 7 при  х = –5.

2. Выполните действия.

а) a8 ∙  a16;                 б) a16 : a4;             в) (a3)5;                 г) (2a)3.

3. Упростите выражение.

а) 3a2b ∙  (–2a3b4);     б) (–3a3b2)3.

4. Постройте график функции y = x2. С помощью графика определите значение у при х = 2,5; х = –2,5.

5. Вычислите: .

6. Упростите выражение.

а) ;                       б) am + 1 ∙  a ∙  a3 – m.

Вариант 4

1. Найдите значение выражения –12с3 при c = .

2. Выполните действия.

а) x7 ∙  x12;                  б) x12 : x3;             в) (x6)3;                 г) (3x)4.

3. Упростите выражение.

а) 5x4y ∙  (–3x2y3);      б) (–2xy4)4.

4. Постройте график функции y = x2. С помощью графика функции определите, при каких значения х значение у равно 9.

5. Вычислите: .

6. Упростите выражение.

а) ;                       б) x2n : (xn – 1)2.

Рекомендации по оцениванию.

Задания  1–4  обязательного  уровня.  Их  необходимо  решить  для  получения отметки «3». Для получения отметки «5» необходимо решить все 6 заданий.

В более слабом классе необходимо решить любые три задания для получения отметки «3» и любые пять для получения отметки «5».

Решение заданий контрольной работы

Вариант 1

1. Если х = –4, то 1 – 5х2 = 1 – 5 · (–4)2 = 1 – 5 · 16 = 1 – 80 = –79.

Ответ: –79.

2. а) y7 ∙  y12 = y7 + 12 = y19;                      б) y20 : y5 = y20 – 5 = y15;

    в) (y2)8 = y2 ∙  8 = y16;                            г) (2y)4 = 24y4 = 16y4.

Ответ: а) у19;   б) у15;   в) у16;   г) 16у4.

3. а) –2ab3 ∙  3a2 ∙  b4 = (–2 ∙  3)(aa2)(b3b4) = –6a3b7;

    б) (–2a5b2)3 = (–2)3(a5)3(b2)3 = –8a15b6.

Ответ: а) –6a3b7;   б) –8a15b6.

4.

Ответ: 2,25;   2,25.

5.  = 52 = 25.

Ответ: 25.

6. а)

;

    б) xn – 2 ∙  x3 – n ∙  x = xn – 2 + (3 – n) + 1 = xn – 2 + 3 – n + 1 = x2.

Ответ: а) 13,5x6y20;   б) х2.

Вариант 2

1. Если p = , то .

Ответ: .

2. а) c3 ∙  c22 = c3 + 22 = c25;                      в) (c4)6 = c4 ∙  6 = c24;

    б) c18 : c6 = c18 – 6 = c12;                       г) (3c)5 = 35c5 = 243c5.

Ответ: а) с25;   б) с12;   в) с24;   г) 243с5.

3. а) ;

    б) .

Ответ: а) –12x6y6;   б) 9x4y6.

4.

Ответ: –2; 2.

5.  = 3.

Ответ: 3.

6. а)

;

    б) .

Ответ: а) ;   б) а2.

Вариант 3

1. Если х = –5, то –3х2 + 7 = –3 · (–5)2 + 7 = –3 · 25 + 7 = –75 + 7 = –68.

Ответ: –68.

2. а) a8 ∙  a16 = a8 + 16 = a24;                     б) a16 : a4 = a16 – 4 = a12;

    в) (a3)5 = a3 ∙  5 = a15;                           г) (2a)3 = 23a3 = 8a3.

Ответ: а) а24;   б) а12;   в) а15;   г) 8а3.

3. а) ;

    б) .

Ответ: а) –6a5b5;   б) –27a9b6.

4.

Ответ: 6,25;   6,25.

5.  = 7.

Ответ: 7.

6. а)

;

    б) am + 1 ∙  a ∙  a3 – m = am + 1 + 1 + 3 – m = a5 .

Ответ: а) –7,2a23b8;   б) а5.

Вариант 4

1. Если c = , то  = 1,5.

Ответ: 1,5.

2. а) x7 ∙  x12 = x7 + 12 = x19;                      б) x12 : x3 = x12 – 3 = x9;

    в) (x6)3 = x6 ∙  3 = x18;                            г) (3x)4 = 34x4 = 81x4.

Ответ: а) х19;   б) х9;   в) х18;   г) 81х4.

3. а) ;

    б) .

Ответ: а) –15x6y4;   б) 16x4y16.

4.

Ответ: –3;   3.

5.  = 5.

Ответ: 5.

6. а)

;

    б) x2n : (xn – 1)2 = x2n : x2 (n – 1) = x2n : x2n – 2 = x2n – (2n – 2) = x2n – 2n + 2 = x2.

Ответ: а) 125a20b9;   б) х2.

 

Урок 59
Анализ результатов контрольной работы

Цели: проанализировать  результаты  контрольной  работы,  выявить типичные  ошибки,  допущенные  учащимися;  провести  коррекционную работу.

Ход урока

I. Анализ результатов контрольной работы.

II. Работа над ошибками.

Учащиеся в тетрадях выполняют работу над ошибками. Примеры, вызвавшие наибольшие затруднения, выносятся на доску.

III. Решение заданий повышенной трудности.

Учащиеся, получившие отметки «5» и «4», могут приступить к выполнению дополнительных заданий и заданий повышенной трудности.

1. № 549*, 550*, 551*, 552*.

2. № 560, 561, 564*, 565*, 566.

IV. Итоги урока.

– Какие типичные ошибки допущены во время выполнения контрольной работы?

– Какие основные понятия и правила необходимо знать для выполнения контрольной работы?

Домашнее задание: закончить решение упражнений из 4-го блока.

 

 

Урок 60
О простых и составных числах

Цели: рассмотреть разложение любого числа на простые множители для нахождения НОД и НОК двух и более чисел; научиться представлять любое число в виде произведения степеней простых чисел.

Ход урока

I. Актуализация знаний.

1. Выберите из чисел простые. Объясните свой выбор.

а) 11;               б) ;                    в) 1;                     г) 27;

д) 3;                 е) 29;                    ж) ;                з) 0;

и) ;             к) 37;                   л) 43;                   м) 121.

2. Найдите НОД (наибольший общий делитель) чисел.

а) 11 и 121;     б) 8 и 20;             в) 37 и 45;            г) 35 и 65.

3. Найдите НОК (наименьшее общее кратное) чисел.

а) 15 и 45;       б) 17 и 21;            в) 1 и 20;              г) 60 и 90.

II. Изучение нового материала.

1. Напоминаем определения простого и составного числа. Рассматриваем возможное доказательство утверждения, что простых чисел – бесконечное множество.

2. Рассматриваем разложение любого составного числа на простые множители. Удобен способ разложения «в столбик». Берем простые делители не наугад, а начиная с самого маленького – 2, затем, если необходимо, – 3, 5, 7, 11 и т. д.

Например:

360

180

90

45

15

5

1

2

2

2

3

3

5

504

252

126

63

21

7

1

2

2

2

3

3

7

Запишем числа в виде произведения:

360 = 2 · 2 · 2 · 3 · 3 · 5 = 23 · 32 · 5;

504 = 2 · 2 · 2 · 3 · 3 · 7 = 23 · 32 · 7.

3. Формулируем правила.

 Чтобы найти  наибольший общий делитель (НОД)  двух чисел, надо:

1) разложить каждое число на простые множители;

2) записать числа в виде произведения степеней простых чисел;

3) взять каждый из множителей в степени с наименьшим показателем, с каким он входит в эти числа.

Пример со с. 112–113 учебника.

 Чтобы найти наименьшее общее кратное (НОК) двух чисел, поступают аналогично, но берут степени с наибольшим показателем.

III. Закрепление изученного материала.

№ 500.

Решение:

а2 + а должно быть кратно 17.

При  а = 16:  а2 + а = 162 + 16 = 16 (16 + 1) = 16 · 17,  значит,  делится
на 17.

Ответ: а = 16.

№ 502*.

Решение:

10х + у – двузначное число, где х и у – числа 1, 2, 3,… , 9.

Если  х = 7  (наибольшее  простое  из  перечня),  то удобно взять  у = 7: 10 · 7 + 7 = 7 (10 + 1) = 7 · 11 – произведение двух простых чисел. Значит, это число 77.

Ответ: 77.

№ 506*.

Решение:

а = 1 · 2 · 3 · 4 · 5 · 6 · 7 · 8 · 9 · 10 = 1 · 2 · 3 · 2 · 2 · 5 · 2 · 3 · 7 ×
× 2 · 2 · 2 · 3 · 3 · 2 · 5 = 28 · 34 · 52 · 7.

Ответ: а = 28 · 34 · 52 · 7.

№ 507.

Решение:

а)    765

153

51

17

1

5       765 = 32 · 5 · 17

3

3

17

2315

63

21

7

1

5       315 = 32 · 5 · 7.

3

3

7

НОД (765; 315) = 32 · 5 = 45.

б)    792

396

198

99

33

11

1

2       792 = 23 · 32 · 11

2

2

3

3

11

1936

968

484

242

121

11

1

2       1936 = 24 · 112.

2

2

2

11

11

 

НОД (792; 1936) = 23 · 11 = 88.

Ответ: а) 45; б) 88.

№ 508.

Решение:

а)    294

147

49

7

1

2       294 = 2 · 3 · 72

3

7

7

756

378

189

63

21

7

1

2       756 = 22 · 33 · 7.

2

3

3

3

7

НОК (294; 756) = 22 · 33 · 72 = 4 · 27 · 49 = 5292;

б)    693

231

77

11

1

3       693 = 32 · 7 · 11

3

7

11

1617

539

77

11

1

3      1617 = 3 · 72 · 11.

7

7

11

НОК (693; 1617) = 32 · 72 · 11 = 4851.

Ответ: а) 5292; б) 4851.

№ 510*.

Решение:

НОК (15; а) = 90.

15

5

1

3       15 = 3 · 5

5

90

30

10

5

1

3       90 = 2 · 32 · 5.

3

2

5

Очевидно, что в разложение искомого числа на простые множители должны входить двойка, две тройки и не более одной пятерки, значит, это число либо 2 · 32 = 18, либо 2 · 32 · 5 = 90.

Ответ: 18 или 90.

IV. Итоги урока.

 

 

 

 

 

 

 

 

 

 

Урок 25
Понятие функции.
(Область определения. таблицы)

Цели: ввести понятие функциональной зависимости; дать определения независимой переменной (аргумента), зависимой переменной, области определения функции, области значений функции.

Ход урока

I. Устная работа.

1. Найдите значение выражения.

а) 3x – (2 + 3x)  при х = 7,862;

б) 2a – (a – 0,3)  при а = 0,7;

в)   при у = 7,62;

г) 0,5(2 + a)  при а = 0,3.

2. Решите уравнение.

а) 3х = –9;                 б) ;                              в) 5а – 15 = 0;

г) 3х = 3х + 11;         д) (x – 8);              е) 3y +  = 0.

II. Объяснение нового материала.

1. Основная задача первого занятия: показать, что функция – это математическая модель, позволяющая описывать и изучать разнообразные зависимости между реальными величинами.

Функция имеет общекультурное, мировоззренческое значение. При её изучении учащиеся знакомятся с идеей всеобщей связи, идеей непрерывности, бесконечности, интерполяции.

2. Объяснение проводить согласно пункту 12 учебника. Необходимо привести достаточно примеров функциональной зависимости (учебник, с. 51–53). Также нужно не только показывать зависимости, но и сразу обсуждать, в какой области человеческой деятельности применяются такие функциональные зависимости.

3. Вводим понятия независимой и зависимой переменных и определение функции как зависимости одной переменной от другой. На примерах показываем, что область определения функции может быть бесконечным и конечным множеством чисел.

III. Формирование умений и навыков.

Все задания, решаемые на этом уроке, направлены на усвоение как самого понятия функции, так и различных способов её задания (словесный, с помощью формулы, табличный, графический). Ученики должны уметь переходить от одного вида задания к другому и находить значения функции при каждом способе задания.

1. № 258, № 260.

2. Функция задана формулой у = 2 – 5х, верны ли равенства:

а) у = 12 при х = –2;                    б) у = 3 при х = ;

в) у = 20 при х = 4;                      г) у = –0,5 при х = ?

3. № 261.

4. Функция задана графиком:

а) Найти значения функции при х = 0; 2; 3,5; –1.

б) При каком значении х значение функции равно 1; 2; 0?

в) Назвать несколько значений х, при которых значение функции положительно.

г) Назвать несколько значений х, при которых значение функции отрицательно.

5. Устно.

Результаты измерений температуры воздуха за сутки даны в следующей таблице:

Время,

ч

0

2

4

6

8

10

12

14

16

18

20

22

24

Температура, °С

–1

+1

–3

–4

2

5

8

10

11

9

6

3

1

а) Назовите температуру в 6 ч, 8 ч, 24 ч.

б) В какое время температура была равна +1°,  –4°,  11°?

в) Почему эту зависимость можно назвать функцией?

6. № 263.

Решение:

Если r – остаток от деления натурального числа п на 4, то можно записать n = 4 · x + r, где 0 ≤ r < 4.

Найдем соответствующие значения r:

а) Если п = 13,          то 13 = 3 · 4 + 1,            то есть r = 1;

б) если п = 34,          то 34 = 8 · 4 + 2,            то есть r = 2;

в) если п = 43,          то 43 = 10 · 4 + 3,          то есть r = 3;

г) если п = 100,        то 100 = 25 · 4 + 0,        то есть r = 0.

В рассматриваемой функциональной зависимости аргументом является переменная п.

Областью определения является множество чисел {13; 34; 43; 100}.

Значениями функции служат числа 0; 1; 2; 3.

IV. Итоги урока.

– Что называется функцией?

– Что называется аргументом?

– Приведите пример функциональной зависимости одной переменной от другой. Укажите независимую и зависимую переменную.

– Какими способами можно задать функцию? Назовите преимущества каждого из них.

Домашнее задание: 1. № 259; № 262; № 264.

2. Функция задана графиком:

а) Найти значения функции при значениях аргумента 0; –2; 1; 3.

б) При каком значении х значение функции равно 2; 0; 1; –1?

в) Назвать несколько значений х, при которых значение функции положительно.

г) Назвать несколько значений х, при которых значение функции отрицательно.

 

 

 

 

 

Урок 26
чТО ТАКОЕ ФУНКЦИЯ

(Аналитический способ задания функции)

Цели: продолжить работу по усвоению понятия функции и связанных с функцией понятий (область определения функции, область значений функции и др.); формировать умение находить значения функций, заданных аналитически (с помощью формулы).

Ход урока

I. Устная работа.

1. Задайте формулой функцию, сопоставляющую каждому числу третью степень этого числа; сумму этого числа с числом 5.

2. Велосипедист едет со скоростью 15 км/ч и за t ч проходит расстояние s км (зависимость s от t). Найдите значение функции, соответствующее значению аргумента, равному ; 2; 2.

II. Объяснение нового материала.

Цель этого и последующих занятий – в упорядочении имеющихся представлений о функции, развертывании системы понятий, характерных для функциональной линии. Значительное место должно быть отведено усвоению важного представления – однозначности соответствия аргумента и определенного по нему значения функции. Для рассмотрения этого вопроса привлекаются различные способы задания функции.

Чаще других в математике и её приложениях применяется задание функции формулой. Все другие способы играют подчиненную роль. Однако сопоставление разных способов задания выполняет важную роль:

1) и  таблицы,  и  графики  служат  для  удобного  в  определенных обстоятельствах представления функции, имеющей аналитическую форму записи;

2) необходимо для усвоения всего многообразия аспектов понятия функции.

Объяснение проводить согласно пункту 13 учебника. Разбираем пример  № 1  со  с. 55  учебника.  Показываем,  что  для  того,  чтобы  найти значение функции, необходимо подставить некоторое значение аргумента в формулу.

Также объясняем, что в случае, когда область определения функции явно не задана, считают, что она состоит из всех значений независимой переменной, при которых эта формула имеет смысл.

III. Формирование умений и навыков.

Задания, выполняемые на этом уроке, можно разбить на группы:

1-я группа. Нахождение значения функции по формуле при заданном значении аргумента.

2-я группа. Составление таблицы значений некоторой функции.

3-я группа. Нахождение области определения функции.

1-я группа.

1. № 267.

2. Вычислить  значение  следующих  функций  при  х,  равном  –2; –1; 0; 1; 2.

а) у = 3х;          б) у = –2х;            в) у = –х – 3;            г) у = 20х + 4.

2-я группа.

1. № 270.

2. № 271.

Решение:

у = х (х – 3,5)

х

0

0,5

1

1,5

2

2,5

3

3,5

4

у

0

–1,5

–2,5

–3

–3

–2,5

–1,5

0

2

3-я группа.

1. Найдите область определения функции, заданной формулой:

а) у = 3х + 2;             б) у = ;                  в) у = x7 + 2x – 3;

г) у = ;            д) у = ;                  е) у = .

2. № 351.

Решение:

а) у = .

Область  определения  функции  –  все  числа,  кроме  тех,  при  которых х – 4 = 0, то есть х2 = 4. Значит, не входят в ООФ х = 2 и х = –2.

б) у = .

Область  определения  функции  –  все  числа,  кроме  тех,  при  которых х2 + 4 = 0, то есть х2 = –4.  Уравнение  не  имеет  решения,  значит, ООФ – любое число.

Ответ: а) любое число, кроме 2 и –2; б) любое число.

3. Дополнительные задания (для сильных учащихся).

3.1. Найдите область определения функции.

а) у =

б) у =

3.2. Задайте формулой какую-нибудь функцию, область определения которой:

а) все действительные числа;

б) все действительные числа, кроме –11;

в) все действительные числа, кроме 3 и 5;

г) все неотрицательные действительные числа;

д) все неположительные действительные числа.;

е) только одно число.

IV. Проверочная работа.

Вариант 1

1. Дана  функция  у = 2х2 – 4х.  Найдите  значение  функции  при  х = 0
и х = –1.

2. Найдите область определения функции.

а) у = 2х – 7;                       б) у = ;

в)* у =

Вариант 2

1. Дана  функция  у = 5х2 + х.  Найдите  значение  функции  при  х = 0
и х = 1.

2. Найдите область определения функции.

а) у = 3х + 6;                      б) у = ;

в)* у =

V. Итоги урока.

– В каком смысле употребляется термин «функция»?

– Что называется областью определения функции? Как найти ООФ?

– Какими способами можно задать функцию?

– Каким образом находится значение функции, заданной формулой?

Домашнее задание: 1. № 268; № 269; № 272.

2. Составьте  таблицу  значений  функции,  заданной  формулой 
у = 3х2 – 2х + 1, где –1 ≤ х ≤ 0, с шагом 0,1.

 

 

 

 

 

 

 

 

 

 

 

 

 

 

 

 

 

 

 

 

 

 

 

 

 

 

Урок                                                   Дата
Тема: ВЫЧИСЛЕНИЕ   значениЙ функции ПО ФОРМУЛЕ

Цели: продолжить формировать умение находить значение функции по формуле, а также формировать умение находить значение аргумента, соответствующее заданному значению функцию, умение решать практические задачи с использованием функциональной терминологии.

Ход урока

I. Устная работа.

1. Найдите значение функции у = 2х – 1 для значений аргумента, равного 0; 1; 2; –1.

2. Найдите область определения функции:

а) у = 3х – 7;                б) у = ;                                            в) у = x3 – 2x2 – 1;

г) у = ;                            д) у = ;                 е) у = ;

ж) у = ;                           з) у = .

II. Формирование умений и навыков.

1. На  данном  уроке  учащиеся  продолжают  выполнять  задания,  на-правленные на усвоение понятия функции и связанных с функцией понятий (область определения функции, область значения функции и др.). Также  учащиеся  учатся  выполнять  действие,  обратное  нахождению  значения функции по формуле, а именно, нахождение соответствующего аргумента.

Эти задания составляют первую группу. Вторая группа – задания на составление формулы функциональной зависимости по условию практической текстовой задачи. При решении таких задач формируются межпредметные связи.

Также следует указывать учащимся на используемые внутрипредметные связи: при нахождении значения функции работа сводилась к нахождению значения выражения с переменной; при нахождении значения аргумента следовало решать некоторое уравнение (см. пример 2 со с. 56 учебника).

2. Затем приступаем к выполнению упражнений.

1-я группа

1. № 273, № 274.

2. Функция задана формулой у = 2х – 1.

а) Какое значение у соответствует х, равному 10; –4,5; 15; 251; 600?

б) При каком значении х соответствующее значение у равно: –19; –57; 205; –3?

Решение:

а) Если х = 10,                            то    у = 2 · 10 – 1 = 19;

    если х = –4,5,          то    у = 2 · (–4,5) – 1 = –10;

    если х = 15,                             то    у = 2 · 15 – 1 = 29;

    если х = 251,           то    у = 2 · 251 – 1 = 501;

    если х = 600,           то    у = 2 · 600 – 1 = 1199.

б) Если у = –19,         то    2х – 1 = –19;

                                                               2х = –19 + 1;

                                                               2х = –18;

                                                               х = –9; то есть у = –19, при х = –9.

    Если у = –57,          то    2х – 1 = –57;

                                                               2х = –57 + 1;

                                                               2х = –56;

                                                               х = – 28, то есть у = –57 при х = – 28.

    Если у = 205,          то    2х – 1 = 205;

                                                               2х = 205 + 1;

                                                               2х = 206;

                                                               х = 103, то есть у = 205 при х = 103.

    Если у = –3,      то    2х – 1 = –3;

                                                               2х = –3,5 + 1;

                                                               2х = –2,5;

                                                               х = –1,25, то есть у = –3 при х = –1.

2-я группа

1. Из формулы равномерного движения s = хt выразить скорость х как функцию пути s и времени t. Вычислить по этой формуле среднюю скорость полета пули, если s = 3 км, t = 6 с.

2. № 276.

Решение:

Обозначим  за  т  массу  пробки  в  граммах,  а  за  V – объем  в  см3. Тогда зависимость массы куска пробки от объема можно выразить формулой т = 0,18 · V.

а) Если V = 240,         то    т = 0,18 · 240 = 43,2 (г);

б) если т = 64,8,        то    0,18 · V = 64,18;

                                                              V = 64,18 : 0,18;

                                                              V = 360 (см3).

Ответ: а) 43,2 г; б) 360 см3.

3. № 278.

Решение:

Анализ условия:

s = 12 · t.

а) Если t = 3,5,                           то   s = 12 · 3,5 = 42 (км);

б) если s = 30,                            то   12 · t = 30;

                                                              t = 30 : 12;

                                                              t = 2,5 (ч).

Ответ: а) 42 км; б) 2,5 ч.

4. № 352.

Решение:

Анализ условия:

у = 1,5х + 150.

а) если х = 10,                            то   у = 1,5 · 10 + 150 = 15 + 150 = 165;

б) если у = 180,          то   1,5х + 150 = 180;

                                                              1,5х = 180 – 150;

                                                              1,5х = 30;

                                                              х = 30 : 1,5;

                                                              х = 20, значит, у = 180 при х = 20.

Ответ: а) у = 165; б) х = 20.

III. Проверочная работа.

Вариант 1

1. Функция задана формулой у = 3х – 7. Найдите значение аргумента, при котором значение функции равно нулю.

2. Найдите значение аргумента, при котором функция у = –3х – 2 принимает значение 0,3.

3. Запишите  область  определения  функции,  заданной  формулой
у = .

Вариант 2

1. Функция задана формулой у = 5 + 2х. Найдите значение аргумента, при котором значение функции равно нулю.

2. Найдите значение аргумента, при котором функция у = –5х + 11 принимает значение 0,2.

3. Запишите  область  определения  функции,  заданной  формулой
у = .

IV. Итоги урока.

– Дайте определение функции. Что называется аргументом, значением функции?

– Объясните на примере функции, заданной формулой у = 3х + 18:

а) как  по  значению  аргумента  найти  соответствующее  значение функции;

б) как найти значения аргумента, которым соответствует указанное значение функции.

Домашнее задание: № 275; № 277; № 279; № 353.

 

 

 

 

 

 

 

 

 

 

 

 

 

 

 

 

 

 

 

 

 

 

Урок                                                    Дата:
Тема: График функции. графики реальных процессов

Цели: формировать понятие «график функции», умение строить график  функции,  заданной  аналитически,  а  также  с  помощью  графика  находить  значение  функции,  соответствующее  заданному  значению  аргумента, и значения аргумента, которым соответствует данное значение функции.

Ход урока

I. Проверочная работа.

Вариант 1

1. Найдите значения функции, заданной формулой у =  для значений аргумента, равных –6; 1,5.

2. Найдите значение аргумента, при котором функция у = 4х + 3 принимает значение, равное .

3*. Найдите значения переменной b, соответствующие значениям переменной а, равным –5; 0, если b = | a | – 4.

Вариант 2

1. Найдите значения функции, заданной формулой у =  – 6 для значений аргумента, равных –8; 0,8.

2. Найдите значение аргумента, при котором функция у = 5х + 4 принимает значение, равное 1,5.

3*. Найдите значения переменной u, соответствующие значениям переменной v, равным –25; 0, если u = | v | – 8.

II. Устная работа.

На рисунке изображен график зависимости некоторой величины у от некоторой величины х.

Ответьте на вопросы:

а) Чему равное значение у, если х = –3; –1; 2; 5?

б) Чему равны значения х, если у = 3; 0; 1?

в) Какое минимальное и какое максимальное значения принимает величина у?

III. Объяснение нового материала.

1. На предыдущих занятиях учащиеся уже познакомились с основными способами задания функции. Особое внимание было уделено связи аналитического и табличного способов. На этом уроке наша задача – показать, что эти два способа тесно связаны с графическим, причем его особенность в том, что с помощью графика мы можем наглядно представлять функциональную зависимость не только для точечной, но и бесконечной области определения функции:

задание функциональной зависимости

В соответствии с этими положениями объяснение нового материала проводится в несколько этапов:

1) Формирование представления о графике функции на основе связи аналитического, табличного и графического способов задания функции.

2) введение определения понятия графика функции.

3) Построение графика функции по точкам.

4) Работа по изображенному графику функции.

2. На первом этапе предлагаем учащимся такое задание.

На рисунке изображены точки на координатной плоскости, выражающие результаты наблюдений за атмосферным давлением. Построить график зависимости давления от времени в промежутке 12 ≤ t ≤ 18, соединив эти точки плавной линией.

Затем рассматриваем пример со с. 58 учебника, в котором показано, как по точкам строится график функции y = , где –2 ≤ х ≤ 3.

Необходимо  сделать  выводпо  точкам  можно  построить  график любой  функции,  заданной  таблично  или  аналитически  (с  помощью формулы).

Вводим определение:

Графиком функции называется множество всех точек координатной плоскости, абсциссы которых равны значениям аргумента, а ординаты – соответствующим значениям функции.

На примере 2 со с. 60 учебника показываем работу по изображенному графику на нахождение значения функции по заданному значению аргумента и обратное задание.

IV. Формирование умений и навыков.

1. № 283.

Можно задать учащимся дополнительные вопросы:

а) Сколько точек пересечения с осью х имеет график? Каково значение у в этих точках?

б) Сколько точек пересечения с осью у имеет график? Каково значение х в этой точке?

в) Сравните значения функции в точках –2 и 1.

г) Назовите координаты какой-нибудь точки графика, у которой значения аргумента и функции положительны; значение аргумента положительно, а функции – отрицательно и т. д.

2. № 284, № 285.

3. Используя график функции, заполните таблицу значений функции для –2 ≤ х ≤ 3 с шагом 0,5.

4. № 354*.

V. Итоги урока.

– Что называется графиком функции?

– Как построить график функции, заданной формулой?

– Как по графику найти значение функции, соответствующее данному значению аргумента?

– Как по графику функции найти значение аргумента, которому соответствует данное значение функции?

– Как по графику зависимости определить, является ли она функцией?Домашнее задание: 1. № 286; № 287; № 288. 2. Постройте  график  функции,  заданной  формулой  y = ,  где
1 ≤ x ≤ 6, предварительно заполнив таблицу с шагом 1.

 

 

Урок                                                                       Дата:
Тема:Решение задач по теме «График функции»

Цели: продолжить формировать умения строить график функции и находить значение функции по заданному аргументу с помощью графика; формировать умение интерпретировать в несложных случаях графики реальных зависимостей между величинами, отвечая на поставленные вопросы практической задачи.

Ход урока

I. Устная работа.

1. Какие из графиков, изображенных на рисунках, являются графиками функций?

а)                             б) 

в)                          г) 

д)               е) 

2. По графику, изображенному на рисунке д), найдите:

а) значение функции, соответствующее значению аргумента, равному –3; –2; 1; 2;

б) значения аргумента, при которых значение функции равно –1; 2; 3;

в) координаты точек пересечения с осью х;

г) координаты точек пересечения с осью у.

II. Формирование умений и навыков.

1. № 289.

2. № 291, № 293.

3. На рисунке изображен график зависимости потребления районом электрической энергии р (%) от времени суток t (ч).

а) В какое время суток электрическая нагрузка была максимальной?

б) В какое время суток нагрузка не превосходила 20 % от максимума?

в) Какова была нагрузка в 18 ч?

г) Какое событие может отражать участок графика АВ?

д) Возрастала или убывала нагрузка с 4 до 8 ч; с 18 до 20 ч?

4. В таблице представлено население (млрд) земного шара в различные годы.

Год, t

1900

1940

1950

1970

1990

2000

Население, Н

1,63

2,25

2,53

3,64

5,3

6,1

По этим данным постройте график. Оцените приближенно по графику население земли в 1981, 1987, 2010 гг.

5. (Криминальная история.)  В  11 ч  вечера  слуга  зажег  хозяину  две свечи,  а  утром  в 7 ч  обнаружил  его  убитым.  Одна  свеча лежала на полу потухшая, а вторая догорала. В какое время произошло убийство, если длина целой свечи 21 см, опрокинутой во время убийства 16 см, а непотухшего огарка 1 см? Постройте график зависимости длины горения свечи от времени.

III. Итоги урока.

– Как по графику найти значение функции, соответствующее данному значению аргумента?

– Как по графику найти значения аргумента, которым соответствует данное значение функции?

– Как, не строя график, выявить принадлежность ему точки с данными координатами?

– Как, не строя график, определить, в каких точках он пересекает ось абсцисс; ось ординат?

Домашнее задание: № 290; № 292; № 355; № 356*.

 

 

 

 

 

 

 

 

 

 

 

 

Урок                                   Дата
ТЕМА:Понятие прямой пропорциональности

Цели: ввести понятие прямой пропорциональности как функции определенного вида; формировать умение распознавать прямую пропорциональность и вычислять значение функции по формуле; повторить тему «Построение точек в координатной плоскости» для последующего изучения графика прямой пропорциональности.

Ход урока

I. Устная работа.

1. Найдите  значение  функции  у =  для  следующих  значений аргумента:

а) 0;                       б) 4;                            в) –4;                          г) –2.

2. Проверьте,  принадлежат  ли  графику  функции,  заданной  формулой у = 2х + 14, следующие точки:

а) А (0; 14);          б) В (–2; 8);                в) С (–7; 0);                г) D (7; 0).

3. Решите уравнение.

а) 3х = 12; б) –2х + 14 = 0;          в) х – 15 = 2;  г) х + 2 = х.

II. Объяснение нового материала.

1. Введение понятия основывается на рассмотрении конкретных практических примеров. Желательно их привести несколько. Так, следует рассмотреть пример со с. 65 учебника. Кроме того, представить уже знакомую учащимся задачу: «Вычислить площадь прямоугольника, основание которого равно 3, а высота равна х». Если искомую площадь обозначить буквой у, то ответ можно записать формулой:

у = 3х.

Если основание прямоугольника равно k, то зависимость между высотой х и площадью у выразится формулой

у = .

Каждое  заданное  значение  числа  k  определяет  некоторую функцию у = .

Затем формулируем четкое определение:

Прямой пропорциональностью называется функция, которую можно задать формулой вида у = , где х – независимая переменная, k – не равное нулю число.

2. Просим учащихся привести примеры прямой пропорциональности и примеры функций, не являющихся прямой пропорциональностью. Также рассматриваем примеры со с. 66 учебника. Показываем, что число k называется коэффициентом прямой пропорциональности, а само название функции связано со следующей пропорцией:

у = .

III. Формирование умений и навыков.

Задания, решаемые на этом уроке, можно разбить на 2 блока.

В первом блоке представлены упражнения на усвоение понятия прямой  пропорциональности  и  выполнение  основных  действий  по формуле.

Второй блок носит  повторительный  характер  и  направлен  на актуализацию  знаний  по  теме  «Построение  точек  в  координатной плоскости».

1-й блок

1. № 297, № 298 (устно).

2. Книга  стоит  150  рублей.  Выразите  формулой  зависимость  между купленным  количеством  (п)  данных  книг  и  уплаченной  суммой  (у) в рублях.

3. Автомобиль «Лада» движется по шоссе со скоростью
80 км/ч. Записать формулу, выражающую зависимость длины пути s (в км) от времени движения t (в ч). Чему равно s (3), s (5,4)?

4. Зависимость между переменными х и у выражена формулой у = . Определить k, если у = –5 при х = 2,5.

5. Дана таблица значений функции у = :

х

0,5

 

1,4

2,1

3

 

у

 

1

4,2

 

 

9,6

Найти k и заполнить пропущенные клетки.

2-й блок

Самостоятельная работа

Вариант 1

1. Постройте систему координат. Отметьте в координатной плоскости точки:  (2,5; 1), (2,5; –1), (0,4; 3,5), (–0,4; 3,5).

2. Запишите координаты точек:

3. В каких координатных четвертях расположены точки:

А (–87; 98); В (0,1; –0,01); С (–1,25; –3,48)?

4. Постройте в координатной плоскости прямую проходящую через точки С (–4; 3) и D (3; –1). Найдите координаты точек, в которых эта прямая пересекает ось х и ось у.

Вариант 2

1. Постройте систему координат. Отметьте в координатной плоскости точки: (4; 3,5), (4; –3,5), (–5,3; –1,5), (5,3; –1,5).

2. Запишите координаты точек:

3. В каких координатных четвертях расположены точки:

А (25; 360); В (–2,5; –100); С ?

4. Постройте в координатной плоскости прямую, проходящую через точки А (3; 4) и В (–5; –1). Найдите координаты точек, в которых эта прямая пересекает ось х и ось у.

IV. Итоги урока.

– Сформулируйте определение прямой пропорциональности.

– Приведите примеры прямой пропорциональности.

– Как называется число k в записи формулы прямой пропорциональности у = ? Какое это число?

– Почему данная функция получила свое название?

Домашнее задание: 1. № 299.

2. Один килограмм конфет стоит 98 рублей. Записать правило, выражающее зависимость стоимости у (в р.) от массы конфет х (в кг).

3. Дана функция у = 4х. Заполнить таблицу:

х

–2

 

0

0,5

 

2

у

 

0

 

 

–2

 

4. № 310; № 311.

 

 

 

 

 

 

 

 

 

Урок                                   Дата
ТЕМА: График прямой пропорциональности

Цели: определить график прямой пропорциональности как прямую, проходящую через начало координат; выявить расположение прямой в зависимости от знака коэффициента пропорциональности; формировать умение строить график прямой пропорциональности по формуле и выполнять обратное действие – записывать по графику формулу функции.

Ход урока

I. Устная работа.

1. Найдите область определения функции.

а) y = 3x + 2;                    б) y = ;                 в) y = ;

г) y = ;                  д) y = x;              е) y = 2x2 + 6x + 1.

2. Является ли функция прямой пропорциональностью:

а) y = 182x;                      б) y = ;                 в) y = x;

г) y = –17x2;                     д) y = ;                 е) y = 3x + 11?

3. Функция задана формулой у = . Найдите коэффициент прямой пропорциональности k, если:

а) х = 2; у = 4;                  б) x = ; y = –4;

в) х = 3; у = ; г) х = 0; у = 0.

II. Объяснение нового материала.

При объяснении нового материала необходимо подчеркнуть, что у функций одного вида должны быть и графики одного вида. Следует выяснить, что представляет собой график прямой пропорциональности.

Начинаем с рассмотрения конкретной функции (см. учебник, с. 66). Можно предложить учащимся лабораторную работу: подобрать функции, заданные формулами:

у = 0,5х;                            у = –0,5х;

у = х;                                 у = –х;

у = 1,5х;                            у = –1,5х;

у = 2х;                               у = –2х;

у = 2,5х;                            у = –2,5х;

у = 3х;                               у = –3х;

у = 3,5х;                            у = –3,5х;

у = 4х;                               у = –4х.

Затем заполнить таблицу значений функции при –4 ≤ х ≤ 4 с шагом 0,5.

Учащиеся заполняют каждый свою таблицу и отмечают в координатной плоскости точки, координаты которых помещены в таблице.

Учитель проходит по рядам и следит, чтобы учащиеся не допустили ошибок.

После выполнения этого задания и обсуждения результатов ученики с учителем делают следующие выводы:

1) График прямой пропорциональности является прямой, проходящей через начало координат.

2) Если коэффициент пропорциональности k > 0, то график расположен в первой и третьей координатных четвертях.

3) Если коэффициент пропорциональности k < 0, то график расположен во второй и четвертой координатных четвертях.

На основе этих выводов учащиеся выводят простейший алгоритм построения графика прямой пропорциональности:

1-й шаг. Для х1 ¹ 0 вычислить у1 по формуле у = .

2-й шаг. Отметить  в  координатной  плоскости  точки  с  координатами (0; 0) и (х1; у1).

3-й шаг. Провести прямую через построенные точки.

III. Формирование умений и навыков.

Упражнения, выполняемые на этом уроке, направлены на отработку алгоритма построения графика прямой пропорциональности и нахождения значений функции по графику.

1. № 300, № 302.

№ 302.

Решение:

у = –0,5х

 

Пусть х = 3, тогда у = –0,5 · 3 = –1,5. Проведем прямую, проходящую через начало координат и точку с координатами (3; –1,5).

а) Если х = –2, то у = 1;                          б) у = –1 при х = 2;

    если х = 4, то у = –2;                              у = 0 при х = 0;

    если х = 1, то у = –0,5.                                       у = 2,5 при х = –5.

Если у = –150, то найдем х, решив уравнение:

–0,5х = –150;

х = –150 : (–0,5);

х = 300.

При выполнении этого задания повторяем с учащимися правило нахождения по графику значения функции по данному значению аргумента и наоборот (отмечаем точку на оси абсцисс; проводим прямую, перпендикулярную оси абсцисс, до пересечения с графиком функции; из полученной точки опускаем перпендикуляр на ось ординат и находим соответствующее числовое значение ординаты).

Также на этом примере показываем, что очень важен выбор правильной величины единичного отрезка. Если взять в качестве единицы измерения одну клеточку, то будет очень неудобно строить график, точки будут «слипаться», чертеж будет грязным и нефункциональным.

При больших значениях аргумента или функции (у = –150) удобнее работать с формулой и выполнять действия аналитически (решить уравнение; вычислить по формуле).

2. № 303 (устно).

Выполняем работу по предыдущему чертежу.

3. № 305, № 306.

№ 305.

Решение:

а) у = 1,7х;

б) у = –3,1х;

в) у = 0,9х;

г) у = –2,3х;

д) у = , где k > 0;

е) у = , где k < 0.

После выполнения этого задания обсудить с учащимися, почему график а) расположен в первой четверти выше графика в).

№ 306. Решение:

Все графики являются прямыми, проходящими через начало координат,  значит,  функции  являются  прямыми  пропорциональностями  и  их можно  задать  формулой  у = .  Задача  сводится  к  нахождению  коэффициента k.

Выберем  на  каждом  графике  произвольную  точку  с  целыми  координатами:

I    (2; 6), значит, 6 = k · 2; k = 3; у = 3х;

II   (4; 1), значит, 1 = k · 4; k = 0,25; у = 0,25х;

III (2; –2), значит, –2 = k · 2; k = –1; у = –х;

IV (2; –6), значит, –6 = k · 2; k = –3; у = –3х.

Ответ: у = 3х; у = 0,25х; у = –х; у = –3х.

IV. Проверочная работа.

Вариант 1

1. График функции у = проходит через точку В (–30; 3). Найдите k.

2. Построить графики функций:

а) у = 5х;               б) у = –5х.

В каждом случае указать координаты двух точек графика, лежащих выше оси абсцисс.

Вариант 2

1. График функции у = проходит через точку А (4; –80). Найдите k.

2. Построить графики функций:

а) у = 6х;               б) у = –6х.

В каждом случае указать координаты двух точек графика, лежащих ниже оси абсцисс.

V. Итоги урока.

– Сформулируйте определение прямой пропорциональности.

– Что является графиком прямой пропорциональности?

– Каков алгоритм построения графика прямой пропорциональности?

– Как расположен в координатной плоскости график функции у = при k > 0 и k < 0?

Домашнее задание: 1. № 301; № 304.

2. № 357.

3. Построить график функции, заданной формулой у = 0,2х. Найти по графику:

а) значение у, соответствующее значению х, равному –5; 0; 5;

б) при каком значении х значение функции равно –2; 0; 2;

в) несколько значений х, при которых значения у неотрицательны.

 

 

Урок                                   Дата
ТЕМА: Решение задач по теме «Прямая пропорциональность»

Цели: продолжить формировать умение строить график прямой пропорциональности и работать с ним; формировать умение решать практические задачи с использованием прямой пропорциональности.

Ход урока

I. Математический диктант.

Вариант 1

1. Формулой какого вида задается прямая пропорциональность?

2. В каких координатных четвертях проходит график прямой пропорциональности у = 50х?

3. На графике функции лежит точка (0; 1). Может ли эта функция быть прямой пропорциональностью?

4. В каких координатных четвертях проходит график прямой пропорциональности у = х?

5. На графике прямой пропорциональности лежит точка (3; –1,5). Запишите формулу этой прямой пропорциональности.

6. Укажите две какие-нибудь точки, через которые проходит график прямой пропорциональности с коэффициентом –4.

7. Постройте график функции у = 2,5х.

Вариант 2

1. График функции проходит через точку (5; 0). Может ли эта функция быть прямой пропорциональностью?

2. В каких координатных четвертях проходит график прямой пропорциональности у = –40х?

3. Формулой какого вида задается прямая пропорциональность?

4. В каких координатных четвертях проходит график прямой пропорциональности у = х?

5. На графике прямой пропорциональности лежит точка (–1; 2,3). Запишите формулу этой прямой пропорциональности.

6. Укажите две какие-нибудь точки, через которые проходит график прямой пропорциональности с коэффициентом .

7. Постройте график функции у = –3,5х.

II. Формирование умений и навыков.

Упражнения, решаемые на этом уроке, направлены на формирование умения интерпретировать графики прямой пропорциональности, отражающие реальные зависимости.

Задания можно комбинировать по тематике и уровню сложности. Они отражают разные предметные области. Это задачи физико-технического, химико-биологического и гуманитарного содержания.

1. № 307, № 308 (устно).

2. Задайте  зависимость  формулой  и  выберите  её  график  из  трёх данных:

а) Зависимость массы молока т от его объема V. (Плотность молока равна 1028 кг/м3.)

    

б) Зависимость суточной дозы d лекарства эритромицина от веса ребенка т (эритромицин назначают детям по 50 мг в сутки на 1 кг веса).

  

в)  Зависимость  количества  слов  k,  сказанных  человеком  в  споре, от времени спора t (считать, что за 1 минуту человек в споре произносит 50 слов).

        

3. а) Если вес человека на Земле равен 600 Н, то его вес на Луне – 100 Н. Задайте формулой и графиком зависимость лунного веса человека от его земного веса. Найдите лунный вес человека, у которого земной вес равен 420 Н.

б) Пропорциональный подоходный налог составляет определенный процент от дохода человека. Задайте формулой и графиком зависимость 10 %-ного налога от дохода. Найдите налог на доход в 900 рублей.

в) Длительность звучания музыкальных нот выражается числами: целая нота – 1, половинная –  и т. д. Увеличим длительность каждой ноты некоторой мелодии в 2 раза. Задайте формулой и графиком зависимость длительностей нот полученной мелодии от длительностей нот исходной. Найдите длительность ноты полученной мелодии, если она соответствует ноте исходной мелодии длительностью в .

4. Сжатие  х  пружины  пропорционально  приложенной  силе  F  (то есть F = kx). Для сжатия пружины на 3 см нужна сила 10 Н. Какая сила потребуется для сжатия пружины на 5 мм? Постройте график зависимости длины пружины от приложенной силы (для 0 ≤ F ≤ 10 Н).

Замечание. обратите внимание учащихся на то, что х – сжатие, а не длина пружины. Сообщите им, что закон F = kx верен лишь при не очень больших деформациях пружины, отсюда ограничение на величину силы.

III. Итоги урока.

– Функция какого вида называется прямой пропорциональностью?

– Как построить график прямой пропорциональности?

– В каких координатных четвертях расположен график функции у = kx при k > 0, k < 0?

– Задачи какого характера отражают прямую пропорциональность?

Домашнее задание: 1. № 309.

2. Задайте прямую пропорциональность формулой, если известно, что её график проходит через точку:

а) А (2; 9);            б) В (3; –7).

3. Постройте график функции.

а) y = ;                  б) y = .

4*. Постройте график функции y =

 

 

 

 

 

 

 

 

 

 

 

 

 

Урок                                                     дата
Понятие линейной функции и её график

Цели: ввести понятие линейной функции; формировать умение выделять линейную функцию из множества функций; определить график линейной функции и выявить роль параметров k и b в расположении графика линейной функции.

Ход урока

I. Устная работа.

1. Какие из функций являются прямой пропорциональностью:

а) у = 13х;             б) у = ;                             в) у = ;

г) у = 13(х – 2);   д) у = 13х2;                             е) у = ?

2. Какая  из  точек  принадлежит  графику  функции,  заданной  формулой у = :

а) (0; –2);              б) ;                    в) (4; –2);

г) (0; 0);                            д) ;                      е) ?

3. График линейной пропорциональности проходит через точку А. Найдите коэффициент пропорциональности, если:

а) А ;                  б) А (2; –6);                            в) А ;

г) А ; д) А (0; 0);                              е) А (3; –0,3).

II. Объяснение нового материала.

Весь материал целесообразно разбить на несколько логических частей и на каждом уроке изучать одну из них.

На этом уроке целесообразно рассмотреть два вопроса: понятие линейной функции и влияние параметров k и b на расположение графика линейной функции.

В соответствии с этим объяснение проводится в два этапа.

1. Введение понятия линейной функции.

Понятие линейной функции начинаем изучать с рассмотрения реальных процессов и реальных ситуаций.

Необходимо привести примеры из учебника и вынести полученные формулы на доску:

s = 50t + 20,         где    t ≥ 0;

y = 3x + 5, где    x Î N.

Далее можно спросить учащихся: что общего во всех этих формулах? Затем сообщить им, что зависимости такого вида называются линейными функциями, и дать четкое определение.

На доску может быть вынесена запись:

Линейной функцией называется функция, которую можно задать формулой вида у = kx + b, где x – независимая переменная, k и b – некоторые числа.

2. Определение прямой пропорциональности как частного случая линейной функции.

Обращаем внимание учащихся, что в отличие от определения прямой пропорциональности, где k ¹ 0, в формуле линейной функции коэффициенты k и b – любые числа, то есть могут равняться нулю. Причем как по отдельности, так и одновременно.

В случае если k ¹ 0 и b = 0, функция у = kx + b принимает вид у = kx, то есть является прямой пропорциональностью. Сразу делаем вывод: графиком линейной функции в этом случае является прямая, проходящая через начало координат, и для её построения необходимо вычислить по формуле координаты ещё одной точки.

3. График линейной функции и роль параметров k и b в её расположении.

а) Следующим шагом целесообразно рассмотреть случай k ¹ 0 и b ¹ 0. Заполняем таблицу со с. 71 учебника для функций у = 0,5х и у = 0,5х + 2. Анализируя полученные данные, учащиеся делают вывод: графиком функции у = 0,5х + 2 является прямая, параллельная прямой, являющейся графиком функции у = 0,5х, и любая точка графика получается сдвигом по оси у на 2 единицы вверх.

Устное упражнение.

Что является графиком  функции  у = 3х + 1;  у = –1,5х + 2;  у = 2х – 14; у = –3х – 1,5?

б) Рассматриваем случай k = 0, b ¹ 0. Функция у = kx + b принимает вид у = b. Получаем, что, независимо от значения х, у всегда равно b. Значит, графиком функции является прямая, параллельная оси х и проходящая через точку (0; b).

в) Рассматриваем случай k = 0, b = 0. Функция у = kx + b принимает вид у = 0, то есть графиком является сама ось х.

После этого на доску можно вынести запись:

Графиком линейной функции является прямая:

а) при k ¹ 0 и b = 0, проходящая через начало координат
и совпадающая с графиком функции у = kx;

б) при k ¹ 0 и b ¹ 0, параллельная графику функции у = kx;

в) при k = 0, b ¹ 0, параллельная оси х;

г) при k = 0, b = 0, совпадающая с осью х.

4. Последним шагом формулируем простейший алгоритм построения графика линейной функции:

1-й шаг. По формуле найти координаты двух точек графика.

2-й шаг. Отметить полученные точки на координатной плоскости.

3-й шаг. Провести через построенные точки прямую.

III. Формирование умений и навыков.

1. Рассматриваем примеры 3–5 со с. 72–73 учебника. Во время работы учащиеся должны называть значения коэффициентов k и b.

2. Определите, какие из следующих функций являются линейными. Назовите для них значения коэффициентов k и b.

а) у = 2,5x – 7;                 б) у = 4 – x;            в) у = 4x – 5x2;

г) у = ;                          д) у = –3х;                              е) у = ;

ж) у = 3x2 + 2;                  з) у = –5;                                и) у = 0.

3. Что является графиком линейной функции и как он расположен?

а) у = –3x + 5;                  б) у = x;                              в) у = –3;                   

г) у = ;                 д) у = ;                                е) у = 0.

4. На рисунках изображены графики функций. Какие из этих функций являются линейными?

а)                     в) 

б)                 г) 

5. № 313, 315.

6. № 319, 321.

IV. Итоги урока.

– Дайте определение линейной функции.

– Что является графиком линейной функции?

– Как  влияют  параметры  k и b  на  расположение  графика  линейной функции?

– Каков алгоритм построения графика линейной функции?

Домашнее задание: № 314; № 316 (устно); № 318; № 320.

 

 

 

 

 

 

 

 

 

 

Урок                                         дата
ТЕМА: Взаимное расположение графиков
линейных функций

Цели: продолжить формировать умение строить график линейной функции и определять по графику значение функции по данному аргументу и наоборот; ввести понятие углового коэффициента прямой и выявить случаи взаимного расположения графиков линейных функций в зависимости от значений угловых коэффициентов.

Ход урока

I. Устная работа.

1. Является ли линейной функция, заданная формулой?

а) у = –2;                               в) у = x2 – 1;                  д) у = 2х;

б) у = x + 11;      г) у = ;                  е) у = 0,5x.

2. Какой из графиков расположен выше?

а) у = 3х или у = 3х – 2;              б) у = –х или у = –х + ;

в) у = 2 или у = 4.

II. Проверочная работа.

Вариант 1

1. Постройте график функции, заданной формулой у = –2х+ 0,5.

2. Линейная функция задана формулой у = 5х – 12. Найдите:

а) значение у, если х = 1,2; –3;

б) значение х, при котором у = 0; –1,5.

Вариант 2

1. Постройте график функции, заданной формулой у = –3х– 1,5.

2. Линейная функция задана формулой у = –4х + 7. Найдите:

а) значение у, если х = –1,3; 8;

б) значение х, при котором у = –2,8; 0.

III. Актуализация знаний.

1. Назовите координаты точек пересечения графиков функций с осями координат. Какие особенности этих точек?

а)   б)   в)

г)     д)   е)

2. № 322, № 324.

№ 322.

Решение:

а) у = –2,4х + 9,6.

Точка пересечения с осью х имеет ординату, равную нулю. Найдем её абсциссу, решив уравнение:

–2,4х + 9,6 = 0;

–2,4х = – 9,6;

х = – 9,6 : (–2,4);

х = 4.

(4; 0) – точка пересечения с осью х.

Точка пересечения с осью у имеет абсциссу, равную нулю. Найдем её ординату по формуле:

Если х = 0, то у = –2,4 · 0 + 9,6 = 9,6.

(0; 9,6) – точка пересечения с осью у.

б) у = –0,7х – 28.

Если у = 0,           то   –0,7х – 28 = 0;

                                     –0,7х = 28;

                                     х = 28 : (–0,7);

                                     х = –40.

(–40; 0).

Если х = 0,           то   у = –0,7 · 0 – 28 = –28.

(0; –28).

в) у = 1,2х + 6.

Если у = 0,           то   1,2х + 6 = 0;

                                     1,2х = –6;

                                     х = –6 : 1,2;

                                     х = –5.

(–5; 0).

Если х = 0,           то    у = 1,2 · 0 + 6 = 6.

(0; 6).

г) у = –5х + 2.

Если у = 0,           то   –5х + 2 = 0;

                                     –5х = –2;

                                     х = –2 : (–5);

                                     х = 0,4.

(0,4; 0).

Если х = 0, то у = –5 · 0 + 2 = 2.

(0; 2).

Ответ:  а) (4; 0), (0; 9,6);  б) (–40; 0), (0; –28);  в) (–5; 0), (0; 6);  г) (0,4; 0), (0; 2).

При выполнении этого задания обращаем внимание учащихся, что находить точку пересечения с осью у, подставляя значение в формулу, не рационально. Данное действие можно выполнять устно, рассуждая так: график линейной функции у = –2,4х + + 9,6 получен перемещением графика прямой пропорциональности у = –2,4х на 9,6 единиц по оси у вверх. Значит, если исходный график пересекал ось у в точке (0; 0), то полученный график пересекает её в точке (0; 9,6).

3. № 325.

При  выполнении  этого  задания  учащиеся  замечают,  что  для  построения  графика  линейной  функции  частного  вида  y = b  достаточно построить точку с координатами (0; b) и провести прямую, параллельную оси х (если выполняем задание в тетради в клеточку), либо построить 2 точки с координатами (0; b) и (х0; b), где х0 – любое число, и провести через них прямую.

IV. Объяснение нового материала.

1. Напоминаем, что график прямой пропорциональности y = kx располагается в I и III или в II и IV координатных четвертях в зависимости от знака коэффициента k. Посмотрев в тетради выполненные ранее построения, замечаем, что графики линейных функций пересекают ось х либо под острым углом (с положительным направлением оси х), либо под тупым. Угол зависит от знака k. Если k = 0, то прямая параллельна оси х. Так как от k зависит угол, то k называют угловым коэффициентом прямой.

2. Затем рассматриваем и анализируем рис. 36, 37 со с. 73 учебника. Делаем вывод: если угловые коэффициенты прямых, являющихся графиками двух линейных функций, равны, то эти прямые параллельны, а если угловые коэффициенты различны, то прямые пересекаются.

3. Рассматриваем случай, когда у линейных функций k различны, а b – одинаковые. Во время актуализации знаний мы вспомнили, что графики этих функций все проходят через точку (0; b), значит, они все пересекаются в этой точке.

V. Формирование умений и навыков.

1. Постройте в одной системе координат графики функций:

у = x + 1;   у = x – 2;   у = x.

Ответьте на вопросы:

1) Чему равен угловой коэффициент каждой прямой?

2) Каково взаимное расположение графиков функций?

3) Каковы координаты точек пересечения каждого графика с осями координат?

2. Пересекаются ли графики функций у = 2х – 4 и у = –4х + 2; у = 2х – 3 и у = 2х + 3?

В том случае, когда графики пересекаются, постройте их. Определите по графику координаты точки пересечения и проверьте результаты вычислением.

3. № 327.

4. Постройте прямую, если её угловой коэффициент равен –0,5 и она проходит через точку (–6; 4). Задайте формулой линейную функцию, график которой параллелен этой прямой и пересекает ось у в точке (0; 5).

VI. Итоги урока.

– Дайте определение линейной функции.

– Что является графиком линейной функции? Как его построить?

– Почему коэффициент k называется угловым? Как от k зависит расположение графика линейной функции?

– В каком случае графики двух линейных функций пересекаются и в каком случае они являются параллельными прямыми?

Домашнее задание: № 323; № 326; № 328; № 329.

 

 

 

Урок                             дата
ТЕМА: Решение задач по теме «Линейная функция и её график»

Цели: формировать умение использовать знания о линейной функции и её график при решении практических задач; интерпретировать полученные результаты.

Ход урока

I. Устная работа.

Разгадайте кроссворд.

В жирной рамочке прочтите ключевое слово – вещество, без которого организм не будет никогда здоровым.

1. Одно из условий задания координатной плоскости.

2. Множество точек в координатной плоскости, удовлетворяющих некоторому условию.

3. Способ задания функции.

4. Независимая переменная.

5. График линейной функции.

6. Вид функции.

7. Зависимость между величинами.

Ответы: 1. направление.

2. график. 3. таблица. 4. аргумент.

5. прямая. 6. линейная. 7. функция.

Ключевое слово: витамин.

II. Тест.

1. Отметьте  знаком  «+»  пары  функций,  графики  которых  пересекаются:

а) у = х и у = 0,4x – 1;             б) у = 4,2х + 2 и у = –4,2х – 2;

в) у = 3х + 1 и у = х + 1;              г) у = 2х + 5 и у = 2х – 10.

2. Даны функции:

а) у = 0,75х;                     б) у = х – 5;                    в) у = х;

г) у = –4х + 3;                  д) у = –8х + 5;            е) у = ;

выпишите  функции,  графики  которых  параллельны  графику  функции
у = 0,75х – 5.

3. Найдите  координаты  точки  пересечения  графиков  функций 
у = 37х – 8 и у = 25х + 4.

III. Формирование умений и навыков.

Упражнения, решаемые на этом уроке, содержат задания на построение и чтение графиков линейных функций, описывающих реальные процессы. Особое внимание следует уделять интерпретации полученных результатов.

1. № 330, 332.

2. Дорожный просвет – это расстояние между днищем автомобиля и дорогой, на которой он стоит. Для некоторого легкового автомобиля дорожный просвет можно вычислить по формуле h = 40 – , где h – дорожный просвет (в см), т – масса груза (в кг), погруженного в автомобиль.

а) Вычислите дорожный просвет, если масса груза в автомобиле равна: 100 кг; 150 кг; 200 кг; 0 кг.

б) Является ли зависимость величины дорожного просвета от массы груза, погруженного в автомобиль, линейной функцией? Чему в этом случае равны коэффициенты k и b?

в) Начертите координатные оси, выбрав на них подходящий масштаб, и постройте график функции h = 40 – , где 0 ≤ т ≤ 600.

г) С помощью построенного графика найдите, какой груз погружен в автомобиль, если дорожный просвет равен: 33 см; 38 см; 35 см; 40 см.

д) С помощью графика определите:

1) на сколько сантиметров уменьшится дорожный просвет, если к грузу в 50 кг добавить груз в 25 кг; к грузу в 100 кг добавить груз в 25 кг;

2) на сколько сантиметров увеличится дорожный просвет, если с машины с грузом в 150 кг снять груз в 50 кг.

3. № 1201*.

Решение:

а) y = | x | – 3. Данную функцию можно переписать в виде:

y =

Функция «кусочная», на каждом промежутке области определения является линейной.

б) y = 4 – | x |.

Если х ≥ 0, то 4 – | x | = 4 – x = –x + 4;

если х < 0,

то  4 – | x | = 4 – (–x) = 4 + x = x + 4.

y =

IV. Итоги урока.

– Какая функция является линейной?

– Что является графиком линейной функции?

– Как называется коэффициент k? Что он показывает в формуле линейной функции?

– Как расположен график функции y = x + 2; у = –3х; y = ?

– Назовите  признак  параллельности  графиков  двух  линейных  функций.

Домашнее задание: № 332; 333; 335; 366.

 

 

 

 

 

 

 

 

 

 

 

 

 

 

 

 

 

 

 

 

 

 

 

 

 

Урок                                    Дата
ТЕМА: Обобщающий урок по теме «Линейная функция»

Цели: обобщить и систематизировать знания по теме «Линейная функция»; подготовиться к написанию контрольной работы.

Ход урока

I. Игра-слалом.

Игра состоит из теоретического и практического конкурсов. Задания выполняются на время.

1. Теоретический конкурс

1. Какую зависимость называют функциональной или функцией?

2. Что такое аргумент и что такое функция?

3. Что называют областью определения функции?

4. Что такое график функции?

5. Какую функцию называют линейной?

6. Что является графиком линейной функции?

7. Что является графиком прямой пропорциональности?

8. В чём их сходство и различие?

9. От чего зависит расположение графика линейной функции?

10. Сколько  точек  необходимо  для  построения  графика  линейной функции?

11. А для графика прямой пропорциональности? Почему?

12. Что такое угловой коэффициент?

13. Как расположен график функции y = kx при k > 0 и k < 0?

14. Как найти координаты точки пересечения графиков двух линейных функций?

15. В каком случае графики двух линейных функций являются параллельными прямыми?

2. Практический конкурс

1. Заполните таблицу для функции, заданной формулой у = –0,5(8 – x).

х

–1,4

 

2,6

 

8,8

 

у

 

–3,4

 

–1,8

 

2,4

2. Какова область определения функции?

а) у = ;                               в) у = ;

б) у = 7х + 6;                                г) у = .

3. Является ли линейной функция:

а) у = ;                            в) у = х(6 – х);

б) у = 3(х + 8);                             г) у = 2(1 – 3х)(х – 3)?

4. Постройте график функции, заданной формулой у = 2х + 3.

5. Постройте график функции, заданной формулой у = 0,5х + 3. С помощью графика найдите:

а) значение у, если х = –4;

б) значение х, если у = 6;

в) координаты точек пересечения графика с осями координат;

г) корень уравнения 0,5х + 3 = 0.

6. Не выполняя построения, выясните, проходит ли график функции, заданной формулой у = 1,25х – 5, через точку:

а) А (20; 20);                                б) В (20; 10).

7. Функция задана формулой у = 0,25х + 3, где х принадлежит промежутку от –4 до 8.

Постройте график этой функции и укажите все целые значения, которые может принимать эта функция.

8. Пересекает ли ось х график линейной функции, и если пересекает, то в какой точке? Функция задана формулой:

а) у = 7х + 49;                              б) у = 15.

9. График некоторой линейной функции вида y = kx + 1 параллелен графику функции у = –0,4х. Найдите значение коэффициента k и выясните, принадлежит ли этому графику точка М (50; –19).

10. Не выполняя построения, найдите координаты точки пересечения графиков линейных функций: у = 4х + 9 и у = 6х – 5.

11. Отметьте точки А (–4; 3) и В (2; –6). Проведите прямую АВ и найдите координаты точек пересечения этой прямой с осью х и осью у.

12. Постройте график функций:

а) у = –5;                          б) х = 3.

13. Какие из графиков функций параллельны, а какие пересекаются:

а) у = –3х + 4;                              в) у = –(2 + 3х);

б) у = –х + 3;                                г) у = х + 3?

14. В одной и той же координатной плоскости постройте графики функции: у = 5, у = х – 2, у = –2х + 4, у = 0.

15. В каких координатных четвертях расположен график прямой пропорциональности, параллельный графику линейной функции, заданной формулой:

а) у = 0,8х – 1,6;              б) у = – 0,4х + 1?

II. Итоги урока.

Домашнее задание: повторить п. 15, п. 16. № 360; № 363; № 372.

 

 

 

 

 

 

 

 

 

 

 

 

Урок
Контрольная работа № 3

Вариант 1

1. Функция задана формулой у = 6х + 19. Определите:

а) значение у, если х = 0,5;

б) значение х, при котором у = 1;

в) проходит ли график функции через точку А (–2; 7).

2. а) Постройте график функции у = 2х – 4.

б) Укажите с помощью графика, чему равно значение у при х = 1,5.

3. В одной и той же системе координат постройте графики функций:

а) у = –2х; б) у = 3.

4. Найдите  координаты  точки  пересечения  графиков  функций
у = 47х – 37 и у = –13х + 23.

5. Задайте формулой линейную функцию, график которой параллелен прямой у = 3х – 7 и проходит через начало координат.

Вариант 2

1. Функция задана формулой у = 4х – 30. Определите:

а) значение у, если х = –2,5;

б) значение х, при котором у = –6;

в) проходит ли график функции через точку В (7; –3).

2. а) Постройте график функции у = –3х + 3.

б) Укажите  с  помощью  графика,  при  каком  значении  х  значение у равно 6.

3. В одной и той же системе координат постройте графики функций:

а) у = 0,5х;            б) у = –4.

4. Найдите  координаты  точки  пересечения  графиков  функций
у = –38х + 15 и у = –21х – 36.

5. Задайте формулой линейную функцию, график которой параллелен прямой у = –5х + 8 и проходит через начало координат.

Вариант 3

1. Функция задана формулой у = 5х + 18. Определите:

а) значение у, если х = 0,4;

б) значение х, при котором у = 3;

в) проходит ли график функции через точку С (–6; –12).

2. а) Постройте график функции у = 2х + 4.

б) Укажите с помощью графика, чему равно значение у при х = –1,5.

3. В одной и той же системе координат постройте графики функций:

а) у = –0,5х;          б) у = 5.

4. Найдите  координаты  точки  пересечения  графиков  функций
у = –14х + 32 и у = 26х – 8.

5. Задайте формулой линейную функцию, график которой параллелен прямой у = 2х + 9 и проходит через начало координат.

Рекомендации по оцениванию.

Задания 1–3 относятся к базовому уровню знаний по теме. Верное выполнение любых трех заданий оценивается отметкой «3». Для получения отметки «5» необходимо выполнить верно все пять заданий.

Решение заданий контрольной работы

Вариант 1

1. у = 6х + 19.

а) Если х = 0,5,    то   у = 6 · 0,5 + 19 = 3 + 19 = 22;

б) если у = 1,                   то   6х + 19 = 1;

                                                6х = 1 – 19;

                                                6х = –18;

                                                х = –18 : 6;

                                                х = –3;

в) 7 = 6 · (–2) + 19;

    7 = –12 + 19;

    7 = 7 – верно,  значит,  график  функции  проходит  через  точку
А (–2; 7).

Ответ: а) 22; б) –3; в) проходит.

2. а) у = 2х – 4.

Построим две точки, принадлежащие графику.

Если х = 0, то у = 2 · 0 – 4 = –4;

если х = 2, то у = 2 · 2 – 4 = 0.

(0; –4), (2; 0).

б) При х = 1,5 у = –1.

3. а) у = –2х. Графиком является прямая, проходящая через начало координат и точку (2; –4).

б) у = 3. Графиком является прямая, проходящая через точку (0; 3) и параллельная оси х.

 

4. Решим уравнение:

47х – 37 = –13х + 23.

47х + 13х = 23 + 37;

60х = 60;

х = 1, значит, абсцисса точки пересечения графиков функций равна 1. Найдем соответствующее значение ординаты:

если х = 1, то у = 47 · 1 – 37 = 10.

Точка пересечения имеет координаты (1; 10).

Ответ: (1; 10).

5. График параллелен прямой у = 3х – 7, значит, угловые коэффициенты равны. Так как прямая проходит через начало координат, то это прямая пропорциональность. Значит, у = 3х.

Ответ: у = 3х.

Вариант 2

1. у = 4х – 30.

а) Если х = –2,5,  то    у = 4 · (–2,5) – 30 = –10 – 30 = –40;

б) если у = –6,                 то   4х – 30 = –6;

                                                4х = –6 + 30;

                                                4х = 24;

                                                х = 24 : 4;

                                                х = 6;

в) –3 = 4 · 7 – 30;

    –3 = 28 – 30;

    –3 = –2 – неверно, значит, график функции не проходит через точку В (7; –3).

Ответ: а) –40; б) 6; в) не проходит.

2. а) у = –3х + 3.

Построим две точки, принадлежащие графику.

Если х = 0, то у = –3 · 0 + 3 = 3;

если х = 2, то у = –3 · 2 + 3 = –3;

(0; 3), (2; –3)

 

б) Если у = 6, то х = –1.

3. а) у = 0,5х. Графиком является прямая, проходящая через начало координат и точку (4; 2).

б) у = –4. Графиком является прямая, проходящая через точку (0; –4) и параллельная оси х.

4. Решим уравнение:

–38х + 15 = –21х – 36;

–38х + 21х = –36 – 15;

–17х = –51;

х = (–51) : (–17);

х = 3, значит, абсцисса точки пересечения графиков функций равна 3.

Найдем соответствующее значение ординаты:

если х = 3, то у = –38 · 3 + 15 = –99.

Точка пересечения имеет координаты (3; –99).

Ответ: (3; –99).

5. График параллелен прямой у = –5х + 8, значит, угловые координаты равны. Так как прямая проходит через начало координат, то это прямая пропорциональность. Значит, у = –5х.

Ответ: у = –5х.

Вариант 3

1. у = 5х + 18.

а) Если х = 0,4,    то   у = 5 · 0,4 + 18 = 2 + 18 = 20;

б) если у = 3,                   то   5х + 18 = 3;

                                                5х = 3 – 18;

                                                5х = –15;

                                                х = –15 : 5;

                                                х = –3;

в) –12 = 5 · (–6) + 18;

    –12 = –30 + 18;

    –12 = –12 – верно, значит, график функции проходит через точку
С (–6; –12).

Ответ: а) 20; б) –3; в) проходит.

2. а) у = 2х + 4.

Построим две точки, принадлежащие графику.

Если х = 0, то у = 2 · 0 + 4 = 4;

если х = –2, то 2 · (–2) + 4 = 0.

(0; 4), (–2; 0)

б) Если х = –1,5, то у = 1.

3. а) у = –0,5х. Графиком является прямая, проходящая через начало координат и точку (4; –2).

б) у = 5. Графиком является прямая, проходящая через точку (0; 5) и параллельная оси х.

4. Решим уравнение:

–14х + 32 = 26х – 8;

–14х – 26х = –8 – 32;

–40х = –40;

х = 1, значит, абсцисса точки пересечения графиков равна 1. Найдем соответствующее значение ординаты:

если х = 1, то у = –14 · 1 + 32 = 18.

Точка пересечения имеет координаты (1; 18).

Ответ: (1; 18).

5. График параллелен прямой у = 2х + 9, значит, угловые коэффициенты равны. Так как прямая проходит через начало координат, то это прямая пропорциональность. Значит, у = 2х.

Ответ: у = 2х.

 

 

 

Урок 41
Анализ результатов контрольной работы.
Обобщение материала по теме «Функции»

Цели: проанализировать результаты контрольной работы, выявить типичные ошибки, допущенные учащимися; провести работу над ошибками, обобщить изученный материал по теме «Функции»; приобрести опыт решения заданий повышенной трудности по теме «Функции».

Ход урока

I. Анализ результатов контрольной работы.

Учитель в целом характеризует полученные результаты, указывает на типичные ошибки, истоки их возникновения и способы преодоления, раздает тетради с контрольной работой.

II. Работа над ошибками.

Учащиеся самостоятельно в тетрадях выполняют работу, получая консультацию учителя. Если какое-то задание решено неверно третью класса и более, оно выносится на доску.

III. Обобщение материала и решение заданий повышенной трудности.

№ 361*, № 362*, № 364*, № 368*, № 373*.

№ 361.

Решение:

а) Функция  линейная  задается  формулой  y = kx + b.  Так  как  график проходит  через  точку  (0; –8),  значит, он получен сдвигом прямой y = kx на 8 единиц внизу по оси у. Значит, в формуле коэффициент b = –8, то есть y = kx – 8. Осталось найти коэффициент k. Так как график проходит также через точку (2; 12), то

k · 2 – 8 = 12;

2k = 20;

k = 10.

Следовательно, формула, задающая линейную функцию, – у = 10х – 8.

Заполним таблицу, подставляя соответствующие значения в формулу:

при х = –2, у = 10 · (–2) – 8 = –28;

при х = 4, у = 10 · 4 – 8 = 32;

при х = 6, у = 10 · 6 – 8 = 52.

х

–2

0

2

4

6

у

–28

–8

12

32

52

б) Рассуждаем аналогично.

В формуле y = kx + b коэффициент b равен 5, то есть y = kx + 5.

Точка (10; 6) принадлежит графику функции, значит,

k · 10 + 5 = 6;

10k = 1;

k = 0,1.

Функция задана формулой у = 0,1х + 5.

Если у = –15,  то   0,1х + 5 = –15;

                               0,1х = –20;

                               х = –200;

если х = –10,          у = 0,1 · (–10) + 5 = 4;

если х = 30,            у = 0,1 · 30 + 5 = 8;

если у = 15,     то   0,1х + 5 = 15;

                               0,1х = 10;

                               х = 100.

х

–200

–10

0

10

30

100

у

–15

4

5

6

8

15

№ 362.

Решение:

х

1

2

3

4

5

6

7

у

11

21

31

41

51

61

71

у = 10х + 1.

Ответ: у = 10х + 1.

№ 364.

Решение:

Если точка А (а; –1,4) принадлежит графику прямой пропорциональности у = 3,5х, то –1,4 = 3,5 · а.

Найдем значение а, решив это уравнение:

3,5а = –1,4;

а = –1,4 : 3,5;

а = –0,4.

Ответ: при а = –0,4.

№ 368.

Решение:

а) у = ах, где а > 0

    у = bx, где b > 0

Обе  функции  –  прямые  пропорциональности  с  положительными угловыми  коэффициентами,  значит,  графиками  являются  прямые,  расположенные в I и III координатных четвертях и проходящие через начало координат.

 

Так как a > b, то для х > 0 значение ах > bx, значит, в I координатной четверти прямая у = ах лежит выше прямой у = bx.

б) Рассуждаем аналогично.

а < 0, b < 0 и | a | < | b |.

Графики функции у = ах и у = bx – прямые, проходящие через начало координат и расположенные во II и IV координатных четвертях.

| a | < | b |, значит, для х > 0 значение ах < bx, то есть в IV четверти график функции у = ах лежит ниже графика функции у = bx.

 

Замечание. при выполнении этого упражнения, если у учащихся возникают трудности с буквенными неравенствами, можно подбирать конкретные числовые значения параметров a и b и строить графики функций с числовыми коэффициентами. Но затем, после анализа, все равно необходимо обосновать обобщенный вывод.

№ 373.

Решение:

Решим это задание графически. Построим графики данных функций.

а) у = 3х + 2.

Если х = 0,      то   у = 3 · 0 + 2 = 2;                (0; 2);

если х = 2,       то   у = 3 · 2 + 2 = 8.                (2; 8).

б) у = –2х + 3.

Если х = 0,      то   у = –2 · 0 + 3 = 3;              (0; 3);

если х = 2,       то   у = –2 · 2 + 3 = –1.            (2; –1).

в) у = 0,5х – 2.

Если х = 0,      то   у = 0,5 · 0 – 2 = –2;           (0; –2);

если х = 2,       то   у = 0,5 · 2 – 2 = –1.           (2; –1).

Ответ: да.

IV. Итоги урока.

Домашнее задание: повторить п. 12–16.

 

 

Урок 42
Задание функции несколькими формулами

Цели: ознакомиться с понятием «кусочной» функции, задаваемой несколькими формулами; приобрести навыки построения и работы с графиками «кусочных» функций.

Ход урока

I. Актуализация знаний.

1. Найдите область определения функции:

а) у = 3х – 14;                     б) у = 3х2 + x;             в) у = ;  

г) у = 5;                              д) у = ;                  е) у = .

2. Какая из данных функций является линейной:

а) у = 3х2 + 4;                     б) у = ;                в) у = x;

г) у = –15х + 0,1;                д) у = ;                е) у = ?

3. Найдите  значение  функции  у = –3х – 4,  если  значение  аргумента равно:

а) 0;                 б) ;                 в) 3;                     г) 18.

4. Какому значению аргумента функции у = 3х + 2 соответствует значение функции:

а) у = 0;           б) у = 14;             в) у = –1;              г) у = 0,4?

5. Найдите значение функции при х = 5, если:

а) у = | x |;                           б) у =  – | x |;               в) у = | x – 6 |;        

г) у = – ∙  | 3 – x |;                      д) у = ;              е) у = –3 –  | 4x |.

II. Изучение нового материала.

Изучение следует начинать с рассмотрения практических задач, в которых описываются ситуации функциональной зависимости, но различной на разных частях области определения.

Указываем, что вместо того, чтобы для каждого промежутка записывать новую функцию, мы используем для записи фигурную скобку.

Например:

s =

Обязательно подчеркиваем, что каждый промежуток не пересекается с другими, но все вместе они составляют область определения функции.

Каждый «кусочек» области определения можно задать неравенством, двойным неравенством, либо это будет вообще отдельная точка или конечное множество точек.

Рассматриваем примеры 2–3 учебника. Особенно обращаем внимание на то, что, задавая функцию несколькими формулами, необходимо следить  за  тем,  чтобы  каждому  значению  х  соответствовало  единственное значение у. В противном случае такая зависимость не будет являться функцией.

III. Закрепление изученного материала.

№ 339.

Решение:

Разобьем на 3 интервала область определения функции:

х < 0;

0 ≤ x ≤ 1;

x > 1.

На каждом интервале заданы линейные функции y = kx + b. Зададим их аналитически, используя зависимость расположения графика от значения коэффициентов k и b:

y =

№ 340.

Решение:

Из бака всего может вытечь 20 · 0,9 = 18 л воды. Время t изменяется от 0 до 12. Так как вода вытекает со скоростью 2 л/мин, то 18 л вытекут за 9 мин, а в оставшееся время объем воды в баке не будет меняться, а будет составлять 2 л. Значит, функцию можно задать следующим образом:

V =

№ 341.

№ 342.

Решение:

а) y = 0,25 | x | + 1. Запишем функцию несколькими формулами, используя определение абсолютной величины:

y =

в) y = (x – 2).

ОДЗ – все числа, кроме нуля.

Если x > 0, то y =  ∙  (x – 2) = x – 2;

если x < 0, то y =  ∙  (x – 2) = –(x – 2) = –x + 2.

y =

На каждом интервале графиком является прямая.

№ 343.

Решение:

Если x < 0, то | x | = –x, значит, –x + 2 = | x | + 2;

если х ≥ 0, | x | = x, значит, x + 2 = | x | + 2.

Таким образом, y = | x | + 2.

Ответ: y = | x | + 2.

№ 344.

Решение:

Область определения функции разбита на три интервала:

–2 ≤ х ≤ 1;

1 < х ≤ 3;

3 < х ≤ 6.

На каждом интервале задана линейная функция, запишем её формулу:

y =

№ 345.

Решение:

Запишем полученные данные в виде таблицы:

t

0

20

30

45

60

90

T

60

100

100

95

90

80

Если 0 ≤ t < 20, то вода в баке нагревается;

если 20 ≤ t ≤ 30, то вода кипит;

если 30 < t ≤ 90, то вода остывает до 80 °С.

№ 346.

Решение:

Запишем полученные данные в виде таблицы.

t

0

1

1,5

2

T

0

3

5

5

2,5

0

№ 347.

Решение:

s =

Скорость автомобиля до остановки 60 км/ч. Скорость автомобиля после остановки 90 км/ч.

IV. Итоги урока. Урок                                           Дата
ТЕМА: Понятие многочлена

Цели: ввести понятие многочлена, подобных членов многочлена, стандартного вида многочлена; формировать умение приводить многочлен к стандартному виду.

Ход урока

I. Устная работа.

1. Является ли одночленом выражение:

а) 7х2у2;                            в) у3 + у;                                 д) 5(a + b)3;

б) a ∙  ;                  г) ;                        е) a2ba?

2. Представьте  одночлен  в  стандартном  виде  и  назовите  его  коэффициент:

а) 4х3х;                             в) 10х2 · (–0,1х2);                   д) –2р5 · 5р3;

б) –3aba7;                    г)  ∙  4c;                            е) xy2 ∙  (–3x7).

II. Объяснение нового материала.

С позиции выполнения упражнений, предложенных в учебнике, данная тема не является сложной для учащихся. Однако при её изучении появляется много новых понятий, которые они должны усвоить.

Необходимо акцентировать внимание учащихся на этих понятиях, а также на формулировках типа «приведите подобные члены многочлена», «представьте многочлен в стандартном виде». Иначе впоследствии школьники, встречая такие задания, могут не понять, что от них требуется. Поэтому в течение урока нужно как можно больше проговаривать изучаемые понятия, их определения и просить учащихся пояснять, что требуется сделать в том или ином задании.

Из-за большого количества новых понятий определение степени многочлена можно отложить до следующего урока.

Объяснение материала проводится в несколько этапов, каждый из которых закрепляется примерами и устными заданиями.

1. Введение понятия многочлена.

При выполнении устной работы у учащихся была возможность вспомнить понятие одночлена, поэтому определение многочлена не должно вызывать у них затруднений.

Задание. Назовите каждый член многочлена и определите вид многочлена (одночлен, двучлен, трёхчлен).

а) –6a3 + 1,3b2;                            г) 4ab + 7ab2;

б) c8;                                   д) xyz + x2z;

в) 5x2 + 7x – 8;                             е) 3a2b2c3.

2. Приведение подобных членов многочлена.

Можно предложить учащимся определить вид многочлена 3y4 + 2y
– 2y4. Некоторые из них скажут, что это трёхчлен. Тогда следует обратить внимание на то, что слагаемые 3у4 и –2у4 являются подобными, и после их приведения получится многочлен  у4 + 2у, который является двучленом.

Делается вывод, что приведение подобных членов многочлена является важной операцией, которая должна предшествовать многим заданиям, связанным с многочленами. Рассмотреть пример 1 из учебника.

3. Стандартный вид многочлена.

Сначала необходимо вспомнить, что называется стандартным видом одночлена, а затем рассмотреть вопрос о приведении многочлена к стандартному виду.

Обратить внимание учащихся на то, что для приведения многочлена к стандартному виду нужно выполнить две операции:

– каждый член многочлена записать в стандартном виде;

– привести подобные члены многочлена.

Пример. Привести многочлен 3х5 – 2х2 + 3х · (–2) + 4х2 к стандартному виду.

3х5 – 2х2 + 3х · (–2) + 4х2 = 3х5 – 2х2 – 6х + 4х2 = 3х5 + 2х2 – 6х.

Как уже говорилось, вопрос о степени многочлена лучше рассмотреть на следующем уроке.

III. Формирование умений и навыков.

Как при объяснении нового материала, так и при формировании умений и навыков рассматриваются три основные группы вопросов:

1) понятие многочлена;

2) приведение подобных членов многочлена;

3) стандартный вид многочлена.

1-я группа

1. № 567.

2. Определите количество членов многочлена и назовите его (двучлен, трёхчлен).

а) x5 + 2ab;                                          в) 8ab + b5 – 9;

б) xy2 + x – 2y + 5;                                   г) 5x3y2 – 5x3.

2-я группа

1. Приведите подобные члены многочлена.

а) 2a + 4ab – 6ab;                                    в) 2x3 – 5x2 + 4xx3 + 3x2;

б) 5x2 + 6x – 9x2;                         г) 4a5 – 7a3 + 2 – 2a3 – 10.

2. № 569.

3-я группа

1. Запишите в стандартном виде многочлен:

а) 3x7 + 2x ∙  (–5) + 5y;                в) 5a4 – 2a ∙  a2a2 + 7a3;

б) 2p3p2 + 7p + 9p2;                 г) 2y2 ∙  (–4y3) + 5y ∙  y3 – 3y5.

2. № 571.

IV. Итоги урока.

– Что называется многочленом? членом многочлена?

– Приведите примеры двучленов, трёхчленов.

– Что такое подобные члены многочлена?

– Как записать многочлен в стандартном виде?

– Записан ли многочлен –3x7 + 2x3 + 4x ∙  (–x2) + x в стандартном виде? Почему?

Домашнее задание: № 568, № 570.

 

Урок 62
Нахождение значений многочлена

Цели: ввести понятие степени многочлена; формировать умение определять степень многочлена и находить значения многочлена; продолжить формирование умения записывать многочлен в стандартном виде.

Ход урока

I. Устная работа.

Записаны ли многочлены в стандартном виде?

а) 3ab2 – 7y – 9;                           б) x5 + 2x2abc;

в) 3y5 – 7y2 + 2y – 9y5;                г) x4 – 3x  x2 + 5;

д) 4xy – 8x2y + 2xy2x2y2;                     е) 2a4 + 3a (–4) + a3 + 8a.

Приведите к стандартному виду все многочлены.

II. Формирование умений и навыков.

Все задания можно разбить на две группы. В 1-ю группу войдут задания на нахождение значений многочлена, а во 2-ю группу – на определение степени многочлена.

1-я группа

1. № 572.

Решение:

(Важно, чтобы учащиеся поняли, что перед подстановкой данного значения в многочлен необходимо привести подобные члены многочлена.)

а) 5x6 – 3x2 + 7 – 2x6 – 3x6 + 4x2 = x2 + 7

при х = –10:                     х2 + 7 = (–10)2 + 7 = 107.

б) 4a2bab2 – 3a2b + ab2ab + 6 = a2bab + 6

при а = –3, b = 2: a2bab + 6 = 9 ∙  2 + 3 ∙  2 + 6 = 30.

2. № 574.

2-я группа

Сначала необходимо ввести понятие степени многочлена. учащиеся уже умеют определять степень одночлена, поэтому данный вопрос не должен вызывать у них затруднений. особое внимание следует обратить на то, что перед определением степени многочлена необходимо сначала привести его к стандартному виду.

1. № 577 (а), № 578 (а).

2. Определите степень многочлена.

а) 3x2x5 + 8x3;                          г) 2a3b – 5b5 + 2a4b2;

б) 8 – 6а;                                      д) 5t2 – 3t + 8 – 4t ∙  t2;

в) 5xy + 2y – 3xy2;                                   е) 3a2x2 + 2axa2x2 + 5 – 2a2x2.

3. Вместо значка * запишите такой одночлен, чтобы получился многочлен четвертой степени.

а) 3x3 – 5x2 + 7 – *;                                 б) 5a – 4a4 + 1 + *;

в) x5 + 2x4 – 3x2 + *;                                г) 4a3b2 + 3a2b2 + ab + *.

Решение:

а) Данный многочлен содержит одночлен второй и третьей степени. Чтобы многочлен был четвертой степени, вместо * нужно записать любой одночлен четвертой степени. Например, 7х4, 3а4, х2у2, abи т. п.

б) Данный многочлен содержит одночлены первой и четвертой степени. Чтобы он был четвертой степени, вместо * достаточно записать любой одночлен не выше четвертой степени. Например, 2а2, xz2, 8у  и т. п.

в) Данный многочлен содержит одночлены второй, четвертой и пятой степени. Чтобы он был четвертой степени, нужно вместо * записать такой одночлен, который взаимно уничтожиться с одночленом х5, то есть – х5.

г) Аналогично предыдущему заданию вместо * нужно записать одночлен –4a3b2.

III. Итоги урока.

– Что называется многочленом? Членом многочлена?

– Как записать многочлен в стандартном виде?

– Как найти значение многочлена при данных значениях переменных?

– Что называется степенью многочлена? Как определить степень произвольного многочлена?

Домашнее задание: № 573, № 577 (б); № 578 (б); № 579.

 

 

Урок 63
Правило сложения и вычитания
многочленов

Цели: рассмотреть вопрос о сложении и вычитании многочленов; формировать умение выполнять эти действия.

Ход урока

I. Проверочная работа.

Вариант 1

1. Приведите многочлен к стандартному виду.

а) 5x ∙  8y ∙  (–7x2) + (–6x) ∙  3y2;

б) 5a2 + 3a – 7 – 5a3 – 3a2 + 7a – 11;

в) 6a2b – 5ab2 + 5a3 + 2ab2 – 8a3 – 3a2b.

2. Найдите значение многочлена.

а) –15ab – 2 + 14a при а = –29, b = –2;

б) m4 – 3m3n + m2n2m3n – 4mn3 при т = –1, п = 1.

Вариант 2

1. Приведите многочлен к стандартному виду.

а) 8x ∙  3y ∙  (–5y) – 7x2 ∙  (–4y);

б) 3t2 – 11t – 5t2 + 5t – 3t2 + 11;

в) 3a2x + 3ax2 + 5a3 + 3ax2 – 8a2x – 10a3.

2. Найдите значение многочлена.

а) –x – 3y – 4 + 2y при х = –15, у = –4;

б) 3uv3 + u2v2 – 2uv3 + u3vu4 при u = 1, v = –1.

II. Устная работа.

1. Назовите выражение, которое получится после раскрытия скобок.

а) x + (yz);                                в) x – (ab);

б) a – (b + c);                               г) 2p – (p + q).

2. Найдите значение выражения разными способами.

а) 17 + (2 – 10);               в) 10 + (–3 + 8);

б) 4 – (5 + 2);                               г) 12 – (4 – 7).

III. Объяснение нового материала.

Если учащиеся хорошо усвоили материал о раскрытии скобок и приведении подобных слагаемых, то данная тема не должна вызывать у них затруднений. Достаточно актуализировать знания учащихся и рассмотреть примеры из учебника.

IV. Формирование умений и навыков.

1. № 585.

2. № 587 (а, в, д); № 589 (а, в).

3. № 588 (а, в).

4. № 591.

Решение:

а) Любое нечетное число можно записать в виде 2п + 1, тогда следующее за ним нечетное число будет равно 2п + 3.

Найдем сумму этих чисел:

2п + 1 + 2п + 3 = 4п + 4.

Первое слагаемое этой суммы делится на 4 и второе слагаемое делится на 4. Значит, вся сумма 4п + 4 делится на 4.

б) Пусть 2п + 1, 2п + 3, 2п + 5 и 2п + 7 – четыре последовательных нечетных числа. Найдем их сумму:

2п + 1 + 2п + 3 + 2п + 5 + 2п + 7 = 8п + 16.

Оба слагаемых этой суммы делятся на 8, значит, и вся сумма делится на 8.

V. Итоги урока.

– Что называется многочленом? степенью многочлена?

– Как привести многочлен к стандартному виду?

– Как раскрыть скобки, перед которыми стоит знак «+»? знак «–»?

– Как выполнить сложение или вычитание многочленов?

Домашнее задание: № 586; № 587 (б, г, е); № 588 (б, г); № 589 (б, г).

 

 

Урок 64
Решение различных упражнений
на сложение и вычитание многочленов

Цели: продолжить формирование умения выполнять сложение и вычитание многочленов.

Ход урока

I. Устная работа.

Выполните сложение или вычитание многочленов.

а) (1 + a) + (3 + 2a);                                г) (5 – y) + (3y – 2);

б) (ab) – (a + b);                                  д) 4p2 – (p2 + 2);

в) 3x – (1 – 2x);                            е) (2 + x) – (x – 3).

II. Формирование умений и навыков.

1. № 590, № 592.

2. № 593.

Решение:

Учащиеся должны понять, что для выполнения этого задания нужно в левой и правой частях равенства отыскивать подобные слагаемые и подбирать выражение М таким образом, чтобы они были равны.

а) Если упражнение вызовет затруднения, то можно представить его более наглядно:

M + (5x2 – 2xy) = 6x2 + 9xyy2.

Слева:                              Справа:

5х2                                    6х2

–2ху                                  9ху

0                                       –у2

Нужно найти такие одночлены, которые в сумме с одночленами из левой части дадут одночлены, равные стоящим в правой части. Получаем их:  х2, 11ху, у2.

Значит, вместо М нужно записать многочлен х2 + 11ху + у2.

б) M – (4ab – 3b2) = a2 – 7ab + 8b2.

Сначала раскроем скобки:  М – 4ab + 3b2 = a2 – 7ab + 8b2.

Слева:                              Справа:

–4ab                                   –7ab

   3b2                                    8b2

   0                                        a2

Находим недостающие одночлены: –3ab, 5b2, –а2. Получаем многочлен: 5b2 – 3abа2.

Если задание не вызывает затруднений у учащихся, то они могут выполнять его устно.

3. Запишите во втором столбце многочлен, сумма которого с многочленом из первого столбца равна многочлену из третьего столбца.

1) 3х + 5

2) 7х + 3

3) a3 + 3a2b + b3

4) 2x2y – 3xy2 – 8

5) x2 + 2xy + y2

6) 3x + 2a

 

8х – 11

х2 + 7х – 15

a3 + 3a2b + b3

0

x2 – 2xy + y2

2x + b

4. № 605.

Решение:

Необходимо объяснить учащимся, что решение любого уравнения начинается с его преобразования.

в) (3,2у – 1,8) – (5,2у + 3,4) = –5,8;

    3,2у – 1,8 – 5,2у – 3,4 = –5,8;

    3,2у – 5,2у = 1,8 + 3,4 – 5,8;

    –2у = –0,6;

    у = –0,6 : (–2);

    у = 0,3.

Ответ: 0,3.

д) 3,8 – 1,5у + (4,5у – 0,8) = 2,4у + 3;

    3,8 – 1,5у + 4,5у – 0,8 = 2,4у + 3;

    – 1,5у + 4,5у – 2,4у = 3 – 3,8 + 0,8;

    0,6у = 0;

    у = 0.

Ответ: 0.

III. Итоги урока.

– Что называется многочленом? степенью многочлена?

– Как раскрыть скобки, перед которыми стоит знак «+»? знак «–»?

– Как выполнить сложение или вычитание многочленов?

Домашнее задание: № 594; № 596; № 606.

 

 

 

Урок 65
Заключение многочлена в скобки

Цели: формировать умение представлять многочлен в виде суммы или разности многочленов; закрепить умение складывать и вычитать многочлены.

Ход урока

I. Проверочная работа.

Вариант 1

1. Составьте сумму и разность многочленов и приведите их к стандартному виду:

а) 2y2 + 8y – 11   и   3y2 – 6y + 3;

б) 2p2 + 3pq + 8q2   и   6p2pq – 8q2.

2. Упростите выражение.

а) (3x + 10y) – (6x + 3y) + (6y – 8x);

б) (8c2 + 3c) + (–7c2 – 11c + 3) – (–3c2 – 4).

Вариант 2

1. Составьте сумму и разность многочленов и приведите их к стандартному виду.

а) 5y2 – 3y – 1   и   8y2 + 2y – 11;

б) 8x2 + 2px – 3p2   и   2x2 + 3px – 3p2.

2. Упростите выражение.

а) (3a + 5b) – (9a – 7b) + (–5a + 11b);

б) (3x2 + 2x) + (2x2 – 3x – 4) – (–x2 + 19).

II. Формирование умений и навыков.

Все задания можно разбить на две группы. В 1-ю группу войдут упражнения на закрепление умения складывать и вычитать многочлены. А во 2-ю группу – упражнения на представление многочлена в виде суммы или разности многочленов.

1-я группа

1. № 597; № 599.

2. № 602.

Решение:

Выполним вычитание многочленов.

 = 0,6х2 – 0,4ху – 1,5у +
+ 1 – у2 + 0,4ху – 0,6х2 = 1 – 1,5уу2.

В полученный многочлен не входит переменная х, значит, исходное выражение не зависит от этой переменной.

3. № 610.

Решение:

а) Пусть п, п + 1 и п + 2 – три последовательных натуральных числа. Найдем их сумму:

п + п + 1 + п + 2 = 3п + 3.

Каждое  слагаемое  этой  суммы  кратно  трём,  значит,  и  вся  сумма кратна 3.

б) Пусть п, п + 1, п + 2, п + 3 – четыре последовательных натуральных числа. Найдем их сумму:

п + п + 1 + п + 2 + п + 3 = 4п + 6.

Первое слагаемое этой суммы кратно четырём, а второе – нет, значит, вся сумма не кратна четырем.

2-я группа

Представление многочлена в виде суммы или разности многочленов является обратной задачей к сложению и вычитанию многочленов. Поэтому начинать выполнение соответствующих упражнений можно, только убедившись, что учащиеся хорошо овладели умением находить сумму и разность многочленов.

Начать рассмотрение данного вопроса лучше с задачи.

Задача. После сложения одночлена с двучленом был получен многочлен 2а – 3b + 4с. Какой одночлен с каким многочленом был сложен?

Решение:

Важно, чтобы учащиеся поняли, что существует несколько вариантов, каждый из которых нужно рассмотреть:

2а + (–3b + 4с),    – 3b + (2а + 4с),                    4с + (2а – 3b).

После  этого  можно  переформулировать  задачу,  сказав,  что  много-член 2а – 3b + 4с был получен в результате вычитания многочлена из одночлена. Здесь также нужно рассмотреть все варианты:

2а – (3b – 4с),                  – 3b – (–2а – 4с),                   4с – (–2а + 3b).

В результате делается вывод о том, как представлять многочлен в виде суммы или разности многочленов, а затем приступить к выполнению соответствующих заданий.

1. Представьте выражение в виде суммы каких-нибудь двучленов:

а) 2а3 – 5а2а + 8;

б) –3у5 + 2у3 + 7у – 5.

2. Представьте выражение в виде разности одночлена и трех-члена:

а) у3 + 3у2 – 4у – 7;

б) 2р4 + р2 + 7р – 8.

3. Представьте многочлен в виде суммы двух многочленов, один из которых содержит переменную b, а другой нет:

а) bx2x + 1 – b;

б) a2b2 – 2ab – 1.

Решение:

а) bx2x + 1 – b = (bx2b) + (1 – х);

б) a2b2 – 2ab – 1 = (а2 – 1) + (–b2 – 2ab).

4. Представьте многочлен в виде разности двух многочленов с положительными коэффициентами:

а) рс + рс – 1;                          в) 3z – 5y – 2;

б) 8х – 3а – 1 + 24ах;                  г) –3а – 5b + 8.

Решение:

а) рс + рс – 1 = (рс + р) – (с + 1);

б) 8х – 3а – 1 + 24ах = (8х + 24ах) – (3а + 1);

в) 3z – 5y – 2 = 3z – (5y + 2);

г) –3а – 5b + 8 = 8 – (3а + 5b).

III. Итоги урока.

– Что называется многочленом? Степенью многочлена?

– Как выполнить сложение или вычитание многочленов?

– Как представить многочлен в виде суммы или разности двух многочленов?

– В многочлене 2а2а + 1 заключите в скобки два последних члена, поставив перед скобками сначала знак «+», а потом знак «–».

Домашнее задание: № 598; № 603; № 607; № 608.

 

 

 

 

 

 

 

 

 

 

 

 

 

Урок                                    Дата
ТЕМА: Правило умножения одночлена на многочлен

Цели: изучить правило умножения одночлена на многочлен; формировать умение применять это правило при преобразовании выражений.

Ход урока

I. Устная работа.

1. Раскройте скобки.

а) 3 (2х – 5);                     в) (4 + 2y);                      д)  · (–3);

б) (5а – 1) 4;                    г) –5 (3р – 8);             е) 0,7 (3а – 10).

2. Упростите выражение.

а) а5 · а7;                          в) аа2а3;                                 д) (п3)2 п4;

б) х8 : х3;                           г) (х2)5;                                   е) у2 у3(у4)2.

II. Объяснение нового материала.

В процессе выполнения устной работы у учащихся была возможность актуализировать необходимые знания. Поэтому при объяснении этого материала достаточно привести несколько примеров умножения одночлена на многочлен и сформулировать соответствующее правило.

Вопрос о решении уравнений с применением умножения одночлена на многочлен целесообразно рассмотреть на следующем уроке. Поэтому примеры 3 и 4 из учебника приводить не нужно.

III. Формирование умений и навыков.

На этом уроке основное внимание следует уделить формированию у учащихся умения непосредственно применять правило умножения одночлена на многочлен. Необходимо следить за грамотностью их записей, за обоснованием каждого шага, поддерживать внимание учащихся.

1. № 614; № 615 (а, в, д).

2. № 616.

Решение:

(На первых порах желательно, чтобы учащиеся (особенно слабые) вели подробные записи, это позволит избежать ошибок в преобразованиях.)

в)

г)

3. № 618 (а, в).

Решение:

(Здесь важно ещё раз напомнить учащимся о том, что перед нахождением значения любого выражения его сначала упрощают.)

в) 4у – 2 (10у – 1) + (8у – 24) = 4у – 20у + 2 + 8у – 24 = –8у – 22

при у = –0,1:       –8у – 22 = –8 · (–0,1) – 22 = 0,8 – 22 = –21,2.

4. № 619.

IV. Итоги урока.

– Как выполнить умножение одночлена на одночлен?

– Перемножьте одночлены –2х2 и 5х4.

– Сформулируйте правило умножения одночлена на многочлен.

– Умножьте одночлен 4а3 на многочлен 2а – 3.

Домашнее задание: № 617;  618 (б, г); № 620.

 

                                                                              

 

 

 

 

 

 

 

 

 

 

 

 

 

 

 

 

 

 

 

 

Урок                                          ДАТА
ТЕМА: Решение уравнений

Цели: продолжить формирование умения умножать одночлен на многочлен; формировать умение выполнять данное действие при решении уравнений.

Ход урока

I. Устная работа.

1. Выполните умножение одночленов.

а) 2х5 · 3х2;                       в) (–3b) · (–7b);                      д) (х2)3 · 5х;               

б) –4a3 · a;                   г) y7 · (–3y);                        е) .

2. Упростите выражение.

а) 2x (x2 – 4x);                  в) 4y;

б) –а2 (а + 8);                   г) p2(2p – 4).

II. Формирование умений и навыков.

Все задания можно разбить на три группы: в 1-ю и 2-ю группы войдут уравнения с целыми и дробными коэффициентами соответственно, а в 3-ю группу – задания на преобразование выражений.

1-я группа

1. № 630 (а, в, д, ж).

2. № 631 (а, в).

Решение:

а) 3х (2х – 1) – 6х (7 + х) = 90;

    6х2 – 3х – 42х – 6х2 = 90;

    –45х = 90;

    х =;

    х = –2.

Ответ: –2.

в) 5x (12x – 7) – 4x (15x – 11) = 30 + 29x;

    60x2 – 35x – 60x2 + 44x = 30 + 29x;

    –35х + 44 х – 29х = 30;

    –20х = 30;

    х = ;

    х = –1,5.

Ответ: –1,5.

2-я группа

Учащиеся должны осознать, что если в уравнении встречается дробь, то необходимо выполнить такое преобразование, которое приведёт к равносильному уравнению с целыми коэффициентами. Для этого обе части уравнения нужно умножить на наименьшее общее кратное знаменателей входящих в уравнение дробей.

1. № 634 (а, в, д, и).

2. № 636.

3. № 637.

Решение:

б) .

Умножим обе части уравнения на 30:

;

3 (a + 13) – 6 · 2a = 2 (3 – a) + 15a;

3а + 39 – 12а = 6 – 2а + 15а;

–9а – 13а = 6 – 39;

–22а = –33;

а = ;

а = 1,5.

Ответ: 1,5.

г

3-я группа

1. № 622.

2. № 629.

Решение:

Преобразуем данное выражение:

2x (x – 6) – 3 (x2 – 4x + 1) = 2x2 – 12x – 3x2 + 12x – 3 = – x2 – 3.

Очевидно, что при любом значении х значение выражения –х2 будет неположительным, тогда значение выражения –х2 – 3 будет отрицательным при любом значении х.

III. Итоги урока.

– Как выполнить умножение одночлена на одночлен?

– Сформулируйте правило умножения одночлена на многочлен.

– Как решить уравнение, в котором встречаются дроби?

Домашнее задание: № 632; № 634 (б, г, е, з); № 638; № 627.

 

 

 

 

 

 

Урок                                          Дата
Решение задач с помощью уравнений

Цели: формировать умение решать задачи с помощью уравнений; закрепить умение выполнять умножение одночлена на многочлен; проверить степень усвоения учащимися изученного материала.

Ход урока

I. Устная работа.

1. Выполните умножение одночленов.

а) 3а2 · (–2а);                                           г) x6 · (–4x);

б) 7b3 · b2;                                           д) (а2)4 · 2а;

в) –4с · (–2с5);                                         е) .

2. Упростите выражение.

а) 3а (4 – а2);                                           в) 2n ;

б) –х3 (х + 2);                                           г) y2(5 + 2y).

II. Проверочная работа.

Вариант 1

1. Упростите выражение.

а) 3p (8c + 1) – 8c (3p – 5);                                б) 5n2 (3n + 1) – 2n (5n2 – 3).

2. Решите уравнение.

а) 6x – 5 (3x + 2) = 5 (x – 1) – 8;             б)  = 2.

3. Преобразуйте в многочлен стандартного вида:

xt (x2t2xt – 3) · p.

III. Формирование умений и навыков.

1. № 642.

Решение:

Составим таблицу:

 

Было

Стало

1-й сарай

3х т

(3х – 2) т

2-й сарай

х т

(х + 2) т

Составим и решим уравнение.

x + 2 = (3x – 2);

7 (х + 2) = 5 (3х – 2);

7х + 14 = 15х – 10;

–8х = –24;

х = 3.

Значит, во втором сарае было 3 т сена, а в первом 9 т сена.

Ответ: 9 т, 3 т.

2. № 643.

Решение:

Составим таблицу:

 

А

k

t

По плану

х га

50 га/день

 дн.

Реально

х га

60 га/день

 дн.

Составим и решим уравнение:

 = 1;

300 = 300;

6х – 5х = 300;

х = 300.

Значит, площадь луга равна 300 га.

Ответ: 300 га.

3. № 646.

Решение:

Составим таблицу:

 

s

х

t

Велосипедист

х км

12 км/ч

 ч

Мотоциклист

(х + 60) км

30 км/ч

 ч

Составим и решим уравнение:

;

;

5х = 2 (х + 60);

5х = 2х + 120;

3х = 120;

х = 40.

Значит,  велосипедист  проехал  40 км  до  того,  как  его  догнал  мотоциклист.

Ответ: 40 км.

4. № 648.

Решение:

Представим наглядно описанную в задаче ситуацию.

Пусть первоначально в растворе было х г соли, то есть её концентрация была равна  ∙  100 % =  %.

В новом растворе уже имеется (х + 10) г соли, значит, её концентрация стала равна  ∙   100 % =  %. По условию концентрация соли в новом растворе повысилась на 4,5 %.

Составим и решим уравнение:

 = 4,5;

19(х + 10) – 20х = 38 · 4,5;

19х + 190 – 20х = 171;

х = –19;

х = 19.

Ответ: 19 г.

IV. Итоги урока.

– Сформулируйте правило умножения одночлена на многочлен.

– Умножьте одночлен –3х4 на многочлен 2х – 5.

– Как начать решение уравнения, в котором есть дроби?

– Как узнать концентрацию какого-либо вещества в растворе?

Домашнее задание: № 640; № 644; № 647; № 649.

 

 

 

Урок                                               Дата
ТЕМА: Разложение многочлена на множители способом вынесения
общего множителя за скобки

Цели: ввести понятие разложения многочлена на множитель; изучить способ вынесения общего множителя за скобки и формировать умение его применять.

Ход урока

I. Устная работа.

1. Выполните умножение.

а) 3x (2x2 – 5);                              в) 5y4 ;

б) a2 (a + 2);             г) –ab (a2b).

2. Найдите наибольший общий делитель чисел.

а) 10, 15 и 25                               в) 8, 12 и 16;

б) 6, 9 и 21;                                  г) 12, 18 и 30.

II. Объяснение нового материала.

Вынесение общего множителя за скобки является обратной задачей к умножению одночлена на многочлен. Поэтому данный материал будет понят учащимися только в том случае, если они хорошо усвоили предыдущую тему.

Объяснение проводится в несколько этапов.

1. Начать лучше с постановки проблемной задачи.

Задача. После умножения некоторого одночлена на некоторый многочлен был получен многочлен 4х2 – 6х4. Какой одночлен на какой многочлен умножали?

Учащиеся подбирают варианты:

2 (2х2 – 3х4), х (4х – 6х3), 2х2 (2 – 3х2) и т. п.

Можно рассмотреть ещё несколько подобных задач. Главное, чтобы учащиеся осознали, что такие задачи всегда имеют решение и являются обратными к выполнению умножения одночлена на многочлен.

2. Сообщить учащимся, что представление многочлена в виде произведения двух или нескольких многочленов называется разложением многочлена на множители.

Данная операция является очень полезной при решении ряда задач, которые впоследствии будут рассмотрены.

3. Вернуться к разложенным на множители многочленам и обратить внимание  учащихся,  что  для  задач  наиболее  целесообразным  является нахождение  «наибольшего»  общего  множителя  каждого  члена  много-члена. Поэтому в рассмотренном примере лучше записать следующее равенство:

4х2 – 6х4 = 2х2 (2 – 3х2).

Данный способ разложения многочлена на множители называется вынесением общего множителя за скобки.

4. Разобрать  несколько  примеров  вынесения  за  скобки  общего множителя:

а) 8х2у – 6х;

б) 3а4 + 9а2 – 6а;

в) пример 1 из учебника.

Сделать вывод: при вынесении общего множителя за скобки среди модулей коэффициентов берут их наибольший общий делитель, а переменные, выносимые за скобки, берут с наименьшим показателем.

III. Формирование умений и навыков.

1. № 654; № 655 (а, в, д, ж, и); № 656 (а, в, д).

В данных заданиях у многочленов общим множителем является либо только число, либо только буква. Необходимо, чтобы учащиеся сначала научились находить такие простые общие множители.

2. № 657 (а, в, д, и, л); № 659.

Здесь общие множители находить сложнее. Важно, чтобы учащиеся отыскивали правильно «наибольшие» общие множители.

№ 659.

Решение:

а) 14x + 21y = 7 (2x + 3y);

б) 15a + 10b = 5 (3a + 2b);

в) 8ab – 6ac = 2a (4b – 3c);

г) 9xa + 9xb = 9x (a + b);

д) 6ab – 3a = 3a (2b – 1);

е) 4x – 12x2 = 4x (1 – 3x);

ж) m4m2 = m2 (m2 – 1);

з) c3 + c4 = c3 (1 + c);

и) 7x – 14x3 = 7x (1 – 2x2);

к) 16y3 + 12y2 = 4y2 (4y + 3);

л) 18ab3 – 9b4 = 9b3 (2ab);

м) 4x3y2 – 6x2y3 = 2x2y2 (2x – 3y).

IV. Итоги урока.

– Что называется разложением многочлена на множители?

– Какой способ разложения многочлена на множители мы узнали на этом уроке?

– В чём состоит способ вынесения общего множителя за скобки?

– Как отыскивать выносимый за скобки общий множитель?

Домашнее задание: № 655 (б, г, е, з); № 656 (б, г, е); № 657 (б, г, е, з, м); № 658.

 

 

 

 

 

 

 

 

Урок                                        ДАТА
ТЕМА: Вынесение общего множителя за скобки при решении различных задач

Цели: продолжить  формирование  умения  выносить  за  скобки общий множитель;  проверить  степень  усвоения учащимися изученного материала.

Ход урока

I. Устная работа.

Найдите общий множитель членов многочлена.

а) 3a + 6b;                        г) 5а4 – 10а2;

б) х3 – 2х;                         д) –3а2сас;

в) 4xy + 6xz;                                 е) 12x – 16x2y.

Если его вынести за скобки, то какое выражение останется?

II. Объяснение нового материала.

На этом уроке учащиеся впервые встречаются с новым для них видом уравнений, поэтому данному вопросу следует уделить особое внимание.

Начать можно с рассматривания примера 4 из учебника и сделать соответствующие выводы. После этого учащиеся должны проговорить своими словами, как решаются подобные уравнения.

Следует обратить внимание учащихся, что по-другому такие уравнения решить нельзя. Это указывает на важность умения выносить за скобки общий множитель.

III. Формирование умений и навыков.

Все задания можно разбить на три группы.

1-я группа. задания на решение уравнений.

2-я группа. задания на нахождение значений выражений.

Эти группы заданий отражают, как может быть использован способ вынесения общего множителя за скобки.

3-я группа. задания на закрепление умения выносить общий множитель за скобки.

1-я группа

№ 661.

Решение:

г) 3х2 – 1,2х = 0;

    х (3х – 1,2) = 0;

    х = 0   или   3х – 1,2 = 0;

                           3х = 1,2;

                           х = 0,4.

Ответ: 0; 0,4.

                          

2-я группа

№ 660 (а, г).

Решение:

Важно, чтобы учащиеся увидели, как вынесение общего множителя за скобки помогает при нахождении значений выражений рациональным способом.

а) 3,28хх2 = х (3,28 – х)

при х = 2,28:       

х (3,28 – х) = 2,28 (3,28 – 2,28) = 2,28 · 1 = 2,28;

г) –mbm2 = –m (b + m)

при m = 3,48 и b = 96,52:

m (b + m) = –3,48 (96,52 + 3,48) = –3,48 · 100 = –348.

3-я группа

№ 664 (а, г); № 666.

Если на прошлом уроке учащиеся выносили за скобки общий множитель у двучленов, то в этих заданиях им придётся работать с трёхчленом.

№ 666.

Решение:

а) x3 – 3x2 + x = x (x2 – 3x + 1);

б) m2 – 2m3m4 = m2 (1 – 2mm2);

в) 4a5 – 2a3 + a = a (4a4 – 2a2 + 1);

г) 6x2 – 4x3 + 10x4 = 2x2 (3 – 2x + 5x2);

д) 15a3 – 9a2 + 6a = 3a (5a2 – 3a + 2);

е) –3m2 – 6m3 + 12m5 = –3m2 (1 + 2m – 4m3).

IV. Проверочная работа.

Вариант 1

1. Разложите на множители многочлен.

а) 5ab + 10a2;

б) 6x2 – 3x3 – 9x4;

в) 6c2x3 – 4c3x2 + 2c2x2.

2. Решите уравнение.

а) 2х2 + 4х = 0;

б) 3х – 5х2 = 0.

V. Итоги урока.

– Что называется разложением многочлена на множители?

– В чём состоит способ вынесения общего множителя за скобки?

– Как отыскать выносимый за скобки общий множитель?

– При решении каких заданий пригодится умение выносить за скобки общий множитель?

– Как решаются уравнения с помощью вынесения за скобки общего множителя?

Домашнее задание: № 660 (б, в); № 662; 664 (б, в); № 667.

 

                                          

 

 

 

                                   

 

                                           Урок                     Дата
ТЕМА: Вынесение общего множителя за скобки при решении различных задач

Цели: закрепить умение выносить за скобки общий множитель; рассмотреть, как используется это умение при решении вопроса о делимости и кратности чисел; формировать умение выносить за скобки двучлен.

Ход урока

I. Устная работа.

1. Вынесите за скобки общий множитель.

а) 5ab + 5ac;                    в) а3 + а5;                               д) 6х2 – 9х4;

б) х2ху;              г) n2m + nm2;             е) 8р3 – 12р.

2. Найдите корни уравнения:

а) (х + 1) (х – 1) = 0;                               в) х2 – 2х = 0;

б) (х – 3) (х + 2) = 0;                               в) х2 + 4х = 0.

II. Формирование умений и навыков.

Все задания можно разбить на три группы. В 1-ю группу войдут задания, в которых требуется применить умение выносить за скобки общий множитель для выяснения вопроса о делимости и кратности чисел. Во 2-й группе будут задания на закрепление умения выносить за скобки общий множитель. А 3-я группа состоит из заданий, в которых за скобки нужно вынести двучлен.

1-я группа

1. № 663 (а, в).

Решение:

а) Вынесем в сумме 165 + 164 за скобки общий множитель:

165 + 164 = 164 (16 + 1) = 164 · 17.

Так как в произведении 164 · 17 встречается множитель 17, то данное произведение кратно 17.

в) Преобразуем выражение и вынесем за скобки общий множитель:

365 – 69 = (62)5 – 69 = 610 – 69 = 69 (6 – 1) = 69 · 5 = 68 · 30.

Очевидно, что полученное произведение кратно 30.

2. № 665 (а, в).

а) Вынесем за скобки общий множитель:

78 – 77 + 76 = 76 (72 – 7 + 1) = 76 · 43.

Так как один из множителей полученного произведения делится на 43, то и всё произведение делится на 43.

в) Преобразуем выражение и вынесем за скобки общий множитель:

274 – 95 + 39 = (33)4 – (32)5 + 39 = 312 – 310 + 39 = 39 (33 – 3 + 1) = 39 · 25.

Так как один из множителей полученного произведения делится на 25, то и все произведение делится на 25.

2-я группа

№ 668.

Решение:

а)

б)

в)

г)

д)

е)

3-я группа

Прежде чем приступить к решению задач этой группы, нужно рассмотреть примеры 2 и 3 из учебника.

1. № 670.

Решение:

б) y (ab) – (ab) = (ab) (y – 1);

г)

д)
= (b – 2) (–3b + 7b – 14) = (b – 2) (4b – 14).

2. № 671.

Решение:

б) x (y – 5) – y (5 – y) = x (y – 5) + y (y – 5) = (y – 5) (x + y);

г)

е) (3 – b) (2 + 5 (3 – b)) =
= (3 – b) (2 + 15 – 5b) = (3 – b) (17 – 5b).

III. Итоги урока.

– Что называется многочленом? Стандартным видом многочлена?

– Сформулируйте правило сложения и вычитания многочленов.

– Как умножить одночлен на многочлен?

– Какое  преобразование  называется разложением многочлена на множители?

– В чём состоит способ вынесения общего множителя за скобки?

– Какой  общий  множитель  имеют  слагаемые  суммы  3х (а – 3) +
+ 2 (3 – а)2?

Домашнее задание: № 663 (б, г); № 665 (б, г); № 669; № 672.

 

 

 

 

 

 

 

 

Урок
ТЕМА: Контрольная работа

Вариант 1

1. Выполните действия.

а) (3a – 4ax + 2) – (11a – 14ax);

б) 3y2 (y3 + 1).

2. Вынесите общий множитель за скобки.

а) 10ab – 15b2;                             б) 18а3 + 6а2.

3. Решите уравнение  9х – 6 (х – 1) = 5 (х + 2).

4. Пассажирский поезд за 4 ч прошёл такое же расстояние, какое товарный за 6 ч. Найдите скорость пассажирского поезда, если известно, что скорость товарного на 20 км/ч меньше.

5. Решите уравнение  .

6. Упростите выражение  2a (a + bc) – 2b (abc) + 2c (ab + c).

Вариант 2

1. Выполните действия.

а) (2a2 – 3a + 1) – (7a2 – 5a);

б) 3х (4х2х).

2. Вынесите общий множитель за скобки.

а) 2ху – 3ху2;                               б) 8b4 + 2b3.

3. Решите уравнение  7 – 4 (3х – 1) = 5 (1–2х).

4. В трех шестых классах 91 ученик. В 6 «А» на 2 ученика меньше, чем в 6 «Б», а в 6 «В» на 3 ученика больше, чем в 6 «Б». Сколько учащихся в каждом классе?

5. Решите уравнение  .

6. Упростите выражение  3x (x + y + c) – 3y (xyc) – 3c (x + yc).

Решение заданий контрольной работы

Вариант 1

1. а) (3a – 4ax + 2) – (11a – 14ax) = 3a – 4ax + 2 – 11a + 14ax =

= 10ax – 8a + 2;

    б) 3y2 (y3 + 1) = 3y5 + 3y2.

2. а) 10ab – 15b2 = 5b (2a – 3b);

    б) 18а3 + 6а2 = 6а2 (3а + 1).

3. 9х – 6 (х – 1) = 5 (х + 2);

    9х – 6х + 6 = 5х + 10;

    3х – 5х = 10 – 6;

    –2х = 4;

    х = –2.

Ответ: –2.

4. Составим таблицу:

 

s

х

t

Пассажирский поезд

4х км

х км/ч

4 ч

Товарный поезд

6 (х – 20) км

(х – 20) км/ч

6 ч

Известно,  что  поезда  прошли  одинаковое  расстояние.  Получим уравнение:

4х = 6 (х – 20);

4х = 6х – 120;

–2х = –120;

х = 60.

Ответ: 60 км/ч.

5. .

Умножим обе части уравнения на 18:

;

3 (3х – 1) – 6х = 2 (5 – х);

9х – 3 – 6х = 10 – 2х;

3х + 2х = 10 + 3;

5х = 13;

х = ;

х = 2,6.

Ответ: 2,6.

6.
–2ac – 2ab + 2b2 + 2bc + 2ac – 2bc + 2c2 = 2a2 + 2b2 + 2c2.

Вариант 2

1. а) (2a2 – 3a + 1) – (7a2 – 5a) = 2a2 – 3a + 1 – 7a2 + 5a = –5a2 + 2a + 1;

    б) 3х (4х2х) = 12х3 – 3х2.

2. а) 2ху – 3ху2 = ху (2 – 3у);

    б) 8b4 + 2b3 = 2b3 (4b + 1).

3. 7 – 4 (3х – 1) = 5 (1–2х);

    7 – 12х + 4 = 5 – 10х;

    – 12х + 10х = 5 – 11;

    –2х = –6;

    х = 3.

Ответ: 3.

4. Пусть в 6 «Б» классе всего х учеников. Тогда в 6 «А» (х – 2) ученика, а в 6 «В» (х + 3) ученика.

По  условию  всего  в  трех  классах  91  ученик.  Составим  и  решим уравнение.

х + (х – 2) + (х + 3) = 91;

х + х – 2 + х + 3 = 91;

3х = 90;

х = 30.

Значит, в 6 «Б» классе 30 учеников. Тогда в 6 «А» 28 учеников, а в 6 «В» 33 ученика.

Ответ: 28, 30 и 33 ученика.

5. .

Умножим обе части уравнения на 20.

4 (х – 1) = 10 (5 – х) + 15х;

4х – 4 = 50 – 10х + 15х;

4х – 5х = 50 + 4;

х = 54;

х = –54.

Ответ: –54.

6.
+ 3хc – 3хy + 3y2 + 3yc – 3хc – 3yc + 3c2 = 3х2 + 3y2 + 3c2.

 

 

 

 

 

 

 

 

 

 

 

 

 

 

 

 

Урок                                        Дата
ТЕМА: Изучение правила умножения многочлена на многочлен

Цели: вывести правило умножения многочлена на многочлен и формировать умение применять это правило.

Ход урока

I. Устная работа.

Выполните умножение.

а) а (ху);                                   б) p (3 – q);             в) –2х (х – 4);

г) 4y ;             д) c2 (c3 + 2);                  е) –5х (3х2 – 4);

ж) 2a4 ;                       з) –q7 (q3q5).

II. Объяснение нового материала.

Объяснение проводится в несколько этапов согласно материалу учебника.

1. Вывести правило умножения многочлена на многочлен и наглядно представить его на доске:

2. Сформулировать полученное правило, попросить нескольких учащихся повторить его.

3. Разобрать примеры применения правила.

Поскольку данная тема является новой для учащихся, целесообразно привести несколько несложных примеров непосредственного применения правила умножения двух многочленов. Примеры использования этого правила при решении ряда задач лучше рассмотреть на следующих уроках.

Пример 1.

Пример 2.

Пример 3.

III. Формирование умений и навыков.

За урок следует опросить как можно больше учащихся, чтобы убедиться, что они усвоили правило умножения многочлена на многочлен. Поэтому для выполнения каждого задания к доске можно вызывать сразу трёх учащихся.

1. № 677, № 678.

В этих заданиях на умножение многочленов каждый из множителей является линейным. Важно, чтобы учащиеся следили за точностью применения соответствующего правила и не ошибались в знаках.

2. № 680.

Эти  задания  несколько  сложнее,  поскольку  помимо  применения правила  умножения  многочленов  учащиеся  должны  помнить  свойства степеней.

Решение:

а) ;

б) ;

в) 12a4a2b2b4;

г) ;

д)

е) 56p3 – 51p2 + 10p.

3. № 682 (а, в).

Решение:

а) (х + 10)2 = (х + 10) (х + 10) = х2 + 10х + 10х + 100 = х2 + 20х + 100;

в) (3а – 1)2 = (3а – 1) (3а – 1) = 9а2 – 3а – 3а – 1 = 9а2 – 6а + 1.

IV. Итоги урока.

– Как умножить одночлен на многочлен?

– Сформулируйте правило умножения многочлена на многочлен.

– Какие знаки будут иметь слагаемые, полученные при умножении многочленов: а) (х + у) (аb);    б) (n m) (pq)?

Домашнее задание: № 679; № 681; № 682 (б, г).

 

 

 

 

 

 

 

 

 

 

 

 

 

Урок                                     дата
ТЕМА: Применение правила умножения многочлена на многочлен

Цели: продолжить формирование умения умножать многочлены; проверить уровень усвоения изучаемого материала.

Ход урока

I. Устная работа.

1. Выполните умножение.

а) 3х2 · 4х3;                                  в) –0,4а2 · (–2а4);                  д) –5у2 (2у – 3);

б) –12y · y5;                             г) x (3x2 + 1);                     е) 2p5 .

2. Сколько слагаемых получится со знаком «плюс» (+) и сколько со знаком «минус» (–) при умножении следующих многочленов:

а) (2 + а) (х + 4);              в) (с – 8) (1– d);

б) (у – 4) (а2 + 5);            г) (–а – 3) (b – 2)?

II. Формирование умений и навыков.

На этом уроке учащимся предстоит выполнить более сложные преобразования. Сначала необходимо рассмотреть примеры 1 и 2 из учебника.

1. № 683 (а, в, д, ж).

Важно, чтобы учащиеся осознали, что при умножении многочлена, содержащего т членов, на многочлен, содержащий п членов, в произведении должно получиться тп членов (до приведения подобных).

Решение:

а) x3 + 2x2yy3;

в) a3 – 2ax2x3;

д) (a2 – 2a + 3) (a – 4) = a3 – 4a2 – 2a2 + 8a + 3a – 12 = a3 – 6a2 +
+ 11a – 12;

ж) x3 + 3x2
– 8x + 10.

2. Представьте в виде многочлена.

а) x2 (x + 3) (x – 2);

б) –2y3 (y – 1) (y + 4);

в) (a + 1) (a – 2) (a + 5).

Решение:

а)
= x4 + x3 – 6x2.

б)
= –8y5 – 6y4 + 8y3;

в) (a + 1) (a – 2) (a + 5) = (a2 – 2a + a – 2) (a + 5) = (a2a – 2) (a + 5) =
= a3 + 5a2a2 – 5a – 2a – 10 = a3 + 4a2 – 7a – 10.

3. № 687 (а, в, д).

Важно, чтобы учащиеся были внимательны при раскрытии скобок, перед которыми стоит знак «–». Если это вызывает у них затруднения, то можно сначала выполнять умножение многочленов, а потом раскрывать скобки.

Решение:

в) + 9x = 9x;

д) (ab) (a + 2) – (a + b) (a – 2) = a2 + 2aab – 2b – (a2 – 2a +
+ ab – 2b) = a2 + 2aab – 2ba2 + 2aab + 2b = 4a – 2ab.

4. № 689.

Решение:

Согласно условию запишем выражение acbd:

III. Проверочная работа.

Вариант 1

1. Выполните умножение.

а) (a + 3) (b – 7);                          в) (x + 2) (x2x – 3);

б) (3x2 – 1) (2x + 1);                                г) –4 (y – 1) (y + 5).

2. Упростите выражение.

8p – (3p + 8) (2p – 5).

Вариант 2

1. Выполните умножение

а) (x + 4) (y – 5);                          в) (a – 3) (a2 + a – 2);

б) (5y2 + 1) (3y – 2);                                г) –3 (x + 4) (x – 1).

2. Упростите выражение

5y2 – (3y – 1) (5y – 2).

IV. Итоги урока.

– Сформулируйте правило умножения многочлена на многочлен.

– Как перемножить три многочлена?

– Сколько слагаемых получится при умножении многочлена, содержащего т членов, на многочлен, содержащий п членов?

Домашнее задание: № 684; № 685; № 686; № 687 (б, г).

 

 

 

 

 

 

 

Урок                                       Дата
ТЕМА: Доказательство тождеств и утверждений

Цели: продолжить  формирование  умения  умножать  многочлены; применять  это  умение  для  доказательства  тождеств  и  некоторых утверждений.

Ход урока

I. Устная работа.

1. Выполните умножение.

а) x2 · 7x5;                                            г) 2х (х2 – 7х);

б) –8а · 4а4;                                             д) –4p4 ;

в) –6y3 · ;                               е) –3п5 (п3 – 2п).

2. Сколько слагаемых получится со знаком «+» и сколько со знаком «–» при умножении многочленов:

а) (a + 2) (b + 5);                         в) (n2 – 3) (m – 5);

б) (х – 3) (у + 7);                          г) (–а – 2) (с – 4)?

II. Формирование умений и навыков.

Все задания можно разбить на две группы. В 1-ю группу войдут задания на доказательство тождеств, а во 2-ю группу – на доказательство утверждений о делимости, кратности и др.

1-я группа

Прежде чем приступить к выполнению заданий этой группы, нужно вспомнить логику доказательства тождеств.

Для наглядности можно вынести на доску схему:

1)              2)               3) 

То есть существует три основных приема доказательства тождеств:

1) преобразовать левую часть тождества в правую или правую часть тождества в левую;

2) показать, что левая и правая части исходного равенства тождественно равны одному и тому же выражению;

3) показать, что разность левой и правой части исходного равенства тождественно равна нулю.

1. № 690 (а), № 691 (а).

При доказательстве этих тождеств используется первый прием, то есть мы будем преобразовывать одну часть равенства до тех пор, пока она не станет тождественно равной другой части равенства.

2. № 692 (а).

При доказательстве этого тождества используется второй прием.

Решение:

а) (x – 3) (x + 7) – 13 = (x + 8) (x – 4) – 2.

Преобразуем левую часть равенства:

Преобразуем правую часть равенства:

Получаем следующее: левая и правая части равенства тождественно равны одному и тому же выражению, значит, исходное равенство является тождеством.

2-я группа

1. № 693.

Решение:

а) Упростим данное выражение:

Получаем, что исходное выражение равно числу –36, значит, не зависит от переменной х.

б)

2. № 699 (а).

Решение:

а) Упростим данное выражение:

Поскольку каждое слагаемое суммы 6п + 6 кратно 6, то и вся сумма кратна 6.

3. № 696.

Решение:

Четыре последовательных нечётных числа можно записать в следующем виде:

а = 2п + 1,   b = 2п + 3,   с = 2п + 5   и   d = 2п + 7.

Составим разность cdab:

(2n + 5) (2n + 7) – (2n + 1) (2n + 3).

Преобразуем это выражение:


– 6n – 2n – 3 = 16n + 32 = 16 (n + 2).

Очевидно, что полученное выражение кратно 16.

III. Итоги урока.

– Сформулируйте правило умножения многочлена на многочлен.

– Как перемножить три многочлена?

– Какие существуют приемы доказательства тождеств?

Домашнее задание:  № 690 (б);  № 691 (б);  № 692 (б);  № 694;
№ 695 (б).

 

 

 

 

 

Урок                                   Дата
ТЕМА: Решение уравнений и задач на составление уравнений

Цели: закрепить умение умножать многочлены; рассмотреть применение данного умения при решении уравнений и текстовых задач; проверить уровень усвоения материала.

Ход урока

I. Устная работа.

Выполните умножение.

а) 2a3 · a5;                           г) 4а2 (2а – 7);                ж) (а + 2) (b – 7);

б) –0,7х2 · 5х8;                        д) x (2x – 5x2);                     з) (х – 3) (2 – у);

в) y · (–6y4);         е) –3р4 (2р2 – 5р3);                     и) (2х2 – 1) (х4 + 3);

к) (–2 – п) (т – 5).

II. Формирование умений и навыков.

Задания можно разбить на две группы. 1-я группа – это задания, в которых требуется использовать умение выполнять умножение многочленов для решения уравнений. А во 2-ю группу войдут задачи на составление уравнений.

1-я группа

№ 697.

Решение:

б) (1 – 2х) (1 – 3х) = (6х – 1) х – 1;

    1 – 3х – 2х + 6х2 = 6х2х – 1;

    6х2 – 5х + 1 – 6х2 + х + 1 = 0;

    –4х = –2;

    х = .

Ответ: .

г) (х + 4) (х + 1) = х – (х – 2) (2 – х);

    х2 + х + 4х + 4 = х – 2х + х2 + 4 – 2х;

    х2 + 5х + 4 – х2 + 4х – 4 = 0;

    9х = 0;

    х = 0.

Ответ: 0.

2-я группа

1. № 701.

Решение:

Пусть  даны  три  последовательных  нечётных  числа:  2п + 1,  2п + 3,
2п + 5. Найдем произведение двух больших из них: (2п + 3) (2п + 5) и произведение двух меньших: (2п + 1) (2п + 3). По условию разность между этими произведениями равна 76.

Составим и решим уравнение.

(2п + 3) (2п + 5) – (2п + 1) (2п + 3) = 76.

4п2 + 10п + 6п + 15 – 4п2 – 6п – 2п – 3 = 76;

8п + 12 = 76;

8п = 64;

п = 8.

Найдем числа:                2п + 1 = 2 · 8 + 1 = 17.

                                         2п + 3 = 2 · 8 + 3 = 19.

                                         2п + 5 = 2 · 8 + 5 = 21.

Ответ: 17, 19 и 21.

2. № 702.

Решение:

Пусть  длина  прямоугольника  равна  х см,  тогда  его  ширина  равна
(35 – х) см. Значит, этот прямоугольник имеет площадь х (35 – х) см2.

Длину уменьшили на 5 см, и она стала равна (х – 5) см, а ширину увеличили на 5 см, и она стала равна (40 – х) см. Тогда площадь нового прямоугольника стала (х – 5) (40 – х) см2. По условию эта площадь на 50 см2 больше, чем площадь данного прямоугольника.

Составим и решим уравнение:

(х – 5) (40 – х) – х (35 – х) = 50;

40хх2 – 200 + 5х – 35х + х2 = 50;

10х – 200 = 50;

10х = 250;

х = 25.

Значит, длина исходного прямоугольника равна 25 см, тогда его ширина равна 10 см.

Ответ: 25 см и 10 см.

В процессе решения задач сильным учащимся дополнительно можно предложить выполнить задания на карточках.

Карточка № 1

1. Преобразуйте произведение в многочлен стандартного вида:

2. Докажите, что значение выражения (163 – 83) (43 + 23) делится на 63.

3. Докажите, что произведение двух средних из четырех последовательных целых чисел на 2 больше произведения крайних чисел.

Карточка № 2

1. Преобразуйте произведение в многочлен стандартного вида:

2. Докажите,  что  значение  выражения  (1252 + 252) (52 – 1)  делится
на 39.

3. Докажите, что квадрат среднего из трёх последовательных нечётных чисел на 4 больше произведения двух крайних чисел.

Решение заданий на карточках

Карточка № 1

1.

+ 56m3 – 7m4 + 12m – 32m2 + 4m3 = 2m5 – 23m4 + 64m3 – 53m2 + 12m.

2. Преобразуем  данное  выражение  и  вынесем  за  скобки  общий множитель:

(163 – 83) (43 + 23) = (212 – 29) (26 + 23) = 29(23 – 1) · 23 (23 + 1) =
= 212 · 7 · 9 = 212 · 63.

Очевидно, что данное произведение делится на 63.

3. Пусть даны четыре последовательных целых числа:  пп + 1,  п + 2, п + 3. Произведение средних чисел равно (п + 1) (п + 2), а произведение крайних чисел равно п (п + 3).

Составим разность и упростим её:

(п + 1) (п + 2) – п (п + 3) = п2 + 2п + п + 2 – п2 – 3п = 2.

Утверждение доказано.

Карточка № 2

1.

2. Преобразуем  данное  выражение  и  вынесем  за  скобки  общий множитель:

(1252 + 252) (52 – 1) = (56 + 54) (52 – 1) = 54 (52 + 1) (52 – 1) =
= 54 · 26 · 24 = 54 · 2 · 13 · 8 · 3 = 54 · 16 · 39.

Очевидно, что данное произведение делится на 39.

3. Пусть даны три последовательных нечётных числа:  2п + 1,  2п + 3,
2п + 5. Квадрат среднего из них равен (2п + 3)2, а произведение крайних равно (2п + 1) (2п + 5).

Составим разность и упростим её:

(2п + 3)2 – (2п + 1) (2п + 5) = (2п + 3) (2п + 3) – (2п + 1) (2п + 5) =
= 4п2 + 6п + 6п + 9 – 4п2 – 10п – 2п – 5 = 4.

Утверждение доказано.

III. Проверочная работа.

Вариант 1

1. При каком значении х равны значения следующих выражений:

(3х + 5) (4х – 1)    и    (6х – 3) (2х + 7)?

2. Упростите выражение.

а)

б)

Вариант 2

1. При каком значении а равны значения следующих выражений:

(5а + 1) (2а – 3)    и    (10а – 3) (а + 1)?

2. Упростите выражение.

а)

б)

IV. Итоги урока.

– Сформулируйте правило умножения многочлена на многочлен.

– Как перемножить три многочлена?

Домашнее задание: № 698; № 700; № 703.

 

 

Урок 77
Изучение способа группировки разложения
многочлена на множители

Цели: познакомить учащихся со способом группировки разложения многочлена на множители; формировать умение применять этот способ.

Ход урока

I. Устная работа.

1. Вычислите.

а) (–0,1)2 + (–0,2)2;             в) – (0,1 – 0,2)2;             д) ;

б) (–0,1 – 0,2)2;                   г) ;                       е) .

2. Разложите многочлен на множители.

а) aba2b;                в) 6у5 – 9у2;                    д) 3 (ab) – x (ab);

б) 2х3 + 4х;                г) n2m3 + n3m;                е) (у + 2)2х (у + 2).

II. Объяснение нового материала.

Данная тема зачастую вызывает затруднения у учащихся. Связано это с формальным усвоением способа группировки, с непониманием его сути и того, что этот способ является обратной задачей к умножению многочлена на многочлен.

Поэтому, прежде чем изучать способ группировки, целесообразно будет вынести на доску пример, отражающий пошаговое умножение двучлена на двучлен. А потом на этом же примере рассмотреть обратную задачу.

(b + 3) (а – 2)

1-й шаг.    b (а – 2) + 3(а – 2)

2-й шаг.    (аb – 2b) + (3а – 6)

3-й шаг.    аb – 2b + 3а – 6

аb – 2b + 3а – 6

1-й шаг.    (аb – 2b) + (3а – 6)

2-й шаг.    b (а – 2) + 3(а – 2)

3-й шаг.    (а – 2) (b + 3)

Затем можно рассмотреть пример 2 из учебника.

Важно, чтобы учащиеся поняли, что из трёх возможных вариантов группировки первого члена два являются верными, а один не даёт результата. Учащиеся могут убедиться в этом на конкретном примере:

1) ху + 4х – 2у – 8 = (ху + 4х) – (2у + 8) = х (у + 4) – 2 (у + 4) =
= (у + 4) (х – 2).

2) ху + 4х – 2у – 8 = (ху – 2у) + (4х – 8) = у (х – 2) + 4 (х – 2) =
= (х – 2) (у + 4).

3) ху + 4х – 2у – 8 = (ху – 8) + (4х – 2у) – не даёт результата.

Пример 3 из школьного учебника лучше рассмотреть на следующем уроке.

III. Формирование умений и навыков.

На этом уроке нужно опросить как можно больше учащихся, чтобы убедиться, что они усвоили способ группировки разложения многочлена на множители.

К доске на одно задание можно вызывать двух учащихся, которые будут группировать члены многочлена по-разному, а затем убеждаться, что результат получен одинаковый.

1. № 708, № 709.

2. № 711 (а, в, д, з).

Решение:

(На первых порах нужно требовать от учащихся подробных записей.)

а) х3 + х2 + х + 1 = (х3 + х2) + (х + 1) = х2 (х + 1) + (х + 1) = (х + 1) (х2 + 1).

в) а4 + 2а3а – 2 = (а4 + 2а3) – (а + 2) = а3 (а + 2) – (а + 2) =
= (а + 2) (а3 – 1).

д) а2ab – 8а + 8b = (а2ab) – (8а – 8b) = а (ab) – 8 (аb) =
= (ab) (а – 8).

з) knmnn2 + mk = (kn + mk) – (mn + n2) = k (n + m) – n (m + n) =
= (m + n) (k n).

IV. Итоги урока.

– Как умножить многочлен на многочлен?

– Что является обратной задачей к умножению многочленов?

– Опишите алгоритм способа группировки разложения многочлена на множители.

– Сколько существует вариантов группировки первого члена многочлена, содержащего 4 слагаемых? Сколько из этих вариантов дадут возможность разложить многочлен на множители?

Домашнее задание: № 710; № 711 (б, г, е); № 712.

 

 

Урок 78
Применение способа группировки
разложения многочлена на множители

Цели: продолжить формирование умения применять способ группировки при разложении многочлена на множители; проверить уровень усвоения материала.

Ход урока

I. Проверочная работа.

Вариант 1

1. Вынесите за скобки общий множитель.

а) a (b + c) + p (b + c);                  в) 3 (x – 2) + y (2 – x)2.

б) 7 (xc) + (cx) xc;

2. Разложите многочлен на множители (проверьте полученный результат умножением).

а) ax + bx + ac + bc;                     в) 2x2 – 3x + 4ax – 6a.

б) 6x + 7y + 42 + xy;

Вариант 2

1. Вынесите за скобки общий множитель.

а) a (x + c) – b (x + c);                  в) 2 (x – 7) – p (7 – x)2.

б) 9 (ab) – (ba) ab;

2. Разложите многочлен на множители (проверьте полученный результат умножением).

а) axay + bxby;                     в) ay – 12bx + 3ax – 4by.

б) 2x + 7y + 14 + xy;

II. Формирование умений и навыков.

Все задания можно разбить на две группы. В 1-ю группу войдут задания на применение способа группировки при доказательстве тождеств и нахождении значений выражений. А во 2-ю группу войдут сложные задания, в которых нужно разложить на множители многочлены способом группировки.

1-я группа

1. № 713.

Важно, чтобы учащиеся поняли, что непосредственная подстановка данных значений переменных приведет к громоздким вычислениям.

Решение:

а) p2q2 + pqq3p3 = (p2q2q3) + (pqp3) = q2 (p2q) + p (qp2) =
= q2 (p2q) – p (p2q) = (p2q) (q2p).

При p = 0,5 и q = –0,5:

(p2q) (q2p) = (0,25 + 0,5) (0,25 – 0,5) = 0,75 · (–0,25) =
=.

б) 3х3 – 2у3 – 6х2у2 + ху = (3х3 – 6х2у2) – (2у3 ху) = 3х2 (х – 2у2) –
у (2у2х) = 3х2 (х – 2у2) + у (х – 2у2) = (х – 2у2) (3х2 + у).

При x =  и у = :

(х – 2у2) (3х2 + у) =

2. № 715.

Заметим, что, исходя из логики доказательства тождеств, можно преобразовать левую часть равенства в правую (для этого многочлен нужно разложить на множители), а можно преобразовать правую часть в левую (для этого нужно перемножить двучлены).

2-я группа

1. № 716.

До этого учащиеся использовали способ группировки для разложения на множители многочленов, состоящих из четырёх членов. Нужно обратить внимание учащихся, что это самый распространенный случай применения данного способа. Но иногда способ группировки может быть использован при разложении на множители многочленов с другим количеством членов.

Решение:

а) (bdadcd) =

б) (bx2 + by2b) =

в) (cn2cp + cp2) =

г) (axab + a) =

2. № 718 (а, в).

Прежде  чем  решать  этот  номер,  нужно  рассмотреть  пример  3  из учебника.

Решение:

а) x (x + 1) +
+ 5 (x + 1) = (x + 1) (x + 5).

в) a (a – 1) –
– 4 (a – 1) = (a – 1) (a – 4).

III. Итоги урока.

– Какие вы знаете способы разложения многочлена на множители?

– Опишите алгоритм способа группировки.

– Сколько членов содержали многочлены, которые мы раскладывали на множители способом группировки?

Домашнее задание: № 714; № 717; № 718 (б, г).

 

Урок 79
Контрольная работа № 6

Вариант 1

1. Выполните умножение.

а) (с + 2) (с – 3);                          в) (5х – 2у) (4ху);

б) (2а – 1) (3а + 4);                      г) (а – 2) (а2 – 3а + 6).

2. Разложите на множители.

а) а (а + 3) – 2 (а + 3);                 б) ахау + 5х – 5у.

3. Упростите выражение  –0,1х (2х2 + 6) (5 – 4х2).

4. Представьте многочлен в виде произведения.

а) х2ху – 4х + 4у;                      б) abacbx + cx + cb.

5. Из прямоугольного листа фанеры вырезали квадратную пластинку, для чего с одной стороны листа отрезали полосу шириной 2 см, а с другой, соседней, – 3 см. Найдите сторону получившегося квадрата, если известно, что его площадь на 51 см2 меньше площади прямоугольника.

Вариант 2

1. Выполните умножение.

а) (а – 5) (а – 3);                          в) (3р + 2с) (2р + 4с);

б) (5х + 4) (2х – 1);                      г) (b – 2) (b2 + 2b – 3).

2. Разложите на множители.

а) x (xy) + a (xy);                   б) 2a – 2b + cacb.

3. Упростите выражение  0,5x (4x2 – 1) (5x2 + 2).

4. Представьте многочлен в виде произведения.

а) 2aac – 2c + c2;                      б) bx + byxyaxay.

5. Бассейн имеет прямоугольную форму. Одна из его сторон на 6 м больше другой. Он окружен дорожкой, ширина которой 0,5 м. Найдите стороны бассейна, если площадь окружающей его дорожки 15 м2.

Вариант 3

1. Выполните умножение.

а) (х – 8) (х + 5);                          в) (6а + х) (2а – 3х);

б) (3b – 2) (4b – 2);                      г) (с + 1) (с2 + 3с + 2).

2. Разложите на множители.

а) 2x (x – 1) – 3 (x – 1);                б) ab + ac + 4b + 4c.

3. Упростите выражение  –0,4a (2a2 + 3) (5 – 3a2).

4. Представьте многочлен в виде произведения.

а) a2 + ab – 3a – 3b;                     б) kpkcpx + cx + cp.

5. Из квадратного листа фанеры вырезали прямоугольную дощечку, одна из сторон которой на 2 см, а другая на 3 см меньше стороны квадрата. Найдите сторону квадратного листа, если его площадь на 24 см2 больше площади получившейся дощечки.

Вариант 4

1. Выполните умножение.

а) (а – 4) (а – 2);                          в) (3у – 2с) (у + 6с);

б) (3х + 1) (5х – 6);                      г) (b + 3) (b2 + 2b – 2).

2. Разложите на множители.

а) 2x (ab) + a (ab);                б) 3x + 3y + bx + by.

3. Упростите выражение  0,2y (5y2 – 1) (2y2 + 1).

4. Представьте многочлен в виде произведения.

а) 3xxy – 3y + y2;                      б) axay + cycxx + y.

5. Клумба прямоугольной формы окружена дорожкой, ширина которой 1 м. Площадь дорожки 26 м2. Найдите стороны клумбы, если одна из них на 5 м больше другой.

Решение заданий контрольной работы

Вариант 1

1. а) (с + 2) (с – 3) = с2 – 3с + 2с – 6 = с2с – 6.

    б) (2а – 1) (3а + 4) = 6а2 + 8а – 3а – 4 = 6а2 + 5а – 4.

    в) (5х – 2у) (4ху) = 20х2 – 5ху – 8ху + 2у2 = 20х2 – 13ху + 2у2.

    г) (а – 2) (а2 – 3а + 6) = а3 – 3а2 + 6а – 2а2 + 6а – 12 =
= а3 – 5а2 + 12а – 12.

2. а) а (а + 3) – 2 (а + 3) = (а + 3) (а – 2).

    б) ахау + 5х – 5у = (ахау) + (5х – 5у) = а(ху) + 5(ху) =

= (ху) (а + 5).

3. –0,1х (2х2 + 6) (5 – 4х2) = –0,1х (10х2 – 8х4 + 30 – 24х2) = –х3 +
+ 0,8х5 – 3х + 2,4х3 = 0,8х5 + 1,4х3 – 3х.

4. а) х2ху – 4х + 4у = (х2ху) – (4х – 4у) = х(ху) – 4(ху) =
= (ху) (х – 4).

    б) abacbx + cx + cb = (abac) – (bxcx) – (bc) =
= a (bc) – x (bc) – (bc) = (bc) (ax – 1).

5. Пусть сторона получившегося квадрата равна х см, тогда его площадь равна х2 см2. Стороны прямоугольника равны (х + 2) см и (х + 3) см, значит, его площадь равна (х + 2) (х + 3) см2.

Составим и решим уравнение:

(х + 2) (х + 3) – х2 = 51;

х2 + 3х + 2х + 6 – х2 = 51;

5х = 45;

х = 9.

Ответ: 9 см.

Вариант 2

1. а) (а – 5) (а – 3) = а2 – 3а – 5а + 15 = а2 – 8а + 15.

    б) (5х + 4) (2х – 1) = 10х2 – 5х + 8х – 4 = 10х2 + 3х – 4.

    в) (3р + 2с) (2р + 4с) = 6p2 + 12cp + 4cp + 8c2 = 6p2 + 16cp + 8c2.

    г) (b – 2) (b2 + 2b – 3) = b3 + 2b2 – 3b – 2b2 – 4b + 6 = b3 – 7b + 6.

2. а) x (xy) + a (xy) = (xy) (x + a).

    б) 2a – 2b + cacb = (2a – 2b) + (cacb) = 2 (ab) + c (ab) =
= (ab) (2 + c).

3. 0,5x (4x2 – 1) (5x2 + 2) = 0,5x (20x4 + 8x2 – 5x2 – 2) = 10x5 + 4x3
– 2,5x3x = 10x5 + 1,5x3x.

4. а) 2aac – 2c + c2 = (2a – 2c) – (acc2) = 2 (ac) – c (ac) =
= (ac) (2 – c).

    б) bx + byxyaxay = (bx + by) – (x + y) – (ax + ay) =
= b (x + y) – (x + y) – a (x + y) = (x + y) (b a – 1).

5. Пусть одна сторона бассейна х м, тогда другая его сторона (х + 6) м. Значит, площадь бассейна х (х + 6) м2.

Найдем площадь бассейна вместе с окружающей его дорожкой. Фигура является прямоугольником, стороны которого равны (х + 1) м и (х + 7) м. Значит, площадь прямоугольника равна (х + 1) (х + 7) м2.

Составим и решим уравнение:

(х + 1) (х + 7) – х (х + 6) = 15;

х2 + 7х + х + 7 – х2 – 6х = 15;

2х = 8;

2х = 4.

Ответ: 4 м и 10 м.

Вариант 3

1. а) (х – 8) (х + 5) = х2 + 5х – 8х – 40 = х2 – 3х – 40.

    б) (3b – 2) (4b – 2) = 12b2 – 6b – 8b + 4 = 12b2 – 14b + 4.

    в) (6а + х) (2а – 3х) = 12a2 – 18ax + 2ax – 3x2 = 12a2 – 16ax – 3x2.

    г) (с + 1) (с2 + 3с + 2) = с3 + 3с2 + 2с + с2 + 3с + 2 = с3 + 4с2 + 5с + 2.

2. а) 2x (x – 1) – 3 (x – 1) = (x – 1) (2x – 3).

    б) ab + ac + 4b + 4c = (ab + ac) + (4b + 4c) = a (b + c) + 4 (b + c) =
= (b + c) (a + 4).

3. –0,4a (2a2 + 3) (5 – 3a2) = –0,4a (10a2 – 6a4 + 15 – 9a2) = –0,4a3 +
+ 2,4a5 – 6a + 3,6a3 = 2,4a5 – 0,4a3 – 6a.

4. а) a2 + ab – 3a – 3b = (a2 + ab) – (3a + 3b) = a (a + b) – 3 (a + b) =
= (a + b) (a – 3).

    б) kpkcpx + cx + cp = (kpkc) – (pxcx) – (pc) =
= k (pc) – x (pc) – (pc) = (pc) (kx – 1).

5. Пусть сторона квадрата равна х см, тогда его площадь равна х2 см2. По условию стороны полученного прямоугольного листа равны (х – 2) см и (х – 3) см, значит, его площадь равна (х – 2) (х – 3) см2.

Составим и решим уравнение:

х2 – (х – 2) (х – 3) = 24;

х2х2 + 3х + 2х – 6 = 24;

5х = 30;

х = 6.

Ответ: 6 см.

Вариант 4

1. а) (а – 4) (а – 2) = а2 – 2а – 4а + 8 = а2 – 6а + 8.

    б) (3х + 1) (5х – 6) = 15х2 – 18х + 5х – 6 = 15х2 – 13х – 6.

    в) (3у – 2с) (у + 6с) = 3у2 + 18су – 2су – 12с2 = 3у2 + 16су – 12с2.

    г) (b + 3) (b2 + 2b – 2) = b3 + 2b2 – 2b + 3b2 + 6b – 6 = b3 + 5b2 +
+ 4b – 6.

2. а) 2x (ab) + a (ab) = (ab) (2x + a).

    б) 3x + 3y + bx + by = (3x + 3y) + (bx + by) = 3 (x + y) + b (x + y) =
= (x + y) (3 + b).

3. 0,2y (5y2 – 1) (2y2 + 1) = 0,2y (10y4 + 5y2 – 2y2 – 1) = 2y5 + y3
– 0,4y3 – 0,2y = 2y5 + 0,6y3 – 0,2y.

4. а) 3xxy – 3y + y2 = (3xxy) – (3yy2) = x (3 – y) – y (3 – y) =
= (3 – y) (xy).

    б) axay + cycxx + y = (axay) + (cycx) – (xy) =
= a (xy) – c (xy) – (xy) = (xy) (ac – 1).

5. Пусть  одна  сторона  клумбы  равна  х м,  тогда  другая  сторона равна (х + 5) м. Значит, площадь клумбы равна х (х + 5) м2.

Найдем площадь участка, состоящего из клумбы и дорожки. Этот участок имеет прямоугольную форму, его стороны равны (х + 2) м и (х + 7) м. Значит, площадь участка равна (х + 2) (х + 7) м2.

Составим и решим уравнение:

(х + 2) (х + 7) – х (х + 5) = 26;

х2 + 7х + 2х + 14 – х2 – 5х = 26;

4х = 12;

х = 3.

Ответ: 3 м и 8 м.

Урок 80
Анализ результатов контрольной работы

Цели: обобщить и систематизировать знания учащихся; проанализировать ошибки, сделанные в контрольной работе.

Ход урока

I. Анализ результатов контрольной работы.

Самые распространенные ошибки разбираются на доске с обсуждением, а затем каждый из учащихся делает работу над своими ошибками под контролем учителя.

II. Обобщение и систематизация знаний.

Те учащиеся, которые допустили ошибки в контрольной работе, после их  исправления  решают  номера  из  учебника:  № 754 (а, в);  № 761;
№ 778 (а, в); № 786.

Сильным учащимся можно предложить задания повышенного уровня сложности.

1. № 774.

Решение:

а) Запишем числа  и  в виде многочлена:

Найдем их сумму и преобразуем её:

Очевидно, что число 11 (a + b) делится на a + b.

б)

Очевидно, что это число кратно 9.

2. № 760.

Решение:

Рассмотрим процесс движения мотоциклистов до их встречи. Пусть скорость первого мотоциклиста х км/ч, тогда скорость второго 1,5х км/ч. До встречи они вместе проедут расстояние, равное 240 км.

Заполним таблицу:

 

s

х

t

Первый мотоциклист

2,4х км

х км/ч

2,4 ч

Второй мотоциклист

2,4 · 1,5х км

1,5х км/ч

2,4 ч

Составим и решим уравнение:

2,4х + 2,4 · 1,5х = 240;

2,4 (х + 1,5х) = 240;

2,5х = 100;

х = 40.

Получаем, что скорости мотоциклистов  равны  40 км/ч  и  40 · 1,5 =
= 60 км/ч.

Расстояние от пункта А до места встречи равно 2,4х = 2,4 · 40 = 96 км. Тогда расстояние от места встречи до В равно 120 – 96 = 24 км.

Ответ: 40 км/ч, 60 км/ч, 24 км.

3. № 766.

Решение:

Сделаем рисунок к задаче:

Пусть в растворе первоначально было х г соли, значит, её концентрация была равна  · 100 %. В новом растворе стало (х + 20) г соли, то есть её концентрация стала равна  · 100 %.

По условию концентрация соли повысилась на 3,75 %. Составим и решим уравнение:

 · 100 –  · 100 = 3,75;

 = 3,75;

.

Домножим обе части уравнения на 120.

24 (х + 20) – 25х = 30 · 15;

24х + 480 – 25х = 450;

х = –30;

х = 30.

Значит, первоначально в растворе было 30 г соли.

Ответ: 30 г.

4. № 797.

Решение:

Преобразуем левую часть равенства:

(10a + b) (10a + c) = 100a2 + 10ac + 10ab + bc = 100a2 + 10a (b + c) + bc.

Преобразуем правую часть равенства:

100а (а + 1) + bc = 100a2 + 100a + bc.

У полученных выражений есть одинаковые слагаемые. Это 100a2 и bc. Но если b + c = 10, то 10а (b + c) = 10а · 10 = 100а, то есть все слагаемые у этих выражений равны. Значит, данное равенство верно при условии, что b + c = 10.

а) 23 · 27.

Здесь а = 2, bc = 3 · 7 = 21. Имеем:

23 · 27 = 100 · 2 · 3 + 21 = 621.

б) 42 · 48.

Здесь а = 4, bc = 2 · 8 = 16. Имеем:

42 · 48 = 100 · 4 · 5 + 16 = 2016.

III. Итоги урока.

– Что называется многочленом? Степенью многочлена?

– Как умножить одночлен на многочлен?

– Как умножить многочлен на многочлен?

– Какие вы знаете способы разложения многочлена на множители?

– Опишите алгоритм способа группировки разложения многочлена на множители.

Домашнее задание: № 754 (г, е); № 762; № 778 (б, г); № 787.

 

 

Урок 81
Деление с остатком

Цели: изучить, как может быть представлено любое целое число при делении его с остатком на некоторое натуральное число; использовать данное представление при решении задач на делимость чисел.

Ход урока

I. Актуализация знаний.

Учащиеся уже умеют делить с остатком натуральные числа. Можно дать им выполнить несколько таких заданий и записать полученные результаты:

17 : 2 = 8 (ост. 1);               20 : 3 = 6 (ост. 2);          23 : 5 = 4 (ост. 3).

Затем предложить учащимся записать числа 17, 20 и 23, используя делитель, частное и остаток:

17 = 8 · 2 + 1;                     20 = 6 · 3 + 2;                23 = 4 · 5 + 3.

II. Изучение нового материала.

1. Рассмотреть деление целых чисел на натуральные с остатком и снова прийти к равенствам, подобным тем, которые были получены на этапе актуализации. Например:

–13 = 5 · (–3) + 2;               –20 = 7 · (–3) + 1;          –32 = 3 · (–11) + 1.

2. Делается вывод о том, что любое целое число а при делении на натуральное число b может быть записано в виде:

a = bq + r, где q – частное от деления,

                                 r – остаток, 0 ≤ r < b.

Данное утверждение доказывается.

3. Рассматривается вопрос о разбиении чисел на классы при делении с остатком.

III. Закрепление изученного материала.

1. № 724.

Решение:

Если число а при делении на 7 даёт в остатке 3, то оно может быть записано в виде:

а = 7q + 3, где q – частное от деления.

Перебирая различные q, будем получать искомые числа:

q = 0, а = 7 · 0 + 3 = 3;

q = 1, а = 7 · 1 + 3 = 10;

q = 2, а = 7 · 2 + 3 = 17 (не удовлетворяет условию);

q = –1, а = 7 · (–1) + 3 = –4;

q = –2, а = 7 · (–2) + 3 = –11;

q = –3, а = 7 · (–3) + 3 = –18 (не удовлетворяет условию).

Ответ: –11, –4, 3, 10.

2. № 726.

Решение:

Если число т при делении на 35 даёт в остатке 15, то оно может быть записано в виде:

т = 35q + 15, где q – частное от деления.

Каждое слагаемое этой суммы делится на 5, значит, и вся сумма делится на 5.

Первое слагаемое суммы делится на 7, а второе не делится, значит, вся сумма не делится на 7.

Ответ: на 5 делится, на 7 не делится.

3. № 727.

Решение:

Согласно условию число а может быть записано в виде:

a = bc + d.

Если числа b, c и d нечётные, то приходим к следующим выводам:

1) число bc – нечётное (как произведение двух нечётных чисел);

2) число bc + d – чётное (как сумма двух нечётных чисел).

Получится следующее: если числа b, c и d нечётные, то число а будет только чётным. Значит, числа а, b, c и d не могут быть одновременно нечётными.

4. № 728.

Решение:

Если числа а и b при делении на 3 дают различные остатки, то они могут быть записаны в виде:

а = 3q + 1    и    b = 3р + 2.

Найдем число аb + 1:

аb + 1 = (3q + 1) (3р + 2) + 1 = 9рq + 6q + 3р + 3.

Каждое слагаемое полученной суммы делится на 3, значит, и вся сумма делится на 3.

5. № 730.

Решение:

Если при делении числа а на 12 получается остаток 5, то число а может быть записано в виде:

а = 12q + 5, где q – частное от деления.

При делении числа а = 12q + 5 на 4 первое слагаемое суммы разделится на 4 без остатка, а второе даст в остатке 1. Значит, число а при делении на 4 даст в остатке 1.

Ответ: 1.

6. № 732.

Решение:

Если число а при делении на 5 даёт в остатке 1, то оно может быть записано в виде:

а = 5q + 1, где q – частное от деления.

Если же это число при делении на 7 даёт в остатке 1, то его можно записать так:

а = 7р + 1, где р – частное от деления.

По условию q больше р на 4. Получим систему уравнений:

5р + 20 + 1 = 7р + 1;

2р = 20;

р = 10.

Найдем число а:

а = 7р + 1 = 7 · 10 + 1 = 71.

Ответ: 71.

IV. Итоги урока.

Домашнее задание: № 723; № 725; № 731; № 733.

 

 

 

Урок                                                Дата
ТЕМА: Формулы квадрата суммы и разности
двух выражений

Цели: вывести формулы квадрата суммы и разности двух выражений; формировать умение использовать эти формулы.

Ход урока

I. Устная работа.

1. Выполните возведение в степень.

а) (–2х)2;                           в) ;                         д) (–7х3у2)2;

б) (5а2)2;                           г) ;                           е) (–0,6п4т5)2.

2. Выполните умножение.

а) 2х2 · 3х7;                       в) 3а (2а2 – 5а);                     д) (х – 3) (у + 4);

б) y5 · (–4y3);                г) –2x4 ;           е) (2a – 1) (b – 5).

II. Объяснение нового материала.

Объяснение нового материала следует производить в несколько этапов. При этом стремиться, чтобы учащиеся самостоятельно вывели формулы квадрата суммы и разности двух выражений.

1. Предложить учащимся представить выражение (a + b)2 в виде многочлена. Они уже встречали подобные задания, когда умножали многочлен на многочлен. Одного из учащихся нужно вызвать к доске, а остальные записывают у себя в тетрадях:

(a + b)2 = (a + b) (a + b) = a2 + ab + ab + b2 = a2 + 2ab + b2.

Аналогично возводится в квадрат выражение ab:

(ab)2 = (ab) (ab) = a2abab + b2 = a2 – 2ab + b2.

2. Сообщить учащимся, что полученные тождества называются формулами квадрата суммы и разности двух выражений. Они нужны, чтобы сделать проще преобразования.

Далее  предложить  учащимся  самостоятельно  сформулировать  правила,  по  которым  выполняется  возведение  в  квадрат  суммы  и  разности выражений.

3. Разобрать примеры 1 и 2 из учебника. Остальные примеры приводить пока не нужно.

III. Формирование умений и навыков.

Основное внимание на этом уроке следует уделить тому, чтобы учащиеся запомнили формулы квадрата суммы и разности, научились их правильно применять. На первых порах следует требовать от учащихся подробных записей и комментирования выполняемых действий.

1. № 799.

(К доске на одно и то же задание желательно вызывать сразу несколько учащихся.)

2. № 803.

решение:

(Во избежание ошибок следует вести подробные записи.)

а) (2x + 3)2 = (2x2) + 2 · 2x · 3 + 32 = 4x2 + 12x + 9;

д)

е)

з) (10с + 0,1у)2 = (10с)2 + 2 · 10с · 0,1у + (0,1у)2 = 100с2 + 2су + 0,01у2.

3. № 812.

Это более сложный номер, поскольку помимо формул квадрата суммы и разности учащимся нужно помнить свойства степеней.

Решение:

а) (а2 – 3а)2 = (а2)2 – 2а2 · 3а + (3а)2 = а4 – 6а3 + 9а2;

б)

в)  = c4 – 1,4c5 + 0,49c6;

г)  = 16y6 – 4y5 +
+ 0,25y4;

д)  + 24a7 +
+ 64a4;

е) = 0,36b2 – 72b3 +
+ 3600b4.

IV. Итоги урока.

– Как возвести в квадрат сумму двух выражений?

– Как возвести в квадрат разность двух выражений?

– Зачем нужны формулы квадрата суммы и разности двух выражений?

– Выполните возведение в квадрат:   а) (3а + 1)2;   б) (х – 5)2.

Домашнее задание: № 800; № 804; № 813.

 

 

 

 

Урок                                                дАТА
тема: Преобразование выражений с использованием
формул квадрата суммы и разности

Цели: продолжить формирование умения возводить в квадрат двучлен; преобразовывать выражения, используя соответствующие формулы; проверить уровень усвоения материала.

Ход урока

I. Устная работа.

Выполните возведение в квадрат.

а) (c + d)2;            б) (x + 1)2;                  в) (a – 2)2;                  г) (y – 5)2.

II. Формирование умений и навыков.

Сначала необходимо разобрать, как возводить в квадрат выражения вида –a + b и –ab. Затем перейти к упрощению выражений с использованием формул квадрата суммы и разности. В соответствии с этим задания делятся на две группы.

1-я группа

Сначала  предложить  учащимся  преобразовать  выражения  (–x + 3)2
и (–y + 7)2. Согласно известным формулам преобразования будут выглядеть следующим образом:

(–x + 3)2 = (–x)2 + 2 ∙  (–x) ∙  3 + 32 = x2 – 6x + 9;

(–y + 7)2 = (–y)2 + 2 ∙  (–y) ∙  7 + 72 = y2 – 14y + 49.

Учащиеся должны осознать, что в таком виде возведение в квадрат проводить неудобно, лучше поменять местами выражения:

(3 – x)2 = 32 – 2 ∙  3 ∙  x + x2 = 9 – 6x + x2;

(7 – y)2 = 72 – 2 ∙  7 ∙  y + y2 = 49 – 14x + y2.

Затем следует выполнить № 807. После этого сделать соответствующие выводы:

(–a + b)2 = (ba)2;

(ab)2 = (ba)2;

(–ab)2 = (a + b)2.

Нужно объяснить учащимся, что применение этих равенств упрощает возведение в квадрат двучлена и пригодится им при дальнейших преобразованиях выражений.

После этого можно перейти к выполнению заданий.

1. № 805, 806.

2. № 809.

Решение:

а)

б)

в)

г)

д)

е)

2-я группа

1. № 815.

2. № 817 (а, в, д).

Решение:

а)

в)

д)
= –2a2 + 4a + 14.

III. Проверочная работа.

Вариант 1

1. Преобразуйте в многочлен.

а) (у + 4)2;     б) (2х – 3у)2;                  в) (–3а + 5)2;                   г) (–х2 – 2х)2.

2. Упростите выражение.

а) (8а b)2 – 64а2;                                         б) а (4 – а) + (4 – а)2.

Вариант 2

1. Преобразуйте в многочлен.

а) (х – 6)2;     б) (7т + 3п)2;                в) (–2у + 3)2;                   г) (–х3 – 4х)2.

2. Упростите выражение.

а) 81х2 – (9х + 2у)2;                                        б) х (х – 7) + (х + 3)2.

IV. Итоги урока.

– Как возвести в квадрат сумму (разность) двух выражений?

– Как возвести в квадрат выражения вида  –а + b  и  –аb?

Домашнее задание: № 808; № 816; № 817 (б, г, е).

 

 

 

 

 

 

 

 

 

Урок                                                     дАТА
тема: Применение формул квадрата суммы и разности

Цели: закрепить умение возводить в квадрат двучлен по формуле; рассмотреть ряд задач, при решении которых применяется это умение.

Ход урока

I. Устная работа.

Выполните возведение в квадрат.

а) (–3х2у3)2;                                  г) ;                   ж) (–п + 3)2;

б) ;                              д) (х – 8)2;                              з) (–а – 10)2.

в) (–0,7p2q4)2;                              е) (2у + 5)2.

II. Формирование умений и навыков.

1. № 814 (устно).

2. № 818 (а, в).

3. № 819.

Решение:

а) (х – 6)2х (х + 8) = 2;

    х2 – 12х + 36 – х2 – 8х = 2;

    –20х = –34;

    х = ;

    х = 1,7.

б) 9х (х + 6) – (3х + 1)2 = 1;

    9х2 + 54х – 9х2 – 6х – 1 = 1;

    48х = 2;

    х = .

Ответ: 1,7.

Ответ: .

в) у (у – 1) – (у – 5)2 = 2;

    у2уу2 + 10у – 25 = 2;

    9у = 27;

    у = 3.

г) 16у (2 – у) + (4у – 5)2 = 0;

    32у – 16у2 + 16у2 – 40у + 25 = 0;

    –8у = –25;

    у = .

Ответ: 3.

Ответ: 3.

4. № 821.

При решении этого номера учащимся предстоит выполнять более сложные преобразования. Зачастую они делают очень распространенную ошибку: сначала умножают число на выражение в скобках, а потом результат возводят в квадрат.

Необходимо напомнить учащимся, что действие возведения в степень является приоритетным среди всех остальных, поэтому его выполняют в первую очередь.

Решение:

а) 7 (4а – 1)2 = 7 (16а2 – 8а + 1) = 112а2 – 56а + 7;

в)

д) 9с2 – 4 + 6 (с – 2)2 = 9с2 – 4 + 6 (с2 – 4с + 4) = 9с2 – 4 + 6с2 – 24с +
+ 24 = 15с2 – 24с + 20.

5. № 823 (а, в).

Решение:

а)

в) + 2 =
= a3 – 3a + 2.

III. Итоги урока.

– Как возвести в квадрат сумму (разность) двух выражений?

– Каким  из  следующих  выражений  тождественно  равно  выражение (х – 2)2:  (х + 2)2,   (2 – х)2,   (–2 – х)2,   (–2 + х)2?

– Как выполнить следующие преобразования:

а) –2 (х – 4)2;                               б) (у + 3) (у – 2)2?

Домашнее задание: № 818 (б, г); № 820; № 822; № 823 (б, г).

 

 

 

 

 

 

 

 

 

 

 

 

 

Урок                                                         дАТА
тема: Изучение способа разложения на множители с помощью формул квадрата суммы и разности

Цели: показать, как применяются формулы квадрата суммы и квадрата разности при разложении на множители трехчленов; формировать умение выполнять данное действие.

Ход урока

I. Устная работа.

1. Выполните возведение в квадрат.

а) (х – 2)2;        б) (2 + х)2;                       в) (–х + 2)2;                г) (–х – 2)2.

2. Будут ли тождественно равны следующие выражения:

а) (а – 2)2 и (2 – а)2;                                           в) (3 – с)2 и (–с + 3)2;

б) (х – 1)2 и (1 + х)2;                                           г) (–у – 5)2 и (у + 5)2?

3. Представьте выражение в виде квадрата одночлена.

а) 25а2;                        в) y2;             д) 2,25т4;

б) 121х2;                      г) 0,64с4;             е) n6.

II. Объяснение нового материала.

1. Чтобы учащиеся увидели место данной темы среди других тем и поняли её важность, необходимо сначала актуализировать их знания о разложении многочлена на множители.

– Что значит «разложить на множители многочлен»?

– Какие вы знаете способы разложения многочлена на множители?

– При решении каких задач пригодится умение раскладывать многочлен на множители?

2. Затем сообщить учащимся, что на этом уроке они познакомятся с ещё одним способом разложения многочлена на множители. Этот способ состоит в применении формул квадрата суммы и разности.

3. Согласно пункту 33 учебника ознакомить учащихся с новым материалом, привести примеры и сделать следующие выводы:

1) с помощью формул квадрата суммы и разности можно раскладывать на множители только трёхчлены;

2) чтобы трёхчлен раскладывался на множители, два его члена должны являться квадратами некоторых одночленов, а третий член должен быть удвоенным произведением этих одночленов.

III. Формирование умений и навыков.

1. № 833, № 834.

Чтобы в дальнейшем избежать ошибок, учащиеся должны всегда проверять, правильно ли они выполнили представление трехчлена в виде квадрата двучлена. Для этого на первых порах можно требовать от них письменную проверку полученных результатов.

Пример:           .

Проверка:     

                             

Затем проверку можно будет делать устно.

2. № 836, № 837.

Важно, чтобы учащиеся не просто осуществляли подбор, а поняли, как это делается. Если у них возникают затруднения, можно решение данных заданий расписывать подробно.

№ 836.

Решение:

а) * + 56а + 49.

   

б) 36 – 12х + * .

   

в)

   

г) 0,01b2 + * + 100c2.

   

3. № 839 (а, в, г).

Перед выполнением этого номера следует привести пример. Спросить учащихся, можно ли разложить на множители трёхчлен –х2 + 2х – 1? Ясно, что в таком виде он сразу на множители не разложится, так как «мешают минусы». Поэтому сначала нужно вынести знак «–» за скобки, а затем применить формулу:

х2 + 2х – 1 = –(х2 – 2х + 1) = –(х – 1)2.

Решение:

а) –1 + 4а – 4а2 = –(1 – 4а + 4а2) = – (1 – 2а)2;

в) 24 ab – 16a2 – 9b2 = –(16a2 – 24 ab + 9b2) = – (4a – 3b)2;

г) –44ах + 121а2 + 4х2 = (11а – 2х)2.

4. № 840 (б).

Решение:

(Важно, чтобы учащиеся поняли, что непосредственная подстановка данных значений переменной приведет к громоздким вычислениям.)

4х2 – 20х + 25 = (2х – 5)2

при х = 12,5:        (2х – 5)2 = (25 – 5)2 = 400;

при х = 0: (2х – 5)2 = (0 – 5)2 = 25;

при х = –2:           (2х – 5)2 = (–4 – 5)2 = 81.

IV. Итоги урока.

– Какие существуют способы разложения многочлена на множители?

– Какие многочлены могут быть разложены на множители с помощью формул квадрата суммы и разности?

– Можно ли разложить на множители следующие трёхчлены:

а) х2 – 6х + 9;                               в) а2 – 2а – 1;

б) х2 + 4х + 6;                              г) 4т2 – 4т + 1?

Домашнее задание: № 835; № 838; № 839 (б, д, е); 840 (в).

 

 

 

 

 

 

 

 

 

 

 

 

 

 

 

 

 

Урок 86
Применение способа разложения на множители
с помощью формул квадрата суммы и разности
при решении различных задач

Цели: продолжить формирование умения раскладывать на множители многочлены с помощью формул квадрата суммы и разности; применять это умение при решении различных задач.

Ход урока

I. Устная работа.

1. Представить выражение в виде квадрата одночлена.

а) 81т2;                              в) y4;                           д) 0,04х8;

б) x2;                             г) 25а6;                          ж) 144р14.

2. Преобразуйте трёхчлен в квадрат двучлена.

а) х2 + 4х + 4;                     в) 9у2 + 6у + 1;

б) а2 – 2а + 1;                     г) п2 – 10п + 25.

II. Формирование умений и навыков.

1. № 841, № 842.

2. Поставьте вместо многоточия один из знаков ≥ или ≤ так, чтобы получившееся неравенство было верно при любом значении х.

а) х2 – 10х + 25 … 0;          в) –х2 + 6х – 9 … 0;

б) 4 + 4х + х2 … 0;              г) –49 – 14хх2 … 0.

3. № 844.

До этого в заданиях учащимся предлагали представить в виде квадрата двучлена только те трёхчлены, которые возможно представить таким образом. В этом номере все трёхчлены содержат выражения вида а2 и b2, но не все содержат удвоенное произведение 2ab.

При выполнении этого номера учащимся можно дать дополнительное задание: исправить один из членов трёхчлена так, чтобы полученный трёхчлен можно было представить в виде квадрата двучлена.

Решение:

а) x2 + 3x + 9.

   

б) 25a2 – 30ab + 9b2.

     2 ∙  5a ∙  3b = 30ab, то есть

    25a2 – 30ab + 9b2 = (5a – 3b)2.

в) p2 – 2p + 4.

    нельзя представить; вместо –2p должно стоять –4р.

г)

    xy, то есть

   

д) 100b2 + 9c2 – 60bc = (10b – 3c)2.

е) 49x2 + 12xy + 64y2.

    нельзя представить;

    вместо 12xy должно стоять 112ху.

4. № 845.

Решение:

а)

б)

в)

г) a2x2 – 2abx + b2 = (axb)2.

№ 848 (можно предложить выполнить сильным учащимся дополнительно).

Решение:

а) x2 + 2x + 2 = x2 + 2x + 1 + 1 = (x + 1)2 + 1.

Так как (х + 1)2 ≥ 0 при любом х, то (х + 1)2 + 1 > 0.

б) 4у2 – 4у + 6 = 4у2 – 4у + 1 + 5 = (2у – 1)2 + 5;

    (2у – 1)2 ≥ 0 Þ (2у – 1)2 + 5 > 0.

в) a2 + b2 – 2ab + 1 = (ab)2 + 1;

    (ab)2 ≥ 0 Þ (ab)2 + 1 > 0.

г) 9x2 + 4 – 6 + 4у2 = 9x2 – 6 + 1 + 3 + 4у2 = (3x – 1)2 + 3 + 4у2.

     (3x – 1)2 + 3 + 4у2 > 0.

III. Проверочная работа.

Вариант 1

1. Представьте многочлен в виде квадрата двучлена.

а) 4a2 + 4ab + b2;                         в) a2 + 9c2 + 6ac;

б) 25x2 – 10x + 1;                         г) a2 + ab + b2.

2. Замените знак * одночленом так, чтобы получившийся трёхчлен можно было представить в виде квадрата двучлена.

а) 16x2 + * + y2;                            в) a2 + 18a + * ;

б) 49 – * + x2;                               г) * – 12x + 9x2.

Вариант 2

1. Представьте многочлен в виде квадрата двучлена.

а) 16a2 + 8ab + b2;                       в) 4x2 + y2 + 4xy;

б) 36x2 – 12x + 1;                         г) p2 – 2pq + 4q2.

2. Замените знак * одночленом так, чтобы получившийся трёхчлен можно было представить в виде квадрата двучлена.

а) 9a2 + * + b2;                             в) x2 + 14x + * ;

б) 81 – * + y2;                               г) * – 24a + 16a2.

IV. Итоги урока.

– Какие существуют способы разложения многочлена на множители?

– Приведите пример трёхчлена, который можно представить в виде:

а) квадрата суммы;

б) квадрата разности.

– Какие значения могут принимать следующие выражения:

а) а2 + 5;                                      в) –3 – х2;

б) х2 – 2х + 1;                               г) –п2 + 4п – 4?

Домашнее задание: № 843; № 846; № 975 (а, в, д, ж).

 

 

 

 

 

 

 

 

 

 

 

 

 

 

 

 

 

 

 

 

 

Урок                                               Дата
ТЕМА: Вывод формулы умножения разности двух выражений на их сумму

Цели: вывести формулу умножения разности двух выражений на их сумму; формировать умение применять эту формулу.

Ход урока

I. Устная работа.

1. Выполните возведение в квадрат.

а) (–3х2у)2;                                   г) ;                   ж) (–3m + 2)2;

б) ;                            д) (2х – 1)2;                            з) (–у – 9)2.

в) (0,9p4q10)2;                              е) (а + 11)2.   

2. Выполните умножение.

а) –3a2 (5aa4);              в) (y – 3) (x + 4);

б) x3 (2xx5);              г) (a – 1) (2b – 5).

II. Объяснение нового материала.

Объяснение  проводить  согласно  пункту  34  учебника  в  несколько этапов.

1. Сначала необходимо, чтобы учащиеся вспомнили уже известную им формулу сокращенного умножения. Вынести её на доску.

Это позволит при выводе новой формулы сопоставить её с ранее изученной, чтобы не путать их в дальнейшем.

2. Спросить учащихся, чем важна эта формула и когда она применяется. Затем сообщить им, что на этом уроке они познакомятся с ещё одной формулой сокращенного умножения, и предложить им выполнить умножение (a b) (a + b).

Один  из  учащихся  выполняет  умножение  на  доске,  остальные  –
в тетрадях.

3. Сделать выводы, сформулировать правило умножения разности двух выражений на их сумму, разобрать примеры 1 и 2 из учебника.

III. Формирование умений и навыков.

Важно, чтобы учащиеся в течение урока выучили новую формулу наизусть и не путали её с ранее изученными. На первых порах следует требовать от учащихся подробных записей.

1. № 854.

После преобразования нескольких выражений учащиеся зачастую начинают делать распространенную ошибку: возводят в квадрат выражения в том порядке, в котором они записаны в первой скобке. Например:

е) (7 + 3y) (3y – 7) = 72 – (3y)2 = 49 – 9y2.

Важно, чтобы учащиеся осознали суть этой ошибки и не делали её в дальнейшем. С этой целью после выполнения данного номера можно дать им несколько дополнительных заданий.

1) (x + 2y) (2yx);                      3) (4a + 1) (1 – 4a);

2) (6 + 5n) (5n – 6);                     4) .

2. № 859.

Решение:

а)

б)

в)

г) 25a4 – 0,16y6;

д) 1,44c4 – 49a4;

е)

3. № 858 (устно).

4. № 860.

При выполнении этого номера у учащихся появляется возможность увидеть, как новая формула сокращенного умножения позволяет выполнить рационально вычисления.

Решение:

г) 74 · 66 = (70 + 4) (70 – 4) = 702 – 42 = 4900 – 16 = 4884;

е) 1,05 · 0,95 = (1 + 0,5) (1 – 0,5) = 1 – 0,52 = 1 – 0,25 = 0,75.

IV. Итоги урока.

– Для чего нужны формулы сокращенного умножения?

– С какой формулой вы познакомились на этом уроке?

– Выполните умножение:

а) (х + 1) (1 – х);

б) (3у + 1) (1 – 3у);

в) (п + 7) (7 – п).

Домашнее задание: № 855; № 857; № 861 (б, г, е).

 

 

 

 

Урок                                                 Дата
ТЕМА: Применение формулы умножения разности двух выражений на их сумму

Цели: продолжить формирование умения применять формулу ; проверить уровень усвоения материала.

Ход урока

I. Устная работа.

Представьте в виде многочлена.

а) (х – 1) (х + 1);        г) (с + 2) (2 – с);         ж) (a2b) (b + a2);

б) (а + 5) (а – 5);        д) ;           з) .

в) ;          е) ;

II. Формирование умений и навыков.

1. № 862.

Учащиеся должны осознать, что до применения нужной формулы следует преобразовать выражения, поменяв местами слагаемые или вынеся знак «минус» (–) за скобки.

Решение:

а)

б)

в)

г)

д)

е)

2. № 864.

3. № 866.

Решение:

а) (5a – 0,2) (0,2 + 5a) = 25a2 – 0,04.

Выражение 25а2 может принимать только неотрицательные значения, значит, наименьшее значение выражения 25а2 – 0,04 равно –0,04, а наибольшего значения нет.

б) (12 – 7у) (7у + 12) = 144 – 49у2.

Выражение –49у2 не имеет наименьшего значения, а наибольшее значение равно 0, то есть наибольшее значение выражения 144 – 49у2 равно 144.

4. № 867 (а, в, д).

Решение:

а) 2 (х – 3) (х + 3) = 2 (х2 – 9) = 2х2 – 18;

в) 5х (х + 2) (х – 2) = 5х (х2 – 4) = 5х3 – 20х;

д) (0,5х – 7) (7 + 0,5х) (–4х) = (0,25х2 – 49) (–4х) = 196хх3.

5. № 869 (а, в, д, ж).

Решение:

а)

в)

д)

ж) a4 – 50a2 + 625.

III. Проверочная работа.

Вариант 1

Выполните умножение двучленов.

а) (х – 7) (х + 7);                          д) (а2 – 3) (а2 + 3);

б) ;                            е) (2ху3)(у3 + 2х);

в) (0,3 + b) (b – 0,3);                               ж) ;

г) ;                         з) (x3y4 – 1) (1 + x3y4).

Вариант 2

Выполните умножение двучленов.

а) (a – 9) (a + 9);                          д) (x2 – 4) (x2 + 4);

б) ;                             е) (3ab4) (b4 + 3a);

в) (0,2 + х) (х – 0,2);                                ж) ;

г) ;            з) (a3b5 – 2) (2 + a3b5).

IV. Итоги урока.

– Чему равно произведение разности двух выражений и их суммы?

– Как преобразовать в многочлен следующие выражения:

а) (–3 + х) (х + 3);                                   в) 3 (a – 7) (a + 7);

б) (–у – 2) (2 – у);                        г) (т – 2)2 (т + 2)2?

Домашнее задание: № 863; № 865; № 867 (б, г, е); № 869 (б, г, е, з).

 

 

Урок                                                        Дата
ТЕМА: Применение формул (a ± b)2 = a2 ± 2ab + b2   и   (ab) (a + b) = a2b2
к преобразованию выражений

Цели: закрепить умение применять формулы сокращенного умножения к преобразованию выражений.

Ход урока

I. Устная работа.

Какие из следующих равенств верные:

а) ;                     е) (a2 + 3) (3 – a2) = 9 – a4;

б) (y + 2) (2 – y) = y2 – 4;                        ж) ;

в) (с – 3)2 = с2 – 9;                                              з) (а – 4)2 = а2 – 8а – 16;

г) (x – 2)2 = x2 – 2x + 4;                           и) (3 – y)2 = y2 – 6y + 9;

д) (n + 1)2 = n2 + 2n + 1;                         к) (–x + 5) (5 – x) = 25 – x2?

II. Формирование умений и навыков.

Все задания можно разбить на две группы.  В  1-ю группу войдут задания  на  закрепление  формулы  (ab) (a + b) = a2b2,  а во 2-ю группу – задания, в которых эта формула применяется с ранее изученной формулой (a ± b)2 = a2 ± 2ab + b2.

1-я группа

1. № 870.

2. № 875.

Решение:

а) 5a2 – 40a – 3a2 +
+ 12 = 2a2 – 40a + 12;

б) – 10b2 – 12b + 1;

в)

г)

3. № 876.

Решение:

а) 8m (1 + 2m) – (4m + 3) (4m – 3) = 2m;

    8m + 16m2 – 16m2 + 9 = 2m;

    8m – 2m = –9;

    6m = –9;

    m = .

Ответ: –1,5.

б) x – 3x (1 – 12x) = 11 – (5 – 6x) (6x + 5);

    x – 3x + 36x2 = 11 – 25 + 36x2;

    –2х = –14;

    х = 7.

Ответ: 7.

2-я группа

1. Упростите выражение.

а) (x + 7)2 – 10 (x + 4);                в) (a + 3c)2 – (b + 3c) (b – 3c);

б) 5b2 – (a – 2b)2;                        г) (x + 3)2 – (x – 3)2.

2. № 978 (б, г).

Решение:

б) (x + 6)2 – (x – 5) (x + 5) = 79;

    x2 + 12x + 36 – x2 + 25 = 79;

    12x = 79 – 36 – 25;

    12х = 18;

    х = ;

    х = 1,5.

Ответ: 1,5.

г) (5х – 1)2 – (1 – 3х)2 = 16х (х – 3);

    25х2 – 10х + 1 – 1 + 6х – 9х2 = 16х2 – 48х;

    16х2 – 4х – 16х2 + 48х = 0;

    44х = 0;

    х = 0.

Ответ: 0.

Некоторым сильным учащимся дополнительно можно предложить выполнить задания на карточках.

Карточка 1

1. Выполните действия, применив нужную формулу.

а) (2х + 2y) (3х – 3y);                              б) (3a – 3х) (7a – 7х).

2. Разложите на множители.

а)          б)

3. Найдите значение выражения

Решение заданий на карточках

Карточка 1

1. а) (2х + 2y) (3х – 3y) = 2 (х + y) ∙  3 (хy) = 6 (х + y) (хy) =

= 6 (х2y2) = 6х2 – 6y2;

    б)

2. а) 34x2 + 10x =
= 2x (17x + 5);

    б) 6pkk2 =
= 6pk – 5k2 = k (6p – 5k).

3.

III. Итоги урока.

– Чему равно произведение разности двух выражений и их суммы?

– Как возвести в квадрат разность двух выражений?

– Объясните, какая ошибка допущена в преобразованиях:

а) (x + 3) (3 – x) = x2 – 9;            в) (x – 5)2 = x2 + 10x + 25;

б) (x – 4)2 = x2 – 16;                                г) (–x + 2) (x – 2) = x2 – 4.

Домашнее задание: № 871, № 874, № 877.

 

 

 

 

 

 

 

 

 

 

 

 

 

 

 

 

Урок                                              Дата
ТЕМА: Изучение формулы разности квадратов

Цель: изучить формулу разности квадратов и формировать умение её применять при разложении на множители многочленов.

Ход урока

I. Устная работа.

1. Представьте в виде квадрата двучлена.

а) 81х2;                             в) 4с10;                        д) b12;

б) a4;                          г) 0,0009п8;                            е) 1,44а2х6.

2. Выполните умножение.

а) (х – 8) (х + 8);                          в) (2х2 – 1) (1 + 2х2);

б) ;                             г) (c3 + 5) (5 – c3).

II. Проверочная работа.

Вариант 1

Упростите выражение.

а) (а + 11)2 – 20а;                                    в)

б)               г) (х – 1) (х + 1) – (y + 1) (y – 1).

Вариант 2

Упростите выражение.

а) 4х2 – (х – 3y)2;                         в)

б)                 г) (a + 2) (a – 2) – (b – 2) (2 + b).

III. Объяснение нового материала.

1. Актуализация знаний.

Вопросы учащимся:

– Что значит «разложить многочлен на множители»?

– Какие вы знаете способы разложения многочлена на множители?

– Как разложить на множители трёхчлен, используя формулу квадрата суммы или разности?

На доску выносится запись

2. Вывод формулы разности квадратов.

Важно, чтобы учащиеся поняли, что они изучают не новую формулу, а просто меняют местами левую и правую части известной формулы.

Следует указать учащимся на аналогию с изучением формул квадрата суммы и разности. Сначала их использовали для упрощения выражений, а затем – для разложения на множители. Так же и формула разности квадратов сначала применялась, чтобы раскрыть скобки, а теперь будет изучаться с целью разложения на множители многочленов.

На доску выносится запись

Учащиеся должны несколько раз проговорить словесную формулировку этого тождества.

3. Рассмотрение примеров.

Учитель демонстрирует примеры 1 и 2 из учебника.

IV. Формирование умений и навыков.

Все задания можно разбить на две группы. Сначала учащиеся отрабатывают умение использовать новую формулу для разложения многочлена на множители, а затем рассматривают применение этой формулы для рационального нахождения значений выражений.

1-я группа

1. № 883.

К доске вызываются сразу несколько учащихся, остальные выполняют задания в тетрадях.

2. № 885.

На первых порах следует требовать от учащихся подробных записей.

Решение:

а)

г)

е) (0,8x – 0,7y) (0,8x + 0,7y);

ж)

2-я группа

1. № 886.

Важно, чтобы учащиеся осознали, что применение формулы разности квадратов значительно упрощает вычисления.

Решение:

г)

е)

2. № 887.

Для выполнения этого номера учащиеся помимо формулы разности квадратов должны помнить об основном свойстве дроби, поэтому перед решением заданий имеет смысл его повторить.

Решение:

а)

б)

в)

г)

V. Итоги урока.

– Какие существуют способы разложения многочленов на множители?

– Как разложить на множители разность квадратов?

– Можно ли разложить на множители следующие многочлены:

а)  – x2;                         в) –п2 + 121;

б) а2 + 9;                                      г) –x2y2 – 49?

Домашнее задание: № 884, № 888.

 

 

 

 

 

 

 

 

 

 

 

 

 

 

 

Урок                                                  Дата
ТЕМА: Применение формулы разности квадратов
для разложения многочлена на множители

Цели: продолжить формирование умения применять формулу разности квадратов для разложения многочлена на множители; проверить уровень усвоения материала.

Ход урока

I. Устная работа.

Какие из следующих многочленов можно разложить на множители:

а) а2 – 16;             в) 4у2 – 1;                               д)  – x2;

б) x2 + ;             г) –25 + п2;                            е) a2b2 – 9?

Выполните  разложение  на  множители  в  тех  случаях,  когда  это возможно.

II. Формирование умений и навыков.

Все задания можно разбить на две группы. В 1-ю группу войдут задания, в которых нужно разложить на множители двучлен выше второй степени, а во 2-ю группу – задания на решение уравнений.

1-я группа

1. № 889.

2. № 892.

Если у учащихся возникнут затруднения, то нужно требовать от них подробной записи и комментирования решения.

Решение:

а)

в)

д)

ж)

и)

2-я группа

№ 890.

Учащиеся уже решали уравнения, в которых к нулю приравнивалось произведение нескольких многочленов. Тем не менее, следует напомнить им о принципе решения таких уравнений. Особое внимание нужно уделить уравнениям, не имеющим решений.

Решение:

а) х2 – 16 = 0.

   (х – 4) (х + 4) = 0;

    х – 4 = 0   или   х + 4 = 0;

    х = 4         или   х = –4.

Ответ: –4; 4.

в)  – x2 = 0.

     = 0;

     – x = 0   или    + x = 0;

     x =         или    x = –.

Ответ: –; .

д) b2 + 36 = 0.

Выражение b2 + 36 > 0
при любом значении b.

Ответ: решений нет.

ж) 4х2 – 9 = 0.

    (2х – 3) (2х + 3) = 0;

     2х – 3 = 0   или    2х + 3 = 0;

     x =          или    x = –.

Ответ: –1,5; 1,5.

III. Проверочная работа.

Вариант 1

1. Разложите на множители.

а) 9х2 – 1;                         в) –100a2 + b2;                       д) n4;

б)  – 16c2;                              г) x2y2 – 4;                              е) х6у8.

2. Найдите значение дроби: .

3. Решите уравнение.

а) х2 – 64 = 0;

б) х2 + 9 = 0.

Вариант 2

1. Разложите на множители.

а) 4р2 – 9;                         в) –121х2 + у2;                        д) c4;

б)  – 25у2;                              г) a2b2 – 49;                            е) a10b6.

2. Найдите значение дроби: .

3. Решите уравнение.

а) х2 – 100 = 0;

б) х2 + 25 = 0.

IV. Итоги урока.

– Как разложить на множители разность квадратов двух выражений?

– Как решить уравнение х2 – 4 = 0?

– Можно ли разложить на множители выражения:

а) х2;                                  в) –у2 + 25;

б) а2 + 36;                        г) –n2?

Домашнее задание: № 891, № 893.

 

 

 

 

 

 

 

 

 

 

 

 

 

 

 

Урок                                               Дата
ТЕМА: Применение формулы разности квадратов
при решении различных задач

Цели: закрепить изученный материал; выполнить более сложные задания на применение формулы разности квадратов; подготовить учащихся к контрольной работе.

Ход урока

I. Устная работа.

Какие из следующих многочленов можно разложить на множители? Если возможно, сделайте это.

а) а2 – 49;                         г) х2 + 4х – 4;             ж) 16х2 – 8х + 1;

б) х2 + ;                        д)  + n2;                           з)  – y2;

в) х2 – 2х + 1;                               е) х2 + 3х + 9;

II. Формирование умений и навыков.

Все задания можно разбить на две группы. В 1-ю группу войдут задания на применение формулы разности квадратов. А во 2-ю группу – задания на все известные учащимся формулы сокращенного умножения, чтобы подготовить их к контрольной работе.

1-я группа

1. № 894.

Решение:

(Записи лучше вести подробно, Скобки, перед которыми стоят знаки «минус» (–) и «плюс» (+), открывать не сразу.)

а)

б)
= (8 – b – 1) (8 + b + 1) = (7 – b) (9 + b);

в)
= (4a – 7) (4a + 1);

г)
= (5 – a – 7) (5 + a + 7) = (–a – 2) (a + 12) = – (a + 2) (a + 12).

2. № 897 (а, б).

Решение:

а) (x – 2y)) =
= (2x + yx + 2y) (2x + y + x – 2y) = (x + 3y) (3xy);

б)
= (a + bbc) (a + b + b + c) = (ac) (a + 2b + c).

3. № 898.

Решение:

Разложим на множители данное выражение:

Поскольку один из множителей произведения 7 (2п + 7) делится на 7, то и всё произведение делится на 7.

2-я группа

1. Упростите выражение.

а)                                    г) (3a + 4) (4 – 3a) – a (5 – 9a);

б)                           д)

в)                              е) y (4 – y) – 2 (y + 3) (y – 3).

2. Решите уравнение.

а)

б)

Некоторым сильным учащимся дополнительно можно предложить выполнить задания на карточках.

Карточка 1

1. Вычислите наиболее рациональным способом.

.

2. Решите уравнение  x3 + 2x2 – 4x – 8 = 0.

3. Докажите, что разность квадратов двух последовательных целых чисел равна сумме этих чисел.

Карточка 2

1. Вычислите наиболее рациональным способом.

.

2. Решите уравнение  x3 + 5x2 – 25x – 125 = 0.

3. Докажите, что разность квадратов двух последовательных чётных чисел равна удвоенной сумме этих чисел.

Решение заданий на карточках

Карточка 1

1.

2. x3 + 2x2 – 4x – 8 = 0.

   (x3 + 2x2) – (4x + 8) = 0;

    x2 (x + 2) – 4 (x + 2) = 0;

   (x + 2) (x2 – 4) = 0;

   (x + 2) (x – 2) (x + 2) = 0;

    х + 2 = 0    или    х – 2 = 0;

    х = –2        или    х = 2.

Ответ: –2; 2.

3. Обозначим два последовательных целых числа за п и п + 1. Согласно условию задачи нужно доказать следующее тождество:

Преобразуем левую часть равенства:

Доказано.

Карточка 2

1.

2. x3 + 5x2 – 25x – 125 = 0.

   (x3 + 5x2) – (25x + 125) = 0;

    x2 (x + 5) – 25 (x + 5) = 0;

   (x + 5) (x2 – 25) = 0;

   (x + 5) (x – 5) (x + 5) = 0;

    х + 5 = 0    или    х – 5 = 0;

    х = –5        или    х = 5.

Ответ: –5; 5.

3. Пусть 2п и 2п + 2 – два последовательных чётных числа. Найдем разность их квадратов.

Найдем удвоенную сумму чисел 2п и 2п + 2:

2 (2n + 2n + 2) = 2 (4n + 2).

Получили одинаковые выражения, то есть утверждение доказано.

III. Итоги урока.

– Назовите известные вам формулы сокращенного умножения.

– Когда  эти  формулы  применяются  слева  направо,  а  когда  справа налево?

– Всегда ли можно разложить на множители разность квадратов двух выражений?

– Когда можно разложить на множители трёхчлен?

Домашнее задание: № 896; № 897 (в, г); № 899; № 975 (б, г, е, з).

 

 

Урок 93
Контрольная работа № 7

Вариант 1

1. Преобразуйте в многочлен:

а) (у – 4)2;                           в) (5с – 1) (5с + 1);

б) (7х + а)2;                        г) (3a + 2b) (3a – 2b).

2. Упростите выражение  (a – 9)2 – (81 + 2a).

3. Разложите на множители:

а) х2 – 49;                           б) 25х2 – 10хy + y2.

4. Решите уравнение  (2 – х)2х (х + 1,5) = 4.

5. Выполните действия.

а) (y2 – 2a) (2a + y2);           б) (3х2 + х)2;                в) (2 + m)2 (2 – m)2.

6. Решите уравнение.

а) (2х – 5)2 – (2х – 3) (2х + 3) = 0;                         б) 9у2 – 25 = 0.

7. Разложите на множители.

а) 4x2y2 – 9a4;                     б) 25a2 – (a + 3)2.

Вариант 2

1. Преобразуйте в многочлен.

а) (3a + 4)2;                        в) (b + 3) (b – 3);

б) (2xb)2;                        г) (5y – 2x) (5y + 2x).

2. Упростите выражение  (c + b) (cb) – (5c2b2).

3. Разложите на множители.

а) 25y2a2;                        б) c2 + 4bc + 4b2.

4. Решите уравнение  12 – (4 – x)2 = x (3 – x).

5. Выполните действия.

а) (3x + y2) (3xy2);           б) (a3 – 6a)2;                в) (ax)2 (x + a)2.

6. Решите уравнение.

а) (4x – 3) (4x + 3) – (4x – 1)2 = 3x;                        б) 16с2 – 49 = 0.

7. Разложите на множители.

а) 100a4b2;                   б) 9x2 – (x – 1)2.

Вариант 3

1. Преобразуйте в многочлен.

а) (х + 6)2;                          в) (3y – 2) (3y + 2);

б) (3а – 1)2;                        г) (4а + 3k) (4а – 3k).

2. Упростите выражение  (b – 8)2 – (64 – 6b).

3. Разложите на множители.

а) 25 – у2;                           б) a2 – 6ab + 9b2.

4. Решите уравнение  36 – (6 – х)2 = х (2,5 – х).

5. Выполните действия.

а) (c2 – 3а) (3а + c2);           б) (3х + х3)2;                в) (3 – k)2 (k + 3)2.

6. Решите уравнение.

а) (3х – 2)2 – (3х – 4) (4 + 3х) = 0;                          б) 25у2 – 64 = 0.

7. Разложите на множители:

а) 36a4 – 25a2b2;                 б) (х – 7)2 – 81.

Вариант 4

1. Преобразуйте в многочлен.

а) (2х – 1)2;                         в) (y – 5) (y + 5);

б) (3a + c)2;                        г) (4b + 5c) (4b – 5c).

2. Упростите выражение  (x + y) (xy) – (x2 + 3y2).

3. Разложите на множители.

а) 16у2 – 0,25;                    б) a2 + 10ab + 25b2.

4. Решите уравнение  (5 – x)2x (2,5 + x) = 0.

5. Выполните действия.

а) (2ab2) (2a + b2);          б) (x – 6x3)2;                          в) (у + b)2 (уb)2.

6. Решите уравнение.

а) (5x – 2) (5x + 2) – (5x – 1)2 = 4;                          б) 100х2 – 16 = 0.

7. Разложите на множители:

а) a2 – 0,09c4;                б) (b + 8)2 – 4b2.

Решение заданий контрольной работы

Вариант 1

1. а) (у – 4)2 = у2 – 8у + 16;

    б) (7х + а)2 = 49х2 + 14ах + а2;

    в) (5с – 1) (5с + 1) = 25с2 – 1;

    г) (3a + 2b) (3a – 2b) = 9a2 – 4b2.

2. (a – 9)2 – (81 + 2a) = a2 – 18a + 81 – 81 – 2a = a2 – 20a.

3. а) х2 – 49 = (х – 7)(х + 7);

    б) 25х2 – 10хy + y2 = (5хy)2.

4. (2 – х)2х (х + 1,5) = 4.

    4 – 4х + х2х2 – 1,5х = 4;

    –5,5х = 0;

    х = 0.

Ответ: 0.

5. а)

    б)

    в) 16 – 8m2 + m4.

6. а) (2х – 5)2 – (2х – 3) (2х + 3) = 0.

        4х2 – 20х + 25 – 4х2 + 9 = 0;

        –20х = –34;

        х = ;

        х =  = 1,7.

Ответ: 1,7.

    б) 9у2 – 25 = 0.

        (3y – 5) (3y + 5) = 0;

        3у – 5 = 0    или    3у + 5 = 0;

        y =           или    y = –.

Ответ: –1;  1.

7. а)

    б) (a + 3)) =
= (5aa – 3) (5a + a + 3) = (4a – 3) (6a + 3).

Вариант 2

1. а) (3a + 4)2 = 9a2 + 24a + 16;

    б) (2xb)2 = 4x2 – 4bx + b2;

    в) (b + 3) (b – 3) = b2 – 9;

    г) (5y – 2x) (5y + 2x) = 25y2 – 4x2.

2. (c + b) (cb) – (5c2b2) = c2b2 – 5c2 + b2 = –4c2.

3. а) 25y2a2 = (5ya) (5y + a);

    б) c2 + 4bc + 4b2 = (c + 2b)2.

4. 12 – (4 – x)2 = x (3 – x).

    12 – 16 + 8xx2 = 3xx2;

    5х = 4;

    х = .

Ответ: 0,8.

5. а)

    б)

    в) a4 – 2a2x2 + x4.

6. а) (4x – 3) (4x + 3) – (4x – 1)2 = 3x.

        16x2 – 9 – 16x2 + 8x – 1 = 3x;

        5х = 10;

        х = 2.

Ответ: 2.

    б) 16с2 – 49 = 0.

        (4с – 7) (4с + 7) = 0;

        4с – 7 = 0    или    4с + 7 = 0;

        с =           или    с = –.

Ответ:  –1;  1.

7. а)

    б)
= (3xx + 1) (3x + x – 1) = (2x + 1) (4x – 1).

Вариант 3

1. а) (х + 6)2 = х2 + 12х + 36;

    б) (3а – 1)2 = 9а2 – 6а + 1;

    в) (3y – 2) (3y + 2) = 9y2 – 4;

    г) (4а + 3k) (4а – 3k) = 16а2 – 9k2.

2. (b – 8)2 – (64 – 6b) = b2 – 16b + 64 – 64 + 6b = b2 – 10b.

3. а) 25 – у2 = (5 – у) (5 + у);

    б) a2 – 6ab + 9b2 = (a – 3b)2.

4. 36 – (6 – х)2 = х (2,5 – х).

    36 – 36 + 12хх2 = 2,5хх2;

    9,5х = 0;

    х = 0.

Ответ: 0.

5. а)

    б)

    в) 81 – 18k2 + k4.

6. а) (3х – 2)2 – (3х – 4) (4 + 3х) = 0;

        9х2 – 12х + 4 – 9х2 + 16 = 0;

        –12х = –20;

        х = .

Ответ:  1.

    б) 25у2 – 64 = 0;

        (5у – 8)(5у + 8) = 0;

        5у – 8 = 0    или    5у + 8 = 0;

        у =           или     у = –.

Ответ: –1,6; 1,6.

7. а)

    б) (х – 7)2 – 81 = (х – 7)2 – 92 = (х – 7 – 9) (х – 7 + 9) = (х – 16) (х + 2).

Вариант 4

1. а) (2х – 1)2 = 4х2 – 4х + 1;

    б) (3a + c)2 = 9a2 + 6ac + c2;

    в) (y – 5) (y + 5) = y2 – 25;

    г) (4b + 5c) (4b – 5c) = 16b2 – 25c2.

2. (x + y) (xy) – (x2 + 3y2) = x2y2x2 – 3y2 = –4y2.

3. а) 16у2 – 0,25 = (4у – 0,5) (4у + 0,5);

    б) a2 + 10ab + 25b2 = (a + 5b)2.

4. (5 – x)2x (2,5 + x) = 0.

    25 – 10x + x2 – 2,5xx2 = 0;

    –12,5х = –25;

    х = 2.

Ответ: 2.

5. а)

    б)

    в) y4 – 2y2b2 + b4.

6. а) (5x – 2) (5x + 2) – (5x – 1)2 = 4.

        25x2 – 4 – 25x2 + 10x – 1 = 4;

        10х = 9;

        х = 0,9.

Ответ: 0,9.

    б) 100х2 – 16 = 0.

        (10х – 4) (10х + 4) = 0;

        10х – 4 = 0    или    10х + 4 = 0;

        х = 0,4           или    х = –0,4.

Ответ: –0,4; 0,4.

7. а)

    б)
= (8 – b) (3b + 8).

 

 

Урок 94
Разложение на множители
суммы и разности квадратов

Цели: вывести формулы суммы и разности кубов; формировать умение применять их при разложении многочлена на множители.

Ход урока

I. Устная работа.

1. Представьте выражение в виде куба одночлена:

а) 8а3;                       в) 125у3;              д) x12;

б) x3;                    г) a9b6;                 е) 8n6y15.

2. Выполните возведение в квадрат.

а) (2x – 1)2;               б) ;         в) (9 – n)2;            г) (–3a + 5)2.

II. Объяснение нового материала.

Объяснение проводить согласно пункту 36 учебника. Необходимо, чтобы учащиеся умели давать формулировки выведенным формулам.

После изучения новых формул следует вынести их на доску, объединив в одну:

III. Формирование умений и навыков.

Желательно, чтобы в течение урока учащиеся выучили новые формулы наизусть.

1. № 905, № 907.

На каждое задание к доске лучше вызывать по одному ученику, который должен комментировать свои действия.

2. № 909 (а, в, д).

3. № 911.

Решение:

а)

б)

в)

г)

д)

е)

4. № 912 (а, в, д).

5. № 914.

Решение:

а) 383 + 373 = (38 + 37) (382 – 38 ∙  37 + 372) = 75 (382 – 38 ∙  37 + 372).

Поскольку один из множителей произведения делится на 75, то и всё произведение делится на 75.

IV. Итоги урока.

– Назовите формулы суммы и разности кубов.

– Когда применяются эти формулы?

– Какие ещё формулы позволяют разложить многочлен на множители? Назовите их.

Домашнее задание: № 906; № 908; № 910; № 912 (б, г, е).

 

 

 

 

 

Урок                                                         Дата                                                   
Тема:  Понятие целого выражения

Цели: ввести понятие целого выражения; формировать умение преобразовывать целые выражения.

Ход урока

I. Устная работа.

Преобразуйте в многочлен.

а) x (2x2 – 4);         в) (x + 4)2;                         б) (x + 3) (x – 3);

г) ;          д) (a – 1) (a2 + a + 1);       ж) (x – 3) (y – 2);

е) ;                        з) (–1 – 2n)2.

II. Объяснение нового материала.

Объяснение  проводить  согласно  пункту  37  учебника  в  несколько этапов.

1. Введение понятия целого выражения.

Сначала необходимо напомнить учащимся о том, что такое математическое выражение, а затем дать определение целого выражения. Сделать вывод: математическое выражение может быть целым или нецелым.

После этого привести примеры и выполнить № 918.

2. Целое выражение и многочлен.

На основе изученного учащиеся сами смогут сделать вывод, что любой многочлен является целым выражением. После этого следует задать им вопрос: любое ли целое выражение является многочленом?

Делаются соответствующие выводы, приводятся примеры, показывающие, как целое выражение представляется в виде многочлена.

3. Преобразование целых выражений.

Сообщить учащимся, что преобразование целых выражений является одним из основных действий в математике. Чтобы выполнять такие преобразования, нужно уметь следующее:

– выполнять умножение одночлена на многочлен и многочлена на многочлен;

– приводить подобные слагаемые;

– знать формулы сокращенного умножения.

Далее привести пример 1 из учебника.

III. Формирование умений и навыков.

Для преобразования целых выражений учащиеся выполняют действия, которые уже должны быть у них отработаны в процессе изучения предыдущих тем. По сути, задания, предложенные в учебнике, служат для обобщения и систематизации знаний и умений учащихся.

1. Упростите выражение.

а) (4ab) (a – 6b) + a (25b – 3a);

б) 2c (5c – 3) – (c – 2) (c – 4);

в) (y – 3) (5 – y) – (4 – y) (y + 6).

2. Преобразуйте в многочлен.

а)                              в)

б) 4b (3b + 6) – (3b – 5) (5 + 3b);                       г)

3. Найдите значение выражения.

а)    при  х = –3,5;

б)          при  a = 1b = 0,7.

4. Упростите выражение.

а)

б)

Решение:

а) Можно выполнять возведение в квадрат и раскрывать скобки, но это будет нерационально. Заметим, что данное выражение является полным квадратом.

(4a3 – 1))2 =
= (4a3 + 5 – 4a3 + 1)2 = 62 = 36.

б)
= 4x3 – 4x2 + x – 2x3 – 2 = 2x3 – 4x2 + x – 2.

IV. Итоги урока.

– Какие математические выражения называются целыми?

– Приведите примеры целых выражений и выражений, которые не являются целыми.

– Являются ли многочлены целыми выражениями?

– Любое ли целое выражение можно представить в виде многочлена?

Домашнее задание: № 920, № 921, № 922.

 

 

 

 

 

 

 

 

 

Урок                                                         Дата                                                 
Тема:  Преобразование целых выражений                                                                                           
Цели: продолжить формирование умения преобразовывать целые выражения; проверить уровень усвоения материала.

Ход урока

I. Устная работа.

1. Какие из следующих выражений являются целыми:

а) 3x2 – 2a;                             в) ;                      д)  – 4;

б) ;                г) ;                                  е) ?

2. Преобразуйте в многочлен.

а)                   в) (x – 5) (y – 2);

б) (–x – 4)2;                            г) .

II. Формирование умений и навыков.

1. № 923.

Решение:

Преобразуем данное выражение:

При любом целом п первое слагаемое полученной суммы делится на 6, а  второе  слагаемое  не  делится  на  6.  Значит,  ни  при  каком  целом п сумма 6п + 10 не делится на 6.

2. № 925.

Решение:

а) x (x + 2) (x – 2) – x (x2 – 8) = 16.

    x (x2 – 4) – x3 + 8x = 16;

    x3 – 4xx3 + 8x = 16;

    4х = 16;

    х = 4.

Ответ: 4.

б) 2y (4y – 1) – 2 (3 – 2y)2 = 48.

    8y2 – 2y – 2 (9 – 12y + 4y)2 = 48;

    8y2 – 2y – 18 + 24y – 8y2 = 48;

    22у = 66;

    у = 3.

Ответ: 3.

3. № 927 (а).

Решение:

а) Упростим данное выражение:


a4 + 2a2 – 1 – 2a2 + 6 = a4 – 1 – a4 + 5 = 4.

Значит, значение выражения не зависит от а.

4*. № 999 (а).

Решение:

а)

a4 – 4a2 + 11 –
a4a3 + 2,5a2 – 1,5a + 6 = –a3 – 1,5a2 – 1,5a + 17.

III. Проверочная работа.

Вариант 1

1. Преобразуйте в многочлен.

а) (c + 2) (c – 3) – (c + 1) (c + 3);

б)

в)

2. Найдите значение выражения

(3a + b)2 – (3ab)2    при    a = 3b = –0,3.

3. Упростите выражение  8 (5y + 3)2 + 9 (3y – 1)2.

IV. Итоги урока.

– Какое выражение называется целым?

– Приведите примеры целых и нецелых выражений.

– Являются ли многочлены целыми выражениями?

– Какие действия и в каком порядке надо выполнить, чтобы представить целое выражение 2x (x – 5)2 – (x + 1) (x – 1) в виде многочлена?

Домашнее задание: № 924; № 926; № 928 (а); № 929 (а).

 

 

 

 

 

Урок                                                         Дата                                                 
Тема:  Преобразование целых выражений . Три способа разложения многочлена на множители.                                                                                                                                                              
Цели: повторить известные способы разложения многочлена на множители и закрепить умение их применять.                 

 

Ход урока

I. Устная работа.

Разложите многочлен на множители.

а) 5х3 – 10х;                     г) y2 + 6y + 9;             ж) а3 + 1;

б) а2 – 4;                           д) 4х2 – 4х + 1;                       з) 49p2q4.

в)  – х2;                      е)    

II. Объяснение нового материала.

Сначала необходимо актуализировать знания учащихся. Следует задать им вопрос о том, какие существуют способы разложения многочлена на множители. Сделать вывод, что таких способов три:

1) вынос общего множителя за скобки;

2) способ группировки;

3) применение формул сокращенного умножения.

Затем можно привести примеры, которые демонстрируют каждый из этих способов:

1)

2)

3) 5 (x2 + 3) =
= (x2 + 3) (2x – 5).

После этого сообщить учащимся, что иногда для разложения многочлена на множители нужно последовательно применить несколько способов. Разобрать примеры 1 и 2 из учебника, остальные примеры лучше рассмотреть на следующем уроке.

III. Формирование умений и навыков.

1. № 934 (а, в, д), № 935.

2. № 937.

Решение:

Это  тождество  можно  доказывать  как  слева  направо,  так  и  справа налево.

Разложим на множители левую часть равенства:

Доказано.

3. № 938.

4. № 939 (а, в, д).

Решение:

а)

в)

д)

5. № 942 (а, в).

Решение:

а) 4xy + 12y – 4x – 12 = (4xy – 4x) + (12y – 12) = 4x (y – 1) + 12 (y – 1) =

= (y – 1) (4x + 12) = 4 (y – 1) (x + 3);

в) –abc – 5ac – 4ab – 20a = –a (bc + 5c + 4b + 20) = –a ((bc + 4b) +

+ (5c + 20)) = –a (b (c + 4) + 5 (c + 4)) = –a (c + 4) (b + 5).

IV. Итоги урока.

– Какие вы знаете способы разложения на множители?

– Чему равна разность квадратов двух выражений?

– Как разложить на множители сумму (разность) кубов?

– В чём состоит способ группировки разложения многочлена на множители?

Домашнее задание:  № 934  (б, г, е);  № 936;  № 939  (б, г, е);  № 942
(б, г).

 

 

 

 

 

 

 

 

 

 

 

 

Урок
Тема: Разложение многочлена на множители
разными способами

Цели: закрепить  умение  раскладывать  многочлен  на  множители; рассмотреть  особенности  применения  способа  группировки  в  сочетании с формулами сокращенного умножения; проверить уровень усвоения материала.

Ход урока

I. Устная работа.

Разложите многочлен на множители.

а) 4a2 – 8a;                            г) n2 + 8n + 16;                 ж) х3 – 1;

б) х2 – 100;                            д) 9х2 – 6х + 1;                              з) 225a2c6.

в)  – a2;                           е) 25p2q2

II. Формирование умений и навыков.

Все задания можно разбить на две группы. В 1-й группе будут задания на обобщение различных способов разложения многочлена на множители, а во 2-ю группу следует отдельно включить задания, в которых используется метод группировки в сочетании с формулами сокращенного умножения.

1-я группа

1. № 940.

2. № 943 (а, в).

3. № 1009.

Решение:

а)

б)

в)

г)

4. № 1010.

Решение:

а)

б)

в)

г)

2-я группа

Сначала  следует  разобрать  пример 3  из учебника и сделать вывод о том, что не всегда члены многочлена группируются по два.

1. № 944.

Решение:

а)
= (xcd) (xc + d);

б)
= (c + 1 – a) (c + 1 + a);

в)
= (p – (x – 3)) (p + (x – 3)) = (px + 3) (p + x – 3);

г)
= (x – (a + 5)) (x + (a + 5)) = (xa – 5) (x + a + 5).

2. № 946 (а, г).

Решение:

а)
= (x + y) (xy – 1);

г)
= (k + p) (kp – 1).

III. Проверочная работа.

Вариант 1

1. Разложите на множители.

а) 3х2 – 12;                                   в) ax2 + 4ax + 4a;

б) –3a3 + 3ab2;                             г) –3x2 + 12x – 12.

2. Представьте в виде произведения.

а)                 б)

3*. Какой многочлен надо записать вместо *, чтобы получившееся равенство было тождеством:

(x + 1) ∙  * = x2 + 3x + 2?

IV. Итоги урока.

– Какие вы знаете способы разложения многочлена на множители?

– В чём состоит каждый из этих способов?

– Как способ группировки применяется в сочетании с формулами сокращенного умножения?

Домашнее задание: № 941; № 943 (б, г); № 945; № 947.

 

 

 

 

 

Урок 99
Разложение многочлена на множители
при решении различных задач

Цели: закрепить умение использовать различные способы разложения многочлена на множители; рассмотреть решение некоторых задач с применением разложения на множители.

Ход урока

I. Устная работа.

Разложите многочлен на множители.

а) 4y5 – 6y8;                      б) 4900 – а2;                     в) x2;

г) y2 – 6y + 9;                   д) 81x2y2;                е) 25a2 – 10a + 1;

ж) у3 + 8;                          з) 121n2m10.

II. Формирование умений и навыков.

На этом уроке следует рассмотреть, как могут быть применены различные способы разложения на множители при решении задач. Можно выделить три направления такого применения:

1) для упрощения вычислений на калькуляторе;

2) для решения уравнений;

3) для доказательства некоторых утверждений.

В соответствии с этим все задания можно разделить на три группы.

1-я группа

Сначала необходимо рассмотреть пример 4 из учебника, показывающий, как можно рационально выполнить вычисления на калькуляторе, если использовать разложение на множители. Для закрепления следует выполнить № 948.

2-я группа

1. № 949.

Решение:

а) х3х = 0.

    х (х2 – 1) = 0;

    х (х – 1) (х + 1) = 0;

    х = 0,   или   х – 1 = 0,   или   х + 1 = 0.

Ответ: 0; –1; 1.

б) 9хх3 = 0.

    х (9 – х2) = 0;

    х (3 – х) (3 + х) = 0;

    х = 0,   или   3 – х = 0,   или   3 + х = 0.

Ответ: –3; 0; 3.

в) х3 + х2 = 0.

    х2 (х + 1) = 0;

    х2 = 0   или   х + 1 = 0;

    х = 0    или   х = –1.

Ответ: –1; 0.

г) 5х4 – 20х2 = 0.

    5х2 (х2 – 4) = 0;

    5х2 (х – 2) (х + 2) = 0;

    5х2 = 0,  или   х – 2 = 0,   или   х + 2 = 0;

    х = 0,     или   х = 2,         или    х = –2.

Ответ: –2; 0; 2.

2. Можно предложить учащимся решить более сложные уравнения.

а) 2x3x2 – 18x + 9 = 0;                        б) 2x3 + 3x2 = 2x + 3.

Решение:

а) 2x3x2 – 18x + 9 = 0.

    (2x3x2) – (18x – 9) = 0;

    x2 (2x – 1) – 9 (2x – 1) = 0;

    (2x – 1) (x2 – 9) = 0;

    (2x – 1) (x – 3) (x + 3) = 0;

    2х – 1 = 0,    или   х – 3 = 0,   или   х + 3 = 0;

    х = ,          или   х = 3,         или   х = –3.

Ответ: –3; ; 3.

б) 2x3 + 3x2 = 2x + 3.

    (2x3 + 3x2) – (2x + 3) = 0;

    x2 (2x + 3) – (2x + 3) = 0;

    (2x + 3) (x2 – 1) = 0;

    2х + 3 = 0,   или   х – 1 = 0,    или   х + 1 = 0;

    х = ,      или   х = 1,          или   х = –1.

Ответ: –1,5; –1; 1.

3-я группа

1. № 951.

Решение:

Разложим данный многочлен на множители:

Получили произведение трёх последовательных целых чисел. Так как числа последовательные, то хотя бы одно из них чётно, то есть кратно 2, а другое кратно 3. Это означает, что всё произведение кратно 6.

2. № 952.

Решение:

Пусть 2п + 1 и 2п + 3 – два последовательных нечётных числа. Найдем разность их квадратов.

(2п + 3)2 – (2п + 1)2 = ((2п + 3) – (2п + 1)) ((2п + 3) + (2п + 1)) =

= (2п + 3 – 2п – 1) (2п + 3 + 2п + 1) = 2 (4п + 4) = 8 (п + 1).

Значит, исходное выражение делится на 8.

III. Итоги урока.

– Какие вы знаете способы разложения на множители?

– Опишите суть каждого способа.

– При решении каких задач пригодится умение раскладывать многочлен на множители?

Домашнее задание: № 950; № 953; № 998 (а); № 1012 (а, г).

 

 

Урок                                     Дата
ТЕМА: Контрольная работа по теме «Преобразование целых выражений»

Вариант 1

1. Упростите выражение.

а) (x – 3) (x – 7) – 2x (3x – 5);                 в) 2 (m + 1)2 – 4m.

б) 4a (a – 2) – (a – 4)2;

2. Разложите на множители.

а) х3 – 9х;                                                 б) –5a2 – 10ab – 5b2.

3. Упростите выражение

4. Разложите на множители.

а) 16х4 – 81;                                                        б) х2хy2y.

5. Докажите, что выражение х2 – 4х + 9 при любых значениях х принимает положительные значения.

Вариант 2

1. Упростите выражение.

а) 2х (х – 3) – 3х (х + 5);                         в) 3 (y + 5)2 – 3y2.

б) (a + 7) (a – 1) + (a – 3)2;

2. Разложите на множители.

а) с3 – 16с;                                                           б) 3a2 – 6ab + 3b2.

3. Упростите выражение

4. Разложите на множители.

а) 81а4 – 1;                                                          б) y2х2 – 6х – 9.

5. Докажите, что выражение –а2 + 4а – 9 может принимать лишь отрицательные значения.

Вариант 3

1. Упростите выражение.

а) 2c (1 + c) – (c – 2) (c + 4);                  в) 30х + 3 (х – 5)2.

б) (y + 2)2 – 2y (y + 2);

2. Разложите на множители.

а) 4аа3;                                                б) ax2 + 2ax + a.

3. Упростите выражение

4. Разложите на множители.

а) 16 – y4;                                                      б) a + a2bb2.

5. Докажите, что выражение c2 – 2c + 12 может принимать лишь положительные значения.

Вариант 4

1. Упростите выражение

а) 5a (2 – a) + 6a (a – 7);                        в) 20x + 5 (x – 2)2.

б) (b – 3) (b – 4) – (b + 4)2;

2. Разложите на множители.

а) 25уу3;                                                          б) –4x2 + 8 – 4у2.

3. Упростите выражение

4. Разложите на множители.

а)  – b4;                                                          б) a2x2 + 4x – 4.

5. Докажите, что выражение –у2 + 2у – 5 при любых значениях у принимает отрицательные значения.

Решение заданий контрольной работы

Вариант 1

1. а)
= –5x2 + 21;

    б)

    в)
= 2m2 + 2.

2. а) х3 – 9х = х (х2 – 9) = х (х – 3) (х + 3);

    б)

3.

4. а) 16х4 – 81 =
× (4x2 + 9);

    б)
= (x + y) (xy – 1).

5. Выделим из данного трёхчлена квадрат двучлена:

Выражение (х – 2)2 не может быть отрицательным ни при каких значениях х. Значит, выражение (х – 2)2 + 5 принимает положительные значения при любых х.

Вариант 2

1. а)

    б)

    в) 75 – 3y2 =
= 30y + 75.

2. а) с3 – 16с = с (с2 – 16) = с (с – 4) (с + 4);

    б)

3.

4. а) 81а4 – 1 =

    б)
= (y – (x + 3)) (y + (x + 3)) = (yx – 3) (y + x + 3).

5. Выделим из данного трёхчлена квадрат двучлена:

Выражение –(а – 2)2 не может принимать положительных значений ни при каком значении а. Значит, выражение –(а – 2)2 – 5 может принимать только отрицательные значения.

Вариант 3

1. а)

    б)

    в) 30x + 75 =
= 3x2 + 75.

2. а) 4аа3 = а (4 – а2) = а (2 – а) (2 + а);

    б)

3.

4. а)

    б)
= (ab) (a + b + 1).

5. Выделим из данного трёхчлена квадрат двучлена:

Выражение (с – 1)2 не может принимать отрицательных значений ни при каком значении с. Значит, выражение (с – 1)2 + 11 может принимать только положительные значения.

Вариант 4

1. а)

    б)
= –15b – 4;

    в) 20 =
= 5x2 + 20.

2. а) 25уу3 = у (25 – у2) = у (5 – у) (5 + у);

   б)

3.

4. а)

    б)
= (a – (x – 2)) (a + (x – 2)) = (ax + 2) (a + x – 2).

5. Выделим из данного трёхчлена квадрат двучлена:

Выражение –(у – 1)2 не может принимать положительных значений ни при каком значении у. Значит, выражение –(у – 1)2 – 4 может принимать только отрицательные значения.

 

 

 

 

 

 

 

 

 

 

 

 

 

 

 

 

 

Урок                                               Дата
ТЕМА: Анализ результатов контрольной работы

Цели: обобщить и систематизировать знания учащихся; проанализировать ошибки, сделанные в контрольной работе.

Ход урока

I. Анализ результатов контрольной работы.

Самые распространенные ошибки разбираются на доске с обсуждением, а затем каждый из учащихся делает работу над своими ошибками.

II. Обобщение и систематизация знаний.

Те учащиеся, которые допустили ошибки в контрольной работе, после их исправления решают номера из учебника:

№ 990 (а); № 992 (г); 1012 (а, в); 1023 (а).

Сильным учащимся можно предложить задания повышенного уровня сложности.

1. № 1005*.

Решение:

Преобразуем данное выражение в многочлен:


– (a + 1) x + a.

а) Чтобы многочлен не содержал х2, коэффициент, стоящий перед ним, должен быть равен нулю:

1 – а = 0,

а = 1.

б) Чтобы многочлен не содержал х, должно выполняться
условие:

а + 1 = 0,

а = –1.

Ответ: а = 1; а = –1.

2. № 1014*(в).

Решение:

2y3y2 – 32y + 16 = 0.

(2y3y2) – (32y – 16) = 0;

y2 (2y – 1) – 16 (2y – 1) = 0;

(2y – 1) (y2 – 16) = 0;

(2y – 1) (y – 4) (y + 4) = 0;

2у – 1 = 0,   или   у – 4 = 0,   или   у + 4 = 0;

у = ,        или   у = 4,         или   у = –4.

Ответ: –4; ; 4.

3. № 1018*(а, в, д).

Решение:

а)
= (ab – 5) (ab + 5);

в)
= (7 – (a + x)) (7 + (a + x)) = (7 – ax) (7 + a + x);

д) (c – 3b)2 =
= (9a – (c – 3b)) (9a + (c – 3b)) = (9a + 3bc) (9a – 3b + c).

4. № 1019* (а, в, д).

Решение:

а)

в)

д)

5. № 1020* (а, в).

Решение:

а)

в)

III. Итоги урока.

Домашнее задание: № 990 (б); № 992 (в); № 1012 (б, г); № 1023 (б).

Дополнительно: № 1006; № 1019 (б, г, е).

 

 

 

 

 

 

Урок                                     ДАТА
ТЕМА: Возведение двучлена в степень

Цели: рассмотреть, как возводятся двучлены в степень выше третьей; формировать умение строить и использовать треугольник Паскаля для возведения двучлена в степень.

Ход урока

I. Актуализация знаний.

Попросить учащихся записать формулу квадрата и куба двучлена:

С помощью этих формул выполнить возведение в степень:

а) (7x + 3)2;                                  в) (3a + 1)3;

б) (x + 2)3;                        г) (n – 4)3.

II. Изучение нового материала.

Объяснение проводится согласно пункту 39 учебника.

1. На доске и в тетрадях учащихся уже записаны формулы квадрата и куба двучлена. Предложить учащимся самостоятельно вывести формулы для возведения двучлена в четвёртую и пятую степень. Вынести все эти формулы на доску, найти закономерность в коэффициентах.

2. Изучить, как могут быть получены коэффициенты в формулах возведения двучлена в п-ю степень с помощью треугольника Паскаля.

Предложить учащимся самостоятельно построить треугольник Паскаля до восьмой степени, чтобы потом выполнять задания из учебника:

п = 0                              1

п = 1                           1    1

п = 2                         1   2    1

п = 3                      1   3     3   1

п = 4                   1   4     6    4   1

п = 5                1   5   10   10   5   1

п = 6             1   6   15   20   15   6   1

п = 7          1   7   21   35   35   21   7   1

п = 8       1   8   28   56   70   56   28   8   1

III. Закрепление изученного материала.

1. № 958, № 959.

Эти задания учащиеся выполняют, используя построенный треугольник Паскаля.

2. № 961.

Решение:

а) 9a4b3 +
+ 27a2b6 + 27b9;

б) (2xy)4 =
= 1– 8xy + 24x2y2 – 32x3y3 + 16x4y4.

3. № 962 (а).

Решение:

а) 6xy5 +
+ y6 + x6 – 6x5y + 15x4y2 – 20x3y3 + 15x2y4 – 6xy5 + y6 = 2x6 + 30x4y2 +
+ 30x2y4 + 2y6.

4. № 963.

Решение:

Представим данное выражение в виде многочлена:

4y3 + y4 +
+ 1 + 5y + 10y2 + 10y3 + 5y4 + y5 = y5 + 6y4 + 15y3 + 19y2 + 12y + 3.

Получаем:

а) при у2 стоит коэффициент 19;

б) при у3 – коэффициент 15.

Ответ: а) 19; б) 15.

5. № 964.

Представим число 1476 как (145 + 2)6. Если мы будем возводить данную сумму в шестую степень, то получим 6 слагаемых, первые пять из которых содержат множитель 145, то есть делятся на 145. Шестое слагаемое будет равно 26, это и будет остатком от деления 1476 на 145.

Ответ: 64.

6. № 965 (а).

Решение:

Представим число 834 как (81 + 2)4. Выполним возведение в степень:

(81 + 2)4 = 814 + 4 · 813 · 2 + 6 · 812 · 22 + 4 · 81 · 23 + 24.

Найдем значение выражения 834 + 65:

834 + 65 = 814 + 4 · 813 · 2 + 6 · 812 · 22 + 4 · 81 · 23 + 81.

Получим, что каждое слагаемое суммы делится на 81, значит, и вся сумма делится на 81.

IV. Итоги урока.

Домашнее задание: № 960; № 962 (б); № 965 (б).

 

 

Урок                                    ДАТА
ТЕМА: Понятие линейного уравнения с двумя переменными

Цели: ввести понятие линейного уравнения с двумя переменными и его решения; формировать умение подбором находить решение таких уравнений.

Ход урока

I. Устная работа.

1. Прочитайте каждое из выражений.

а) x2y2;                     г) ;                             ж) p3q3;

б) (xy)2;                    д) ;                          з) (2x + 3)3;

2. Решением каких уравнений является число 2?

а) x +  = 2;              б) x = 2;               в) 4 – x = 2;      г) 3 – x = .

II. Объяснение нового материала.

На этом уроке целесообразно разобрать следующие вопросы:

– уравнение с двумя переменными;

– линейное уравнение с двумя переменными;

– решение уравнения с двумя переменными.

Вопрос о том, как находить решение линейного уравнения с двумя переменными с помощью выражения одной переменной через другую, лучше рассмотреть на следующем уроке.

Объяснение материала проводится в несколько этапов.

1. Понятие уравнения с двумя переменными.

Изучение данного вопроса следует начать с рассмотрения задачи.

Задача. Обозначив за х первое число и за у второе число, составьте соотношение по следующим условиям:

а) Первое число на 5 больше второго:

ху = 5.

б) Сумма квадрата первого числа и удвоенного второго числа равна 17:

х2 + 2у = 17.

в) Утроенное произведение чисел равно 24:

3ху = 24.

г) Разность куба первого числа и половины второго числа равна 12:

х3у = 12.

Когда все полученные равенства будут вынесены на доску, сообщить учащимся, что они называются уравнениями с двумя переменными.

2. Понятие линейного уравнения с двумя переменными.

На откидной части доски заранее выписать в два столбика уравнения:

2x + y2 = 10;                                 7x + 2y = ;

х3у = 5;                                 3x – 5y = 11;

x2y2 = 8;                                    x + 0,8y = 0;

xy4 + y = 7;                               –4x + 3y = 9;

4x – 5xy = 2;                                 –2x – 10y = .

Спросить учащихся: в чём характерная особенность уравнений из второго столбца? Замечаем, что эти уравнения имеют вид ax + by = c. Сообщаем учащимся, что такие уравнения называются линейными уравнениями с двумя переменными, и предлагаем им самим сформулировать определение этих уравнений.

Затем даём задание учащимся: назвать коэффициенты a, b и с в линейных уравнениях из второго столбца.

3. Решение уравнений с двумя переменными.

Сначала следует вспомнить, что называется решением уравнения с одной переменной, а затем перейти к определению решения уравнения с двумя переменными.

Дать учащимся задание: проверить, какие из пар чисел являются решениями уравнения х + у = 3.

а) (1; 2);                б) (–2; 4);                   в) (5; –2);                    г) (–7; 11).

Предложить  учащимся  найти  ещё  какие-либо  решения  этого  урав-нения.

После этого сделать вывод: линейное уравнение с двумя переменными может иметь бесконечно много решений.

Затем ещё раз следует повторить определения всех понятий, изученных на этом уроке, и перейти к выполнению заданий.

III. Формирование умений и навыков.

1. № 1025, № 1026 (устно).

2. № 1027 (а).

Данный номер можно выполнить устно или письменно в зависимости от уровня подготовки класса.

3. № 1028.  (Это  задание  лучше  сделать  письменно,  вызвав  одного ученика к доске. Предложить учащимся форму записи решения такого задания.)

Решение:

10х + у = 12.

(3; 20):                  10 · 3 + (–20) = 10                     10  12 – не является;

(–2; 12):                10 · (–2) +12 = –8                      –8  12 – не является;

(0,1; 11):               10 · 0,1 + 11 = 12                      12 = 12 – является;

(1; 2):                    10 · 1 + 2 = 10                           12 = 12 – является;

(2; 1):                    10 · 2 + 1 = 21                           21  12 – не является.

4. № 1029 (а).

Каждый из учащихся должен составить своё уравнение, некоторые из которых выносятся на доску.

IV. Итоги урока.

– Приведите пример уравнения с двумя переменными.

– Какое из уравнений с двумя переменными называется линейным?

– Что называется решением уравнения с двумя переменными?

– Как проверить, является ли пара чисел  решением уравнения x + 2y = 7?

– Сколько решений может иметь линейное уравнение с двумя переменными?

Домашнее задание: № 1027 б); № 1029 (б); № 1137; № 1138 (а, в).

 

 

 

 

 

 

 

 

 

 

 

 

 

 

Урок                                               ДАТА
ТЕМА:Решение линейных уравнений с двумя переменными

Цели: формировать умение решать линейные уравнения с двумя переменными с помощью выражения одной переменной через другую; проверить уровень усвоения материала.

Ход урока

I. Устная работа.

1. Какие из следующих уравнений являются линейными
с двумя переменными?

а) 5xy + 3 = 7;                              в) yx = 10;               г) 7x = 5;

б) 3x – 7y = ;                            г) 5x + 2x2 = 1;                       е) –4x + 0,8y = –2.

В линейных уравнениях назовите коэффициенты a, b и с.

2. Решением какого уравнения является пара чисел ?

а) 2x + y = 1;                            в) x + 2y = 2;

б) xy = ;                                г) x – 3y = 0.

II. Объяснение нового материала.

На этом уроке следует разобрать, как в линейных уравнениях выражать одну переменную через другую и как с помощью этого можно находить решения таких уравнений. также нужно рассмотреть вопрос о решении уравнений в целых числах.

1. Выражение одной переменной через другую.

Сначала необходимо актуализировать знания учащихся, задав им следующие вопросы:

– Какие два уравнения называются равносильными?

– Будут ли уравнения 3х = 9 и 2х = 6 равносильны?

– Какие преобразования можно совершать при решении линейных уравнений с одной переменной?

– Поясните каждое из проводимых преобразований на примере решения уравнения 1 – 2х = 5.

Затем сообщить учащимся, что уравнения с двумя переменными обладают такими же свойствами, как и уравнения с одной переменной, а значит, при их решении можно выполнять аналогичные преобразования. Благодаря этому появляется возможность выражать в таких уравнениях одну переменную через другую.

Примеры:

а) х + 2y = 4.

Выразим переменную х через у:

х = 4 – 2y.

Выразим переменную у через х:

2у = 4 – х;

у = .

б) 3х – 5y = 2.

3х = 2 + 5y;                      –5y = 2 – 3х;

х = .                    y = .

2. решение линейных уравнений с двумя переменными.

Замечаем, что с помощью выражения одной переменной через другую можно находить разнообразные решения линейных уравнений с двумя переменными. Рассмотреть по учебнику решение уравнения 5x + 2y = 12. Ещё раз сделать вывод о том, что подобные уравнения могут иметь бесконечно много решений.

3. решение уравнений в целых числах.

Учащиеся уже разобрали, как решаются линейные уравнения с двумя переменными, и выяснили, что они могут иметь бесконечно много решений. Среди всех решений есть пары, включающие в себя дробные числа.

Однако при решении некоторых задач возникает необходимость отыскать только целые или натуральные пары решений уравнений с двумя переменными. Рассмотреть пример решения такой задачи из учебника.

III. Формирование умений и навыков.

Основная цель на этом уроке состоит в том, чтобы учащиеся научились в линейных уравнениях с двумя переменными выражать одну переменную через другую и отыскивать решения таких уравнений.

К решению задач на составление уравнений можно перейти только в том случае, если данное умение будет сформировано в полной мере.

1. № 1030.

2. № 1032.

Решение:

а) 3х + 2у = 12.

    2у = 12 – 3х;

    у = ;

    у = 6 – 1,5х;

если х = 2,            то у = 6 – 1,5 · 3 = 3              (2; 3);

если х = –4,          то у = 6 – 1,5 · (–4) = 12                   (–4; 12);

если х = 10,          то у = 6 – 1,5 · 10 = –9                      (10; –9).

3. № 1035.

4. № 1037.

Решение:

Пусть было взято х двухрублёвых и у пятирублёвых монет. Получим уравнение:

2х + 5у = 28.

Требуется найти все пары натуральных значений переменных х и у, удовлетворяющих этому уравнению.

Выразим переменную х через у:

2х = 28 – 5у;

х = .

Имеем:

если у = 2, то х = 9;

если у = 4, то х = 4.

Других пар натуральных чисел, удовлетворяющих уравнению 2х + 5у =
= 28, нет.

Ответ: 4 или 9 монет.

5. № 1039.

Решается аналогично предыдущей задаче.

№ 1042 (можно предложить сильным учащимся выполнить дополнительно).

Решение:

Пусть п – некоторое натуральное число. Если оно при делении на 5 даёт остаток 1, то справедливо следующее равенство:

п = 5q + 1, где q – частное от деления на 5.

Аналогично можно записать равенство:

п = 6p + 2, где р – частное от деления на 6.

Получим уравнение:

5q + 1 = 6p + 2;

5q – 6p = 1.

Выразим переменную q через переменную р:

5q = 6p + 1;

q = .

Нужно подобрать наименьшую натуральную пару (p; q), удовлетворяющую уравнению.

Если р = 4, то q =  = 5. Найдем п:

п = 5q + 1 = 5 ∙  5 + 1 = 26.

Ответ: 26.

IV. Проверочная работа.

Вариант 1

1. Решением каких уравнений является пара чисел (–2; 3):

а) 2x + y = 1;                                в) x2 + y = 1;

б) xy = –5;                                г) 3x + y2 = 3?

2. Выразите из уравнения 2x – 3y = 7 переменную х через у и найдите три решения этого уравнения.

V. Итоги урока.

– Какие уравнения называются линейными с двумя переменными?

– Что называется решением уравнения с двумя переменными?

– Какими свойствами обладают уравнения с двумя переменными?

– Как  можно  найти  решение  линейного  уравнения  с  двумя  переменными?

– Сколько  может  иметь  решений  линейное  уравнение  с  двумя  переменными?

Домашнее задание: № 1031, № 1034, № 1036, № 1038.

Дополнительно: № 1041.

 

 

 

 

 

 

 

 

 

 

 

 

 

 

 

 

 

Урок                                            ДАТА
тЕМА:Понятие графика линейного уравнения
с двумя переменными

Цели: ввести понятие графика линейного уравнения с двумя переменными; формировать умение строить такие графики и находить по ним решения уравнений.

Ход урока

I. Устная работа.

1. Что является графиком функции y = 3x? Как называется эта функция и как построить её график?

2. Графикам каких функций принадлежит точка А (3; –2):

а) y = 2x + 1;                                в) y = –x + 2;

б) y = x – 5;                                  г) y = 4 – 2x?

II. Объяснение нового материала.

Объяснение проводить согласно пункту 41 учебника в несколько этапов.

1. Ввести понятие графика уравнения с двумя переменными.

2. Выяснить, что представляет собой график линейного уравнения с двумя переменными.

3. Рассмотреть  случаи,  когда  коэффициенты  при  х  или  у  в  уравнении ax + by = c равны нулю.

4. Сделать выводы и рассмотреть примеры построения графиков линейных уравнений с двумя переменными.

III. Формирование умений и навыков.

1. № 1045.

Необходимо, чтобы учащиеся могли ответить на вопрос: как определить, принадлежит ли графику уравнения какая-либо точка с данными координатами? Вызвать к доске одного из учащихся и предложить способ оформления решения подобных задач.

Решение:

3x + 4y = 12.

а) А (4; 1):     3 · 4 + 4 · 1 = 16           16  12 – не принадлежит;

б) В (1; 3):                3 · 1 + 4 · 3 = 15           15  12 – не принадлежит;

в) С (–6; –7,5):         3 · (–6) + 4 · (–7,5) = –48   –48  12 – не принадлежит;

г) D (0; 3):                3 · 0 + 4 · 3 = 12           12 = 12 – принадлежит.

2. № 1047.

3. По данному графику линейного уравнения с двумя переменными найдите три какие-либо его решения:

4. № 1048 (а, б).

5. В одной системе координат постройте графики уравнений:

а) x = 1;                                    в) 1,2у = –2,4;

б) 2х = –8;                                    г) y = 3.

IV. Итоги урока.

– Что называется графиком уравнения с двумя переменными?

– Что является графиком линейного уравнения с двумя переменными?

– Как построить график линейного уравнения с двумя переменными?

– Как определить, принадлежит ли точка с данными координатами графику уравнения с двумя переменными?

Домашнее задание: № 1046; № 1048 (в, г, д, е).

 

 

 

 

 

 

 

 

 

 

Урок                                                   Дата
ТЕМА: Построение графика линейного уравнения с двумя переменными

Цели: продолжить формирование умения строить графики линейных уравнений с двумя переменными; проверить уровень усвоения материала.

Ход урока

I. Устная работа.

1. Является ли решением уравнения х – 2у = 3 пара чисел:

а) (3; 1);                б) (7; 2);                     в) (–1; –1);                  г) (–1; –2)?

Принадлежит  ли  графику  этого  уравнения  точки  с  такими  координатами?

2. Принадлежит ли графику уравнения 3х + у = 5 точка:

а) А (1; 2); б) В (2; –3);                в) С (–1; 8);                г) D (–2; 1)?

Являются ли решением этого уравнения данные пары чисел?

II. Формирование умений и навыков.

1. Дан график некоторого линейного уравнения с двумя переменными:

а) Определите  по  графику, какие из пар чисел  (1; –2), (–2; 0), (–3; –1), (–1; –1) являются решениями этого уравнения.

б) Найдите несколько решений этого уравнения.

2. В одной системе координат постройте графики уравнений:

а) 2x + y = 3;                                б) x = 2;              в) 0,7у = 2,1.

3. № 1050 (а, в).

4. № 1051, № 1052.

Сильным учащимся можно предложить выполнить дополнительно № 1154 (а, в).

Решение:

а) (x – 2) (y – 3) = 0.

Произведение  равно  нулю,  когда  хотя  бы  один  из  множителей  равен нулю:

х – 2 = 0;  или           у – 3 = 0;

х = 2                                      у = 3.

Значит, графиком данного уравнения служат две прямые: х = 2 и у = 3.

III. Проверочная работа.

Вариант 1

1. Принадлежит ли графику уравнения 2х – 5у = 1 точка:

а) А (3; 1);

б) В (–1; –1);

в) С (–2; –1)?

2. Постройте график линейного уравнения –4x + 3y = 6.

3. Известно, что график уравнения x + 2y = 2 проходит через точку А, абсцисса которой равна 2. Найдите ординату этой точки.

IV. Итоги урока.

– Что называется графиком уравнения с двумя переменными?

– Как построить график линейного уравнения с двумя переменными?

– Как  определить,  принадлежит  ли  точка  А (2; –4)  графику  уравнения 3x + y = 2?

– Как найти абсциссу точки, принадлежащей графику какого-либо уравнения, если известна её ордината?

Домашнее задание: № 1049 (б, в, г); № 1050 (б, г); № 1148.

 

 

 

 

Урок                                                   Дата
ТЕМА: Понятие системы уравненийс двумя переменными

Цели: ввести понятие системы уравнений с двумя переменными; формировать умение решать графически системы линейных уравнений с двумя переменными.

Ход урока

I. Устная работа.

1. Какие из пар чисел являются решениями уравнения –ху = 5?

а) (2; 3);                б) (–2; 3);                   в) (–3; –2);                  г) (1; –6).

2. Даны два уравнения: х + у = 3 и ху = 1. Какие из пар чисел являются одновременно решением каждого из этих уравнений:

а) (1; 2);                б) (–1; 2);                   в) (2; 1);                      г) (–2; 5)?

II. Объяснение нового материала.

На этом уроке следует ввести понятие системы уравнений с двумя переменными и рассмотреть, как графически решаются системы линейных уравнений. Вопрос о возможном количестве решений таких систем целесообразно рассмотреть на следующем уроке.

Объяснение проводить согласно пункту 42 учебника в несколько этапов.

1. Рассмотреть задачу из учебника, подводящую к понятию системы уравнений с двумя переменными. Здесь необходимо добиться чёткого понимания учащимися того, в чём состоит отличие простых уравнений с двумя переменными от их систем.

Можно вернуться ко второму заданию устной работы, обратив внимание учащихся на то, что мы искали общее решение двух уравнений.

2. Ввести понятие решения системы уравнений с двумя переменными. Учащиеся должны уметь формулировать определение этого понятия.

Желательно привести примеры, показывающие, что некоторые пары чисел могут быть решением какого-либо одного уравнения системы, но не являться решением всей системы.

Пример.

(2; 1)   –

является решением 1-го уравнения системы, но не является решением 2-го, значит, не является решением системы
уравнений.

(–1; 1) –

является решением 2-го уравнения системы, но не является решением 1-го, значит, не является решением системы
уравнений.

(1; 3)   –

является решением и 1-го, и 2-го уравнений, значит,
является решением всей системы.

3. Рассмотреть, как можно графически решить любую систему линейных уравнений. При этом обратить внимание учащихся, что данный способ не всегда позволяет находить точные решения системы, поэтому в дальнейшем будут изучены другие способы.

III. Формирование умений и навыков.

1. № 1056.

Необходимо показать учащимся, как следует оформлять решение подобных заданий:

а) х = 3, у = 1:

Ответ: не является.

б) х = 2, у = 2:

Ответ: является.

2. № 1058 (а).

3. № 1059.

Каждый из учащихся составляет систему самостоятельно, а затем некоторые из систем выносятся на доску. Можно устроить конкурс: у кого система получилась «красивее», то есть такая, которую сложнее составить.

Например

4. № 1060 (а, б).

При построении графиков учащиеся могут выражать переменную у через х, а могут просто в каждое из уравнений подставить некоторое значение х и находить соответствующее ему значение у.

IV. Итоги урока.

– Что представляет собой система уравнений с двумя переменными?

– Что называется решением системы уравнений с двумя переменными?

– Является ли пара чисел (1; –2) решением системы уравнений

– Как решить систему линейных уравнений с двумя переменными графически?

Домашнее задание: № 1057; № 1058 (б); № 1060 (в, г).

 

 

 

 

Урок                                    Дата
ТЕМА: Графическое решение систем линейных уравнений с двумя переменными

Цели: продолжить формирование умения решать графически системы линейных уравнений с двумя переменными; рассмотреть вопрос о возможном количестве решений таких систем; проверить уровень усвоения материала.

Ход урока

I. Устная работа.

1. Является ли пара чисел (2; –5) решением уравнения:

а) 2x + y = 9;                                в) –x + y = 3;

б) xy = 7;                                  г) y – 2x = –9?

2. Является ли пара чисел (1; 2) решением системы уравнений:

а)                            б)                     в)

II. Объяснение нового материала.

Сначала необходимо актуализировать знания учащихся. Они должны чётко  сформулировать,  как  графически  решаются  системы  линейных уравнений.

Акцент делаем на то, что решением системы уравнений будет координата точки пересечения двух построенных прямых. После этого ставим вопрос: сколько решений может иметь система линейных уравнений и от чего это зависит? Ясно, что система будет иметь столько решений, в скольких точках пересекутся графики уравнений, входящих в неё.

Спросить у учащихся: может ли система линейных уравнений с двумя переменными иметь два или три решения? Очевидно, что нет, поскольку прямые могут пересечься только в одной точке. Тогда задаём учащимся следующий вопрос: а как ещё могут располагаться прямые?

Далее рассматриваем все случаи расположения двух прямых на плоскости и зависимость этого расположения от уравнений этих прямых. Делаем выводы и даём их учащимся под запись:

1) Если угловые коэффициенты прямых различны, то они пересекаются в одной точке, следовательно, система имеет единственное решение.

2) Если угловые коэффициенты прямых одинаковы, а точки пересечения с осью у различны, то прямые параллельны, следовательно, система не имеет решений.

3) Если уравнения прямых одинаковы, то их графики совпадают, следовательно, система имеет бесконечно много решений.

III. Формирование умений и навыков.

1. Решите графически систему уравнений:

2. № 1062.

Решение:

а)

    , значит, система имеет одно решение.

в)

1,5x = 1 – прямая, параллельная оси y

–3x + 2y = –2 – прямая, непараллельная оси y

  Þ

система имеет
одно решение

г)

–0,5 = –0,5

1,5 ¹ 0

  Þ  система не имеет решений.

3. № 1064 (а).

Сильным учащимся можно дать дополнительное задание.

4. Подберите, если возможно, такое значение k, при котором данная система имеет единственное решение; не имеет решений; имеет бесконечное множество решений.

а)                     б)                     в)

Решение:

а)

Если k = 3, то прямые будут параллельны, то есть система не будет иметь решений. В остальных случаях прямые пересекаются, значит, система имеет единственное решение.

б)

Поскольку коэффициенты при х равны, то прямые будут либо параллельны, либо совпадать, то есть единственное решение система иметь не может.

Если k = –1, то прямые совпадают, значит, система будет иметь бесконечное множество решений. В остальных случаях прямые будут параллельны, то есть система не имеет решений.

в)

Если , то есть k = 3, то уравнения системы будут одинаковы, значит,  прямые  совпадают,  то  есть  система  имеет  бесконечное  множество решений. В остальных случаях система будет иметь единственное решение.

IV. Проверочная работа.

Вариант 1

1. Решите графически систему уравнений:

2. Не выполняя построений, выясните, сколько решений имеет система уравнений.

а)                     б)                       в)

Вариант 2

1. Решите графически систему уравнений:

2. Не выполняя построений, выясните, сколько решений имеет система уравнений.

а)        б)                      в)

V. Итоги урока.

– Что называется решением системы уравнений с двумя переменными?

– Является пара чисел (–1; –1) решением системы уравнений

– Как графически решить систему линейных уравнений
с двумя переменными?

– Сколько решений может иметь система линейных уравнений с двумя переменными?

– Как найти количество решений системы линейных уравнений с двумя переменными?

Домашнее задание: № 1061; № 1063; № 1064 (б).

 

Урок 109
Алгоритм решения систем линейных уравнений
способом подстановки

Цели: разобрать, в чём состоит способ подстановки решения систем линейных уравнений; вывести алгоритм применения этого способа; формировать умение решать системы уравнений способом подстановки.

Ход урока

I. Устная работа.

1. Является ли пара чисел (2; 3) решением системы уравнений:

а)                    б)                  в)

2. Сколько решений имеет система уравнений:

а)                б)                  в)

II. Объяснение нового материала.

Объяснение проводить согласно пункту 43 учебника.

1. Разобрать пример 1, сообщив учащимся, что данный способ решения систем уравнений называется способом подстановки.

2. Дать определение равносильных систем уравнений и привести их геометрическую интерпретацию.

3. Предложить учащимся самостоятельно на основе разнообразного примера сформулировать, в чём состоит способ подстановки решения систем линейных уравнений.

Желательно, чтобы учащиеся записали в тетрадях алгоритм решения систем уравнений способом подстановки. При этом каждый шаг алгоритма должен отражаться соответствующим действием в решении системы уравнений.

Алгоритм

1-й шаг.

Выразить из какого-нибудь уравнения системы
одну переменную через другую

2-й шаг.

Подставить в другое уравнение системы вместо
этой переменной полученное выражение

3-й шаг.

Решить полученное уравнение с одной
переменной

4 (3 + y) + y = 2,

12 + 4у + у = 2,

5у = –10,

у = –2.

4-й шаг.

Найти соответствующее значение второй
переменной

х = 3 + у,

х = 3 + (–2),

х = 1.

Ответ: (1; –2)

Обратить внимание учащихся, что выражать следует ту переменную, при которой стоит более «удобный» коэффициент (в частности ±1).

Пример 2 лучше разобрать на следующем уроке.

III. Формирование умений и навыков.

Желательно, чтобы в течение урока учащиеся запомнили алгоритм решения систем уравнений способом подстановки и могли его применять, не обращаясь к записям в тетрадях и разобранным примерам.

1. Выразите в уравнениях х через у и у через х.

а) х + у = 5;               в) х – 3у = –6;                д) 5х – 2у = 0;

б) ух = –2;             г) –2х + у = 3;                е) 3х + 5у = –7.

2. № 1068.

3. № 1069.

Для решения каждой системы следует вызывать к доске по одному учащемуся. Требовать, чтобы они вслух комментировали все шаги решения.

Можно  предложить  учащимся  такое  оформление  решения  систем уравнений, при котором их не нужно «тянуть» до конца решения, а после получения  уравнения  с  одной  переменной  приступать  отдельно  к  его решению.

а)

    6х – (2х + 1) = 7;

    6х – 2х – 1 = 7;

    4х = 8;

    х = 2;

    у = 2х + 1;

    у = 2 · 2 + 1 = 5.

Ответ: (2; 5).

в)

    3 (6 – у) – 5у = 2;

    18 – 3у – 5у = 2;

    –8у = –16;

    у = 2;

    х = 6 – у;

    х = 6 – 2 = 4.

Ответ: (4; 2).

IV. Итоги урока.

– Что называется решением системы уравнений с двумя переменными?

– Какие вы знаете способы решения систем уравнений?

– Сформулируйте алгоритм решения систем уравнений способом подстановки.

– Из какого уравнения системы лучше выражать переменную?

Домашнее задание: № 1070.

 

 

Урок 110
Решение систем линейных уравнений
способом подстановки

Цели: продолжить формирование умения решать системы уравнений способом подстановки; проверить первоначальный уровень усвоения материала.

Ход урока

I. Устная работа.

Является ли пара чисел (–3; 1) решением системы уравнений:

а)                б)                  в)

II. Проверочная работа.

Вариант 1

1. Выразите в уравнении х через у и у через х.

а) x + y = ;                     б) 2xy = 7;                     в) –3x + 5y = 1.

2. Решите  систему  уравнений  способом  подстановки  и  сделайте  проверку.

а)                  б)

Вариант 2

1. Выразите в уравнении х через у и у через х.

а) xy = ;                     б) x + 3y = 5;                    в) 4x – 5y = –1.

2. Решите  систему  уравнений  способом  подстановки  и  сделайте  проверку.

а)                 б)

III. Формирование умений и навыков.

На этом уроке учащиеся будут решать системы уравнений, в которых ни один коэффициент при переменных не равен ±1. Сначала нужно разобрать пример 2 из учебника, сделать соответствующие выводы, а затем приступить к выполнению заданий.

1. № 1071.

Следует обратить внимание учащихся, что иногда удобнее выражать переменную вместе с её коэффициентом.

Решение:

а)

20v + 15v = 7;

35v = 7;

v = ;

2u = –5 ∙   = –1;

u = .

Ответ: .

б) Здесь не получится сделать, как в предыдущей системе, поскольку коэффициенты при переменных не являются кратными.

3p + 4 ∙  p = 29;

3 · 3р + 4 · 5р = 29 · 3;

9р + 20р = 29 · 3;

29р = 29 · 3;

р = 3;

q = p =  ∙  3 = 5.

Ответ: (3; 5).

в)

5u – (14 – 4u) = 25;

5u – 14 + 4u = 25;

9u = 39;

u = .

3v = 14 – 4 ∙  4;

3v = 14 – 17 = –3;

v = –1.

Ответ: .

г)

5 ∙  (5p + 22) + 7q = –2;

25p + 110 + 7q = –2;

32q = –112;

q = –3,5.

2p = 5 ∙  (–3,5) + 22;

2р = –17,5 + 22 = 4,5;

р = 2,25.

Ответ: (2,25; –3,5).

2. № 1073.

Решение:

Чтобы найти координаты точки пересечения двух прямых, нужно решить соответствующую систему уравнений.

а)

16х – 5 (23 – 7х) = 38;

16х – 115 + 35х = 38;

51х = 153;

х = 3.

Ответ: (3; 0,5).

IV. Итоги урока.

– Что называется решением системы уравнений с двумя переменными?

– Сформулируйте алгоритм решения систем уравнений способом подстановки.

– В каких случаях при решении системы уравнений можно выражать переменную вместе с её коэффициентом?

Домашнее задание: № 1072, № 1074.

 

 

 

Урок 111
Решение систем линейных уравнений
способом подстановки

Цели: закрепить умение учащихся решать системы линейных уравнений способом подстановки; проверить уровень усвоения материала.

Ход урока

I. Устная работа.

1. Является ли пара чисел (–2; –2) решением системы уравнений:

а)                 б)                в)

2. Из какого уравнения системы и какую переменную выразить «удобнее»? Ответ объясните.

а)                 б)                в)

II. Формирование умений и навыков.

На этом уроке учащиеся будут решать системы уравнений, в которых до выражения одной переменной через другую предварительно необходимо провести ряд преобразований.

1. № 1075.

Решение таких систем не должно вызывать затруднений у учащихся. Достаточно открыть скобки, привести подобные слагаемые – и система станет похожей на ранее решённые.

2. № 1171 (а).

Решение:

2 (1 – 2у) + 1 = –3у;

2 – 4у + 1 = –3у;

у = –3;

у = 3;

х = 1 – 2у;

х = 1 – 2 · 3 = –5.

Ответ: (–5; 3).

3. № 1077.

Учащиеся уже знают, что если в линейном уравнении встречаются дроби, то обе части уравнения нужно умножать на наименьший общий знаменатель этих дробей.

Необходимо объяснить учащимся, что таким же приёмом пользуются и при решении систем уравнений.

Решение:

а)

2 (–у – 2) – 3у = –24;

–2у – 4 – 3у = –24;

–5у = –20;

у = 4;

х = –у – 2;

х = – 4 – 2 = –6.

Ответ: (–6; 4).

Замечание. Обращаем внимание на опечатку: во втором уравнении системы вместо –2 должно стоять –1.

в)

2 (35п + 120) + 5п = 15;

70п + 240 + 5п = 15;

75п = –225;

п = –3;

3т = 35 · (–3) + 120;

3т = –105 + 120 = 15;

т = 5.

Ответ: т = 5, п = –3.

4*. Сильным учащимся дополнительно можно предложить выполнить № 1173.

Решение:

а)

Система содержит три уравнения, а переменных всего две. Такая система имеет решение, если общее решение двух любых её уравнений будет являться решением третьего уравнения.

Сначала нужно решить систему из двух уравнений:

Подставим пару чисел  в третье уравнение:

7 · 4 – 5 ·  = 1.

Очевидно, что равенство будет неверным. Поэтому исходная система решений не имеет.

б)

Решим систему уравнений:

11х + 3(3 – 2х) = 1;

11х + 9 – 6х = 1;

5х = –8;

х = –1,6;

у = 3 – 2 · (–1,6);

у = 6,2.

Подставим пару чисел (–1,6; 6,2) в третье уравнение:

5 · (–1,6) + 2 · 6,2 = 4;

–8 + 12,4 = 4;

4,4 = 4 – неверно.

Значит, исходная система решений не имеет.

III. Проверочная работа.

Вариант 1

1. Решите систему уравнений 

2. Не выполняя построений, найдите координаты точки пересечения графиков уравнений 3x + 7y = 2 и 2x – 5y = 1.

3. Решите систему уравнений 

Вариант 2

1. Решите систему уравнений 

2. Не выполняя построений, найдите координаты точки пересечения графиков уравнений 2x – 9y = 1 и 5x + 2y = 3.

3. Решите систему уравнений 

IV. Итоги урока.

– Что называется решением системы уравнений с двумя переменными?

– Сформулируйте алгоритм решения системы уравнений способом подстановки.

– Как, не выполняя построений, найти координаты точки пересечения графиков двух уравнений?

– Как следует начать решение системы уравнений, в которой встречаются дробные коэффициенты?

Домашнее задание: № 1076; № 1171 (б); № 1078.

Урок                                    Дата
ТЕМА: Алгоритм решения систем линейных уравнений способом сложения

Цели: разобрать, в чём состоит способ сложения решения систем линейных уравнений; вывести алгоритм применения этого способа; формировать умение решать системы уравнений способом сложения.

Ход урока

I. Устная работа.

1. Является ли пара чисел (4; –1) решением системы уравнений:

а)                        б)                         в)

2. Являются ли данные системы уравнений равносильными:

      и     

II. Объяснение нового материала.

Объяснение  проводить  согласно  пункту  44  учебника  в  несколько этапов:

1. На примере 1 выявить суть способа сложения решения систем линейных уравнений.

2. Рассмотреть вопрос о равносильности систем уравнений и его геометрическую интерпретацию.

3. Рассмотреть пример 2 из учебника.

4. Вывести алгоритм решения систем линейных уравнений способом сложения.

Так же, как был записан алгоритм решения систем уравнений способом подстановки, учащиеся должны занести в тетради новый алгоритм вместе с примером.

Алгоритм

1-й шаг.

Умножить почленно уравнения системы на такие множители, чтобы коэффициенты при одной
из переменных стали противоположными

2-й шаг.

Сложить почленно левые и правые части
уравнений системы

3-й шаг.

Решить получившееся уравнение с одной
переменной

  –х = –1,

  х = 1.

4-й шаг.

Найти соответствующее значение второй
переменной

 3·1+2у=–1,

 2у=–4,

 у=–2.

 Ответ: (1; –2)

Системы, в которых нужно подбирать множители к обоим уравнениям, на этом уроке решать не нужно, поэтому пример 3 также лучше разобрать на следующем уроке.

III. Формирование умений и навыков.

В течение урока учащиеся должны запомнить алгоритм решения систем линейных уравнений способом сложения.

1. Умножьте одно из уравнений системы на какое-нибудь число так, чтобы с помощью сложения можно было исключить одну из переменных.

а)                б)                       в)

2. № 1082.

Для решения каждой системы следует вызывать к доске по одному учащемуся. Требовать, чтобы они вслух комментировали все шаги решения.

Необходимо показать учащимся вариант оформления решения системы уравнений способом сложения.

Решение:

в)

2у = 60;

у = 30;

4х – 5 · 30 = 90;

4х = 240;

х = 60.

Ответ: (60; 30).

3. № 1084 (а, б, в).

Этот номер несколько сложнее предыдущего.

Учащимся придётся подбирать множитель, который сделает коэффициенты противоположными. Множитель лучше не «держать в уме», а записывать справа от уравнения.

Решение:

а)

15у = 0;

у = 0;

20х – 7 · 0 = 5;

20х = 5;

х = .

Ответ: .

IV. Итоги урока.

– Какие существуют способы решения систем уравнений?

– Сформулируйте алгоритм решения систем линейных уравнений способом сложения.

– Сколько решений может иметь система линейных уравнений?

Домашнее задание: № 1083; № 1085 (а, б).

 

 

 

 

 

 

 

 

 

 

 

 

 

 

 

 

 

 

Урок                                    Дата
ТЕМА: Решение систем линейных уравнений способом сложения

Цели: продолжить формирование умения решать системы уравнений способом  сложения;  проверить  первоначальный  уровень  усвоения  материала.

Ход урока

I. Устная работа.

Являются ли следующие системы уравнений равносильными:

а)           и         

б)           и         

II. Проверочная работа.

Вариант 1

1. Умножьте одно из уравнений системы на такое число, чтобы с помощью сложения можно было исключить одну из переменных.

а)                б)                       в)

2. Решите способом сложения систему уравнений:

а)                 б)

III. Формирование умений и навыков.

На этом уроке следует разобрать с учащимися решение систем уравнений, в которых для применения способа сложения нужно подбирать множители для обоих уравнений системы.

Сначала необходимо рассмотреть пример 3 из учебника, сделать выводы, а затем приступить к выполнению заданий.

1. № 1084 (г, д, е).

Решение:

г)

17х = 34;

х = 2;

11 · 2 – 18у = 4;

–18у = 18;

у = 1.

Ответ: (2; 1).

После решения подобных систем необходимо, чтобы учащиеся сделали вывод: для нахождения множителей нужно сначала узнать наименьшее общее кратное коэффициентов.

2. № 1093.

Прежде чем применять способ сложения для подобных систем уравнений, нужно избавиться от дробных коэффициентов.

Решение:

а)

   

19х = 57;

х = 3;

5 · 3 – у = 11;

у = –4;

у = 4.

Ответ: (3; 4).

г)

   

5v = 75;

v = 15;

2u + 15 = 39;

2u = 24;

u = 12.

Ответ: (12; 15).

3. № 1095 (а, г).

IV. Итоги урока.

– Сформулируйте алгоритм решения систем линейных уравнений способом сложения.

– На какое число нужно умножить каждое из уравнений системы  чтобы её можно было решить способом сложения?

Домашнее задание: № 1085 (в, г); № 1094.

 

Урок                                    Дата
ТЕМА:Составление уравнений прямой, проходящей через две заданные точки

Цели: закрепить умение учащихся решать системы уравнений способом сложения; разобрать, как с помощью системы уравнений можно составить уравнение прямой, проходящей через две заданные точки; проверить уровень усвоения материала.

Ход урока

I. Устная работа.

1. Являются ли следующие системы уравнений равносильными:

а)             и         

б) и         

2. Первое уравнение системы у = 2х – 1. Подберите второе уравнение так, чтобы полученная система:

а) имела единственное решение;

б) не имела решений;

в) имела бесконечное множество решений.

II. Формирование умений и навыков.

Все задания можно разбить на две группы. В 1-й группе будут задания на закрепление умения решать системы уравнений способом сложения. Во 2-ю группу войдут задания, в которых требуется написать уравнение прямой, проходящей через две точки с данными координатами.

1-я группа

1. № 1086 (а, в).

2. № 1092 (а).

2-я группа

1. № 1087 (а, в).

Решение:

а) Чтобы составить уравнение прямой, нужно найти коэффициенты k и b. Подставляя координаты данных точек M (5; 5) и N (–10; –19) в уравнение y = kx + b, получим систему уравнений:

15k = 24;

k = 1,6;

5 · 1,6 + b = 5;

b = 5 – 8;

b = –3.

Получим уравнение: у = 1,6х – 3.

2. № 1088.

3. № 1091.

Решение:

Чтобы задать формулой функцию по её графику, нужно найти на этом графике  две  любых точки и записать их координаты. Например, А (–1; 1) и В (1; –3). Задача свелась к составлению уравнения прямой y = kx + b, проходящей через точки А и В.

2b = –2;

b = –1;

1 = –k – 1;

k = –2.

Получим уравнение: у = –2х – 1.

Сильным учащимся можно предложить дополнительно выполнить задания на карточках.

Карточка 1

Решите систему уравнений:

а)                     б)

Решение заданий на карточке 1

а)

Если сложить первое и третье уравнения системы, то получится уравнение с одной переменной:

2х = 6;

х = 3.

Подставив найденное значение х в первое и второе уравнения, получим и решим систему:

2у = 4;

у = 2;

2 – z = 1;

z = 1.

Ответ: (3; 2; 1).

б) Сделаем замену переменных:  = a,  = b. Получим и решим систему уравнений:

3b = 9;

b = 3;

5a – 6 · 3 = 2;

5a = 20;

a = 4.

Вернёмся к замене:        = 4, значит, x = ;

                                          = 3, значит, y = .

Ответ: .

III. Проверочная работа.

Вариант 1

Решите систему уравнений.

а)                б)

IV. Итоги урока.

– Что называется решением системы уравнений с двумя переменными?

– Какие существуют способы решения систем уравнений?

– Опишите, в чем состоит каждый из трёх способов решения систем уравнений.

– Любую ли линейную систему уравнений можно решить графически? способом подстановки? способом сложения?

Домашнее задание: № 1086 (б, г); № 1087 (б, г); № 1089; № 1092 (б).

 

 

 

Урок                                  дата
ТЕМА: Составление системы уравнений по условию задачи

Цели: изучить способ решения задач с помощью составления систем уравнений; формировать умение составлять системы уравнений по условию задачи и решать их.

Ход урока

I. Устная работа.

Какое из уравнений нужно записать в систему  чтобы она имела единственное решение? не имела решений? имела бесконечное множество решений?

а) y + 3x = 7;                                в) y – 2x = 3;

б) 4x – 2y = 2;                              г) x = 5.

II. Объяснение нового материала.

Сначала следует вспомнить, в чём заключается способ решения задач с помощью составления уравнения, а затем показать, что задачи могут решаться и с помощью составления системы уравнений.

Разобрав примеры решения задач, учащиеся должны сформулировать действия, которые необходимо выполнить, чтобы решить задачу с помощью составления системы уравнений.

III. Формирование умений и навыков.

Сначала необходимо дать учащимся несколько заданий на составление системы уравнений по условию задачи, а затем уже переходить непосредственно к решению задач.

1. Запишите с помощью системы уравнений следующую ситуацию:

а) Сумма двух чисел равна 17. Одно из них на 7 меньше другого.

б) Периметр прямоугольника равен 400 м. Его длина в 3 раза больше ширины.

в) Четыре боксёра тяжёлого веса и пять боксёров лёгкого веса вместе весят 730 кг. Спортсмен тяжелого веса весит на 70 кг больше спортсмена лёгкого веса.

г) Таня заплатила за 3 тетради и 2 карандаша 58 р., а Лена за 3 такие же тетради и 1 карандаш – 78 р.

2. № 1099, № 1101.

3. № 1103.

4. № 1104.

Решение:

Пусть ослица несла х мешков, а мул нёс у мешков. Если ослица отдаст 1 мешок мулу, то у неё останется х – 1 мешок, а у мула станет у + 1 мешок. По условию у мула станет в 2 раза больше мешков, чем у ослицы, то есть получим уравнение: у + 1 = 2(х – 1).

Если мул отдаст 1 мешок ослице, то у него останется у – 1 мешок, а у ослицы станет х + 1 мешок. По условию в этом случае количество мешков у них станет равным, то есть получим уравнение: у – 1 = х + 1.

В итоге имеем систему уравнений:

x + 2 – 2x = –3;

х = –5;

х = 5;

у = 5 + 2;

у = 7.

Ответ: 5 и 7 мешков.

IV. Итоги урока.

– Какие существуют способы решений систем уравнений с двумя переменными? Опишите каждый из них.

– Как решаются задачи с помощью составления системы уравнений?

– Придумайте ситуацию, которая описывается следующей системой уравнений:

Домашнее задание: № 1100, № 1102, № 1105.

 

 

 

 

 

 

 

 

 

 

 

 

 

 

 

Урок                                    Дата
ТЕМА: Решение задач «на движение» с помощью систем уравнений

Цели: продолжить формирование умения решать задачи с помощью систем уравнения, уделив особое внимание задачам «на движение»; проверить уровень усвоения материала.

Ход урока

I. Устная работа.

1. Являются ли данные системы уравнений равносильными:

а)            и       

б)            и       

2. Придумайте ситуацию, которая описывается следующей системой уравнений:

а)                  б)

II. Формирование умений и навыков.

Сначала необходимо актуализировать знания учащихся. Они должны вспомнить, как используется таблица при решении задач «на движение» и какая существует зависимость между величинами s, х и t.

1. № 1108.

2. № 1110.

Решение:

Обозначим скорости автомобилей через х км/ч и у км/ч. Выделим процессы: движение автомобилей навстречу друг другу и движение в одном направлении. Соответственно заполним две таблицы.

Движение навстречу

 

s

х

t

1-й автомобиль

2х км

х км/ч

2 ч

2-й автомобиль

2у км

у км/ч

2 ч

Получаем уравнение: 2х + 2у = 280.

Движение в одном направлении

 

s

х

t

1-й автомобиль

14х км

х км/ч

14 ч

2-й автомобиль

14у км

у км/ч

14 ч

Получаем уравнение: 14х – 14у = 280.

Составим и решим систему уравнений:

2х = 160;

х = 80;

80 – у = 20;

у = 60.

Ответ: 80 км/ч и 60 км/ч.

3. № 1111.

4. № 1113.

Решение:

Пусть х км/ч – собственная скорость теплохода, а у км/ч – скорость течения реки. Выделим процессы: движение теплохода по течению и против течения реки в первом и во втором случаях.

 

s

х

t

по течению

3 (х + у) км

(х + у) км/ч

3 ч

против течения

4 (ху) км

(ху) км/ч

4 ч

Получим уравнение: 3 (х + у) + 4 (ху) = 380.

 

s

х

t

по течению

(х + у) км

(х + у) км/ч

1 ч

против течения

0,5 (ху) км

(ху) км/ч

0,5 ч

Получим уравнение: (х + у) + 0,5 (ху) = 85.

Составим и решим систему уравнений:

10х = 550;

х = 55;

3 · 55 + у = 170;

у = 170 – 165;

у = 5.

Ответ: 55 км/ч и 5 км/ч.

 

 

III. Проверочная работа.

Вариант 1

1. У Толи 18 монет по 2 р. и по 5 р. на сумму 97 р. Сколько монет каждого достоинства у Толи?

2. Поезд прошёл первый перегон за 2 ч, а второй за 3 ч. Всего за это время он прошёл 330 км. Найдите скорость поезда на каждом перегоне, если на втором перегоне она была на 10 км/ч больше, чем на первом.

Вариант 2

1. У Лены 8 монет по 10 р. и 5 р. Сколько у неё десятирублёвых и сколько пятирублёвых монет, если всего у неё 65 р.?

2. Туристы прошли 24 км, причём 3 ч дорога шла в гору, а 2 ч – под гору. С какой скоростью туристы шли в гору и с какой под гору, если на первом участке их скорость была на 2 км/ч меньше, чем на втором?

IV. Итоги урока.

– Как решаются задачи с помощью систем уравнений?

– Как используется таблица при решении задач «на движение»?

Домашнее задание: № 1106, № 1109, № 1112.

 

 

 

 

 

 

 

 

 

 

 

 

 

 

Урок                                 Дата
ТЕМА: Решение задач

Цели: закрепить умение учащихся решать задачи с помощью систем уравнений; подготовить учащихся к контрольной работе.

Ход урока

I. Устная работа.

Придумайте задачу, для решения которой нужно составить систему уравнений:

II. Формирование умений и навыков.

1. № 1107.

Решение:

Пусть первый автомат изготовлял в час х деталей, а второй – у деталей. Заполним таблицу:

 

А

работа

k

производительность

t

время

первый автомат

3х дет.

х дет./ч

3 ч

второй автомат

2у дет.

у дет./ч

2 ч

совместная работа

2 (х + у) дет.

(х + у) дет./ч

2 ч

Составим и решим систему уравнений:

3х + 600 – 2х = 720;

х = 120;

2у = 600 – 2 · 120 = 360;

у = 180.

Ответ: 120 и 180 деталей.

2. № 1115.

Решение:

Пусть слиток золота весит х г, а слиток серебра весит у г. Согласно условию 9 слитков золота и 11 слитков серебра весят одинаково. Получим уравнение: 9х = 11у.

После того как поменяли местами один слиток золота с одним слитком серебра, на левой чаше оказалось 8 слитков золота и 1 слиток серебра, их общая масса равна (8х + у) г. На правой чаше стало 10 слитков серебра и 1 слиток золота, их общая масса равна (10у + х) г. По условию левая чаша на 13 г легче правой, значит, получим уравнение:

(10у + х) – (8х + у) = 13.

Составим и решим систему уравнений:

9yy = 13;

81y – 77y = 117;

4у = 117;

у = 29,25;

х = ;

х = 35,75.

Ответ: 35,75 г и 29, 25 г.

3. № 1118.

Решение:

Пусть первая бригада по плану за месяц должна была изготовить х деталей, а вторая бригада – у деталей. По условию вместе они должны за месяц изготовить 680 деталей, то есть получим уравнение: х + у = 680.

Первая бригада, перевыполняя план, изготовила за месяц на 0,2х деталей больше, а вторая – на 0,15у деталей больше. По условию сверх плана было изготовлено 118 деталей, то есть получим уравнение:

0,2х + 0,15у = 118.

Составим и решим систему уравнений:

0,2 (680 – у) + 0,15у = 118;

136 – 0,2у + 0,15у = 118;

–0,05у = –18;

у = 360;

х = 680 – 360;

х = 320.

Ответ: 320 и 360 деталей.

Если останется время, можно предложить учащимся задачи повышенного уровня сложности.

4*. № 1120.

Решение:

Пусть на вклад «Депозитный» клиент положил х р., а на вклад «До востребования» – у р.

По условию всего клиент положил в банк 45000 р., то есть получим уравнение: х + у = 45000.

Доход от вклада «Депозитный» составил 9 %, то есть 0,09 х р., а от вклада «До востребования» 1 %, то есть 0,01у р. Общий доход клиента по условию равен 3410 р., значит, получим уравнение: 0,09х + 0,01у = 3410.

Составим и решим систему уравнений:

9х + 45000 – х = 341000;

8х = 296000;

х = 37000;

у = 45000 – 37000;

у = 8000.

Ответ: 37000 р. и 8000 р.

5*. № 1121.

Решение:

Пусть 10 %-ного раствора нужно взять х г, а 15 %-ного – у г.

Всего  нужно  получить  80 г  раствора,  то  есть  получим  уравнение:
х + у = 80.

В х г  10 %-ного  раствора  содержится  0,1х г соляной кислоты, а в у г 15 %-ного раствора – 0,15у г соляной кислоты. В результате получили 80 г 12 %-ного раствора, в нём соляной кислоты 80 · 0,12 = 9,6 г.

Получим уравнение: 0,1х + 0,15у = 9,6.

Составим и решим систему уравнений:

80 – у + 1,5у = 96;

0,5у = 16;

у = 32;

х = 80 – 32 ;

х = 48.

Ответ: 48 г и 32 г.

III. Итоги урока.

– Что называется решением системы уравнений с двумя переменными?

– Какие существуют способы решения систем уравнений? Опишите каждый из них.

– Как решить задачу с помощью системы уравнений?

Домашнее задание: № 1114; № 1116; № 1117.

Дополнительно: № 1122.

 

 

Урок 118
Контрольная работа № 9

Вариант 1

1. Решите систему уравнений:

2. Банк продал предпринимателю г-ну Разину 8 облигаций по 2000 р. и 3000 р. Сколько облигаций каждого номинала купил г-н Разин, если за все облигации было заплачено 19000 р.?

3. Решите систему уравнений

4. Прямая y = kx + b проходит через точки А (3; 8) и В (–4; 1). Напишите уравнение этой прямой.

5. Выясните, имеет ли решение система и сколько:

Вариант 2

1. Решите систему уравнений

2. Велосипедист ехал 2 ч по лесной дороге и 1 ч по шоссе, всего он проехал 40 км. Скорость его на шоссе была на 4 км/ч больше, чем скорость на лесной дороге. С какой скоростью велосипедист ехал по шоссе и с какой скоростью по лесной дороге?

3. Решите систему уравнений

4. Прямая y = kx + b проходит через точки А (5; 0) и В (–2; 21). Напишите уравнение этой прямой.

5. Выясните, имеет ли решение система и сколько:

Вариант 3

1. Решите систему уравнений

2. На турбазе имеются палатки и домики, вместе их 25. В каждом домике живут 4 человека, а в палатке – 2 человека. Сколько на турбазе палаток и сколько домиков, если турбаза рассчитана на 70 человек?

3. Решите систему уравнений

4. Прямая y = kx + b проходит через точки А (10; –9) и В (–6; 7). Напишите уравнение этой прямой.

5. Выясните, имеет ли решение система и сколько:

Вариант 4

1. Решите систему уравнений

2. За 15 акций компании «Трансгаз» и 10 акций компании «Суперсталь» заплатили 35000 р. Сколько стоит одна акция каждой компании, если акция «Трансгаза» на 1000 р. дешевле акции «Суперстали»?

3. Решите систему уравнений

4. Прямая y = kx + b проходит через точки А (–2; 11) и В (12; 4). Напишите уравнение этой прямой.

5. Выясните, имеет ли решение система и сколько:

Решение заданий контрольной работы

Вариант 1

1.

6х – 2(3 – 4х) = 1;

6х – 6 + 8х = 1;

14х = 7;

х = 0,5;

у = 3 – 4 · 0,5;

у = 1.

Ответ: (0,5; 1).

2. Пусть  г-н  Разин  купил  х  облигаций  по  2000  р.  и  у  облигаций по 3000 р.

По  условию  всего  он  купил  8  облигаций,  то  есть  получим  уравнение: х + у = 8.

За облигации номинала 2000 р. предприниматель заплатил 2000 х р., а за облигации номинала 3000 р. заплатил 3000у р. Всего за облигации было заплачено 19000 р., то есть получим уравнение: 2000х + 3000у = 19000.

Составим и решим систему уравнений:

2000 (8 – у) + 3000у = 19000;

16000 – 2000у + 3000у = 19000;

1000у = 3000;

у = 3;

х = 8 – 3;

х = 5.

Ответ: 5 облигаций по 2000 р. и 3 облигации по 3000 р.

3.

   

8 (6 – 2у) + 5у = –7;

48 – 16у + 5у = –7;

–11у = –55;

у = 5;

х = 6 – 2 · 5;

х = –4.

Ответ: (–4; 5).

4. Подставляя координаты точек А и В в уравнение y = kx + b, получим систему уравнений:

–4k + 8 – 3k = 1;

–7k = –7;

k = 1;

b = 8 – 3;

b = 5;

у = х + 5.

Ответ: у = х + 5.

5. Выразим в каждом уравнении системы у через х и сравним коэффициенты k и b:

Так как коэффициенты k равны, а b не равны, то прямые параллельны. Значит, система не имеет решений.

Ответ: не имеет.

Вариант 2

1.

2х + 3 (3х – 7) = 1;

2х + 9х – 21 = 1;

11х = 22;

х = 2;

у = 3 · 2 – 7;

у = –1.

Ответ: (2; –1).

2. Пусть по лесной дороге велосипедист ехал со скоростью х км/ч, а по шоссейной – со скоростью у км/ч.

На шоссе его скорость была на 4 км/ч больше, поэтому получим уравнение: ух = 4.

За 2 ч по лесной дороге и 1 ч по шоссе велосипедист проехал (2х + у) км, по условию всего он проехал 40 км. Получим уравнение: 2х + у = 40.

Составим и решим систему уравнений:

3х + 4 = 40;

3х = 36;

х = 12;

у = 4 + 12;

у = 16.

Ответ: 16 км/ч и 12 км/ч.

3.

   

2 (5 – 4х) + х = –11;

10 – 8х + х = –11;

–7х = –21;

х = 3;

у = 5 – 4 · 3;

у = –7.

Ответ: (3; –7).

4. Подставляя координаты точек А и В в уравнение y = kx + b, получим систему уравнений:

–7k = 21;

k = –3;

b = –5 · (–3);

b = 15.

Ответ: у = –3х + 15.

5. Выразим в каждом уравнении системы у через х и сравним коэффициенты k и b:

Получили два одинаковых уравнения, значит, система имеет бесконечное множество решений.

Ответ: имеет бесконечное множество решений.

Вариант 3

1.

4 (4у – 9) + 3у = 2;

16у – 36 + 3у = 2;

19у = 38;

у = 2;

х = 4 · 2 – 9;

х = –1.

Ответ: (–1; 2).

2. Пусть на турбазе х палаток и у домиков.

По условию их всего 25, то есть получаем уравнение: х + у = 25.

В домиках живут 4у человек, а в палатках 2х человек. Всего на турбазе находится 70 человек. Получим уравнение: 2х + 4у = 70.

Составим и решим систему уравнений:

25 + у = 35;

у = 10;

х = 25 – 10;

х = 15.

Ответ: 15 палаток и 10 домиков.

3.

   

8у = 16;

у = 2;

3х + 10 = 26;

3х = 16;

х = 5.

Ответ: .

4. Подставляя координаты точек А и В в уравнение y = kx + b, получим систему уравнений:

–6k – 9 – 10k = 7;

–16k = 16;

k = –1;

b = –9 – 10 · (–1);

b = 1.

Ответ: у = –х + 1.

5. Выразим в каждом уравнении системы у через х и сравним коэффициенты k и b:

Так как коэффициенты k равны, а b не равны, то прямые параллельны. Значит, система не имеет решений.

Ответ: не имеет.

Вариант 4

1.

3 (–4у – 4) – 2у = 16;

–12у – 12 – 2у = 16;

–14у = 28;

у = –2;

х = –4 · (–2) – 4;

х = 4.

Ответ: (4; –2).

2. Пусть одна акция «Трансгаза» стоит х р., а одна акция «Суперстали» стоит у р.

Известно, что акция «Трансгаза» на 1000 р. дешевле, поэтому получим уравнение: ух = 1000.

За  15  акций  «Трансгаза»  было  заплачено  15х р., а за 10 акций «Суперстали» – 10у р. Известно, что всего заплатили 35000. Получим уравнение: 15х + 10у = 35000.

Составим и решим систему уравнений:

15х + 10 (1000 + х) = 35000;

15х + 10000 + 10х = 35000;

25х = 25000;

х = 1000;

у = 1000 + 1000;

у = 2000.

Ответ: 1000 р. и 2000 р.

3.

   

х + 2 (2х + 8) = 6;

х + 4х + 16 = 6;

5х = –10;

х = –2;

у = 2 · (–2) + 8;

у = 4.

Ответ: (–2; 4).

4. Подставляя координаты точек А и В в уравнение y = kx + b, получим систему уравнений:

14k + 11 = 4;

14k = –7;

k = –0,5;

b = 2 · (–0,5) + 11;

b = 10.

Ответ: у = –0,5х + 10.

5. Выразим в каждом уравнении системы у через х и сравним коэффициенты k и b:

Получили два одинаковых уравнения, значит, система имеет бесконечное множество решений.

Ответ: имеет бесконечное множество решений.

 

 

 

Урок 119
Линейные неравенства
с двумя переменными и их системы

Цели: ввести понятия линейного неравенства с двумя переменными и системы таких неравенств; формировать умение их решать.

Ход урока

I. Актуализация знаний.

Необходимо, чтобы учащиеся сначала вспомнили, что такое неравенство, как оно записывается и как читается. Также они должны знать, что неравенства бывают верными и неверными.

Задание. Какие из следующих неравенств верные, а какие неверные:

а) 5 < 7,1;                 в)  < 0,25;                  д) –1,63 > –2,1;

б) –4 > 2;                  г) –10 < –15;                  е)  > –0,9?

Далее нужно дать задание, в котором раскрывается понятие решения неравенства с одной переменной.

Задание. Какие  из  следующих  чисел  являются  решением  неравенства 2x + 1 < 4:

–3;  2;  0;  2,7;  –1;  1;  –2,5?

II. Изучение нового материала.

Изучение  проходит  согласно  пункту  46  учебника  в  несколько
этапов.

1. Ввести понятие неравенства с двумя переменными и его решения.

2. Привести примеры неравенств с двумя переменными и показать графическую иллюстрацию их решения.

3. Ввести понятие системы неравенств с двумя переменными и её решения.

4. Показать, как на координатной плоскости отыскивается решение системы неравенств.

III. Закрепление изученного материала.

1. № 1128.

2. № 1130.

Решение:

а) ух + 1                                            б) у < –0,2х + 3

                        

3. № 1132.

4. № 1133.

Решение:

а)

б)

в)

5. № 1135.

Решение:

IV. Итоги урока.

Домашнее задание: № 1129, № 1131, № 1134, № 1136.

 

 

Уроки 120
Обобщающее повторение

Обобщающее повторение проводится по следующим темам:

1. Линейное уравнение с одной переменной.

2. Системы линейных уравнений с двумя переменными.

3. Линейная функция и её график.

4. Степень с натуральным показателем. Одночлен.

5. Многочлены и действия с ними.

6. Формулы сокращенного умножения.

Желательно обобщающее повторение проводить в виде коллективного решения контрольно-измерительных материалов (КИМ) по выделенным темам (см. список литературы).

 

Урок 126
Итоговая контрольная работа

Вариант 1

1. Упростите выражение (a + 6)2 – 2a (3 – 2a).

2. Решите систему уравнений

3. а) Постройте график функции y = 2x – 2.

    б) Определите, проходит ли график функции через точку А
(–10; –20).

4. Разложите на множители:

а) 2a4b3 – 2a3b4 + 6a2b2;               б) x2 – 3x – 3yy2.

5. Из пункта А вниз по реке отправился плот. Через 1 ч навстречу ему из пункта В, находящегося в 30 км от А, вышла моторная лодка, которая встретилась с плотом через 2 ч после своего выхода. Найдите собственную скорость лодки, если скорость течения реки 2 км/ч.

Вариант 2

1. Упростите выражение (x – 2)2 – (x – 1) (x + 2).

2. Решите систему уравнений

3. а) Постройте график функции y = –2x + 2.

    б) Определите, проходит ли график функции через точку А (10; –18).

4. Разложите на множители:

а) 3x3y3 + 3x2y4 – 6xy2;                 б) 2a + a2b2 – 2b.

5. Из поселка на станцию, расстояние между которыми 32 км, выехал велосипедист. Через 0,5 ч навстречу ему со станции выехал мотоциклист и встретил велосипедиста через 0,5 ч после своего выезда. Известно, что скорость мотоциклиста на 28 км/ч больше скорости велосипедиста. Найдите скорость каждого из них.

Вариант 3

1. Упростите выражение 2x (2x + 3y) – (x + y)2.

2. Решите систему уравнений

3. а) Постройте график функции y = 2x + 2.

    б) Определите, проходит ли график функции через точку А
(–10; –18).

4. Разложите на множители:

а) 2a3x3 – 2a3x2 – 10a2x;               б) a2 + 5a + 5bb2.

5. Из пункта А в пункт В, расстояние между которыми 17 км, вышел пешеход. Через 0,5 ч навстречу ему из пункта В вышел второй пешеход и встретился с первым через 1,5 ч после своего выхода. Найдите скорость каждого пешехода, если известно, что скорость первого на 2 км/ч меньше скорости второго.

Вариант 4

1. Упростите выражение (y – 4) (y + 2) – (y – 2)2.

2. Решите систему уравнений

3. а) Постройте график функции y = –2x – 2.

    б) Определите, проходит ли график функции через точку А (10; –20).

4. Разложите на множители:

а) 3x3y3 – 3x4y2 + 9x2y;                  б) 2xx2 + y2 + 2y.

5. Из пункта А вверх по течению к пункту В, расстояние до которого от пункта А равно 35 км, вышла моторная лодка. Через 0,5 ч навстречу ей из пункта В отплыл плот и встретил моторную лодку через 1,5 ч после своего отправления. Найдите собственную скорость лодки, если скорость течения реки 2 км/ч.

Решение заданий контрольной работы

Вариант 1

1. (a + 6)2 – 2a (3 – 2a) = a2 + 12a + 36 – 6a + 4a2 = 5a2 + 6a + 36.

2.

5х – 2 (4х – 4) = 11;

5х – 8х + 8 = 11;

–3х = 3;

х = –1;

у = 4 · (–1) – 4;

у = –8.

Ответ: (–1; –8).

 

3. а) График функции y = 2x – 2:

    б) А (–10; –20):

    –20 = 2 · (–10) – 2;

    –20 = –22 – неверно.

Ответ: не проходит.

4. а) 2a4b3 – 2a3b4 + 6a2b2 = 2a2b2 (a2bab2 + 3);

    б) x2 – 3x – 3yy2 = (x2y2) – (3x + 3y) = (xy) (x + y) – 3 (x + y) =
= (x + y) (xy – 3).

5. Пусть собственная скорость лодки х км/ч. Выделим процессы: движение плота из пункта А до встречи с лодкой и движение лодки из пункта В до встречи с плотом.

Заполним таблицу:

 

s

х

t

плот

6 км

2 км/ч

3 ч

лодка

2 (х – 2) км

(х – 2) км/ч

2 ч

Составим и решим уравнение:

6 + 2 (х – 2) = 30;

6 + 2х – 4 = 30;

2х = 28;

х = 14.

Ответ: 14 км/ч.

Вариант 2

1. (x – 2)2 – (x – 1) (x + 2) = x2 – 4x + 4 – x2 – 2x + x + 2 = –5x + 6.

2.

3 (2у – 7) + 5у = 12;

6у – 21 + 5у = 12;

11у = 33;

у = 3;

х = 2 · 3 – 7;

х = –1.

Ответ: (–1; 3).

 

3. а) График функции y = –2x + 2:

    б) А (10; –18):

    –18 = –2 · 10 + 2;

    –18 = –18 – верно.

Ответ: проходит.

4. а) 3x3y3 + 3x2y4 – 6xy2 = 3xy2 (x2y + xy2 – 2);

    б) 2a + a2b2 – 2b = (2a – 2b) + (a2b2) = 2 (ab) +
+ (ab) (a + b) = (ab) (2 + a + b).

5. Пусть  скорость  велосипедиста  х км/ч,  тогда скорость мотоциклиста (х + 28) км/ч. Выделим процессы: движение велосипедиста до встречи с мотоциклистом и движение мотоциклиста до встречи с велосипедистом.

Заполним таблицу:

 

s

х

t

велосипедист

х км

х км/ч

1 ч

мотоциклист

0,5 (х + 28) км

(х + 28) км/ч

0,5 ч

Составим и решим уравнение:

х + 0,5 (х + 28) = 32;

х + 0,5х + 14 = 32;

1,5х = 18;

х = 12.

Получаем, что скорость велосипедиста равна 12 км/ч, тогда скорость мотоциклиста равна 12 + 28 = 40 км/ч.

Ответ: 12 км/ч и 40 км/ч.

Вариант 3

1. 2x (2x + 3y) – (x + y)2 = 4x2 + 6xyx2 – 2xyy2 = 3x2 + 4xyy2.

2.

3х + 7 (4х – 9) = –1;

3х + 28х – 63 = –1;

31х = 62;

х = 2;

у = 4 · 2 – 9;

у = –1.

Ответ: (2; –1).

3. а) График функции y = 2x + 2:

    б) А (–10; –18):

   –18 = 2 · (–10) + 2;

   –18 = –18 – верно.

Ответ: проходит.

4. а) 2a3x3 – 2a3x2 – 10a2x = 2a2x (ax2ax – 5);

    б) a2 + 5a + 5bb2 = (a2b2) + (5a + 5b) = (ab) (a + b) +
+ 5 (a + b) = (a + b) (ab + 5).

5. Пусть скорость первого пешехода равна х км/ч, тогда скорость второго пешехода (х + 2) км/ч. Рассмотрим движение обоих пешеходов до встречи.

Заполним таблицу:

 

s

х

t

первый пешеход

2х км

х км/ч

2 ч

второй пешеход

1,5 (х + 2) км

(х + 2) км/ч

1,5 ч

Составим и решим уравнение:

2х + 1,5 (х + 2) = 17;

2х + 1,5х + 3 = 17;

3,5х = 14;

х = 4.

Получаем, что скорость первого пешехода равна 4 км/ч, тогда скорость второго равна 6 км/ч.

Ответ: 4 км/ч и 6 км/ч.

Вариант 4

1. (y – 4) (y + 2) – (y – 2)2 = y2 + 2y – 4y – 8 – y2 + 4y – 4 = 2y – 12.

2.

5 (–8у – 6) – 2у = 12;

–40у – 30 – 2у = 12;

–42у = 42;

у = –1;

х = –8 · (–1) – 6;

х = 2.

Ответ: (2; –1).

3. а) График функции y = –2x – 2:

    б) А (10; –20):

    –20 = –2 · 10 – 2;

    –20 = –22 – неверно.

Ответ: не проходит.

4. а) 3x3y3 – 3x4y2 + 9x2y = 3x2y (xy2x2y + 3);

    б) 2xx2 + y2 + 2y = (2x + 2y) + (y2x2) = 2 (x + y) + (yx) (y + x) =
= (x + y) (2 + yx).

5. Пусть собственная скорость лодки равна х км/ч. Выделим процессы: движение лодки от пункта А до встречи с плотом и движение плота от пункта В до встречи с лодкой.

Заполним таблицу:

 

s

х

t

моторная лодка

2 (х – 2) км

(х – 2) км/ч

2 ч

плот

3 км

2 км/ч

1,5 ч

Составим и решим уравнение:

2 (х – 2) + 3 = 35;

2х – 4 + 3 = 35;

2х = 36;

х = 18.

Ответ: 18 км/ч.

 

 

 

 

 

 

 

 

 

 

 

 

 

 

 

Просмотрено: 0%
Просмотрено: 0%
Скачать материал
Скачать материал "Поурочные планы по алгебре 7 класса"

Методические разработки к Вашему уроку:

Получите новую специальность за 3 месяца

Логопед

Получите профессию

Экскурсовод (гид)

за 6 месяцев

Пройти курс

Рабочие листы
к вашим урокам

Скачать

Скачать материал

Найдите материал к любому уроку, указав свой предмет (категорию), класс, учебник и тему:

6 650 435 материалов в базе

Скачать материал

Другие материалы

Вам будут интересны эти курсы:

Оставьте свой комментарий

Авторизуйтесь, чтобы задавать вопросы.

  • Скачать материал
    • 29.11.2015 4661
    • DOCX 3.7 мбайт
    • Рейтинг: 5 из 5
    • Оцените материал:
  • Настоящий материал опубликован пользователем Ахметшина Расиля Мунаваровна. Инфоурок является информационным посредником и предоставляет пользователям возможность размещать на сайте методические материалы. Всю ответственность за опубликованные материалы, содержащиеся в них сведения, а также за соблюдение авторских прав несут пользователи, загрузившие материал на сайт

    Если Вы считаете, что материал нарушает авторские права либо по каким-то другим причинам должен быть удален с сайта, Вы можете оставить жалобу на материал.

    Удалить материал
  • Автор материала

    Ахметшина Расиля Мунаваровна
    Ахметшина Расиля Мунаваровна
    • На сайте: 9 лет и 5 месяцев
    • Подписчики: 2
    • Всего просмотров: 60910
    • Всего материалов: 33

Ваша скидка на курсы

40%
Скидка для нового слушателя. Войдите на сайт, чтобы применить скидку к любому курсу
Курсы со скидкой

Курс профессиональной переподготовки

Технолог-калькулятор общественного питания

Технолог-калькулятор общественного питания

500/1000 ч.

Подать заявку О курсе

Курс повышения квалификации

Особенности подготовки к проведению ВПР в рамках мониторинга качества образования обучающихся по учебному предмету «Математика» в условиях реализации ФГОС НОО

72 ч. — 180 ч.

от 2200 руб. от 1100 руб.
Подать заявку О курсе
  • Сейчас обучается 70 человек из 29 регионов
  • Этот курс уже прошли 294 человека

Курс повышения квалификации

Внедрение системы компьютерной математики в процесс обучения математике в старших классах в рамках реализации ФГОС

36/72 ч.

от 1700 руб. от 850 руб.
Подать заявку О курсе
  • Сейчас обучается 139 человек из 52 регионов
  • Этот курс уже прошли 493 человека

Курс повышения квалификации

Применение математических знаний в повседневной жизни

36 ч. — 180 ч.

от 1700 руб. от 850 руб.
Подать заявку О курсе
  • Сейчас обучается 29 человек из 16 регионов

Мини-курс

Стартап: от идеи к успеху

6 ч.

780 руб. 390 руб.
Подать заявку О курсе
  • Сейчас обучается 28 человек из 17 регионов

Мини-курс

Самосовершенствование: шаги к личному росту и эмоциональному благополучию

10 ч.

1180 руб. 590 руб.
Подать заявку О курсе
  • Сейчас обучается 255 человек из 60 регионов
  • Этот курс уже прошли 63 человека

Мини-курс

Финансы и управление в медиакоммуникациях

3 ч.

780 руб. 390 руб.
Подать заявку О курсе